Download as pdf or txt
Download as pdf or txt
You are on page 1of 256

ACHIEVE IAS MCQ SERIES, GEOGRAPHY, 50 DAY TIMETABLE

www.achieveias.co.in, YouTube Channel: http://youtube.com/c/AchieveIAS Telegram Channel:


http://t.me/Achieve_Ias, Mail: achieveias21@gmail.com, Contact Number: 8968920720
DAY NCERT CHAPTERS TO BE
COVERED
1. 6th Class Chapters 1-4
2. 6th Class Chapters 5-8
3. 7th Class Chapters 1-3
4. 7th Class Chapters 4-5
5. 7th Class Chapters 6-7
6. 7th Class Chapters 8-9
7. 8th Class Chapter 1
8. 8th Class Chapter 2
9. 8th Class Chapter 3
10. 8th Class Chapter 4
11. 8th Class Chapter 5
12. 8th Class Chapter 6
13. 9th Class Chapter 1
14. 9th Class Chapter 2
15. 9th Class Chapter 3 and 4
16. 9th Class Chapters 5
17. 10th Class Chapter 6
18. 10th Class Chapters 1
19. 10th Class Chapter 2
20. 10th Class Chapter 3
21. 10th Class Chapter 4
22. 10th Class Chapter 5
23. 10th Class Chapter 6
24. 10th Class Chapter 7
25. 11th Class, Chapters 2 & 3
Fundamentals of
Physical Geography
26. 11th Class, Chapters 4 & 5
Fundamentals of
Physical Geography
ACHIEVE IAS MCQ SERIES, GEOGRAPHY, 50 DAY TIMETABLE
www.achieveias.co.in, YouTube Channel: http://youtube.com/c/AchieveIAS Telegram Channel:
http://t.me/Achieve_Ias, Mail: achieveias21@gmail.com, Contact Number: 8968920720
27. 11th Class, Chapters 6 & 7
Fundamentals of
Physical Geography
28. 11th Class, Chapters 8 & 9
Fundamentals of
Physical Geography
29. 11th Class, Chapters 10 & 11
Fundamentals of
Physical Geography
30. 11th Class, Chapters 12 & 13
Fundamentals of
Physical Geography
31. 11th Class, Chapter 14 & 15
Fundamentals
of
Physical Geography
32. 11th Class, Chapter 16
Fundamentals
of
Physical Geography
33. 11th Class, India - Chapter 1
Physical Environment
34. 11th Class, India - Chapter 2
Physical Environment
35. 11th Class, India - Chapter 3
Physical Environment
36. 11th Class, India - Chapter 4
Physical Environment
37. 11th Class, India – Chapter 5
Physical Environment
38. 11th Class, India – Chapters 6
Physical Environment
ACHIEVE IAS MCQ SERIES, GEOGRAPHY, 50 DAY TIMETABLE
www.achieveias.co.in, YouTube Channel: http://youtube.com/c/AchieveIAS Telegram Channel:
http://t.me/Achieve_Ias, Mail: achieveias21@gmail.com, Contact Number: 8968920720
39. 11th Class, India – Chapters 7
Physical Environment
40. 12th Class, India Chapters 1 and 2
People and Economy
41. 12th Class, India – Chapters 3 and 4
People and Economy
42. 12th Class, India – Chapters 5 and 6
People and Economy
43. 12th Class, India – Chapter 7 and 8
People and Economy
44. 12th Class, India – Chapter 9 and 10
People and Economy
45. 12th Class, India – Chapter 11 and 12
People and Economy
46. 12th Class, Chapter 1 and 2
Fundamentals of
Human Geography
47. 12th Class, Chapter 3 and 4
Fundamentals of
Human Geography
48. 12th Class, Chapter 5 and 6
Fundamentals of
Human Geography
49. 12th Class, Chapter 7 and 8
Fundamentals of
Human Geography
50. 12th Class, Chapter 9 and 10
Fundamentals of
Human Geography
www.achieveias.co.in, YouTube Channel: http://youtube.com/c/AchieveIAS Telegram Channel: http://t.me/Achieve_Ias, Mail: achieveias21@gmail.com, Contact
Number: 8968920720

ACHIEVE IAS GEOGRAPHY MCQ SERIES, DAY 1, SOLUTIONS


1. Consider the following statements:
1. Moon can only be seen once in a month.
2. The day of full moon is called Poornima.

Which of the above statements is/are correct:


a. 1 Only b. 2 Only c. Both 1 and 2 d. None of the above

Solution: (b)
You may also see the moon on most of the days. It may, however, appear at different times, in different
shapes and at different positions. You can see the full moon only once in about a month’s time. It is
Full moon night or Poornima. A fortnight later, you cannot see it at all. It is a New moon night or
Amavasya. On this day, you can watch the night sky best, provided it is a clear night.

2. Which of the following planets have rings around them:


1. Jupiter
2. Saturn
3. Mars
4. Uranus

Choose from the following options:


a. 1 and 2 Only b. 1, 2 and 3 Only c. 2, 3 and 4 Only d. 1, 2 and 4 Only

Solution: (d)
Jupiter, Saturn and Uranus have rings around them. These are belts of small debris. These rings may
be seen from the earth with the help of powerful telescopes.

3. Consider the following statements:


1. The imaginary horizontal lines drawn on the surface of earth for representing different locations as
measured from the centre of earth are called latitudes.
2. The axis of earth is one of such latitudes.
3. The latitude of equator divides earth into two equal parts.

Which of the above statements is/are correct:


a. 1 and 2 Only b. 2 and 3 Only c. 1 and 3 Only d. All are Correct

Solution: (c)

You will notice that a needle is fixed through the globe in a tilted manner, which is called its axis. Two
points on the globe through which the needle passes are two poles – North Pole and South Pole. The
globe can be moved around this needle from west to east just as the earth moves. But, remember there
is a major difference. The real earth has no such needle. It moves around its axis, which is an imaginary
line. Another imaginary line running on the globe divides it into two equal parts. This line is known as
www.achieveias.co.in, YouTube Channel: http://youtube.com/c/AchieveIAS Telegram Channel: http://t.me/Achieve_Ias, Mail: achieveias21@gmail.com, Contact
Number: 8968920720

the equator. The northern half of the earth is known as the Northern Hemisphere and the southern half
is known as the Southern Hemisphere. They are both equal halves.
Therefore, the equator is an imaginary circular line and is a very important reference point to locate
places on the earth. All parallel circles from the equator up to the poles are called parallels of latitudes.
Latitudes are measured in degrees.
The equator represents the zero degree latitude. Since the distance from the equator to either of the
poles is one-fourth of a circle round the earth, it will measure ¼th of 360 degrees, i.e. 90°. Thus, 90
degrees north latitude marks the North Pole and 90 degrees south latitude marks the South Pole.

Q4. Consider the following statements:


1. Tropic of Cancer and Capricorn represent lines of latitudes.
2. The region of earth between them is called Torrid Zone.

Which of the above statements is/are correct:


a. 1 Only b. 2 Only c. Both 1 and 2 d. None

Solution: (c)

Besides the equator (0°), the North Pole (90°N) and the South Pole (90° S), there are four important
parallels of latitudes–
(i) Tropic of Cancer (23½° N) in the Northern Hemisphere.
(ii) Tropic of Capricorn (23½° S) in the Southern Hemisphere.
(iii) Arctic Circle at 66½° north of the equator.
(iv) Antarctic Circle at 66½° south of the equator.

HEAT ZONES OF THE EARTH


The mid-day sun is exactly overhead at least once a year on all latitudes in between the Tropic of
Cancer and the Tropic of Capricorn. This area, therefore, receives the maximum heat and is called the
Torrid Zone.
The mid-day sun never shines overhead on any latitude beyond the Tropic of Cancer and the Tropic of
Capricorn. The angle of the sun’s rays goes on decreasing towards the poles. As such, the areas
bounded by the Tropic of Cancer and the Arctic Circle in the Northern Hemisphere, and the Tropic of
Capricorn and the Antarctic Circle in the Southern Hemisphere, have moderate temperatures. These
are, therefore, called Temperate Zones.
Areas lying between the Arctic Circle and the North Pole in the Northern Hemisphere and the Antarctic
Circle and the South Pole in the Southern Hemisphere, are very cold. It is because here the sun does
www.achieveias.co.in, YouTube Channel: http://youtube.com/c/AchieveIAS Telegram Channel: http://t.me/Achieve_Ias, Mail: achieveias21@gmail.com, Contact
Number: 8968920720

not rise much above the horizon. Therefore, its rays are always slanting and provide less heat. These
are, therefore, called Frigid Zones (very cold).

Q5. Prime Meridian is a line of longitude representing:


a. Zero Degree Longitude.
b. 180 Degree Longitude.
c. 360 Degree Longitude.
d. None of the Above

Solution: (a)

Unlike parallels of latitude, all meridians are of equal length. Thus, it was difficult to number the
meridians. Hence, all countries decided that the count should begin from the meridian which passed
through Greenwich, where the British Royal Observatory is located. This meridian is called the Prime
Meridian. Its value is 0° longitude and from it we count 180° eastward as well as 180° westward.
The Prime Meridian and 180° meridian divide the earth into two equal halves, the Eastern Hemisphere
and the Western Hemisphere. Therefore, the longitude of a place is followed by the letter E for the east
and W for the west. It is, however, interesting to note that 180° East and 180° West meridians are on
the same line.

Q6. Consider the following statements:


1. When earth moves around its own axis, it is called Revolution.
2. When earth moves in its orbit, it is called Rotation.
3. Rotation results in change of seasons and Revolution results in Day and Nights on earth.

Which of the above statements is/are correct:


a. 3 Only b. 1 and 2 Only c. 1, 2 and 3 d. None of the above.

Solution: (d)
www.achieveias.co.in, YouTube Channel: http://youtube.com/c/AchieveIAS Telegram Channel: http://t.me/Achieve_Ias, Mail: achieveias21@gmail.com, Contact
Number: 8968920720

Rotation is the movement of the earth on its axis. The movement of the earth around the sun in a fixed
path or orbit is called Revolution.
The axis of the earth which is an imaginary line, makes an angle of 66½° with its orbital plane. The
plane formed by the orbit is known as the orbital plane. The earth receives light from the sun. Due to
the spherical shape of the earth, only half of it gets light from the sun at a time. The portion facing the
sun experiences day while the other half away from the sun experiences night. The circle that divides
the day from night on the globe is called the circle of illumination. This circle does not coincide with
the axis. The earth takes about 24 hours to complete one rotation around its axis. The period of rotation
is known as the earthday. This is the daily motion of the earth.

What would happen if the earth did not rotate? The portion of the earth facing the sun would always
experience day, thus bringing continuous warmth to region. The other half would remain in darkness
and be freezing cold all the time. Life would not have been possible in such extreme conditions.
The second motion of the earth around the sun in its orbit is called revolution. It takes 365¼ days (one
year) to revolve around the sun. We consider a year as consisting of 365 days only and ignore six hours
for the sake of convenience. A year is usually divided into summer, winter, spring and autumn seasons.
Seasons change due to the change in the position of the earth around the sun.

Q7. Equinox refers to:


a. Lunar eclipse b. Solar eclipse.
c. Both a and b d. Equal Day and Night on Earth

Solution: (d)
On 21st March and September 23rd, direct rays of the sun fall on the equator. At this position, neither
of the poles is tilted towards the sun; so, the whole earth experiences equal days and equal nights. This
is called an equinox.

Q8. Indian Standard Meridian is:


a. 77.5 Degree East Longitude. b. 82.5 Degree East Longitude.
c. 82.5 Degree West Longitude. d. 87.5 Degree East Longitude.

Solution: (b)
www.achieveias.co.in, YouTube Channel: http://youtube.com/c/AchieveIAS Telegram Channel: http://t.me/Achieve_Ias, Mail: achieveias21@gmail.com, Contact
Number: 8968920720

The local time of places which are on different meridians are bound to differ. For example, it will be
difficult to prepare a time-table for trains which cross several longitudes. In India, for instance, there will
be a difference of about 1 hour and 45 minutes in the local times of Dwarka in Gujarat and Dibrugarh
in Assam. It is, therefore, necessary to adopt the local time of some central meridian of a country as
the standard time for the country. In India, the longitude of 82½° E (82° 30' E) is treated as the standard
meridian. The local time at this meridian is taken as the standard time for the whole country. It is known
as the Indian Standard Time (IST)

Q9. Which of the following is correctly matched:


1. Brightest Planet: Venus
2. Largest Planet: Jupiter
3. Earth’s Twin: Venus

Choose from the following options:


a. 1 and 2 Only b. 2 and 3 Only c. 1 and 3 Only d. All are correctly matched

Solution: (d)

Q10. Hottest Planet in the Solar System:


a. Venus b. Mercury c. Mars d. Jupiter

Solution: (a)
www.achieveias.co.in, YouTube Channel: http://youtube.com/c/AchieveIAS Telegram Channel: http://t.me/Achieve_Ias,
Mail: achieveias21@gmail.com, Contact Number: 8968920720

ACHIEVE IAS GEOGRAPHY MCQ SERIES, DAY 2, SOLUTIONS

1. Consider the following Statements regarding Heat Zones of the earth:


1. The area between the tropic of cancer and Tropic of Capricorn which receive the maximum heat
are called frigid zones.
2. Areas lying between the Arctic Circle and the North Pole in the Northern Hemisphere and the
Antarctic Circle and the South Pole in the Southern Hemisphere, are very cold and are called torrid
zones.

Identify the correct statements:


A. 1 only B. 2 only C. All are correct D. None

Answer: D

Explanation: The area between the tropic of cancer and Tropic of Capricorn which receive the
maximum heat are called torrid zones. Areas lying between the Arctic Circle and the North Pole in
the Northern Hemisphere and the Antarctic Circle and the South Pole in the Southern Hemisphere,
are very cold and are called frigid zones. Areas lying between the Arctic Circle and the North Pole
in the Northern Hemisphere and the Antarctic Circle and the South Pole in the Southern
Hemisphere, are very cold and are called frigid zones.

2. Which is the only continent that lies in all 4 hemispheres:


A. Asia
B. Europe
C. Africa
D. North America

Answer: C

Explanation: Africa is the only continent which lies in all 4 hemispheres due to equator and Prime
Meridian passing through the continent.

3. Which of the following rivers cross/crosses equator twice:


1. Nile
2. Congo
3. Amazon

Select the correct answer using the code given below


A. 1 only B. 2 only C. 2 and 3 only D. 3 only

Answer: B

Explanation: Congo is the only river in the world that crosses equator twice.

4. With reference to Great Barrier Reef, consider the following statements:


1. It is the world’s largest coral reef system
2. It can be seen from outer space
3. It is presently threatened due to coral bleaching
www.achieveias.co.in, YouTube Channel: http://youtube.com/c/AchieveIAS Telegram Channel: http://t.me/Achieve_Ias,
Mail: achieveias21@gmail.com, Contact Number: 8968920720

Select the correct answer using the code given below


A. 1 only B. 1 and 2 only C. 1 and 3 only D. All of the above

Answer: D

Explanation: Great Barrier Reef, which is world heritage site has been threatened due to coral
bleaching, a result of increasing temperatures of oceans caused by global warming. Coral bleaching
occurs when algae is removed from coral polyps which otherwise has a symbiotic relationship.

5. Which is the only continent without deserts:


A. Australia B. North America C. South America D. Europe

Answer: D

Explanation:
Australia is 2/3rd covered with deserts. Great Basin, Sonoran and Mojave are deserts of North
America. Atacama and Patagonian are deserts of South America.

6. Consider the following pairs:


1. Pampas Argentina
2. Prairies North America

Select the correct answer using the code given below


A. 1 only B. 2 only C. Both 1 and 2 D. Neither 1 nor 2

Answer: C

Explanation:
Grasslands in different areas take different names:
Pampas : Argentina
Prairies : North America
Downs : Australia
Steppes : Asia
Veld : Africa

7. Which of the following river(s) form(s) the Northern plains of India:


1. Indus
2. Ganga
3. Brahmaputra

Select the correct answer using the code given below


A. 1 and 2 only B. 2 only C. 2 and 3 only D. All of the above

Answer: D

Explanation: Indus, Ganga, Brahmaputra bring alluvial deposits forming the Northern Plains of
India.
www.achieveias.co.in, YouTube Channel: http://youtube.com/c/AchieveIAS Telegram Channel: http://t.me/Achieve_Ias,
Mail: achieveias21@gmail.com, Contact Number: 8968920720

8. Consider the following statements:


1. China has only one time zone
2. North Pole and South Pole have no time zones
3. International space station uses GMT

Select the correct answer using the code given below:


A. 1 only B. 2 and 3 only C. 2 only D. All of the above

Answer: D

Explanation: Inspite of its vast geographical extent, China decided to follow a single time zone for
unifying its country and its people soon after the communist party assumed power.
Since all longitudes converge at North and South Pole, they have no time zone. Research stations
follow the time of their countries or from where they set off their expedition.
ISS follows Greenwich Mean Time.

9. Which of the following forests are called monsoon forests:


A. Tropical rain forests B. Tropical deciduous forests
C. Coniferous forests D. Mangrove forests

Answer: B

Explanation: Tropical deciduous forests care called monsoon forests because they shed their
leaves during the dry season and bear leaves at the start of the rainy season or monsoon season.

10. Where was the largest crater made by a meteorite:


A. UK B. UAE C. South Africa D. INDIA

Answer: C

Explanation: The oldest impact crater on Earth is also the largest. Vredefort crater in South Africa,
also called the Vredefort Dome, was originally 185 miles (300 kilometers) across, scientists
estimate. A meteorite or asteroid bigger than South Africa's Table Mountain blasted out the giant
crater 2.02 billion years ago.
www.achieveias.co.in, YouTube Channel: http://youtube.com/c/AchieveIAS Telegram Channel: http://t.me/Achieve_Ias,
Mail: achieveias21@gmail.com, Contact Number: 8968920720

ACHIEVE IAS GEOGRAPHY MCQ SERIES, DAY 3, SOLUTIONS

1. Which of the following are covered under the environment?


1. Man
2. Field topography
3. Man-made items
4. Natural Goods

Code:
A. 1, 2 and 3 B. 1, 3 and 4 C. 2 and 4 D. All of the above

Answer: D

Explanation: The place, people, things and nature that surrounds any living organism is called as
environment. It is a combination of natural and human made phenomena. While the natural
environment refers to both biotic and abiotic conditions existing on the earth, human environment
reveals the activities, creations and interactions among human beings.

2. Which of the following assertions is/are true?


1. The gravitational force of the earth holds the atmosphere around it.
2. The changes in the atmosphere produce changes in the weather and climate.
3. Plant and animal kingdom together make biosphere or the living world.

A. 1 and 2 B. 2 and 3 C. 1 and 3 D. All of the above

Answer: D

Explanation The thin layer of air which surrounds the earth is called the atmosphere. The
gravitational force of the earth holds the atmosphere around it. It protects us from the harmful rays
and scorching heat of the sun. It consists of a number of gases, dust and water vapour. The changes
in the atmosphere produce changes in the weather and climate. Plant and animal kingdom together
make biosphere or the living world. It is a narrow zone of the earth where land, water and air interact
with each other to support life.

3. The term ‘ENVIRONMENT’ is originated from which language?


A. French B. German C. Sanskrit D. English

Answer: A

Explanation: The term ‘environ’ – ‘environment’ is derived from the French word ‘Environer’ /
‘Environner’, meaning “neighbourhood” or to enclose, surround, or encircle.

4. Which of the following assertions is the correct explanation of 'ecosystem'?


A. Community of organisms interacting with each other.
B. The place in the Earth where living organisms live.
C. The community of organisms as well as the environment in which they live.
D. Flora and fauna of any geographical area.
www.achieveias.co.in, YouTube Channel: http://youtube.com/c/AchieveIAS Telegram Channel: http://t.me/Achieve_Ias,
Mail: achieveias21@gmail.com, Contact Number: 8968920720

Answer: C
Explanation: Ecosystem is a system formed by the interaction of all living organisms with each
other and with the physical and chemical factors of the environment in which they live, all linked by
transfer of energy and material.

5. Read the assertions given below carefully?


1. The uppermost layer over the earth surface is called as the “crust” and it is the thinnest of all the
layers.
2. The thickness of the ocean crust is more than the continental crust.

Which of the above assertions is/are true?


A. 1 only B. 2 only C. Both 1 and 2 D. Neither 1 nor 2

Answer: A

Explanation The uppermost layer over the earth’s surface is called the crust. It is the thinnest of all
the layers. It is about 35 km. on the continental masses and only 5 km. on the ocean floors. That is,
the thickness of the continental crust is more than the ocean crust.

6. Consider the following assertions?


1. When the molten lava comes on the earth’s surface, it rapidly cools down and becomes solid.
Rocks formed in such a way on the crust are called extrusive igneous rocks.
2. When the molten magma cools down deep inside the earth’s crust. Solid rocks so formed are
called intrusive igneous rocks.
3. Extrusive igneous rocks form large grains.
4. Intrusive igneous rocks have a very fine grained structure.

Which of the above assertions is/are true?


A. 1, 3 and 4 B. 2, 3 and 4 C. 1 and 2 D. All of the above

Answer: C

Explanation: The assertions 3 and 4 are wrong. The molten lava of the extrusive igneous rocks
gets rapidly cooled downwards. Therefore, they have a very fine grained structure. For example,
basalt. The Deccan plateau is made up of basalt rocks. While the intrusive igneous rocks, cool down
deep inside the earth’s crust. Due to this, they form large grains. Granite is an example of such a
rock. Grinding stones used to prepare paste/powder of spices and grains are made of granite.

7. Which of the following is the speciality of sedimentary rocks?


1. Sedimentary rocks are formed by the deposition of various types of sediments.
2. There is no possibility of finding fossils in these rocks.
3. These rocks are relatively less rigid.

A. 1 and 2 only B. 2 and 3 only C. 1 and 3 only D. 1, 2 and 3

Answer: C
www.achieveias.co.in, YouTube Channel: http://youtube.com/c/AchieveIAS Telegram Channel: http://t.me/Achieve_Ias,
Mail: achieveias21@gmail.com, Contact Number: 8968920720

Explanation: Rocks roll down, crack, and hit each other and are broken down into small fragments.
These smaller particles are called sediments. These sediments are transported and deposited by
wind, water, etc. These loose sediments are compressed and hardened to form layers of rocks.
These types of rocks are called sedimentary rocks. For example, sandstone is made from grains of
sand. These rocks may also contain fossils of plants, animals and other microorganisms that once
lived on them.

8. Which of the following statements is/are true?


1. During the earthquake, the place on the crust where the movement starts is called the epicentre.
2. The vent (opening) in the earth’s crust through which molten material erupts suddenly is called
crater.

A. 1 only B. 2 only C. Both 1 and 2 D. Neither 1 nor 2

Answer: B

Explanation: When the Lithospheric plates move, the surface of the earth vibrates. The vibrations
can travel all around the earth. These vibrations are called earthquakes. The place in the crust where
the movement starts is called the focus. The place on the surface directly above the focus is called
the epicentre. Vibrations travel outwards from the epicentre as waves. Greatest damage is usually
closest to the epicentre and the strength of the earthquake decreases away from the centre.

9. In which region does the river form Ox-bow lakes?


A. In the mountainous areas B. In the coastal parts
C. In the plains D. In the above three parts

Answer: C

Explanation: As the river enters the plain it twists and turns forming large bends known
as meanders. Due to continuous erosion and deposition along the sides of the meander, the ends
of the meander loop come closer and closer. In due course of time the meander loop cuts off from
the river and forms a cut-off lake, also called an ox-bow lake. At times the river overflows its banks.
This leads to the flooding of the neighbouring areas. As it floods, it deposits layers of fine soil and
other material called sediments along its banks. This leads to the formation of a flat fertile floodplain.
The raised banks are called levees. As the river approaches the sea, the speed of the flowing water
decreases and the river begins to break up into a number of streams called distributaries. The river
becomes so slow that it begins to deposit its load. Each distributary forms its own mouth. The
collection of sediments from all the mouths forms a delta.

10. Which of the following coastal landforms is/are created by the sea waves?
1. Sea Caves
2. Sea arches
3. Stacks
4. Sea cliff

Code:
A. 1 and 2 B. 2 and 3 C. 1, 3 and 4 D. All of the above
www.achieveias.co.in, YouTube Channel: http://youtube.com/c/AchieveIAS Telegram Channel: http://t.me/Achieve_Ias,
Mail: achieveias21@gmail.com, Contact Number: 8968920720

Answer: D

Explanation: The erosion and deposition of the sea waves gives rise to coastal landforms. Sea
waves continuously strike at the rocks. Cracks develop. Over time they become larger and wider.
Thus, hollow like caves are formed on the rocks. They are called sea caves. As these cavities
become bigger and bigger only the roof of the caves remain, thus forming sea arches. Further,
erosion breaks the roof and only walls are left. These wall like features are called stacks. The steep
rocky coast rising almost vertically above sea water is called sea cliff. The sea waves deposit
sediments along the shores forming beaches.
www.achieveias.co.in, YouTube Channel: http://youtube.com/c/AchieveIAS Telegram Channel: http://t.me/Achieve_Ias,
Mail: achieveias21@gmail.com, Contact Number: 8968920720

ACHIEVE IAS GEOGRAPHY MCQ SERIES, DAY 4, SOLUTIONS

1. What is the correct sequence of gases found in the atmosphere on the basis of the
percentage in descending order?
A. Nitrogen > oxygen > helium > carbon dioxide
B. Nitrogen > oxygen > carbon dioxide > argon
C. Nitrogen > oxygen > argon > carbon dioxide
D. Nitrogen > oxygen > methane > carbon dioxide

Answer: C

Explanation: Atmosphere is a mixture of many gases in which —


Nitrogen: 78%
Oxygen: 21%
Argon: 0.93%
Carbon dioxide: 0.03%
All other gases: 0.04%

2. Consider the following assertions:


1. Nitrogen is the most plentiful gas in the air.
2. When we inhale, we take some amount of nitrogen into our lungs and exhale it.
3. Plants fulfil their need of nitrogen, directly from the air.

Which of the above assertions is/are true?


A. 1 and 2 B. 2 and 3 C. 1 and 3 D. 1, 2 and 3

Answer: A

Explanation: Nitrogen is the most plentiful gas in the air. When we inhale, we take some amount
of nitrogen into our lungs and exhale it. But plants need nitrogen for their survival. They cannot take
nitrogen directly from the air. Bacteria that live in the soil and roots of some plants take nitrogen
from the air and change its form so that plants can use it.

3. Which of the following combinations is/are right?


1. Winds that blow constantly throughout the year in a particular direction – Local winds
2. Winds that change their direction in different seasons – Seasonal winds
3. Winds that blow only during a particular period of the day or year in a small area – Permanent
winds

A. 1 only B. 2 only C. 1 and 3 D. 2 and 3

Answer: B

Explanation: a. Permanent winds – The trade winds, westerlies and easterlies are the permanent
winds. These blow constantly throughout the year in a particular direction.
b. Seasonal winds – These winds change their direction in different seasons. For example
monsoons in India.
www.achieveias.co.in, YouTube Channel: http://youtube.com/c/AchieveIAS Telegram Channel: http://t.me/Achieve_Ias,
Mail: achieveias21@gmail.com, Contact Number: 8968920720

c. Local winds – These blow only during a particular period of the day or year in a small area. For
example, land and sea breeze; and loo, the hot and dry local wind of northern plains of India.

4. Read the following assertions carefully:


1. Horizontally the distribution of air pressure is influenced by temperature of air at a given place.
2. The low pressure area is associated with cloudy sky and wet weather.
3. High pressure is associated with clear and sunny skies.
4. Air always moves from high pressure areas to low pressure areas.

Which of the above assertions is/are true?


A. 1 and 2 only B. 2, 3 and 4 only C. 1, 3 and 4 only D. All of the above

Answer: D

Explanation: Air pressure is defined as the pressure exerted by the weight of air on the earth’s
surface. The air pressure is highest at sea level and decreases with height. Horizontally the
distribution of air pressure is influenced by temperature of air at a given place. In areas where
temperature is high the air gets heated and rises. This creates a low-pressure area. Low pressure
is associated with cloudy skies and wet weather. In areas having lower temperature, the air is cold.
It is therefore heavy. Heavy air sinks and creates a high pressure area. High pressure is associated
with clear and sunny skies. The air always moves from high pressure areas to low pressure areas.

5. All the weather phenomena like rainfall, fog and hailstorm occur in which layer?
A. Troposphere B. Stratosphere C. Mesosphere D. Thermosphere

Answer: A

Explanation: The troposphere is the most important layer of the atmosphere. Its average height is
13 kilometers. The air we breathe exists here. Almost all the weather phenomena like rainfall, fog
and hailstorm occur in this layer. The troposphere is the most important layer of the atmosphere. Its
average height is 13 km. The air we breathe exists here. Almost all the weather phenomena like
rainfall, fog and hailstorm occur in this layer. The stratosphere is almost free from clouds and
associated weather phenomenon, making conditions most ideal for flying aeroplanes.
Mesosphere is the third layer of the atmosphere. It lies above the stratosphere. It extends up to the
height of 80 km. Meteorites burn up in this layer on entering from the space.
The temperature rises very rapidly with increasing height in the thermosphere. The ionosphere is a
part of the thermosphere. It extends between 80 and 400 km.

6. Which of the following areas is/are considered to be the best fishing zone in the world?
A. The areas where cold currents flows.
B. The areas where hot currents flow.
C. The areas where the warm and cold currents meet.
D. The area of the sea that is around the equator.

Answer: C

Explanation: The areas where the warm and cold currents meet provide the best fishing grounds
of the world. Seas around Japan and the eastern coast of North America are such examples. The
www.achieveias.co.in, YouTube Channel: http://youtube.com/c/AchieveIAS Telegram Channel: http://t.me/Achieve_Ias,
Mail: achieveias21@gmail.com, Contact Number: 8968920720

areas where a warm and cold current meet also experience foggy weather making it difficult for
navigation.

7. Which of the following processes is/are included in the water cycle?


1. Evaporation
2. Condensation
3. Precipitation
A. 1 and 2 only
B. 2 and 3 only
C. 3 and 1 only
D. All of the above

Answer: D

Explanation: The sun’s heat causes evaporation of water into vapour. When the water vapour cools
down, it condenses and forms clouds. From there it may fall on the land or sea in the form of rain,
snow or sleet. The process by which water continually changes its form and circulates between
oceans, atmosphere and land is known as the water cycle.

8. The main cause of the origin of tides in the ocean are:


A. Gravitational force of the sun
B. Moon's gravitational force
C. Due to the force of gravity of both the sun and the moon
D. Sea water streams

Answer: C

Explanation: The rhythmic rise and fall of ocean water twice in a day is called a tide. It is high tide
when water covers much of the shore by rising to its highest level. It is low tide when water falls to
its lowest level and recedes from the shore.The strong gravitational pull exerted by the sun and the
moon on the earth’s surface causes the tides. The water of the earth closer to the moon gets pulled
under the influence of the moon’s gravitational force and causes high tide.

9. Which of the following benefits is/are related to high tide?


1. High tides help in navigation.
2. The high tides also help in fishing.
3. The rise and fall of water due to tides is being used to generate electricity in some places.
A. 1 and 2 only
B. 1 and 3 only
C. 2 and 3 only
D. All of the above

Answer: D

Explanation: High tides help in navigation. They raise the water level close to the shores. This helps
the ships to arrive at the harbour more easily. The high tides also help in fishing. Many more fish
come closer to the shore during the high tide. This enables fishermen to get a plentiful catch. The
rise and fall of water due to tides is being used to generate electricity in some places.
www.achieveias.co.in, YouTube Channel: http://youtube.com/c/AchieveIAS Telegram Channel: http://t.me/Achieve_Ias,
Mail: achieveias21@gmail.com, Contact Number: 8968920720

10. The word tsunami is originated from which language?


A. Chinese language B. French language
C. German language D. Japanese language

Answer: D

Explanation: Tsunami is a Japanese word that means “Harbour waves” as the harbours get
destroyed whenever there is tsunami.
www.achieveias.co.in, YouTube Channel: http://youtube.com/c/AchieveIAS Telegram Channel: http://t.me/Achieve_Ias,
Mail: achieveias21@gmail.com, Contact Number: 8968920720

ACHIEVE IAS GEOGRAPHY MCQ SERIES, DAY 5, SOLUTIONS

1. Regarding the human settlements, consider the following assertions:


1. The places where a building or a settlement develops is called a site.
2. The people living in deep forests, hot and cold deserts and mountains often live in permanent
settlements.

Which of the above assertions is/are true?


A. 1 only B. 2 only C. Both 1 and 2 D. Neither 1 nor 2

Answer: A

Explanation: Settlements are places where people build their homes. The place where a building
or a settlement develops is called its site. The natural conditions for selection of an ideal site are
favourable climate, availability of water, suitable land, fertile soil. Settlements can be permanent or
temporary. Settlements which are occupied for a short time are called temporary settlements. The
people living in deep forests, hot and cold deserts and mountains often dwell in such temporary
settlements.

2. Regarding the waterways, consider the following statements:


1. Waterways are the cheapest for carrying heavy and bulky goods over long distances.
2. Only rivers are used for inland waterways.

Which of the above assertions is/are true?


A. 1 only B. 2 only C. Both 1 and 2 D. Neither 1 nor 2

Answer: A

Explanation: The waterway is the cheapest way to carry heavy and bulky goods over long
distances. The waterways are mainly of two types — inland waterways and sea routes. Navigable
rivers and lakes are used for inland waterways. Some of the important inland waterways are: the
Ganga-Brahmaputra river system, the Great Lakes in North America and the river Nile in Africa. Sea
routes and oceanic routes are mostly used for transporting merchandise and goods from one country
to another. These routes are connected with the ports.

3. Which cities of India are connected by the Golden Quadrilateral highway?


A. Delhi, Mumbai, Chennai and Patna
B. Delhi, Mumbai, Kanpur and Kolkata
C. Mumbai, Ahmedabad, Chennai and Kolkata
D. Delhi, Mumbai, Chennai and Kolkata

Answer: D

Explanation: India's Golden Quadrilateral highway connects Delhi, Mumbai, Chennai and Kolkata.

4. Consider the following grasslands:


1. Savanna 2. Veld
3. Campos 4. Llanos
www.achieveias.co.in, YouTube Channel: http://youtube.com/c/AchieveIAS Telegram Channel: http://t.me/Achieve_Ias,
Mail: achieveias21@gmail.com, Contact Number: 8968920720

Which of the above are examples of tropical grasslands?


A. 1 and 2 B. 2, 3 and 4 C. 1, 3 and 4 D. All of the above

Answer: C

Explanation: The major examples of tropical grasslands in the world are:


Savanna grasslands of East Africa.
The grasslands found in Brazil, are known as Campos.
The grasslands of Venezuela, are known as LIanos.

5. Which areas are mentioned in the context of the following features: Limited growth of
natural vegetation, only mosses, lichens and very small shrubs are found here. It grows
during the very short summer
A. Mediterranean area B. Polar region C. Hot desert region D. Cold desert region

Answer: B

Explanation: The above features are in reference to the vegetation found in the polar region. This
is called Tundra type of vegetation. This vegetation is found in the polar areas of Europe, Asia and
North America.

6. In the context of temperate grasslands, consider the following assertions:


1. These grasslands are found in the mid-latitudinal zones and in the interior part of the continents.
2. Usually, grass here is short and nutritious.

Which of the above assertions is/are true?


A. 1 only B. 2 only C. Both 1 and 2 D. Neither 1 nor 2

Answer: C

Explanation: Temperate grasslands are found in the mid-latitudinal zones and in the interior part of
the continents. Usually, grass here is short and nutritious. Wild buffaloes, bisons, antelopes are
common in the temperate region.

7. In the context of tropical grasslands, consider the following assertions:


1. Tropical grasslands occur only in the north of the equator.
2. Tropical grass usually grow very small.
3. Savannah grasslands of Africa are examples of tropical grasslands.

Which of the above assertions is/are false?


A. 1 only B. 2 only C. 3 only D. 1 and 2

Answer: D

Explanation: Tropical grasslands occur on either side of the equator and extend till the tropics. This
vegetation grows in the areas of moderate to low amount of rainfall. The grass can grow very tall,
about 3 to 4 metres in height. Savannah grasslands of Africa are of this type. Elephants, zebras,
giraffes, deer, leopards are common in tropical grasslands.
www.achieveias.co.in, YouTube Channel: http://youtube.com/c/AchieveIAS Telegram Channel: http://t.me/Achieve_Ias,
Mail: achieveias21@gmail.com, Contact Number: 8968920720

8. In which part of the continents are Mediterranean vegetation found?


1. Western part
2. Eastern part
3. Southwestern part
4. Northern part

Code:
A. 1 and 3 B. 2 and 4 C. 1, 3 and 4 D. 1, 2 and 4

Answer: A

Explanation: Mediterranean vegetation is found on the western and south-western edges of the
continents. It is mostly found in the areas around the Mediterranean Sea in Europe, Africa and Asia,
hence the name. This kind of vegetation is also found outside the actual Mediterranean region in
California in the USA, south West Africa, south western South America and South west Australia.
These regions are marked for hot dry summers and mild rainy winters.

9. In the context of temperate deciduous forests, which of the following assertions is/are
true?
1. As we go towards higher latitudes, more temperate deciduous forests are found.
2. They are found mostly in the North-eastern past of USA, and coastal regions of Western Europe.
3. They shed their leaves in the dry season.

A. 1 and 2 B. 2 and 3 C. 1 and 3 D. All of the above

Answer: D

Explanation: As we go towards higher latitudes, there are more temperate deciduous forests.
These are found in the north eastern part of USA, China, New Zealand, Chile and also found in the
coastal regions of Western Europe. They shed their leaves in the dry season.

10. The growth of any vegetation depends on which factors?


1. Temperature
2. Moisture
3. Thickness of soil
4. Wildlife

Code:
A. 1 and 2 B. 2, 3 and 4 C. 1, 2 and 3 D. All of the above

Answer: C

Explanation: The growth of vegetation depends on temperature and moisture. It also depends on
factors like slope and thickness of soil. With the change in height, the climate changes and that
changes natural vegetation.
www.achieveias.co.in, YouTube Channel: http://youtube.com/c/AchieveIAS Telegram Channel: http://t.me/Achieve_Ias,
Mail: achieveias21@gmail.com, Contact Number: 8968920720

ACHIEVE IAS GEOGRAPHY MCQ SERIES, DAY 6, SOLUTIONS

1. In the context of the Amazon River Basin, which of the following assertions is/are true?
1. The Amazon Basin stretches directly on the equator and is characterized by hot and wet climate
throughout the year.
2. Both day and nights are almost equally hot and humid.
3. It rains almost every day, that too without much warning.

Code:
A.1 and 2 B.2 and 3 C.1 and 3 D.1, 2 and 3

Answer: D

Explanation: All the assertions are right. The Amazon Basin stretches directly on the equator and
is characterized by hot and wet climate throughout the year. Both day and nights are almost equally
hot and humid. The skin feels sticky. It rains almost every day, that too without much warning. The
day temperatures are high with very high humidity. At night the temperature goes down but the
humidity remains high.

2. Parasitic plants such as orchid and bromeliads grow in which type of forest areas?
A. Rain Forests B. Monsoon Forest
C. Mediterranean Forest D. Coniferous Forest

Answer: A

Explanation: As it rains heavily in the rainforests, thick forests grow. The forests are in fact so thick
that the dense “roof” created by leaves and branches does not allow the sunlight to reach the ground.
The ground remains dark and damp. Only shade tolerant vegetation may grow here. Orchids,
bromeliads grow as plant parasites.

3. In the context of the RainForest of Amazon river basin, what is 'Maloca'?


A. A specific type of organism found here.
B. The main food items of its inhabitants.
C. The houses built by the people living here.
D. Agricultural system adopted by the people here.

Answer: C

Explanation: The Rain Forests of Amazon River basin, provide a lot of wood for the houses. Some
families live in thatched houses shaped like beehives. There are other large apartment-like houses
called “Maloca” with a steeply slanting roof.

4. In the fresh waters of which river/river basins, the variety of dolphin, Susu is found?
1. Ganges
2. Brahmaputra
3. Chambal
4. Kaveri
www.achieveias.co.in, YouTube Channel: http://youtube.com/c/AchieveIAS Telegram Channel: http://t.me/Achieve_Ias,
Mail: achieveias21@gmail.com, Contact Number: 8968920720

Code:
A. 1 and 2 B. 3 and 4 C. 1, 2 and 4 D. All of the above
Answer: A

Explanation: In the fresh waters of River Ganga and River Brahmaputra, a variety of dolphin locally
called Susu (also called blind dolphin) is found. The presence of Susu is an indication of the health
of the river. The untreated industrial and urban wastes with high amount of chemicals are killing this
species.

5. Kolkata is situated on the bank of which river?


A. Bhagirathi B. Kosi C. Mahanadi D. Hooghly

Answer: D

Explanation: The waterways, is an effective means of transport particularly along the rivers. Kolkata
is an important port on the River Hooghly.

6. What are known as the 'Prairies'?


A. Tropical grasslands of North America.
B. The temperate grasslands of South America
C. Tropical grasslands of South America.
D. Temperate grasslands of North America.

Answer: D

Explanation: The temperate grasslands of North America are known as the Prairies. It is a region
of flat, gently sloping or hilly land. For the most part, prairies are treeless but, near the low lying
plains, flanking river valleys, woodlands can be found. Tall grass, upto two metres high, dominates,
the landscape. It is actually a “sea of grass.”

7. Regarding the 'Prairie' grasslands, consider the following assertions:


1. These grasslands are bound by Rocky Mountains in the West and the Great Lakes in the East.
2. The grasslands of Prairies were the home of native Americans often called “Red Indians”
3. Bison or the American buffalo is the most important animal of this region.

Use the code given below to choose the correct answer:


A. 1 and 2 only B. 2 and 3 only C. 3 only D. All of the above

Answer: D

8. Which of the following rivers is/are related to the drainage system of the Prairies
grassland?
1. Mississippi River
2. Saskatchewan River
3. Hudson River
4. St. Lawrence River

Code:
www.achieveias.co.in, YouTube Channel: http://youtube.com/c/AchieveIAS Telegram Channel: http://t.me/Achieve_Ias,
Mail: achieveias21@gmail.com, Contact Number: 8968920720

A.1 and 2 only B.1, 2 and 3 only C.3 and 4 only D.1, 3 and 4 only

Answer: A

Explanation: Prairie grasslands, cover parts of United States of America and parts of Canada. In
the USA, the area is drained by the tributaries of Mississippi and the Canadian prairies are drained
by the tributaries of Saskatchewan Rivers.

9. Assertion: The Prairies are also known as the “Granaries of the world.”
Reason: Wheat is highly produced in the Prairie area.
A. Both assertion and reason are correct and the reason, is the correct explanation of the assertion.
B. Both assertion and reason are correct but the reason, is not the correct explanation of the
assertion.
C. The assertion is correct and the reason is wrong.
D. The assertion is wrong and the reason is correct.

Answer: A

Explanation: Scientific methods of cultivation and use of tractors, harvesters and combines has
made North America a surplus food producer. The Prairies are also known as the “Granaries of the
world,” due to the huge surplus of wheat production.

10. Regarding the Ladakh region, consider the following assertions:


1. Groves of willows and poplars are seen in the valleys.
2. Most of the people here are Hindus or Buddhists.
3. Several Buddhists monasteries dot the Ladakh landscape with their traditional ‘gompas’.

Which of the above mentioned assertions is/are correct?


A. 1 and 2 only B. 2 and 3 only C. 1 and 3 only D. 1, 2 and 3 only

Answer: C

Explanation: Due to high aridity, the vegetation is sparse. There are scanty patches of grasses and
shrubs for animals to graze. Groves of willows and poplars are seen in the valleys. The finest cricket
bats are made from the wood of the willow trees.The people here are either Muslims or Buddhists.
In fact several Buddhists monasteries dot the Ladakhi landscape with their traditional ‘gompas’.
Some famous monasteries are Hemis, Thiksey, Shey and Lamayuru.
www.achieveias.co.in, YouTube Channel: http://youtube.com/c/AchieveIAS Telegram
Channel: http://t.me/Achieve_Ias, Mail: achieveias21@gmail.com, Contact Number:8968920720

ACHIEVE IAS GEOGRAPHY MCQ SERIES, DAY 7, SOLUTIONS


1. Resources that we find in nature and are used without much modification are called:
(a) Human Resourcr (b) Natural Resource
(c) Renewable Resource (d) Human Made Resource

Answer: B

Explanation: Natural resources are components that exist in the world without the input of humans.
These natural resources are diverse ranging from renewable resources to non-renewable resources,
living to non-living resources, tangible to intangible resources. Natural resources are essential to the
survival of humans and all other living organisms. All the products in the world use natural resources
as their basic component, which may be water, air, natural chemicals or energy. The high demand for
natural resources around the world has led to their rapid depletion. As a result, most nations are pushing
for proper management and sustainable use of natural resources.

2. Resources which can be renewed or reproduced are known as:


(a) Exhaustible Resource (b) Renewable Resource
(c) Non-Renewable Resource (d) Useful Resource

Answer: B

Explanation: Renewable resources refer to resources that can naturally regenerate after use. They
include resources such as wind, water, natural vegetation, solar energy, and animals. These resources
exist in nature in abundance. There is little concern about depleting renewable resources because their
rate of production exceeds the rate of human consumption. Conservationists throughout the world
advocate for the use of renewable resources because they are readily available and less costly to the
environment.

3. Resources refer to the number and ability of the people. E.g. knowledge, skill, etc are:
(a) Biotic Resource (b) Human Made Resource
(c) Human Resource (d) Abiotic Resource

Answer: C

Explanation: The resource that resides in the knowledge, skills, and motivation of people. Human
resource is the least mobile of the four factors of production, and (under right conditions) it improves
with age and experience, which no other resource can do. It is therefore regarded as the scarcest and
most crucial productive resource that creates the largest and longest lasting advantage for an
organization.

4. Balancing the need to use resources and also conserve them for the future is called:
(a) Sustainable Development (b) Resource Conservation
(c) Resource Development (d) Sustainable Conservation

Answer: A

Explanation: "Sustainable development is development that meets the needs of the present, without
compromising the ability of future generations to meet their own needs." The concept of sustainable
development can be interpreted in many different ways, but at its core is an approach to development
www.achieveias.co.in, YouTube Channel: http://youtube.com/c/AchieveIAS Telegram
Channel: http://t.me/Achieve_Ias, Mail: achieveias21@gmail.com, Contact Number:8968920720

that looks to balance different, and often competing, needs against an awareness of the environmental,
social and economic limitations we face as a society. All too often, development is driven by one
particular need, without fully considering the wider or future impacts. We are already seeing the damage
this kind of approach can cause, from large-scale financial crises caused by irresponsible banking, to
changes in global climate resulting from our dependence on fossil fuel-based energy sources. The
longer we pursue unsustainable development, the more frequent and severe its consequences are
likely to become, which is why we need to take action now.

5. Resources are found in a region but have not been utilized, might be because of the lack of
technology:
(a) Potential Resource (b) Localized Resource
(c) Actual Resource (d) Ubiquitous Resource

Answer: A

Explanation: Potential resources are those which are available in the region but are not fully used. But
these resources could be used in future. For example, uranium present in Ladakh is a potential resource
which could be used in the future.

6. Resources created by human beings are called:


(a) Natural Resource (b) Useful Resource
(c) Industrial Resource (d) Man Made Resource

Answer: D

Explanation: Man-made resources are items or substances that have value to human lives that do not
occur in the natural world. Examples of man-made resources include plastic, paper, soda, sheet metal,
rubber and brass. These contrast with natural resources, such as water, crops, sunlight, crude oil, wood
and gold. Some man-made resources are nearly essential to modern human life, such as medicines.
Without medicines, vaccines and similar manmade chemicals, many people would become sick and
die. Others are not essential, and only exist because people want them. For example, many pesticides
are man-made resources, and while not essential, their use enhances the lives of many people.

7. We enjoy the beauty of mountains, waterfalls, sea, landscapes. Thus, they are resources
which value:
(a) Ethical Value (b) Artistic Value (c) Aesthetic Value (d) Economic Value

Answer: C
ACHIEVE IAS MCQ SERIES, GEOGRAPHY, DAY 7, SOLUTIONS
Explanation: Aesthetic value is the value that an object, event orstate of affairs (most paradigmatically
an art work or the natural environment) possesses in virtue of its capacity to elicit pleasure (positive
value) or displeasure (negative value) when appreciated or experienced aesthetically.

8. Resources which are found everywhere are called:


(a) Biotic Resource (b) Potential Resource
(c) Ubiquitous Resource (d) Renewable Resource

Answer: C
www.achieveias.co.in, YouTube Channel: http://youtube.com/c/AchieveIAS Telegram
Channel: http://t.me/Achieve_Ias, Mail: achieveias21@gmail.com, Contact Number:8968920720

Explanation: A ubiquitous resource is a natural resource that is available just about anywhere you
live. Air, wind, water are all ubiquitous resources. Localized resources are natural resources only found
in certain places.

9. Different ways to conserve natural resources are:


Select the incorrect one:
(a) Preventing wastage (b) Save Water
(c) Deforestation (d) Afforestation

Answer: C

Explanation: Deforestation is the clearing of trees, transforming a wooded area into cleared land. The
first step in turning the wilderness into a shopping center is deforestation. You can see the word forest
in deforestation. The prefix de- means "remove" and the suffix -ation signals the act or state of.

10. All non-living things are known as:


(a) Biotic Resource (b) Exhaustible Resource
(c) Abiotic Resource (d) Human Resource

Answer: C

Explanation: Abiotic resources are resources that are nonliving Human depletion of abiotic resources,
such as water, soil, and minerals is a source of concern for humans, as these resources are not easily
replenished and are being used above the rate that they can be naturally replaced.
www.achieveias.co.in, YouTube Channel: http://youtube.com/c/AchieveIAS Telegram Channel: http://t.me/Achieve_Ias,
Mail: achieveias21@gmail.com, Contact Number: 8968920720

ACHIEVE IAS GEOGRAPHY MCQ SERIES, DAY 8, SOLUTIONS

1. Consider the following statements:


1. The Soil formation is dependent on parent rock and climatic factors.
2. Mulching and terrace farming are methods of soil conservation.

Which of the statements given above is/are correct?


A. 1 only B. 2 only C. Both 1 and 2 D. Neither 1 nor 2

Answer: C

Explanation: The major factors of soil formation are the nature of the parent rock and climatic
factors. Other factors are the topography, role of organic material and time taken for the composition
of soil formation.

2. In the coastal and dry regions, rows of trees are planted to check the wind movement to
protect soil cover, this process is known as :
A. Strip Cropping B. Contour Barriers C. Terrace Cultivation D. Shelter Belts

Answer: D

Explanation: A shelterbelt is defined as a barrier of trees and shrubs that provides protection from
wind and storm and decreases erosion. Shelterbelts may be planted for a specific purpose, and this
determines how many rows of trees are planted, the species planted, and the design of the planting.
The term "field shelterbelt" is used for tree rows planted in the field to protect the soil and growing
crops from erosion. Farmyard shelterbelts are planted for the protection of a yard. Livestock
shelterbelts protect livestock and livestock facilities, and can provide odor control. Roadside
shelterbelts may be planted for snow control along roads, and for dust reduction. Shelterbelts may
also be planted to improve wildlife habitat, and to provide a buffer between agricultural land and
bodies of water. Some of the benefits of shelter belts are:
 Soil erosion
 Protects animals and humans from clod winds, if planted perfectly.
 Decrease wind erosion.
 Increase productivity.

3. The fresh water is continuously being renewed and recharged through the:
A. Hydrological Cycle B. Oxygen Cycle C. Rock Cycle D. Renewable Cycle

Answer: A

Explanation: Water cycle, also called hydrologic cycle, cycle that involves the continuous
circulation of water in the Earth-atmosphere system. Of the many processes involved in the water
cycle, the most important are evaporation, transpiration, condensation, precipitation, and runoff.
Although the total amount of water within the cycle remains essentially constant, its distribution
among the various processes is continually changing. Evaporation, one of the major processes in
the cycle, is the transfer of water from the surface of the Earth to the atmosphere. By evaporation,
water in the liquid state is transferred to the gaseous, or vapour, state. This transfer occurs when
some molecules in a water mass have attained sufficient kinetic energy to eject themselves from
www.achieveias.co.in, YouTube Channel: http://youtube.com/c/AchieveIAS Telegram Channel: http://t.me/Achieve_Ias,
Mail: achieveias21@gmail.com, Contact Number: 8968920720

the water surface. The main factors affecting evaporation are temperature, humidity, wind speed,
and solar radiation. The direct measurement of evaporation, though desirable, is difficult and
possible only at point locations. The principal source of water vapour is the oceans, but evaporation
also occurs in soils, snow, and ice. Evaporation from snow and ice, the direct conversion from solid
to vapour, is known as sublimation. Transpiration is the evaporation of water through minute pores,
or stomata, in the leaves of plants. For practical purposes, transpiration and the evaporation from
all water, soils, snow, ice, vegetation, and other surfaces are lumped together and
called evapotranspiration, or total evaporation. Precipitation that falls to the Earth is distributed in
four main ways: some is returned to the atmosphere by evaporation, some may be intercepted by
vegetation and then evaporated from the surface of leaves, some percolates into the soil by
infiltration, and the remainder flows directly as surface runoff into the sea. Some of the infiltrated
precipitation may later percolate into streams as groundwater runoff. Direct measurement of runoff
is made by stream gauges and plotted against time on hydrographs. Most groundwater is derived
from precipitation that has percolated through the soil. Groundwater flow rates, compared with those
of surface water, are very slow and variable, ranging from a few millimetres to a few metres a day.
Groundwater movement is studied by tracer techniques and remote sensing. Ice also plays a role
in the water cycle. Ice and snow on the Earth’s surface occur in various forms such as frost, sea ice,
and glacier ice. When soil moisture freezes, ice also occurs beneath the Earth’s surface, forming
permafrost in tundra climates. About 18,000 years ago glaciers and ice caps covered approximately
one-third of the Earth’s land surface. Today about 12 percent of the land surface remains covered
by ice masses.

4. It is the uppermost layer, Rich in humus & minerals and Consists of Sand, Silt & Clay:
A. Sub Soil B. Top Soil C. Alluvial Soil D. Black Soil

Answer: B

Explanation: The soil has taken thousands of years to form. Soil formation takes place in the
following ways:
 Big rocks break down into smaller rocks by continuous action of wind and rain. It takes many
years for these rocks to break down into smaller rocks.
 Rocks are mainly broken by two types of weathering- physical weathering and chemical
weathering. A number of natural force, called agents, work to break down the parent rock into
tiny particles of soil. These agents include wind, water, the sun’s heat, and plants and animals.
 These pieces get further broken down to form sand and silt and, ultimately, into finer particles
and the process continues. This process is very slow. It takes thousands of years to form a just
1cm layer of soil. These fine particles form the top layer of the soil.

Soil Profile: The soil is found in layers, which are arranged during the formation of soil. These layers
called horizons, the sequence of layers is the soil profile. The layers of soil can easily be observed
by their color and size of particles. The main layers of the soil are topsoil, subsoil and the parent
rock. These features of the layer of soil play a very important role in determining the use of the soil.
Soil that has developed three layers, is mature soil. It takes many years under a favorable condition
for the soil to develop its three layers. At some places, the soil contains only two layers. Such soil is
immature soil.
Horizons of the Soil
Soil consists of the following horizons:
www.achieveias.co.in, YouTube Channel: http://youtube.com/c/AchieveIAS Telegram Channel: http://t.me/Achieve_Ias,
Mail: achieveias21@gmail.com, Contact Number: 8968920720

1. Horizon A or Topsoil: It is also called the humus layer, which is rich in organic material. This layer
consists of decomposed material and organic matter. This is the reason, the topsoil has a dark
brown color. The hummus makes the topsoil soft, porous to hold enough air and water. In this layer,
the seeds germinate and roots of the plants grow. Many living organisms like earthworms,
millipedes, and centipedes, bacteria, and fungi are found in this layer of soil.
2. Horizon B or Subsoil: Just below the topsoil lies another layer called subsoil or horizon-B. It is
comparatively harder and compact than topsoil. It is lighter in color than the topsoil because there
is less humus in this layer. This layer is less organic but is rich in minerals brought down from the
topsoil. It contains metal salts, especially iron oxide in a large proportion. Farmers often mix horizon-
A and horizon-B when ploughing their fields.
3. Bedrock or Horizon C: Bedrock is also known as parent rock and lies just below the subsoil. It
contains no organic matter and made up of stones and rocks, so it is very hard. This layer represents
a transition zone between the earth’s bedrock and horizon A and B.

What is humus layer in the soil? It is also called topsoil or horizon A, which is rich in organic material.
This layer consists of decomposed material and organic matter. This is the reason, the topsoil has
dark brown color. The humus makes the topsoil soft, porous to hold enough air and water. In this
layer, the seeds germinate and roots of the plants grow. Many living organisms like earthworms,
millipedes, and centipedes, bacteria, and fungi are found in this layer of soil.

5. In the biosphere living beings are inter-related and interdependent on each other for
survival. This life supporting system is known as the:
A. Interdependence B. Ecology C. Ecosystem D. Food Circle

Answer: C

Explanation: The ecosystem is the set of species in a given area that interact among themselves,
through processes such as predation, parasitism, competition and symbiosis, and with their abiotic
environment to disintegrate and become part of cycles of energy and nutrients. The species of the
ecosystem, including bacteria, fungi, plants and animals, are dependent on each other. The
relationships between species and their environment facilitate the flow of matter and energy within
the ecosystem. The concept of the ecosystem has evolved since its origin. The term, coined in the
1930s, belongs to British botanists Roy Clapham (1904-1990) and Sir Arthur Tansley (1871-1955).
It was originally applied to units of diverse spatial scale; from a weathered piece of tree trunk to a
pond, a region or even the entire biosphere of the planet, the only requirement being that organisms,
physical environment and interactions could exist within them. More recently, the ecosystem has
had a geographical focus and has become analogous to formations or vegetation types, e.g., scrub,
pine forest, grassland, etc. This simplification ignores the fact that the limits of some vegetation
types are indistinct, while the boundaries of ecosystems are not. The transition zones between
ecosystems are known as ecotones.

6. A natural area designated to protect the ecological integrity of one or more ecosystems
for present and future generations is known as:
A. Wildlife Sanctuaries B. Bio reserves C. Botanical Gardens D. National Parks

Answer: D
www.achieveias.co.in, YouTube Channel: http://youtube.com/c/AchieveIAS Telegram Channel: http://t.me/Achieve_Ias,
Mail: achieveias21@gmail.com, Contact Number: 8968920720

Explanation: National park, an area set aside by a national government for the preservation of the
natural environment. A national park may be set aside for purposes of public recreation and
enjoyment or because of its historical or scientific interest. Most of the landscapes and their
accompanying plants and animals in a national park are kept in their natural state. The national
parks in the United States and Canada tend to focus on the protection of both land and wildlife,
those in the United Kingdom focus mainly on the land, and those in Africa primarily exist to conserve
animals. Several other countries have large areas reserved in national parks,
notably Brazil, Japan, India, and Australia. The national parks of various countries differ greatly in
their effectiveness in protecting their resources. Some governments provide their park systems with
large enough budgets to make possible strict enforcement of regulations; others do not. Most
national parks have a built-in paradox: although they often depend for their existence
on tourism stimulated by public interest in nature, the preservation of their wildlife depends on its
not being molested. This paradox is usually resolved by allowing visitors to travel only within limited
areas in the park. This lets them see the park while it minimizes their contact with the wildlife. See
also conservation; nature reserve; national forest.

7. Soil is derived from the weathering of the igneous and metamorphic rocks:
A. Red Soil B. Black Soil C. Laterite Soil D. Mountain Soil

Answer: A

Explanation: Red soil is a type of soil that develops in a warm, temperate, moist climate under
deciduous or mixed forests and that have thin organic and organic-mineral layers overlying a
yellowish-brown leached layer resting on an illuvial (see illuviation) red layer. Red soils generally
derived from crystalline rock. They are usually poor growing soils, low in nutrients and humus and
difficult to cultivate because of its low water holding capacity. Red soils denote the second largest
soil group of India covering an area of about 6.1 lakhs sq. km (18.6% of India's area) over the
Peninsula from Tamil Nadu in the south to Bundelkhand in the north and Rajmahal hills in the east
to Kachchh in the west. They surround the black soils on their south, east and north.

8. Species of plants and animals that do not exist now but existed in the past are called:
A. Endangered Species B. Extinct Species C. Dying Species D. Dead Species

Answer: B

Explanation: Extinction happens when environmental factors or evolutionary problems cause a


species to die out. The disappearance of species from Earth is ongoing, and rates have varied over
time. A quarter of mammals is at risk of extinction, according to IUCN Red List estimates.
To some extent, extinction is natural. Changes to habitats and poor reproductive trends are among
the factors that can make a species’ death rate higher than its birth rate for long enough that
eventually, none are left. Humans also cause other species to become extinct by hunting,
overharvesting, introducing invasive species to the wild, polluting, and changing wetlands and
forests to croplands and urban areas. Even the rapid growth of the human population is causing
extinction by ruining natural habitats.

9. Trees in these forests shed their leaves in a particular season in order to conserve loss of
moisture through transpiration:
Select the incorrect one:
www.achieveias.co.in, YouTube Channel: http://youtube.com/c/AchieveIAS Telegram Channel: http://t.me/Achieve_Ias,
Mail: achieveias21@gmail.com, Contact Number: 8968920720

A. Evergreen Forests B. Tundra C. Deforestation D. Deciduous Forests

Answer: D

Explanation: Deciduous forest, vegetation composed primarily of broad-leaved trees that shed all
their leaves during one season. Deciduous forest is found in three middle-latitude regions with a
temperate climate characterized by a winter season and year-round precipitation: eastern North
America, western Eurasia, and north-eastern Asia. Deciduous forest also extends into more arid
regions along stream banks and around bodies of water. For the deciduous forest of tropical
regions, see monsoon forest. Oaks, beeches, birches, chestnuts, aspens, elms, maples,
and basswoods (or lindens) are the dominant trees in mid-latitude deciduous forests. They vary in
shape and height and form dense growths that admit relatively little light through the leafy
canopy. Shrubs are found primarily near clearings and forest edges, where more light is available,
and herbaceous flowering plants are abundant within the forest in the spring, before the trees come
into full leaf.

10. Species of some animals are on the verge of extinction as their population has decreased
considerably. Such species are known as:
A. Endangered Species B. Extinct Species C. Dying Species D. Dead Species

Answer: A

Explanation: Endangered species, any species that is at risk of extinction because of a sudden
rapid decrease in its population or a loss of its critical habitat. Previously, any species
of plant or animal that was threatened with extinction could be called an endangered species. The
need for separate definitions of “endangered” and “threatened” species resulted in the development
of various categorization systems, each containing definitions and criteria by which a species can
be classified according to its risk of extinction. As a rule, a range of criteria must be analyzed before
a species can be placed in one category or another. Often such categorization systems are linked
directly to national legislation, such as the United States Endangered Species Act (ESA) or the
Canadian Species at Risk Act (SARA). In addition, regional agreements, such as the European
Union’s Habitats Directive (Council Directive 92/43/EEC), and
international conservation agreements, such as the Convention on the Conservation of Migratory
Species of Wild Animals (CMS) or the Convention on International Trade in Endangered Species of
Wild Fauna and Flora (CITES), are connected to species-assessment systems. One of the most-
recognized independent international systems of species assessment is the Red List of Threatened
Species, created by the International Union for Conservation of Nature (IUCN).
www.achieveias.co.in, YouTube Channel: http://youtube.com/c/AchieveIAS Telegram Channel: http://t.me/Achieve_Ias,
Mail: achieveias21@gmail.com, Contact Number: 8968920720

ACHIEVE IAS GEOGRAPHY MCQ SERIES, DAY 9, SOLUTIONS

1. Consider the following pairs:


Country Producer
1. Chile : Copper
2. Australia : Bauxite
3. Bolivia : Tin

Which of the pairs given above is/are correctly matched?


A. 1 only B. 1 and 2 only C. 2 and 3 only D. 1, 2 and 3

Answer: D

Explanation: Chile and Peru are leading producers of copper. Brazil and Bolivia are among the
world’s largest producers of tin. Australia is the largest producer of bauxite in the world.

2. Which of the following statements is/are correct?


1. Rice grows best in alluvial clayey soil.
2. Coffee grows well in drained loamy soil and warm, wet climate.
3. Tea grows well in drained loamy soils and gentle slopes.

Select the correct answer using the code given below:


A. 1 and 2 only B. 2 and 3 only C. 1 only D. 1, 2 and 3

Answer: D

Explanation: Coffee requires warm and wet climate and well drained loamy soil. Rice needs high
temperature, high humidity and rainfall. It grows best in alluvial clayey soil, which can retain water.
Tea requires cool climate and well distributed high rainfall throughout the year for the growth of its
tender leaves.

3. A naturally occurring substance that has a definite chemical composition is known as:
A. Ore B. Mineral C. Soil D. Land

Answer: B

Explanation: Minerals are solid substances that are present in nature and can be made of one
element or more elements combined together (chemical compounds). Gold, Silver and carbon are
elements that form minerals on their own. They are called native elements. Instead, ordinary kitchen
salt is a chemical compound that is called rock salt, which is a mineral formed of sodium and chlorine
ions. Atoms, ions and molecules that form a mineral are present in the space in a tidy way and
according to well-defined geometrical shapes, which are called crystal lattices. The structure of the
crystal lattice defines the shape of the crystal as we see it. For example, rock salt or kitchen salt is
a mineral formed of cubic-shaped crystals. Its crystal lattice has the same shape and consists of
sodium and chlorine ions that are present in the space in alternate order.
The order of atoms in the space and the way they combine with each other determine the way a
mineral can laminate or exfoliate. Lamination is the property that some materials have to break
according to their geometrical shape. Its chemical composition also determines the colour of the
www.achieveias.co.in, YouTube Channel: http://youtube.com/c/AchieveIAS Telegram Channel: http://t.me/Achieve_Ias,
Mail: achieveias21@gmail.com, Contact Number: 8968920720

crystal, such as the yellow colour for the topaz, red for ruby, purple for amethyst quartz. Another
characteristic of minerals is their hardness, which is their resistance to scratches. Hardness is
classified by numbers (from 1 to 10), according to the Moh’s scale. At the beginning of the scale
there are very soft minerals that can be scratched with a nail, such as talc, chalk and calcite. At the
end of the scale there is the diamond, which is the hardest mineral in nature.
"A mineral is an element or chemical compound that is normally crystalline and that has been formed
as a result of geological processes" .

4. Non-metallic minerals like Limestone, Sandstone, Marble, etc. are found in the:
A. Metamorphic Rocks B. Igneous Rocks
C. Sedimentary Rocks D. Weathered Rocks

Answer: C

Explanation: Sedimentary rocks are formed from pre-existing rocks or pieces of once-living
organisms. They form from deposits that accumulate on the Earth's surface. Sedimentary rocks
often have distinctive layering or bedding. Many of the picturesque views of the desert southwest
show mesas and arches made of layered sedimentary rock.
Common Sedimentary Rocks:
Common sedimentary rocks include sandstone, limestone, and shale. These rocks often start as
sediments carried in rivers and deposited in lakes and oceans. When buried, the sediments lose
water and become cemented to form rock. Tuffaceous sandstones contain volcanic ash.
Clastic Sedimentary Rocks:
Clastic sedimentary rocks are the group of rocks most people think of when they think of
sedimentary rocks. Clastic sedimentary rocks are made up of pieces (clasts) of pre-existing rocks.
Pieces of rock are loosened by weathering, then transported to some basin or depression where
sediment is trapped. If the sediment is buried deeply, it becomes compacted and cemented,
forming sedimentary rock. Clastic sedimentary rocks may have particles ranging in size from
microscopic clay to huge boulders. Their names are based on their clast or grain size. The
smallest grains are called clay, then silt, then sand. Grains larger than 2 millimeters are called
pebbles. Shale is a rock made mostly of clay, siltstone is made up of silt-sized grains, sandstone is
made of sand-sized clasts, and conglomerate is made of pebbles surrounded by a matrix of sand
or mud.
Biologic Sedimentary Rocks:
Biologic sedimentary rocks form when large numbers of living things die. Chert is a example for
this type of rock, and this is one of the ways limestone can form. Limestone can also form by
precipitating out of the water.

5. It can be obtained by building dams at narrow openings of the sea:


A. Geothermal Energy B. Hydel Energy C. Tidal Energy D. Wind Energy

Answer: C

Explanation: Tidal energy is one of the oldest forms of energy generation. It is a renewable form
of energy that converts the natural rise and fall of the tides into electricity. Tides are caused by the
combined effects of gravitational forces exerted by the Moon, the Sun, and the rotation of the Earth.
Tidal energy presents an evolving technology with tremendous potential . However, it can only be
installed along coastlines. Coastlines often experience two high tides and two low tides on a daily
www.achieveias.co.in, YouTube Channel: http://youtube.com/c/AchieveIAS Telegram Channel: http://t.me/Achieve_Ias,
Mail: achieveias21@gmail.com, Contact Number: 8968920720

basis. The difference in water levels must be at least 5 meters high to produce electricity.
Tidal electricity can be created from several technologies, the main ones being tidal barrages, tidal
fences and tidal turbines. Tidal barrages are the most efficient tidal energy sources. A tidal barrage
is a dam that utilizes the potential energy generated by the change in height between high and low
tides. This energy turns a turbine or compresses air, which generates electricity. Tidal fences are
turbines that operate like giant turnstiles, while tidal turbines are similar to wind turbines only under
water. In both cases, electricity is generated when the mechanical energy of tidal currents turns
turbines connected to a generator. Ocean currents generate relatively more energy than air currents
because ocean water is 832 times denser than air and therefore applies greater force on the
turbines.
Context: Tidal power is an easy to install, renewable source of energy with no direct greenhouse
gas emissions and a low environmental impact. Because the ocean’s tidal patterns are well
understood, tidal energy is a very predictable energy source making it a highly attractive for electrical
grid management. This sets it apart from other renewables that can be more variable. Adoption of
tidal technologies has been slow and the amount of power generated from tidal power plants is very
small. This is largely due to the very specific site requirements necessary to produce tidal electricity.
Additionally, tide cycles do not always match the daily consumption patterns of electricity and
therefore do not provide sufficient capacity to satisfy demand.

6. It is a rock deposit that contains enough mineral to make it economically feasible to extract
and purify to derive a desired product material:
A. Coal B. Mineral C. Bauxite D. Ore

Answer: D

Explanation: Ore is a deposit in Earth’s crust of one or more valuable minerals. The most valuable
ore deposits contain metals crucial to industry and trade, like copper, gold, and iron. Copper ore is
mined for a variety of industrial uses. Copper, an excellent conductor of electricity, is used as
electrical wire. Copper is also used in construction. It is a common material in pipes
and plumbing material. Like copper, gold is also mined for industry. For example, space helmets are
plated with a thin layer of gold to protect astronaut’s eyes from harmful solar radiation. However,
most gold is used to create jewelry. For thousands of years, gold ore was mined as a basis
for currency, or money. Most nations stopped valuing their money on the gold standard in the
twentieth century. Iron ore has been mined for thousands of years. Iron, the second-
most abundant metal on Earth, is the main component of steel. Steel is a strong, valuable building
material. Iron is used in everything from glass to fertilizer to the solid-rocket boosters once used for
the space shuttle to leave Earth’s atmosphere. Metals are often associated with particular
ores. Aluminium, for example, is usually found in the ore called bauxite. Aluminium found in bauxite
is used in containers, cosmetics, and medicines.

7. Heat energy obtained from the earth is called:


A. Solar Energy B. Geothermal Energy
C. Hydel Energy D. Tidal Energy

Answer: B

Explanation: GEOTHERMAL ENERGY HAS been used for thousands of years in some countries
for cooking and heating. It is simply power derived from the Earth’s internal heat. This thermal energy
www.achieveias.co.in, YouTube Channel: http://youtube.com/c/AchieveIAS Telegram Channel: http://t.me/Achieve_Ias,
Mail: achieveias21@gmail.com, Contact Number: 8968920720

is contained in the rock and fluids beneath Earth’s crust. It can be found from shallow ground to
several miles below the surface, and even farther down to the extremely hot molten rock
called magma. These underground reservoirs of steam and hot water can be tapped to generate
electricity or to heat and cool buildings directly. A geothermal heat pump system can take advantage
of the constant temperature of the upper ten feet (three meters) of the Earth’s surface to heat a
home in the winter, while extracting heat from the building and transferring it back to the relatively
cooler ground in the summer. Geothermal water from deeper in the Earth can be used directly for
heating homes and offices, or for growing plants in greenhouses. To produce geothermal-generated
electricity, wells, sometimes a mile (1.6 kilometers) deep or more, are drilled into underground
reservoirs to tap steam and very hot water that drive turbines linked to electricity generators. The
first geothermally generated electricity was produced in Larderello, Italy, in 1904. There are three
types of geothermal power plants: dry steam, flash, and binary. Dry steam, the oldest geothermal
technology, takes steam out of fractures in the ground and uses it to directly drive a turbine. Flash
plants pull deep, high-pressure hot water into cooler, low-pressure water. The steam that results
from this process is used to drive the turbine. In binary plants, the hot water is passed by a secondary
fluid with a much lower boiling point than water. This causes the secondary fluid to turn to vapor,
which then drives a turbine. Most geothermal power plants in the future will be binary plants.

8. It is obtained from energy stored in the nuclei of atoms of naturally occurring radioactive
elements like Uranium and Thorium:
A. Nuclear Power Energy B. Solar Energy
C. Tidal Energy D. Wind Energy

Answer: A

Explanation: Nuclear energy comes from splitting atoms in a reactor to heat water into steam, turn
a turbine and generate electricity. Ninety-six nuclear reactors in 29 states generate nearly 20 percent
of the nation’s electricity, all without carbon emissions because reactors use uranium, not fossil
fuels. These plants are always on: well-operated to avoid interruptions and built to withstand extreme
weather, supporting the grid 24/7. Nuclear energy is energy in the nucleus (core) of an atom. Atoms
are tiny particles that make up every object in the universe. There is enormous energy in the bonds
that hold atoms together. Nuclear energy can be used to make electricity. But first the energy must
be released. It can be released from atoms in two ways: nuclear fusion and nuclear fission. In
nuclear fusion, energy is released when atoms are combined or fused together to form a larger
atom. This is how the sun produces energy. In nuclear fission, atoms are split apart to form smaller
atoms, releasing energy. Nuclear power plants use nuclear fission to produce electricity.

9. It is found with petroleum deposits and is released when crude oil is brought to the surface:
A. Natural Gas B. Crude Oil C. Hydel Energy D. Biogas

Answer: A

Explanation: Natural gas is a mixture of gases which are rich in hydrocarbons. All these gases
(methane, nitrogen, carbon dioxide etc) are naturally found in atmosphere. Natural gas reserves are
deep inside the earth near other solid & liquid hydrocarbons beds like coal and crude oil.
Natural gas is not used in its pure form; it is processed and converted into cleaner fuel for
consumption. Many by-products are extracted while processing of natural gas like propane, ethane,
butane, carbon dioxide, nitrogen etc, which can be further used.
www.achieveias.co.in, YouTube Channel: http://youtube.com/c/AchieveIAS Telegram Channel: http://t.me/Achieve_Ias,
Mail: achieveias21@gmail.com, Contact Number: 8968920720

Description: Natural gas is mainly used as fuel for generating electricity and heat. Natural gas in
compressed form is used as fuel for vehicles which is known as CNG. It is used as fuel for boilers
and air conditioners worldwide. This is used for making fertilizers also, mainly ammonia.
A green field project is going on in Russia to produce LNG to run aircrafts. Russia, USA and Canada
are major producers & consumers. For transporting, two variants are used - LNG for cross countries
exchange and CNG for domestic purpose. Natural gas prices are affected mainly through the US
demand and it's seasonal in nature wherein in winters its prices fluctuate heavily. Also they have
direct correlation with crude oil prices. It is measured in million British thermal units (mmBTUs), but
in some countries it is traded in Gigajoule also. The world's most liquid derivative contracts for
natural gas are traded on NYMEX, while in India it is traded on MCX.

10. Organic waste such as dead plant and animal material, animal dung and kitchen waste
can be converted into a gaseous fuel called:
A. Biogas B. Natural gas C. CNG D.LPG

Answer: A

Explanation: Biogas is a type of biofuel that is naturally produced from the decomposition of
organic waste. When organic matter, such as food scraps and animal waste, break down in an
anaerobic environment (an environment absent of oxygen) they release a blend of gases, primarily
methane and carbon dioxide. Because this decomposition happens in an anaerobic environment,
the process of producing biogas is also known as anaerobic digestion. Anaerobic digestion is a
natural form of waste-to-energy that uses the process of fermentation to breakdown organic matter.
Animal manure, food scraps, wastewater, and sewage are all examples of organic matter that can
produce biogas by anaerobic digestion. Due to the high content of methane in biogas (typically 50-
75%) biogas is flammable, and therefore produces a deep blue flame, and can be used as an energy
source. Biogas is known as an environmentally-friendly energy source because it alleviates two
major environmental problems simultaneously:
1. The global waste epidemic that releases dangerous levels of methane gas every day
2. The reliance on fossil fuel energy to meet global energy demand

By converting organic waste into energy, biogas is utilizing nature’s elegant tendency to recycle
substances into productive resources. Biogas generation recovers waste materials that would
otherwise pollute landfills; prevents the use of toxic chemicals in sewage treatment plants, and saves
money, energy, and material by treating waste on-site. Moreover, biogas usage does not require
fossil fuel extraction to produce energy.
www.achieveias.co.in, YouTube Channel: http://youtube.com/c/AchieveIAS Telegram Channel: http://t.me/Achieve_Ias,
Mail: achieveias21@gmail.com, Contact Number: 8968920720

ACHIEVE IAS GEOGRAPHY MCQ SERIES, DAY 10, SOLUTIONS

1. Slash and burn practice of agriculture is also known as:


(a) Intensive Farming (b) Extensive Farming
(c) Shifting Farming (d) Nomadic Farming

Answer: C

Explanation: Slash and burn farming is a form of shifting agriculture where the natural vegetation
is cut down and burned as a method of clearing the land for cultivation, and then, when the plot
becomes infertile, the farmer moves to a new fresh plat and does the same again. This process is
repeated over and over. The soil loses its fertility because the richness of the rainforest is in the
trees. As leaves fall or trees die everything is broken down by the soil’s organisms, nutrients are
returned to the soil and the tree roots take them up again. Thus continuous recycling keeps
everything fertile and growing. When this no longer happens in a cleared plot it soon becomes
infertile. Torrential tropical rains quickly wash nutrients out of the soil when it is left bare after harvest.
Not only is this devastating the worlds remaining tropical forests (keeping carbon in the trees" and
"saving the rich diverse life of the rainforests") while keeping the farmers in poverty, but it is also
forcing many of them to abandon the land, and migrate to city slums in the hope of feeding their
families. Life in the slums can be very hard indeed. There is not enough work available.

2. Which is also known as golden fiber?


(a) Cotton (b) Wheat (c) Silk (d) Jute

Answer: D

Explanation: Jute is known as the Golden Fibre. That's an appropriate name for the yellowish
brown, shiny, natural vegetable fibre produced from plants of genus Corchorus. It occupies place
next to cotton in the amount produced and the variety of uses. Due to its inherent high tensile
strength, low extensibility, moderate moisture retention and better breathability, jute is used
extensively in agricultural bulk packaging. For centuries, jute, grown in the Ganga Delta, has been
an integral part of India and was widely used for apparel as well as non-textile uses. The importance
and use of jute spread to different parts of the world, where it is called hessian fibre. With the advent
of synthetic fibres in the last mid-century, usage of jute for many of its industrial applications
decreased. However, it is regaining importance based on its inherent sustainable properties. Now,
jute is being used for a number of value added and diversified applications and thus is considered
not to have lost its glitter. The demand for jute is increasing due to its techno-commercial cost
competitiveness as well as eco-friendly biodegradable characteristics. The various end-use
applications of jute are:
 Apparel - its lustre, softness and uniformity make it ideal for production of comfort wear clothes,
particularly pullovers and intimates
 Home textiles - this fibre is traditionally used for floor covering mats, curtains, chair coverings
and carpets
 Industrial/Technical - owing to its long length and tensile strength, jute is widely used in
making rope and twine for industrial use and making baskets and bags for storing grain and
agricultural products
 Other - used as herb and food in cooking of soups in some Middle Eastern and African countries,
leaves eaten in parts of India.
www.achieveias.co.in, YouTube Channel: http://youtube.com/c/AchieveIAS Telegram Channel: http://t.me/Achieve_Ias,
Mail: achieveias21@gmail.com, Contact Number: 8968920720

3. It is also known as "Monoculture", i.e. single crop grown over a large area.
(a) Commercial Grain Farming (b) Plantation Farming
(c) Multiple Farming (d) Mixed Farming

Answer: B

Explanation: Monoculture farming is the raising of a single crop within a specified area. Most of the
commercial farms in the US are now monoculture in nature, with crops like corn and soy taking top
billing. This is in contrast to the traditional method of farming, which relied on multiple crops being
planted within a specific area. Many indoor farms growing medicinal herbs and flowers are
considered to be monoculture farms. Monoculture farming is a relatively new technique, but one that
has taken hold of the US farming system. It is now the basis for most of our agriculture. Really, it is
nothing more than the practice of intensively growing a single crop within a specific area. This is
completely at odds with the traditional method of farming, which allowed the growing of multiple
crops. Many examples of monoculture farming can be found today. Corn and soy production are
both very common, and most farms will “specialize” in one plant type. For instance, an Iowa corn
farmer will grow corn exclusively, while a Georgia farmer might grow soy beans only. The problem
with monoculture farming is that it eliminates biological controls provided by mixed crop gardening,
and it also causes soil degradation. In a traditional farm, different plant species require different
levels of nutrients, and many also replace depleted nutrients. Within a monoculture system, depleted
nutrients must be replaced through the application of fertilizers instead. Because plants are more
susceptible to pests in this type of farming, significant amounts of pesticides must also be applied.
Monoculture farming also hurts biodiversity. For instance, growing biologically identical plants can
create problems when the environmental conditions in a particular area change. Because plants are
not allowed to adapt naturally to those changes, it can result in widespread crop failure.

4. Out of the following which is not a cropping season of India?


(a) Zaid (b) Kharif (c) Kaffir (d) Rabi

Answer: C

Explanation: The agriculture in India is an important topic in Indian geography. 49% of the
population in India is dependent on agriculture. In the total geographical area in India, 141 million
hectares is the net sown area while 195 million hectare is the gross cropped area. Agriculture in
India contributes to 14% of the GDP and the distribution of income and wealth. It provides essential
amenities like food for the people and fodder for the animals. It also provides the major source of
raw materials to the agro-based industries in India. The vast relief of the country, varied climate and
soil conditions cause to the provision of a variety of crops. All tropical, subtropical and temperate
crops are grown in India but predominantly food crops are cultivated in 2/3rd of the total cropped
area. There are three chief cropping seasons in India namely Kharif, Rabi and Zaid. The Kharif
season spreads from the month of July to the month of October and the Rabi season is from October
to the month of March. The crops cultivated between March and June are called Zaid. In this article,
you can read details about the different seasons. Rice and wheat form the staple food in this country.
Good monsoons, especially the southwest monsoons are essential to sow and harvest crops in
India. In order to ensure the availability of food for the country’s growing population, it is imperative
that we have a reasonably good Rabi and Kharif season.
www.achieveias.co.in, YouTube Channel: http://youtube.com/c/AchieveIAS Telegram Channel: http://t.me/Achieve_Ias,
Mail: achieveias21@gmail.com, Contact Number: 8968920720

Types of crops in India


1. KHARIF SEASON IN INDIA (kharif means autumn in Arabic)
July – October
Harvest – September to October
A.k.a Monsoon Crops
Such crops require a lot of water
Example: rice, sorghum, maize, Tea, rubber, coffee, guar, Sesame, cereals such as Arhar Dhal,
pearl millet, soybeans, cotton, oilseeds, etc.

2. RABI CROPS IN INDIA


Sowing between October and November
Harvest – February to April
A.k.a Winter Season Crops
Need cold weather for growth
Need less water
Example: wheat, oats, barley, pulses, cereals, oilseeds, linseed, etc.

3. ZAID SEASON
Sowing between March and June (between Kharif and Rabi)
Requires warm & dry weather for growth and a longer day-length for flowering
Example: Seasonal fruits and vegetable

5. Which one is not a millet crop?


(a) Jowar (b) Ragi (c) Wheat (d) Bajra

Answer: C

Explanation: Millet, any of several species of cereal grasses in the family Poaceae, cultivated for
their small edible seeds. Millets were probably first cultivated in Asia more than 4,000 years ago,
and they were major grains in Europe during the Middle Ages. Today, though they are used chiefly
for pasture or to produce hay in the United States and Western Europe, they remain
important food staples in less-developed countries worldwide. Millet grains are high
in carbohydrates, with protein content varying from 6 to 11 percent and fat varying from 1.5 to 5
percent. They are somewhat strong in taste and are mainly consumed in flatbreads and porridges
or prepared and eaten much like rice.

6. The type of agriculture practiced in India is:


(a) Intensive Farming (b) Extensive Farming
(c) Primitive Farming (d) Mixed Farming

Answer: A

Explanation: Intensive agriculture, in agricultural economics, system of cultivation using large


amounts of labour and capital relative to land area. Large amounts of labour and capital are
necessary to the application of fertilizer, insecticides, fungicides, and herbicides to growing crops,
and capital is particularly important to the acquisition and maintenance of high-efficiency machinery
for planting, cultivating, and harvesting, as well as irrigation equipment where that is required.
Optimal use of these materials and machines produces significantly greater crop yields per unit of
www.achieveias.co.in, YouTube Channel: http://youtube.com/c/AchieveIAS Telegram Channel: http://t.me/Achieve_Ias,
Mail: achieveias21@gmail.com, Contact Number: 8968920720

land than extensive agriculture, which uses little capital or labour. As a result, a farm using
intensive agriculture will require less land than an extensive agriculture farm to produce a similar
profit. In practice, however, the increased economies and efficiencies of intensive agriculture often
encourage farm operators to work very large tracts in order to keep their capital investments in
machinery productively engaged—i.e., busy. On the level of theory, the increased productivity of
intensive agriculture enables the farmer to use a relatively smaller land area that is located close to
market, where land values are high relative to labour and capital, and this is true in many parts of
the world. If costs of labour and capital outlays for machinery and chemicals, and costs of storage
(where desired or needed) and transportation to market are too high then farmers may find it more
profitable to turn to extensive agriculture. However, in practice many relatively small-scale farmers
employ some combination of intensive and extensive agriculture, and many of these operate
relatively close to markets. Many large-scale farm operators, especially in such relatively vast and
agriculturally advanced nations as Canada and the United States, practice intensive agriculture in
areas where land values are relatively low, and at great distances from markets, and farm enormous
tracts of land with high yields. However, in such societies overproduction (beyond market demands)
often results in diminished profit as a result of depressed prices.

7. Cultivation of grapes is also known as:


(a) Viticulture (b) Horticulture (c) Sericulture (d) Pisciculture

Answer: A

Explanation: Viticulture is the process of grape production. Grapes are grown around the globe
apart from the Antarctica, and they have high adaptability properties to different environments.
Grapes are fruits which are used to produce wine. The people who study the science behind grape
production are called viticulturists. Viticulturists study pests and diseases to control, irrigation,
fertilization, fruit development, and characteristics of grapes. They are also responsible in managing
the canopy on which the grapes grow, when to prune, and when to harvest the fruit. Winemakers
also liaise with viticulturists in wine production because the best vines give the best wine. Different
varieties of grapes are now approved by the European Union as the real grapes for wine production,
because of their characteristics. Some of the wines produced using grapes include the red wine,
which is produced from gulp of black and red grapes. Grapes are also used in making raisins.

8. Seasonal migration of people with their animals is called:


(a) Farmers (b) Jhumming (c) Transhumance (d) Labours

Answer: C

Explanation: Transhumance is a pastoral practice that is organized around the movement of


livestock between winter and summer pastures. In montane areas, it means movement between
lower valleys during winters and higher fields in summers. The herders who practice
seasonal pastoralism have a permanent settlement where their families live, and only a small group
of people migrate with the herd. Transhumance is common in the highlands, mountains, and regions
that are too cold to be utilized and inhabited except during summer.

9. Jhumming, Ladang, Milap, Roca & Ray are also known as:
(a) Intensive Farming (b) Commercial Farming
(c) Nomadic Farming (d) Shifting Farming
www.achieveias.co.in, YouTube Channel: http://youtube.com/c/AchieveIAS Telegram Channel: http://t.me/Achieve_Ias,
Mail: achieveias21@gmail.com, Contact Number: 8968920720

Answer: D

Explanation: Slash and burn farming is a form of shifting agriculture where the natural vegetation
is cut down and burned as a method of clearing the land for cultivation, and then, when the plot
becomes infertile, the farmer moves to a new fresh plat and does the same again. This process is
repeated over and over.Shifting Cultivation is known as Ladang in Indonesia, Caingin in Philippines,
Milpa in Central America & Mexico, Ray in Vietnam, Taungya In Myanmar, Tamrai in Thailand,
Chena in Sri Lanka, Conuco in Venezuela, Roca in Brazil, Masole in central Africa.

10. In this farming the land is used for growing food and fodder crops and rearing livestock?
(a) Intensive Farming (b) Plantation Farming
(c) Primitive Farming (d) Mixed Farming

Answer: D

Explanation: Mixed farming exists in many forms depending on external and internal factors.
External factors are weather patterns, market prices, political stability, technological developments,
etc. Internal factors relate to local soil characteristics, composition of the family and farmers'
ingenuity. Farmers can decide to opt for mixed enterprises when they want to save resources by
interchanging them on the farm - because these permit wider crop rotations and thus reduce
dependence on chemicals, because they consider mixed systems closer to nature, or because they
allow diversification for better risk management. There is wide variation in mixed systems. Even
pastoralists practise a form of mixed farming since their livelihood depends on the management of
different feed resources and animal species. At a higher level, a region can consist of individual
specialized farms and service systems that together act as a mixed system. Other forms of mixed
farming include cultivation of different crops on the same field, such as millet and cowpea or millet
and sorghum, or several varieties of the same crop with different life cycles, which uses space more
efficiently and spreads risks more uniformly. Mixed farming systems can be classified in many ways
- based on land size, type of crops and animals, geographical distribution, market orientation, etc.
Three major categories, in four different modes of farming, are distinguished here. The categories
are:
 On-farm versus between-farm mixing
 Mixing within crops and/or animal systems
 Diversified versus integrated systems.
www.achieveias.co.in, YouTube Channel: http://youtube.com/c/AchieveIAS Telegram Channel: http://t.me/Achieve_Ias,
Mail: achieveias21@gmail.com, Contact Number: 8968920720

ACHIEVE IAS GEOGRAPHY MCQ SERIES, DAY 11, SOLUTIONS

1. The factor(s) responsible for population change is (are):


A. Births B. Deaths C. Migration D. All of the above

Answer: D

Explanation: At the small area level, the key factors of population change are the age structure of
the existing population, the housing markets attracted to and away from an area and their associated
demographic characteristics (fertility patterns, household types etc.) and the supply of dwellings and
mix of housing stock in the area.
Dwelling additions: The addition of dwellings is the major driver of population growth, providing
opportunities for new households (such as young people leaving the family home and divorces) or
households relocating from other areas.
Current age structure The age structure of the local population impacts on Coffs Harbour City's
household types and size, the likelihood of the local population having children and to die, as well
as the propensity for people to move. Age specific propensities for a population to have children or
die are applied to each small area's base population. An older population will have fewer births,
more deaths, while a younger population will have vice versa.
Birth rates: Birth rates are especially influential in determining the number of children in an area,
with most inner urban areas having very low birth rates, compared to outer suburban or rural and
regional areas. Birth rates have been changing, with a greater share of women bearing children at
older ages or not at all, with overall increases in fertility rates. This can have a large impact on the
future population profile.
Death rates: Death rates are influential in shaping the numbers of older people in an area's
population. Death rates too have been changing with higher life expectancy at most ages, with men
gaining on women's greater life chances.
Migration: Migration is one of the most important factors of population change. While births and
deaths are relatively easy to predict due to reliable age specific behaviour, migration is volatile, often
changing due to housing market preferences, economic opportunities and changing household
circumstances. Migration patterns vary across Australia and change across time, but most moves
tend to be short and incremental in nature. Regional areas have larger moves due to the distances
between towns and cities, where people often move for economic reasons, mainly the availability of
employment or education and training opportunities. The most mobile age groups in the population
are the young adults. They tend to move to attend educational institutions, seek work and express
a change in lifestyle. It is for this reason that young people often move the greatest distances and
sometimes move against pre-established patterns. Market research has shown that empty nesters
are more likely to move to smaller accommodation if appropriate and affordable alternative housing
is supplied in the local area that is accessible to established social networks.

2. What do you mean by the term density of population of a country?


A. The average number of deaths per annum
B. The average number of births per annum
C. Number of people living per one square kilometer
D. None of the above

Answer: C
www.achieveias.co.in, YouTube Channel: http://youtube.com/c/AchieveIAS Telegram Channel: http://t.me/Achieve_Ias,
Mail: achieveias21@gmail.com, Contact Number: 8968920720

Explanation: Population density is a measurement of population per unit volume or unit area. It is
a key geographical term. So the density of population is the number of people living in a particular
space, say one kilometer or one mile. It is determined by dividing the population of an area by its
land area.

3. The difference between the birth rate and the death rate of a country is called the:
A. Natural growth rate B. Normal growth rate
C. Actual growth rate D. None of the above

Answer: A

Explanation: The rate of natural increase refers to the difference between the number of live births
and the number of deaths occurring in a year, divided by the mid-year population of that year,
multiplied by a factor (usually 1,000).It is equal to the difference between the crude birth rate and
the crude death rate. This measure of the population change excludes the effects of migration.

4. The following is also known as ‘Sunrise industry’:


A. Iron and steel industry B. Cotton textile
C. Information technology D. All of the above

Answer: D.

Explanation: Sunrise industry is a colloquial term for a burgeoning sector or business in its infancy
stage showing promise of a rapid boom. Sunrise industries are typically characterized by high growth
rates, numerous start-ups, and an abundance of venture capital funding. These industries generate
a lot of "buzz" as investors' interest in its long-term growth prospect and public awareness increases.

KEY TAKEAWAYS:
 A sunrise industry is a new business or business sector showing potential for substantial and
rapid growth.
 Notable characteristics of sunrise industries include high-growth rates and a lot of start-ups and
venture capital funding.
 As a sunrise industry develops, it may transition to the maturity stage and then to the sunset
stage.
 To remain relevant and on an upward trajectory, sunrise industries must prove their viability and
sustainability.

5. The following city is often referred to as ‘Manchester of India’?


A. Mumbai B. Ahmadabad C. Surat D. Kolkata

Answer: B

Explanation: The city of Ahmedabad in the Gujarat state is famously known as the “Manchester
City of India”. This name was given to this city by a popular textile center in the Manchester of Great
Britain, adhering to the striking similarities of Ahmedabad's prospering cotton textile industries with
the ones in Manchester.

6. Cotton textile industry requires:


www.achieveias.co.in, YouTube Channel: http://youtube.com/c/AchieveIAS Telegram Channel: http://t.me/Achieve_Ias,
Mail: achieveias21@gmail.com, Contact Number: 8968920720

A. Warm climate B. Moist climate C. Both ‘a’ and ‘b’ D. None of the above

Answer: C
Explanation: Several factors, like availability of raw cotton, market, transport, etc. play a key role in
the localisation of cotton textile industry. The significance of raw cotton is evident from the fact that
80 per cent of the industry is coterminous with the cotton growing tracts of the country.
Some of the important centres such as Ahmedabad, Solapur, Nagpur, Coimbatore and Indore are
located in the areas of large scale cotton cultivation. Mumbai is also not far away from the cotton
producing areas of Maharashtra and Gujarat which have contributed a good deal in the localisation
and growth of cotton textile industry here. It is equally important to note that cotton is a pure raw
material, in the sense that it does not lose much of its weight in the process of manufacturing and
the slight loss in weight is more than compensated by the use of sizing materials. There is not much
of difference between the cost of transporting raw cotton and finished cloth. Both can be transported
with equal ease and without adding much to the total cost of production. Hence, this industry
normally tends to be located at such centres which have favourable transport facilities with respect
to market. In other words, it is primarily a market oriented industry. With tropical and sub-tropical
climate, all parts of India provide vast market potential for cotton textile industry. West Bengal, Bihar,
Uttar Pradesh, Kerala and Orissa do not grow cotton and still have large number of big centres
where cotton textile industry has flourished well. Thus although in earlier stages of industrialisation,
cotton textile manufacturing was concentrated in Mumbai, it has witnessed great spatial spread and
now covers almost the entire country. Since, it was a traditional cottage industry, cheap and skilled
labour was readily available. The most notable feature of the distribution of the industry is that even
within a state, the industry is localised within particular areas and regions, almost to the complete
exclusion of others. Dispersal of industry from the old nuclei started after 1921 with railway lines
penetrating into the peninsular region. New centres like Coimbatore, Madurai, Bangalore, Nagpur,
Indore, Solapur and Vadodara were favourably located in respect to raw material, market and labour
than places of original locations. This industry also reached some places with some additional
advantages, such as nearness to coal (Nagpur), financial facilities (Kanpur) and wide market with
port facilities (Kolkata). Dispersal of cotton textile industry was further boosted with the development
of hydroelectricity. The growth of this industry in Coimbatore, Madurai and Tirunelveli is largely due
to the availability of hydroelectricity from Pykara dam. The industry also tended to shift from areas
of high labour cost to those with low labour cost. The labour cost factor played a crucial role in
establishing this industry at Madurai, Turunelveli, and Coimbatore.

7. Which of the following comes under primary activities?


A. Agriculture B. Manufacturing C. Transportation D. None of the above

Answer: A

Explanation: A primary economic activity is a type of operation or industry that involves extracting
or refining natural resources, such as mining, agriculture, forestry, and fishing. These economic
functions deliver the raw material for other industries, which further refine and develop these
materials into products to sell to customers.
Traditionally, an economy is segregated into three sectors:
1. Primary: processed raw material from the natural resources
2. Secondary: engaged in manufacturing
3. Tertiary: services based
www.achieveias.co.in, YouTube Channel: http://youtube.com/c/AchieveIAS Telegram Channel: http://t.me/Achieve_Ias,
Mail: achieveias21@gmail.com, Contact Number: 8968920720

While each sector holds its unique importance in development of an economy, often this structure
serves as the guideline for the policy makers to make suited strategies for each sector. The
requirements of the primary sector are different from those of the other two and need special focus
for better performance.

8. With reference to population pyramid, consider the following statements:


1. The numbers of children are shown at the bottom.
2. In a country low birth rates make the pyramid narrow at the base.
3. In India, the pyramid is broad in the younger age groups.

Which of the statements given above is/are correct?


A. 1 and 2 only B. 2 only C. 3 only D. 1, 2 and 3

Answer: D

Explanation: In countries where death rates (especially amongst the very young) are decreasing,
the pyramid is broad in the younger age groups, because more infants survive to adulthood. This
can be seen in the pyramid for India. The numbers of children (below 15 years) are shown at the
bottom and reflect the level of births.In countries like Japan, low birth rates make the pyramid narrow
at the base.
www.achieveias.co.in, YouTube Channel: http://youtube.com/c/AchieveIAS Telegram Channel: http://t.me/Achieve_Ias,
Mail: achieveias21@gmail.com, Contact Number: 8968920720

ACHIEVE IAS GEOGRAPHY MCQ SERIES, DAY 12, SOLUTIONS

1. Which among the following does not belong to India's major large scale industries?
A. Cotton textile industry B. Iron and steel industry
C. Jute industry D. Khadi and village industry

Answer: D

Explanation: Under the chairmanship of K.C. Pant, Deputy Chairman of Planning Commission, a
committee was set up on strengthening the khadi and village industries sector.

2. Which unit of the Hindustan Copper Ltd. is the first copper smelting unit in India?
A. Malanjkhand Copper Project (MP B. Khetri Copper Complex (Rajasthan)
C. Indian Copper Complex (Jharkhand) D. Tajola Copper Project (Maharashtra)

Answer: A

Explanation: Malanjkhand Copper Project was established in 1982. Initial project has been set up by
Hindustan Copper Ltd to exploit the copper ore through an open pit mine.

3. Which is called as the heavy engineering industry?


A. Heavy Electricals B. Heavy Machinery C. Glass D. Iron and steel

Answer: B

Explanation: Heavy Machinery is also called the heavy engineering industry. The major plants are
located at Ranchi, Visakhapatnam and Durgapur.

4. Which of the following industries was de-reserved in 1993?


A. Atomic energy B. Atopic minerals C. Mining of copper and zinc D. Railways

Answer: C

5. When was the “Package for Promotion of Micro and Small Enterprises” announced?
A. August 2005 B. February 2007 C. March 2009 D. November 2010

Answer: B

Explanation: In February 2007, a “Package for Promotion of Micro and Small Enterprises” was
announced. This includes measures addressing concerns of credit, fiscal support, cluster-based
development, infrastructure, technology, and marketing. Capacity building of MSME Associations and
support to women entrepreneurs are the other important attributes of this package.

6. Which is the largest sponge iron producer in the world?


A. Algeria B. India C. Iran D. Saudi Arabia
www.achieveias.co.in, YouTube Channel: http://youtube.com/c/AchieveIAS Telegram Channel: http://t.me/Achieve_Ias,
Mail: achieveias21@gmail.com, Contact Number: 8968920720

Answer: B

Explanation: India has been the largest sponge iron producer in the world since 2002.

7. Consider the following statements and identify the right ones.


A. The 1991 industrial reforms exempted all industries from compulsory licensing
B. There are six industries under compulsory licensing today

A. I only B. ii only C. both D. none

Answer: B

Explanation: The 1991 industrial reforms exempted all industries from compulsory licensing except
18 industries.
www.achieveias.co.in, YouTube Channel: http://youtube.com/c/AchieveIAS Telegram Channel: http://t.me/Achieve_Ias,
Mail: achieveias21@gmail.com, Contact Number: 8968920720

ACHIEVE IAS GEOGRAPHY MCQ SERIES, DAY 13, SOLUTIONS

1. A narrow channel of sea which separates two land masses is known as:
(a) Mound (b) Pass (c) Strait (d) Valley

Answer: C

Explanation: A strait is a narrow body of water that connects two larger bodies of water.It may be
formed by a fracture in an isthmus, a narrow body of land that connects two bodies of water. Tectonic
shifts can lead to straits like this. One strait that was formed by tectonic activity is the Strait of
Gibraltar, the only link between the Mediterranean Sea and the Atlantic Ocean. The Strait of
Gibraltar is actually closing, as the African tectonic plate slides north. In a few thousand years, the
Strait of Gibraltar will be the Isthmus of Gibraltar, and the Mediterranean will be a large,
salty, inland sea. If fractures in an isthmus are created by human activity, the straits are usually
called canals. The Suez Canal was constructed in 1869 as a waterway between the Mediterranean
Sea and the Red Sea. The Suez Canal allows transportation between Europe and Asia without
having to go around the entire continent of Africa. It is an important economic strait.
A strait can also be formed by a body of water overflowing land that has subsided or has
been eroded. The Bosporus, which links the Black Sea and the Aegean Sea, was formed this way.
Land at the Southwestern edge of the Black Sea eroded and crumbled, creating a strait. Although
scientists know that the Black Sea was once an enclosed lake, they do not know for sure whether
the Black Sea flooded into the Aegean, or the Aegean flooded into the Black Sea. The Bosporus is
an extremely important strait, separating the continents of Europe and Asia. Besides two entire
continents, the Bosporus also separates a single country. It splits the European part of Turkey,
called Thrace, and the Asian part, called Anatolia.

2. Which of the following is the southernmost latitude of the Indian mainland?


(a) 8o 4' North (b) 8o 4' South
(c) 6o 4' South (d) 6o 4' North

Answer: A

Explanation: The Republic of India is located on the geographic coordinates of 21.0000° N latitude
and 78.0000° E longitude in Asia. For the country of India the latitude of 22° 00' N and longitude of
77° 00' E denote its geographical alignment. The specific latitude of India suggests its position in
Northern Hemisphere. The terra firma of India is bounded by the Bay of Bengal, Arabian Sea,
Pakistan, Bangladesh, Myanmar, Nepal, Bhutan and China. The country of India includes 3,287,590
square kilometers of area in the southern region Asia. The terrain of India is graced with Deccan
plateau, the plain lands beside the rivers. Himalayan mountain ranges are in the northern part of
India and desert is in West. The diversity of the climatic condition in India is featured by tropic rainy
season in the southern region while the northern part is temperate.
The latitude and the longitude of New Delhi the capital of India are 28° 36' N and 77° 12' E
respectively. The time followed in India is noticed as UTC+5.5. Kanyakumari at 8°4′41″N and
37°32′28″E is the southernmost tip of the Indian mainland, while the southernmost point in India
is Indira Point on Great Nicobar Island.

3. When was Indira-Point submerged under water?


(a) 2005 (b) 2004 (c) 2007 (d) 2002
www.achieveias.co.in, YouTube Channel: http://youtube.com/c/AchieveIAS Telegram Channel: http://t.me/Achieve_Ias,
Mail: achieveias21@gmail.com, Contact Number: 8968920720

Answer: B
Explanation: In the year 2004, Indra Point submerged under the water. Located at at Great Nicobar
Island of Andaman and Nicobar Islands, Indra Point is a village in the Nicobar district. Reports of
many missing inhabitants raised after this tsunami attack. Around 20 families along with sea turtles
were also missing. Indira Point comes under the Nicobar district and Great Nicobar Tehsil in
Andaman and Nicobar Islands of India. Administratively; Indira Point comes under the Laxmingar
Panchayat. Indira Point was earlier known as the Pygmalion Point since the time of its inception.
But when late P.M. Rajiv Gandhi visited this place he named it as Indira Point. Indira Point comes
under the Nicobar district and Great Nicobar Tehsil in Andaman and Nicobar Islands of India.
Administratively; Indira Point comes under the Laxmingar Panchayat. Indira Point is known as the
location of the southernmost point of India's territory.

4. Suez Canal was opened in the year?


(a) 1869 (b) 1969 (c) 1769 (d) 1896

Answer: A

Explanation: The Suez Canal is a man-made waterway connecting the Mediterranean Sea to the
Indian Ocean via the Red Sea. It enables a more direct route for shipping between Europe and Asia,
effectively allowing for passage from the North Atlantic to the Indian Ocean without having to
circumnavigate the African continent. The waterway is vital for international trade and, as a result,
has been at the center of conflict since it opened in 1869.The Suez Canal stretches 120 miles from
Port Said on the Mediterranean Sea in Egypt southward to the city of Suez (located on the northern
shores of the Gulf of Suez). The canal separates the bulk of Egypt from the Sinai Peninsula. It took
10 years to build, and was officially opened on November 17, 1869.Owned and operated by the
Suez Canal Authority, the Suez Canal’s use is intended to be open to ships of all countries, be it for
purposes of commerce or war—though that hasn’t always been the case.

5. Kavarati is situated in?


(a) Pondicherry (b) Diu and Daman
(c) Andaman and Nicobar Islands (d) Lakshadweep

Answer: D

Explanation: The Kavaratti is the capital of the Union Territory Lakshadweep in India. The Island of
Kavaratti lies 360Km of the coast of the State of Kerala at 10.57°N 72.64°E. is the closest major city
on the Indian mainland at a distance of 404 km (218 nmi). It has a lagoon area of 3.46 sq mi.
Karavatti has been selected as one of the hundred Indian cities to be developed as asmart city under
PM Narendra Modi’s flagship Smart Cities Mission. The Kavaratti Island is the headquarters of the
Union Territory of Lakshadweep. This island is at a distance of 404 km (218 nautical miles) from
Kochi and is located between Agatti Island on the west and Andrott Island on the east. It lies between
10o 32′ and 10o 35′ N latitude and 72o 35′ and 72o 40′ E longitude, having an area of 4.22 sq km.
Maximum length of the island is 5.8 km and width is 1.6 km. It has a lagoon having a length of about
6 km and an area of 4.96 sq km. The island is 2 to 5 m above the mean sea level on the western
side and 2 to 3 m on the eastern side. It is located in the centre of Lakshadweep archipelago.
Strangely Kavaratti has a small inland lake at its Northern end. The island itself is stretched over an
area of slightly more than 4 sq kms and has the maximum percentage of non – islanders as
www.achieveias.co.in, YouTube Channel: http://youtube.com/c/AchieveIAS Telegram Channel: http://t.me/Achieve_Ias,
Mail: achieveias21@gmail.com, Contact Number: 8968920720

residents. The climate of Kavaratti is similar to the climatic conditions of Kerala. March to May is the
hottest period of the year. The temperature ranges from 25oC to 35oC and humidity ranging from
70 -76 per cent for most of the year. The average rainfall received is 1600 mm a year. Monsoon
prevails here from 15th May to 15th September. The monsoon period raises temperature to the
mercury level between 27- 30 degrees. During the monsoon time, boats are not allowed outside the
lagoon because of the violent sea. The presence of the reef maintains calm at the lagoon.

6. Latitudinal and longitudinal extent of India is?


(a) 33o (b) 35o (c) 30o (d) 32o

Answer: C

Explanation: The Longitudinal & latitudinal extend of India is 68°4'N , 97°25'N and 8°4'E and 37°6'E.

7. The westernmost longitude of India is?


(a) 82o 30' East (b) 68o 7' West (c) 68o 7' East (d) 97o 25' East

Answer: C
Explanation: The westernmost longitude of India is located in Gujarat (Kutch area). India's western
most longitude is just west of Ghuar Mota, Gujarat , 68 degree, 34'min,west. The answer is 68° 7′ E
(East).

8. Tropic of Cancer does not pass through?


(a) Rajasthan (b) Chhattisgarh (c) Orissa (d) Tripura

Answer: C

Explanation: Tropic of Cancer is an imaginary line, at an angle of 23.50 degrees North from the
Equator, that passes through the middle of India. If you run this imaginary line around Earth (Yes,
Flat-Earthlings, the Earth is spherical), it passes through 17 countries, India being one of them. The
Tropic of Cancer passes through eight states in India: Gujarat (Jasdan), Rajasthan (Kalinjarh),
Madhya Pradesh (Shajapur), Chhattisgarh (Sonhat), Jharkhand (Lohardaga), West Bengal
(Krishnanagar), Tripura (Udaipur) and Mizoram (Champhai).

9. What is IST?
(a) Indian Stretchable Time (b) International Standard Time
(c) Indian Stabilized Time (d) Indian Standard Time

Answer: D

Explanation: IST stands for Indian Standard Time. Indian Standard Time is the time observed in
India and Sri Lanka. It passes through Allahabad. India has a time of GMT+5:30. It's main
importance is that it helps us in scheduling partys, functions, etc. which are connected throughout
the country.

10. Indian state with the longest coastline on the eastern coast:
(a) Tamil Nadu (b) Andhra Pradesh (c) Orissa (d) Jharkhand
www.achieveias.co.in, YouTube Channel: http://youtube.com/c/AchieveIAS Telegram Channel: http://t.me/Achieve_Ias,
Mail: achieveias21@gmail.com, Contact Number: 8968920720

Answer: B

Explanation: Coastal India has a coastline of 7516.6 km along the Arabian sea through the Bay of
Bengal, The coastline of India starts from of the Gulf of Kutch across the Gulf of Khambhat then
Konkan to Kanara and through the Malabar along the Arabian sea and goes along the Bay of Bengal
through the Coromandal Coast, Puducherry, Utkala to Sunderbans in Coastal East India. Gujarat
(1600 km) state has the longest Coastline in India, lies on the Kathiawar region and bounded by the
Arabian Sea. The state is dotted with 41 sea ports and most amazing beaches of Gujarat includes
Nagoa beach in Diu, Dwarka beach, Mandvi beach Devka beach and Porbandar beach. Andhra
Pradesh (974 km) state the second longest coastline in India with a length of 974 km, along with
Coromandel Coast on Bay of Bengal. The Coastal Andhra Pradesh consists of Konaseema, wildlife
and bird sanctuaries, estuaries, sandy beaches and several sea ports.
www.achieveias.co.in, YouTube Channel: http://youtube.com/c/AchieveIAS Telegram Channel: http://t.me/Achieve_Ias,
Mail: achieveias21@gmail.com, Contact Number: 8968920720

ACHIEVE IAS GEOGRAPHY MCQ SERIES, DAY 14, SOLUTIONS

1. A landmass bounded by sea on three sides is referred to as:


A. Coast B. Peninsula C. Island D. None of the above

Answer: B

Explanation: A peninsula is a piece of land that is almost entirely surrounded by water but is
connected to the mainland on one side. Peninsulas can be very small, sometimes only large enough
for a single lighthouse, for instance. Lighthouses often sit on peninsulas near rocky coastlines to
warn sailors that they are getting close to land. Peninsulas can also be very large. Most of the U.S.
state of Florida is a peninsula that separates the Gulf of Mexico and the Atlantic Ocean. Peninsulas
are found on every continent. In North America, the narrow peninsula of Baja California, in Mexico,
separates the Pacific Ocean and the Sea of Cortez, also called the Gulf of California. In Europe, the
nations of Portugal and Spain make up the Iberian Peninsula. The so-called Horn of Africa, which
juts into the Arabian Sea on central African east coast, is a huge peninsula. The nations of North
Korea and South Korea make up the Korean Peninsula in eastern Asia. In Australia, the Cape York
Peninsula is only 160 kilometers (99 miles) from the island of New Guinea. The Antarctic Peninsula
seems to point to the tip of South America, several hundred kilometers (miles) away.

2. Which of the following is defined the northern frontiers of India?


A. Himalayan Mountain range B. Karakoram Range
C. Aravali Range D. Satpura Range

Answer: A

Explanation: Great Himalayas, also called Higher Himalayas or Great Himalaya Range, highest
and Northern most section of the Himalayan mountain ranges. It extends south-eastward across
northern Pakistan, northern India, and Nepal before trending eastward across Sikkim state (India)
and Bhutan and finally turning north-eastward across northern Arunachal Pradesh state (India);
throughout nearly all of its length it adjoins to the north the southern Tibet Autonomous Region
of China. The range’s total length is some 1,400 miles (2,300 km), and it has an average elevation
of more than 20,000 feet (6,100 metres). The Great Himalayas contain many of the world’s tallest
peaks, including (from west to east) Nanga Parbat, Annapurna, Mount Everest, and Kanchenjunga.
The most characteristic features of the Himalayas are their soaring heights, steep-sided jagged
peaks, valley and alpine glaciers often of stupendous size, topography deeply cut by erosion,
seemingly unfathomable river gorges, complex geologic structure, and series of elevational belts (or
zones) that display different ecological associations of flora, fauna, and climate. Viewed from the
south, the Himalayas appear as a gigantic crescent with the main axis rising above the snow line,
where snowfields, alpine glaciers, and avalanches all feed lower-valley glaciers that in
turn constitute the sources of most of the Himalayan rivers. The greater part of the Himalayas,
however, lies below the snow line. The mountain-building process that created the range is still
active. As the bedrock is lifted, considerable stream erosion and gigantic landslides occur. The
Himalayan ranges can be grouped into four parallel longitudinal mountain belts of varying width,
each having distinct physiographic features and its own geologic history. They are designated, from
south to north, as the Outer, or Sub-, Himalayas (also called the Siwalik Range); the Lesser, or
Lower, Himalayas; the Great Himalaya Range (Great Himalayas); and the Tethys, or Tibetan,
www.achieveias.co.in, YouTube Channel: http://youtube.com/c/AchieveIAS Telegram Channel: http://t.me/Achieve_Ias,
Mail: achieveias21@gmail.com, Contact Number: 8968920720

Himalayas. Farther north lie the Trans-Himalayas in Tibet proper. From west to east the Himalayas
are divided broadly into three mountainous regions: western, central, and eastern.

3. Consider the following statement(s) is/are related to the Northern Mountains


I. A great arc of mountains, consisting of the Himalayas of Nepal, Hindu Kush, and Patkai ranges
define the northern Indian subcontinent.
II. The mountains in these ranges include some of the world's tallest mountains which act as a
natural barrier to cold polar winds.

Which of the above statement(s) is/are correct?


A. Only I B. Only II C. Both I and II D. Neither I nor II

Answer: C

Explanation: The north and north-eastern India comprises of Himalayas that separates India from
the Tibetan plateau, and other ranges namely Hindu Kush and Patkai at north-eastern border with
Burma. These ranges include world’s tallest mountains that impact the climatic conditions of India.
Some of the major Indian rivers originate from these ranges and flow through the plains, joining
the ocean. The Satpura and the Aravallis are also important mountain ranges running across parts
of central and northern India. It is the world’s highest mountain range bordering India at northeast
that hold the tallest peak known as Mount Everest. Most of the peaks stay covered with snow all-
round the year. Spread over an area of 500000 square kilometer, the Himalayas stretch from the
Indian states of Jammu and Kashmir, Himachal Pradesh, Uttarakhand, Sikkim to Arunachal
Pradesh. They object the flow of downhill winds from Central Asia; this protects the northern India
from extreme cold in winters. The Himalayan range is divided into five types of ranges mentioned
as under:
Pir Panjal Range – They form the southern part of Himalayas touching the height of 5000m. They
start from Gulmarg of the northwest region and extend to the Banihal Pass through the southern
edge of Kashmir. They comprise of Pir Panjal Pass, Banihal Pass and Sythen Pass that connect
Kashmir to Kishtwar.
Ladakh Range – The range lies at northern side of Leh and melds with Kailash range of Tibet.
They include Kardung La Pass and Digar La Pass.
Zanskar Range – Lying at the northern parts of Himalayas, Zanskar range stretches from
Lamarayu in west, running through Lahaul and Spiti, Kinnaur to Uttaranchal. It comprises of Fatu
La Pass, Singge La Pass, Cha Cha La Pass and Rubrang La Pass.
Dhaula Dhar Range – It lies south of Pir Panjal range and is characterized by snow-tipped
mountains forming Beas, Ravi Chenab and Tawi valleys.
East Karakoram Range – Characterized by its vastness, it separates India with Central Asia and
connects Leh, Yarkand and Kashgar. Here, the highest peaks are Rimo, Saltoro Kangri and
Teram Kargari.
Siwalik Hills: Siwalik meaning ‘tresses of Lord Shiva’, the Hills lie in the outer Himalayas
extending from Arunachal Pradesh to West Bengal and from Uttrakhand to Himachal Pradesh and
Kashmir. They reach an average height of 1500m to 2000m including hills of Jammu, Vaishno
Devi and Kangra.
Karakoram Range: It holds the pride of preserving the world’s second highest peak – K2, and
glaciers – Siachen and Biafo, but along with the dilemma of lying in disputed areas of Jammu and
Kashmir. They include Hindu Kush, Pamirs, Kunlun and Himalayas, their geographical location is
www.achieveias.co.in, YouTube Channel: http://youtube.com/c/AchieveIAS Telegram Channel: http://t.me/Achieve_Ias,
Mail: achieveias21@gmail.com, Contact Number: 8968920720

really of great significance as the borders of India, Afghanistan, Pakistan, China and Tajikistan
come together to be a part of them.
Patkai Range: Also known as Purvanchal range, the Patkai range lies in the eastern India-Burma
border. They comprise the tapered peaks, steep mountains and deep valleys. They include the
hills namely, Garo-Khasi-Jaintia in Meghalaya, Patkai-Bum and Lushai.
Vindhya Range: This range spreads in central India covering an area of 1050 km. They are
known to have formed from the Aravalli mountain ranges. Being in central India, they separate
north India with south India.
Satpura Range: This range also covers an area of central India, stretching from Gujarat, running
parallel to Vindhya range through the states of Maharashtra, Madhya Pradesh and reaching
Chhattisgarh. They are adorned with many peaks and separate Indo-Gangetic plain from Deccan
Plateau.
Aravali Range: The oldest mountain range that covers parts of Rajasthan and Haryana near
Delhi, Aravali range has its highest peak of 1,722 m in Mount Abu, known as Guru Shikhar.

4. Consider the following statement(s) is/are related to Vindhya range:


I. Geographically, separates Northern India from Southern India.
II. The western end of the range lies in eastern Gujarat, near its border with Madhya Pradesh, and
runs east and north, almost meeting the Ganges at Mirzapur

Which of the above statement(s) is/are correct?


A. Only I B. Only II C. Both I and II D. Neither I nor II

Answer: C

Explanation: India. From Gujarat state on the west, it extends about 675 miles (1,086 km)
across Madhya Pradesh state to abut on the Ganges (Ganga) River valley near Varanasi, Uttar
Pradesh. The mountains form the southern edge of the Malwa Plateau and then divide into two
branches: the Kaimur Range, running north of the Son River into western Bihar state, and the
southern branch, running between the upper reaches of the Son and Narmada rivers to meet
the Satpura Range in the Maikala Range (or Amarkantak Plateau).The Vindhya Range, at an
elevation from 1,500 to 3,500 feet (450 to 1,100 metres), gives rise to the main southern tributaries
of the Ganges-Yamuna system, including the Chambal, Betwa, Ken, and Tons rivers. Because of
their horizontal sandstone structure, the mountains tend to be flat-topped and plateau like. The 2nd-
century-CE Greek geographer Ptolemy called them the Vindius. The range is regarded as marking
the border between northern and peninsular India. Kaimur Hills, also called Kaimur Range, eastern
portion of the Vindhya Range, starting near Katangi in the Jabalpur district of Madhya Pradesh and
running generally east for a distance of about 300 miles (480 km) to Sasaram in Bihar. Its maximum
width is about 50 miles (80 km).After traversing the northern part of Jabalpur and southeast Maihar,
the range turns to the east and runs through Rewa. There it separates the valley of the Son and
Tons rivers and continues into the Mirzapur district of Uttar Pradesh and the Shahabad district
of West Bengal. The range has a distinctive jagged appearance in Madhya Pradesh,
with metamorphic rock formations and upheaved rock strata. The range almost disappears at
places, being marked by a low chain of rocks, and it never rises more than a few thousand feet
above the plain. The ruined fort of Rohtas is situated in these hills, and archaeological explorations
have uncovered well-preserved prehistoric rock art and paintings in caves and sheltered areas in
Bihar.
www.achieveias.co.in, YouTube Channel: http://youtube.com/c/AchieveIAS Telegram Channel: http://t.me/Achieve_Ias,
Mail: achieveias21@gmail.com, Contact Number: 8968920720

5. Consider the following statement(s) is/are related to the Deccan Plateau


I. It is a large triangular plateau, bounded by the Vindhyas to the north and flanked by the Eastern
and Western Ghats.
II. It slopes gently from west to east and gives rise to several peninsular rivers such as the Godavari,
the Krishna, the Kaveri and the Mahanadi which drain into the Bay of Bengal.

Which of the above statement(s) is/are correct?


A. Only I B. Only II C. Both I and II D. Neither I nor II

Answer: C

Explanation: Deccan, the entire southern peninsula of India south of the Narmada River, marked
centrally by a high triangular tableland. The name derives from the Sanskrit daksina (“south”).
The plateau is bounded on the east and west by the Ghats, escarpments that meet at the plateau’s
southern tip. Its northern extremity is the Satpura Range. The Deccan’s average elevation is about
2,000 feet (600 metres), sloping generally eastward. Its principal rivers—the Godavari, Krishna,
and Kaveri (Cauvery)—flow from the Western Ghats eastward to the Bay of Bengal. The plateau’s
climate is drier than that on the coasts and is arid in places.

6. Consider the following statement(s) is/are related to the Chota Nagpur Plateau
I. It is situated in eastern India, covering much of Jharkhand and adjacent parts of Odisha, Bihar
and Chhattisgarh.
II. The natural vegetation in most of the peninsula is xeric scrub, part of the Northwestern thorn
scrub forests ecoregion.

Which of the above statement(s) is/are correct?


Code:
A. Only I B. Only II C. Both I and II D. Neither I nor II

Answer: C

Explanation: Chota Nagpur, plateau in eastern India, in north-western Chhattisgarh and


central Jharkhand states. The plateau is composed of Precambrian rocks (i.e., rocks more than
about 540 million years old). Chota Nagpur is the collective name for the Ranchi, Hazaribagh, and
Kodarma plateaus, which collectively have an area of 25,293 square miles (65,509 square km). Its
largest division is the Ranchi Plateau, which has an average elevation of about 2,300 feet (700
metres). The Chota Nagpur plateau in its entirety lies between the basins of the Ganges
(Ganga) and Son rivers to the north and the Mahanadi River to the south. Through its centre, from
west to east, runs the coal-bearing, faulted Damodar River valley. Numerous streams have
dissected the uplands into a peneplain (an area reduced almost to a plain by erosion) with isolated
hills. Centuries of heavy cultivation have depleted the plateau of much of its natural vegetation,
though some valuable forests still remain. Forest products, such as tussah silk and lac, are
economically important. The Chota Nagpur area has the most valuable concentration of mineral
resources in India. The Damodar valley has vast coal reserves, and the Hazaribagh region is one
of the main sources of mica in the world. Other minerals are copper, limestone, bauxite, iron ore,
asbestos, and apatite (useful in the manufacture of phosphate fertilizers). A huge thermal plant for
generating electricity and a large steel mill are located at Bokaro in eastern Jharkhand. Railroads
www.achieveias.co.in, YouTube Channel: http://youtube.com/c/AchieveIAS Telegram Channel: http://t.me/Achieve_Ias,
Mail: achieveias21@gmail.com, Contact Number: 8968920720

cross the plateau, connecting Kolkata (Calcutta), West Bengal, to the southeast with Patna, Bihar,
to the north, and also link other cities in the south and west.

7. Consider the following statement(s) is/are related to the Western Ghats or Sahyadri
mountains
I. It run along the western edge of India's Deccan Plateau and separate it from a narrow coastal
plain along the Arabian Sea.
II. The range runs approximately 1,600 km from south of the Tapti River near the Gujarat–
Maharashtra border and across Maharashtra, Goa, Karnataka, Kerala and Tamil Nadu to the
southern tip of the Deccan peninsula.

Which of the above statement(s) is/are correct?


A. Only I B. Only II C. Both I and II D. Neither I nor II

Answer: C

Explanation: The Western Ghats are internationally recognized as a region of immense global
importance for the conservation of biological diversity, besides containing areas of high geological,
cultural and aesthetic values. A chain of mountains running parallel to India’s western coast,
approximately 30-50 km inland, the Ghats traverse the States of Kerala, Tamil Nadu, Karnataka,
Goa, Maharashtra and Gujarat. These mountains cover an area of around 140,000 km² in a 1,600
km long stretch that is interrupted only by the 30 km Palghat Gap at around 11°N.
Older than the great Himalayan mountain chain, the Western Ghats of India are a geomorphic
feature of immense global importance. The Outstanding Universal Value of the Western Ghats is
manifested in the region’s unique and fascinating influence on large-scale biophysical and ecological
processes over the entire Indian peninsula. The mountains of the Western Ghats and their
characteristic montane forest ecosystems influence the Indian monsoon weather patterns that
mediate the warm tropical climate of the region, presenting one of the best examples of the tropical
monsoon system on the planet. The Ghats act as a key barrier, intercepting the rain-laden monsoon
winds that sweep in from the south-west during late summer. A significant characteristic of the
Western Ghats is the exceptionally high level of biological diversity and endemism. This mountain
chain is recognized as one of the world’s eight ‘hottest hotspots’ of biological diversity along with Sri
Lanka. The forests of the Western Ghats include some of the best representatives of non-equatorial
tropical evergreen forests in the world. At least 325 globally threatened (IUCN Red Data List) species
occur in the Western Ghats. The globally threatened flora and fauna in the Western Ghats are
represented by 229 plant species, 31 mammal species, 15 bird species, 43 amphibian species, 5
reptile species and 1 fish species. Of the total 325 globally threatened species in the Western Ghats,
129 are classified as Vulnerable, 145 as Endangered and 51 as Critically Endangered. The Western
Ghats, also known as Sahyadri (Benevolent Mountains), are a mountain range that covers an area
of 140,000 square kilometres (54,000 sq mi) in a stretch of 1,600 kilometres (990 mi) parallel to the
western coast of the Indian peninsula, traversing the states of Kerala, Tamil
Nadu, Karnataka, Goa, Maharashtra and Gujarat. It is a UNESCO World Heritage Site and is one
of the eight "hottest hot-spots" of biological diversity in the world. It is sometimes called the
Great Escarpment of India. It contains a large proportion of the country's flora and fauna, many of
which are only found in India and nowhere else in the world. According to UNESCO, the Western
Ghats are older than the Himalayas. They influence Indian monsoon weather patterns by
intercepting the rain-laden monsoon winds that sweep in from the south-west during late summer.
The range runs north to south along the western edge of the Deccan Plateau, and separates the
www.achieveias.co.in, YouTube Channel: http://youtube.com/c/AchieveIAS Telegram Channel: http://t.me/Achieve_Ias,
Mail: achieveias21@gmail.com, Contact Number: 8968920720

plateau from a narrow coastal plain, called Konkan, along the Arabian Sea. A total of thirty-nine
areas in the Western Ghats, including national parks, wildlife sanctuaries and reserve forests, were
designated as world heritage sites in 2012 – twenty in Kerala, ten in Karnataka, five in Tamil
Nadu and four in Maharashtra.
The range starts near the Songadh town of Gujarat, south of the Tapti river, and runs approximately
1,600 km (990 mi) through the states of Maharashtra, Goa, Karnataka, Kerala and Tamil
Nadu ending at Marunthuvazh Malai, at Swamithope, near the southern tip of India. These hills
cover 160,000 km2 (62,000 sq mi) and form the catchment area for complex riverine drainage
systems that drain almost 40% of India. The Western Ghats block southwest monsoon winds from
reaching the Deccan Plateau. The average elevation is around 1,200 m (3,900 ft).

8. Consider the following statement(s) is/are related to the Eastern Ghats


I. It is a discontinuous range of mountains, which have been eroded and vivisected by the four major
rivers of southern India, the Godavari, Mahanadi, Krishna, and Kaveri.
II. The Nilgiri hills in Tamil Nadu lies at the junction of the Eastern and Western Ghats.

Which of the above statement(s) is/are correct?


A. Only I B. Only II C. Both I and II D. Neither I nor II

Answer: C

Explanation: The Eastern Ghats are a discontinuous range of mountains along India's eastern
coast. The Eastern Ghats run from the northern Odisha through Andhra Pradesh to Tamil Nadu in
the south passing some parts of Karnataka and in the Wayanad district of Kerala. They are eroded
and cut through by four major rivers of peninsular India, viz. Godavari, Mahanadi, Krishna,
and Kaveri. The mountain ranges run parallel to the Bay of Bengal. The Deccan Plateau lies to the
west of the range, between the Eastern Ghats and Western Ghats. The Coastal plains, including
the Coromandel Coast region, lie between the Eastern Ghats and the Bay of Bengcal. The Eastern
Ghats are not as high as the Western Ghats. The Eastern Ghats are older than the Western Ghats,
and have a complex geological history related to the assembly and breakup of the ancient
supercontinent of Rodinia and the assembly of the Gondwana supercontinent. The Eastern Ghats
are made up of charnockites, granite gneiss, khondalites, metamorphic gneisses and quartzite rock
formations. The structure of the Eastern Ghats includes thrusts and strike-slip faults all along its
range. Limestone, bauxite and iron ore are also found in the Eastern Ghats hill ranges. Nilgiri Hills,
mountainous region of Tamil Nadu state, south-eastern India. The peaks of the Nilgiri rise abruptly
from the surrounding plains to an elevation of about 6,000 to 8,000 feet (1,800 to 2,400 metres);
one of them, Doda Betta (8,652 feet [2,637 metres]), is the highest point in Tamil Nadu. Part of the
Western Ghats, the hills are separated from the Karnataka Plateau (north) by the Noyar River and
from the Anaimalai and Palni hills (south) by the Palghat Gap. They are considerably cooler and
wetter than the surrounding plains, the upper hills forming undulating grassy downs. Tea, cinchona
(whose bark yields quinine), coffee, and vegetables are grown extensively.

9. Consider the following statement(s) is/are related to the Thar Desert


I. The desert continues into Pakistan as the Cholistan Desert.
II. Most of the Thar Desert is situated in Rajasthan, covering 61% of its geographic area.

Which of the above statement(s) is/are correct?


A. Only I B. Only II C. Both I and II D. Neither I nor II
www.achieveias.co.in, YouTube Channel: http://youtube.com/c/AchieveIAS Telegram Channel: http://t.me/Achieve_Ias,
Mail: achieveias21@gmail.com, Contact Number: 8968920720

Answer: C

Explanation: Thar Desert, also called Great Indian Desert, arid region of rolling sand hills on the
Indian subcontinent. It is located partly in Rajasthan state, north-western India, and partly
in Punjab and Sindh (Sind) provinces, eastern Pakistan. The Thar Desert covers some 77,000
square miles (200,000 square km) of territory. It is bordered by the irrigated Indus River plain to the
west, the Punjab Plain to the north and northeast, the Aravalli Range to the southeast, and the Rann
of Kachchh to the south. The subtropical desert climate there results from persistent high
pressure and subsidence at that latitude. The prevailing southwest monsoon winds that bring rain
to much of the subcontinent in summer tend to bypass the Thar to the east. The name Thar is
derived from thul, the general term for the region’s sand ridges. The amount of annual rainfall in the
desert is generally low, ranging from about 4 inches (100 mm) or less in the west to about 20 inches
(500 mm) in the east. Precipitation amounts fluctuate widely from year to year. About 90 percent of
the total annual rainfall occurs during the season of the southwest monsoon, from July to September
(see also Indian monsoon). During other seasons the prevailing wind is the dry northeast monsoon.
May and June are the hottest months of the year, with temperatures rising to 122 °F (50 °C). During
January, the coldest month, the mean minimum temperature ranges between 41 and 50 °F (5 and
10 °C), and frost is frequent. Dust storms and dust-raising winds, often blowing with velocities of 87
to 93 miles (140 to 150 km) per hour, are common in May and June.The desert vegetation is mostly
herbaceous or stunted scrub; drought resistant trees occasionally dot the landscape, especially in
the east. On the hills, gum arabic acacia and euphorbia may be found. The khajri (or khejri) tree
(Prosopis cineraria) grows throughout the plains.

10. The origin of Himalayas can best be explained by


A. Continental Drift Theory B. Ocean Floor Mapping
C. Sea Floor Spreading D. Theory of Plate Tectonics

Answer: D

Explanation: Plate tectonics, theory dealing with the dynamics of Earth’s outer shell—
the lithosphere—that revolutionized Earth sciences by providing a uniform context for
understanding mountain-building processes, volcanoes, and earthquakes as well as the evolution
of Earth’s surface and reconstructing its past continents and oceans. The concept of
plate tectonics was formulated in the 1960s. According to the theory, Earth has a rigid outer layer,
known as the lithosphere, which is typically about 100 km (60 miles) thick and overlies a plastic
(moldable, partially molten) layer called the asthenosphere. The lithosphere is broken up into seven
very large continental- and ocean-sized plates, six or seven medium-sized regional plates, and
several small ones. These plates move relative to each other, typically at rates of 5 to 10 cm (2 to 4
inches) per year, and interact along their boundaries, where they converge, diverge, or slip past one
another. Such interactions are thought to be responsible for most of Earth’s seismic and volcanic
activity, although earthquakes and volcanoes can occur in plate interiors. Plate motions
cause mountains to rise where plates push together, or converge, and continents to fracture and
oceans to form where plates pull apart, or diverge. The continents are embedded in the plates and
drift passively with them, which over millions of years results in significant changes in
Earth’s geography. The theory of plate tectonics is based on a broad synthesis of geologic and
geophysical data. It is now almost universally accepted, and its adoption represents a true scientific
revolution, analogous in its consequences to quantum mechanics in physics or the discovery of
www.achieveias.co.in, YouTube Channel: http://youtube.com/c/AchieveIAS Telegram Channel: http://t.me/Achieve_Ias,
Mail: achieveias21@gmail.com, Contact Number: 8968920720

the genetic code in biology. Incorporating the much older idea of continental drift, as well as the
concept of seafloor spreading, the theory of plate tectonics has provided an overarching framework
in which to describe the past geography of continents and oceans, the processes controlling creation
and destruction of landforms, and the evolution of Earth’s
crust, atmosphere, biosphere, hydrosphere, and climates. During the late 20th and early 21st
centuries, it became apparent that plate-tectonic processes profoundly influence the composition of
Earth’s atmosphere and oceans, serve as a prime cause of long-term climate change, and make
significant contributions to the chemical and physical environment in which life evolves. In essence,
plate-tectonic theory is elegantly simple. Earth’s surface layer, 50 to 100 km (30 to 60 miles) thick,
is rigid and is composed of a set of large and small plates. Together, these
plates constitute the lithosphere, from the Greek lithos, meaning “rock.” The lithosphere rests on
and slides over an underlying partially molten (and thus weaker but generally denser) layer of plastic
partially molten rock known as the asthenosphere, from the Greek asthenos, meaning “weak.” Plate
movement is possible because the lithosphere-asthenosphere boundary is a zone of detachment.
As the lithospheric plates move across Earth’s surface, driven by forces as yet not fully understood,
they interact along their boundaries, diverging, converging, or slipping past each other. While the
interiors of the plates are presumed to remain essentially undeformed, plate boundaries are the sites
of many of the principal processes that shape the terrestrial surface, including
earthquakes, volcanism, and orogeny (that is, formation of mountain ranges).
www.achieveias.co.in, YouTube Channel: http://youtube.com/c/AchieveIAS Telegram Channel: http://t.me/Achieve_Ias,
Mail: achieveias21@gmail.com, Contact Number: 8968920720

ACHIEVE IAS GEOGRAPHY MCQ SERIES, DAY 15, SOLUTIONS

1. When the streams flow in different directions from a central peak or dome like structure, a
pattern is developed which is called as:
(a) Dendritic Drainage Pattern (b) Trellis Drainage Pattern
(c) Rectangular Drainage Pattern (d) Radial Drainage Pattern

Answer: D

Explanation: A drainage pattern in which consequent streams radiate or diverge outward, like the
spokes of a wheel, from a high central area; it is best developed on the slopes of a young,
unbreached domal structure or of a volcanic cone.

2. A lake formed when a meandering river is cut off from the mainstream is known as:
(a) Ox-Bow Lakes (b) Glacial Lakes (c) Lagoons (d) Man Made Lakes

Answer: A

Explanation: An oxbow lake starts out as a curve, or meander, in a river. A lake forms as the river
finds a different, shorter, course. The meander becomes an oxbow lake along the side of the river.
Oxbow lakes usually form in flat, low-lying plains close to where the river empties into another body
of water. On these plains, rivers often have wide meanders. Meanders that form oxbow lakes have
two sets of curves: one curving away from the straight path of the river and one curving back. The
corners of the curves closest to each other are called concave banks. The concave banks erode over
time. The force of the rivers flowing water wears away the land on the meanders concave banks.
The banks opposite the concave banks are called convex banks. The opposite of erosion happens
here. Silt and sediment build up on convex banks. This build-up is called deposition. Erosion and
deposition eventually cause a new channel to be cut through the small piece of land at the narrow
end of the meander. The river makes a shortcut. Oxbow lakes are the remains of the bend in the
river. Oxbow lakes are still water lakes. This means that water does not flow into or out of them.
There is no stream or spring feeding the lake, and it doesn’t have a natural outlet. Oxbow lakes often
become swamps or bogs, and they often dry up as their water evaporates. Oxbow lakes can be rich
wildlife habitats. Along the Amazon River in South America, oxbow lakes are common and their still
waters provide a unique habitat for plants and wildlife. Giant river otters frequently live in the
Amazons oxbow lakes, and feed on about 5 kilograms (11 pounds) of fish per day. People often
create oxbow lakes. The Mississippi River is shorter now than it was in the 19th century, for instance,
because engineers have cut off hundreds of meanders. This created hundreds of oxbow lakes.
These lakes eventually dried up to create acres of land for farming, housing, and industry. An oxbow
lake gets its name from the U-shaped collar placed around an oxs neck to which a plough is
attached. It can also be called a horseshoe lake, a loop lake, or a cut off lake.

3. The Sutlej, Beas, Ravi, Chenab and Jhelum are the major tributaries to river:
(a) Ganges (b) Brahmaputra (c) Indus (d) Godavari

Answer: C

Explanation: Indus River, Tibetan and Sanskrit Sindhu, Sindhi Sindhu, or Mehran, great trans-
Himalayan river of South Asia. It is one of the longest rivers in the world, with a length of some 2,000
www.achieveias.co.in, YouTube Channel: http://youtube.com/c/AchieveIAS Telegram Channel: http://t.me/Achieve_Ias,
Mail: achieveias21@gmail.com, Contact Number: 8968920720

miles (3,200 km). Its total drainage area is about 450,000 square miles (1,165,000 square km), of
which 175,000 square miles (453,000 square km) lie in the ranges and foothills of the Himalayas,
the Hindu Kush, and the Karakoram Range; the rest is in the semiarid plains of Pakistan. The river’s
annual flow is about 58 cubic miles (243 cubic km)—twice that of the Nile River and three times that
of the Tigris and Euphrates rivers combined. The river’s conventional name derives from the Tibetan
and Sanskrit name Sindhu. The earliest chronicles and hymns of the Aryan peoples of ancient India,
the Rigveda, composed about 1500 BCE, mention the river, which is the source of the country’s
name.The river rises in the south-western Tibet Autonomous Region of China near Lake Mapam at
an elevation of about 18,000 feet (5,500 metres). For about 200 miles (320 km) it flows northwest,
crossing the southeastern boundary of the disputed Kashmir region at about 15,000 feet (4,600
metres). A short way beyond Leh, in Ladakh (in the Indian-administered state of Jammu and
Kashmir), it is joined on its left by its first major tributary, the Zaskar River. Continuing for 150 miles
(240 km) in the same direction into the Pakistani-administered areas of the Kashmir region, the
Indus is joined by its notable tributary the Shyok River on the right bank. Below its confluence with
the Shyok, as far as the Kohistan region of Pakistan’s Khyber Pakhtunkhwa province, it is fed by
mighty glaciers on the slopes of the Karakoram Range, the Nanga Parbat massif, and the Kohistan
highlands. The Shyok, Shigar, Gilgit, and other streams carry glacial melt water into the Indus.
The Shigar River joins the Indus on the right bank near Skardu in Baltistan. Farther downstream
the Gilgit River is another right-bank tributary, joining it at Bunji. A short distance downstream
the Astor River, running off the eastern slope of Nanga Parbat, joins as a left-bank tributary. The
Indus then flows west and turns south and southwest to enter Khyber Pakhtunkhwa province, in the
process skirting around the northern and western sides of the Nanga Parbat massif (26,660 feet
[8,126 metres]) in gorges that reach depths of 15,000 to 17,000 feet (4,600 to 5,200 metres) and
widths of 12 to 16 miles (19 to 26 km). Trails cling grimly to precipitous slopes overlooking the river
from elevations of 4,000 to 5,000 feet (1,200 to 1,500 metres). The Indus receives its most-notable
tributaries from the eastern Punjab Plain. These five rivers—the Jhelum, Chenab, Ravi, Beas,
and Sutlej—give the name Punjab (“Five Rivers”) to the region divided between Pakistan and India.

4. The Amravati, the Bhavani, the Hemavati, the Kabini, etc. are the main tributaries of:
(a) The Krishna River (b) The Godavari River
(c) The Kaveri River (d) The Mahanadi River

Answer: C

Explanation: Kaveri River, Kaveri also spelled Cauvery, sacred river of southern India. It rises on
Brahmagiri Hill of the Western Ghats in southwestern Karnataka state, flows in a south-easterly
direction for 475 miles (765 km) through the states of Karnataka and Tamil Nadu, and descends the
Eastern Ghats in a series of great falls. Right Bank Tributaries are :- Hemavati , Shimsha , Arkaveri
, Chovrai , Turunananimuttai .Left bank Tributaries are :- Lakshmantirath , Kabini ,Suvarnevati ,
Bhavani , Noyil , Amravati .

5. Climate is an important geographical phenomenon which determines many things like


vegetation, fauna and comfort of living. Consider the following statements in this regard and
pick out the incorrect ones:
1. Weather refers to the sum total of weather conditions and variations over a large area for a long
period of time.
2. Climate refers to the state of the atmosphere over an area at any point of time.
(a) 1 only (b) 2 only (c) Both 1 and 2 (d) Neither 1 nor 2
www.achieveias.co.in, YouTube Channel: http://youtube.com/c/AchieveIAS Telegram Channel: http://t.me/Achieve_Ias,
Mail: achieveias21@gmail.com, Contact Number: 8968920720

Answer: C

Explanation: Weather, state of the atmosphere at a particular place during a short period of time.
It involves such atmospheric phenomena as temperature, humidity, precipitation (type and
amount), air pressure, wind, and cloud cover. Weather differs from climate in that the latter includes
the synthesis of weather conditions that have prevailed over a given area during a long time period—
generally 30 years. For a full discussion of the elements and origins of weather, see climate. For a
treatment of how conditions in space affect satellites and other technologies, see space weather.
Weather, as most commonly defined, occurs in the troposphere, the lowest region of the atmosphere
that extends from the Earth’s surface to 6–8 km (4–5 miles) at the poles and to about 17 km (11
miles) at the Equator. Weather is largely confined to the troposphere since this is where almost all
clouds occur and almost all precipitation develops. Phenomena occurring in higher regions of the
troposphere and above, such as jet streams and upper-air waves, significantly affect sea-level
atmospheric-pressure patterns—the so-called highs and lows—and thereby the weather conditions
at the terrestrial surface. Geographic features, most notably mountains and large bodies of water
(e.g., lakes and oceans), also affect weather. Recent research, for example, has revealed that
ocean-surface temperature anomalies are a potential cause of atmospheric temperature anomalies
in successive seasons and at distant locations. One manifestation of such weather-affecting
interactions between the ocean and the atmosphere is what scientists call the El Niño/Southern
Oscillation (ENSO). It is believed that ENSO is responsible not only for unusual weather events in
the equatorial Pacific region (e.g., the exceedingly severe drought in Australia and the torrential
rains in western South America in 1982–83) but also for those that periodically occur in the mid-
latitudes (as, for example, the record-high summer temperatures in western Europe and unusually
heavy spring rains in the central United States in 1982–83). The ENSO event of 1997–98 was
associated with winter temperatures well above average in much of the United States. The ENSO
phenomenon appears to influence mid-latitude weather conditions by modulating the position and
intensity of the polar-front jet stream.
Climate is defined as an area's long-term weather patterns. The simplest way to describe climate is
to look at average temperature and precipitation over time. Other useful elements for describing
climate include the type and the timing of precipitation, amount of sunshine, average wind speeds
and directions, number of days above freezing, weather extremes, and local geography.
The climate of any particular place is influenced by a host of interacting factors. These
include latitude, elevation, nearby water, ocean currents, topography, vegetation, and prevailing
winds. The global climate system and any changes that occur within it also influence local climate.
Consider how each factor illustrated by the thumbnail images might control climate at your location.

6. Consider the following statements about the characteristics of Indian Monsoon.


1. Monsoon season of India is associated with continuous rain fall without breaks.
2. Monsoon causes flood as well as drought.
3. Monsoon winds are pulsating and unstable in nature and direction.
4. Tropical cyclones originate in Andaman Sea and causes heavy rain in eastern coast of India post
retreat of monsoon.

Which of these is/are not correct?


(a) 1 only (b) 1 and 2 only (c) 4 only (d) 1 and 3 only
Answer: A
www.achieveias.co.in, YouTube Channel: http://youtube.com/c/AchieveIAS Telegram Channel: http://t.me/Achieve_Ias,
Mail: achieveias21@gmail.com, Contact Number: 8968920720

Explanation: Characteristics of the Indian Monsoon


1. These winds generally strike the Indian coast in the first week of June, but their arrival and
departure may be before time or even it may be delayed.
2. Monsoon is not steady winds. They are irregular in nature affected by different atmospheric
conditions i.e. due to regional climatic conditions.
3. Monsoons are not equally distributed. The spatial distribution of rainfall is uneven – some regions
may receive heavy rains while the others will have to be content with meagre or scanty rains.
4. When the monsoon arrives, it gives heavy rainfall which continues for several days. This is known
as ‘burst of monsoon’. This occurs mainly at Kerala coast where it reaches first.
5. Generally, these winds start retreating by the end of September. But, sometimes, their departure
may be delayed till October or they may retreat even much earlier.

MECHANISM OF MONSOON:
Monsoon refers to the seasonal reversal in the wind direction during a year. They flow from sea to
land during the summer and land to sea during winter.

1. Traditional theory: Monsoon is divided into summer monsoon and winter monsoon.
During summer, the interior parts of North Indian Plains covering Rajasthan, Punjab, Haryana, and
Western Uttar Pradesh are intensely hot. Such high temperature heats up the air of that region.
Hot air rises, the low-pressure area is created under it. This low pressure is also known as the
monsoonal trough. On the other hand, the temperature over the Indian Ocean is relatively low, as
water needs more time to get heated as compared to land. So a relatively high-pressure region is
created over the sea. Thus, there is a difference of temperature and resultant pressure over North
Central Indian Plains and the Indian Ocean. Due to this difference, air from the high-pressure region
of the sea starts moving towards the low-pressure region of North India. This gives birth to summer
Monsoon. Thus, by the middle of June, the general movement of air is from equatorial region of
Indian Ocean to the Indian subcontinent and the direction of these winds, in general, is from
southwest to northeast. It is also called South West (SW) monsoon. According to the traditional
belief, the monsoon is caused by the differential heating and cooling of land and sea.
According to modern scientists, the origin of the Indian monsoon is influenced by a number of
factors; important among them is the system of jet streams. Temperature distribution in May and
creation of low-pressure trough .In winter the sun shines vertically over the tropic of Capricorn.
The northern part of India is colder than the Arabian Sea and Bay of Bengal. High pressure is
developed over land than seas; hence the flow of wind is reversed. The winds flow from North East
to South West which is the winter monsoon.

2. Modern Theory of Monsoon: It is also called Air Mass theory. Trade winds meet at equator from
both hemispheres. The meeting place of both trade winds is known as Inter-Tropical Convergence
Zone (ITCZ).The apparent shift of sun also shifts the ITCZ along the direction of movement of the
sun. When in the summer season the sun is shining over the tropics of Cancer, the ITCZ shifts
northwards up to 20-25 N and it is located over Indo-Gangetic plains in India. When the sun is in the
southern hemisphere and shining over the tropics of Capricorn, the ITCZ shifts to 20-25 S and it is
located in the Indian Ocean region. In the summer the Southeast trade winds of the southern
hemisphere cross the equator and start flowing from south-west direction to northeast direction
under the influence of Coriolis force. These displaced trade winds are called south-west monsoons
when they blow over the Indian subcontinent. When the winds blow from North East to the south-
west in winter from land to sea, it is called North East monsoon in winter.
www.achieveias.co.in, YouTube Channel: http://youtube.com/c/AchieveIAS Telegram Channel: http://t.me/Achieve_Ias,
Mail: achieveias21@gmail.com, Contact Number: 8968920720

7. The Cold Weather Season begins from mid-November in Northern India and stays till
February. Consider the following statements about Cold Weather Season?
1. During this season, the Northeast Trade winds prevail over the country which blow from sea to
land and hence, for most part of the country, it is a dry season
2. A characteristic feature of the Cold Weather Season over the Northern Plains is the inflow of
cyclonic disturbances from the west and the northwest
3. The Peninsular region does not have a well-defined cold season

Select the correct answer from the code given below:


(a) 2 and 3 only (b) 1 and 3 only (c) 1 and 2 only (d) All the above

Answer: A

Explanation: The cold weather season begins with November in north India and by the beginning
of December the whole country comes under its grip. In this season the southerly branch of the jet
stream occupies position south of the Himalayas indicating that oil more the Northern Hemisphere
polar dynamics in command of the situation. The return of the westerly jet stream is accompanied
with the restriction of light north-east trade winds (monsoon) toll surface, withdrawal of the ITC,
formation of antic clinic cell over north-western India and dry wealth conditions prevailing over most
of the part of the country. Temperature-during winter season there is general increase of
temperature from north to south and the isotherms run almost parallel to the latitudes. The 21°
isotherm for the month of January runs east- west through the middle of the country connecting Tapi
estuary in the west and the Mahanadi delta in the east. In January the north-west India- Punjab,
Haryana, west Uttar Pradesh and north Rajasthan experience less than 15° C temperature while
the average temperature is less than 10°C over Jammu and Kashmir and northern U.P. South India
the isotherms tend to bend to the and run parallel to the coast.
The western cold warmer than the eastern coast by about 1.7° (J mean daily minimum temperature
varies front in the north-western part of India to 24°C Peninsula. The night temperature in the plains
Punjab and Haryana sometimes goes below freezing point producing ground frost conic such period
of unusually cold weather is gem described as ‘cold wave’. The Peninsular region of the country,
however, does not have well defined cold weather season.
The westerly jet stream plays key role steering these disturbances into India. Their averred
frequency is 4 to 5 depressions per month we greater intensity between December and February
Rainfall-According to C.W.B. Normand if weather, clear skies, low humidity and, thereof® absence
of rainfall, low temperature and a lair diurnal variation in it are the usual features of winey season in
India. However, slight rainfall. During winter season anticyclone conditions Ii are found over north-
western part of the country (from where winds move toward the oceanic low of the south. These
winds blow from north-west in north-west India, from west to east in the middle Ganga plain, from
north-east to south-west in the n Bay of Bengal, Arabian Sea and Peninsula. A charts act eristic
feature of the cold weather season is the s inflow of depression from the west and the northwest.
These low pressure systems, called the western disturbances, originate in West Asia and near”
Mediterranean Sea. They travel eastward across I and Pakistan and reach India during the wanly
season.

8. The latitude that pass through Sikkim does not pass through:
1. Punjab
2. Rajasthan
3. Uttrakhand
www.achieveias.co.in, YouTube Channel: http://youtube.com/c/AchieveIAS Telegram Channel: http://t.me/Achieve_Ias,
Mail: achieveias21@gmail.com, Contact Number: 8968920720

4. Delhi
5. Uttar Pradesh

Code:
a) 1, 3 and 4 only b) 1, 2 and 3 only c) 2, 4 and 5 only d) 1 and 4 only

Answer: A

9. Consider the following statements:


1. Retreating monsoon season is 3 month long.
2. The temperature falls during the retreating monsoon season.

Which of the following statements are correct:


1. Only 2. Only 3. Both 1 and 2 4. None of the above

Answer: C

Explanation: Retreating monsoon season is an important topic in geography in the UPSC civil
services exam. It comes under the climatology portions of UPSC geography syllabus.
Retreating monsoon season commences with the beginning of the withdrawal of the south-west
monsoon [mid-September – November] and lasts till early January. It is a 3-month long process
where it starts from the peninsula in October and from the extreme south-eastern tip by December.
The south-west monsoons withdraw from the Coromandel Coast in mid-December. In Punjab, the
south-west monsoons withdraw from there in the second week of September. During this period,
the temperature comes down sharply. The sky also becomes clear. The most severe and destructive
tropical cyclones are originated in the Indian seas and the Bay of Bengal during the retreating
monsoons. It is during the retreating monsoon season in India that the south eastern coast receives
a lot of rainfall; tropical cyclones also occur during this time. The state of Tamil Nadu receives almost
half of its annual rainfall during this time. This is called the winter monsoon or the northeast
monsoons.

10. Which of the following statement is/are correct?


1. Mahanadi river drainage basin is shared by Maharashtra, Chhattisgarh, Jharkhand and Orissa.
2. Godavari river drainage basin is largest among peninsular rivers.
3. There is no west flowing river in Madhya Pradesh

(a) 1 and 2 only (b) 1 and 3 only (c) 2 and 3 only (d) All the above

Answer: A

Explanation: The Mahanadi basin is the 8th largest basin in the country having total catchment
area of 139681.51 sq. km which is nearly 4.28% of the total geographical area of the country.
 It lies between east longitudes 80° 30' to 86° 50' and north latitudes 19° 21' to 23° 35'.
 The catchment area of the basin extends over major parts of Chhattisgarh and Odisha and
comparatively smaller portions of Jharkhand, Maharashtra and Madhya Pradesh.
 The Mahanadi basin is divided into 3 sub basins, namely upper, middle and lower Mahanadi.
 The upper basin is a saucer shaped and mostly lies in Chhattisgarh state.
www.achieveias.co.in, YouTube Channel: http://youtube.com/c/AchieveIAS Telegram Channel: http://t.me/Achieve_Ias,
Mail: achieveias21@gmail.com, Contact Number: 8968920720

 The basin is circular in shape with a diameter of about 400 km and an exit passage of about 160
km length and 60 km breadth.
 Physiographically, the basin can be divided into four regions, namely, the northern plateau, the
Eastern Ghats, the coastal plain and the erosional plains of central table land. The first two are
hilly regions. The coastal plain is the delta area which is highly fertile. The central table land is
the central interior region of the basin, traversed by the river and its tributaries.
 The basin receives about 90% of its rainfall during the monsoon season. Generally, the
southwest monsoon sets by the middle of June over the entire basin and remains active till the
end of September. The spatial variation in rainfall is moderate in the basin. The formation of
depressions in the Bay of Bengal cause cyclones, which bring about wide spread heavy rains
resulting in floods and destructions.
 The average annual runoff of Mahanadi is 66.9 Billion Cubic Meter. Thee basin receives uneven
rainfall thus drought is prevalent in some districts. Total fourteen districts in the basin are covered
under Drought Prone Area programme.
 Districts Jagatsinghpur, Puri and Kendrapara which are situated near coastal parts of Mahanadi
face water logging problem.
 The main soil types found in the basin are red and yellow soils. Mixed red and black soils occur
in parts of the Bolangir, Sambalpur, and Sundargarh districts of Orissa. Laterite soil is found in
the lower parts of Orissa. The deltaic soil is found in the coastal plains of the Mahanadi. Black
soil and sandy soil with "Kankar" are the main soils found in the part of basin lying in
Chhattisgarh.
 Except in the Chhatisgarh and coastal plains, the basin has an extensive area under forests.
Forest and agriculture are the main stay of the people in the interior parts of the basin. The
Chhatisgarh and coastal plains, with a high incidence of rainfall, are predominantly rice growing
areas.

Godavari River, sacred river of central and southeastern India. One of the longest rivers in India,
its total length is about 910 miles (1,465 km), and it has a drainage basin of some 121,000 square
miles (313,000 square km). The Godavari River rises in northwestern Maharashtra state in the
Western Ghats range, only about 50 miles (80 km) from the Arabian Sea, and flows for most of its
course generally eastward across the broad plateau of the Deccan (peninsular India).
After traversing central Maharashtra it enters northern Telangana state northwest of Nizamabad and
continues through a broad valley and, for a short stretch, forms the border with Madhya
Pradesh state to the north. The river then turns southeastward for the last 200 miles (320 km) of its
course, flowing through a gap in the Eastern Ghats ranges and then across Andhra Pradesh state
before reaching the Bay of Bengal. There it empties via its two mouths: the Gautami Godavari to
the north and the Vasishta Godavari to the south. From its source to the Eastern Ghats, the Godavari
River flows through gentle, somewhat monotonous terrain, along the way receiving the Darna,
Purna, Manjra, Pranhita, and Indravati rivers. Upon entering the Eastern Ghats region, however, the
river flows between steep and precipitous banks, its width contracting until it flows through a
deep cleft only 600 feet (180 metres) wide, known as the Gorge. On either side wooded hills rise
almost vertically from the waters. Having passed through the Eastern Ghats, the river widens again,
traversing wide lowland plains, the low islands in its stream being used to grow a variety of crops,
notably tobacco. At that point the Godavari flows placidly. Just below the city of Rajahmundry in
Andhra Pradesh, a dam was constructed on the river in the mid-19th century by the British
engineer Sir Arthur Thomas Cotton, the first major irrigation project on the Godavari. Since Indian
independence from Britain in 1947, some newer projects have been completed to provide irrigation
www.achieveias.co.in, YouTube Channel: http://youtube.com/c/AchieveIAS Telegram Channel: http://t.me/Achieve_Ias,
Mail: achieveias21@gmail.com, Contact Number: 8968920720

and hydroelectric power, including the Jayakwadi Dam in west-central Maharashtra, and other
projects have been planned.
The important peninsular rivers flowing towards the west are Shetrunji, Bhadra (Bhadar), Vaitarna,
Kalindi, Bedti, Sharavati, Bharsthpuzha, Periyar and Pamba. The rivers flowing towards the Arabian
Sea have short courses.
www.achieveias.co.in, YouTube Channel: http://youtube.com/c/AchieveIAS Telegram Channel: http://t.me/Achieve_Ias,
Mail: achieveias21@gmail.com, Contact Number: 8968920720

ACHIEVE IAS GEOGRAPHY MCQ SERIES, DAY 16, SOLUTIONS

1. Which of the statements is/are correct?


1. Tropical Deciduous Forests are the most widespread forest in India.
2. Pine, Silver fir, Spruce and Cedar are the varieties of coniferous trees found in temperate forest.

a. 1 only b. 2 only c. Both 1 and 2 d. Neither 1 nor 2

Answer: C

Explanation: Tropical deciduous forests occur in regions with heavy rainfall for part of the year
followed by a marked dry season. These forest formations are dense and lush during the wet
summers, but become a dry landscape during the dry winters when most trees shed their leaves.
Shedding their leaves allows trees to conserve water during dry periods. Bare trees open up the
canopy layer, enabling sunlight to reach ground level and facilitate the growth of thick underbrush.
Trees use underground water during the dry seasons. These dramatic changes in water availability
have a great impact on the plants and animals living in this ecosystem. Many trees in deciduous
forests lose their leaves during the drought period, though trees on moister sites with access to
groundwater tend to remain evergreen. Tropical deciduous forests occur in climates that are warm
year-round and may receive several hundred centimeters of rain per year. Though these forests
occur in climates that are warm year-round and may receive several hundred centimeters of rain
per year, they have long dry seasons which last several months and vary with geographic location.
Broadly speaking tropical deciduous forests occur in drier areas north and south of the tropical
rainforest belt, south or north of the subtropical deserts, generally in two bands: one between 10°
and 20°N latitude and the other between 10° and 20°S latitude. The most diverse dry forests in the
world occur in southern Mexico and in the Bolivian lowlands. The dry forests of the Pacific Coast of
north-western South America support a wealth of unique species due to their dry climate.
Deciduous forests are home to trees such as oak, birch, beech, aspen, elm and maple. Tropical and
subtropical forests also have teak trees, palm trees and bamboo. Plants found in
these forests include flowers, ferns, mosses and herbs.

2. Which of the given statements is/are correct?


1. The diurnal range of temperature is more in coastal areas than in Deserts.
2. Kerala gets rainfall from both south-west and north-east monsoon.

a. 1 only b. 2 only c. Both 1 and 2 d. Neither 1 nor 2

Answer: B

Explanation: Temperature lag is an important factor in diurnal temperature variation: peak daily
temperature generally occurs after noon, as air keeps net absorbing heat even after noon, and
similarly minimum daily temperature generally occurs substantially after midnight, indeed occurring
during early morning in the hour around dawn, since heat is lost all night long. The analogous annual
phenomenon is seasonal lag. High desert regions typically have the greatest diurnal-temperature
variations, while low-lying humid areas typically have the least. This explains why an area like
the Snake River Plain can have high temperatures of 38 °C (100 °F) during a summer day, and then
have lows of 5–10 °C (41–50 °F). At the same time, Washington D.C., which is much more humid,
has temperature variations of only 8 °C (14 °F); urban Hong Kong has a diurnal temperature range
www.achieveias.co.in, YouTube Channel: http://youtube.com/c/AchieveIAS Telegram Channel: http://t.me/Achieve_Ias,
Mail: achieveias21@gmail.com, Contact Number: 8968920720

of little more than 4 °C (7.2 °F). South West Monsoon is a major one which occurs from July -
September .Majority of Indian states receives the rainfall from this Monsoon. Rain hits first the state
of Kerala and along the coastal areas of Karnataka/Maharastra (Konkan coastal area). After it gains
momentum it crosses the Western Ghats downpours in all other states of India. North East Monsoon
mostly starts end of September. This monsoon wind get obstructed by Eastern Ghats and
downpours at eastern coastal area (Coromandal Cost) along Andhra Pradesh and Tamil Nadu. The
coastal area of Andhra Pradesh and majority of Tamil Nadu receives most of rainfall from this
Monsoon. It is Kerala which receives first rainfall. If Kerala receives first rainfall in June ( i guess) all
the farmers will be revealed that monsoon is On-time. The quantity of rainfall is something different
matter.

3. Which of the following is/are regions of Tropical forest?


1. Western Ghats
2. Andaman Nicobar Islands
3. West Bengal

Select the correct answer from the following codes


a. Only 3 b. Only 1 and 2 c. Only 2 and 3 d. 1, 2 and 3

Answer: B

Explanation: The Western Ghats and the Andaman Nicobar Islands have tropical rain forests, the
deltaic regions have tropical forests and mangroves; Himalayan heights are marked with temperate
vegetation.

Tropical Rain Forests


 As the name suggests, they are in areas that see heavy rainfall of 200 cm and above.
 The forests are thick and dense, preventing sunlight from reaching the ground.
 Many trees in this vegetation shed their leaves at different times of the year.
 They are also called Evergreen Forest because they appear green year round.
 Some trees found in this region are Ebony, Rosewood, Rubber etc.
 In India, parts that witness this vegetation are Andaman and Nicobar Island, some parts of North
East, a small strip of the western side in the Western Ghats.
 Animals found in Tropical Rain Forests are butterflies, spiders, snakes, frogs, sloths etc.

Mangrove Forests
 Mangrove forests are identified with tangled roots. Sometimes, their roots are under water.
 They can live in areas with less oxygen and can survive in saline water.
 Examples of trees in this region are Nipa Palm, Loop Root etc.
 In India, they are found in Sunderban, West Bengal, and in Andaman and Nicobar Islands.
 Animals found in this region are turtles, snakes, crocodiles etc.

4. Consider the following statements:


1. Natural vegetation depends on variation in climate alone.
2. Tropical evergreen forest are found in eastern slopes of Western Ghats.

Select the correct answer from the following codes


a. Only 1 b. Only 2 c. Both 1 and 2 d. Neither 1 nor 2
www.achieveias.co.in, YouTube Channel: http://youtube.com/c/AchieveIAS Telegram Channel: http://t.me/Achieve_Ias,
Mail: achieveias21@gmail.com, Contact Number: 8968920720

Answer: D

Explanation: Tropical evergreen forest are found in the western slope of the Western Ghats, hills
of the north-eastern region and the Andaman and Nicobar Islands. Depending upon the variations
in the climate and the soil, the vegetation of India changes from one region to another.

5. Which of the following is/are features of tropical evergreen forest?


1. Annual precipitation of over 200 cm
2. No definite time for trees to flowering
3. Absence of stratification

Select the correct answer from the following codes


a. Only 1 b. Only 1 and 2 c. Only 2 and 3 d. 1, 2 and 3

Answer: B

Explanation: Tropical evergreen forest are found in warm and humid areas with an annual
precipitation of over 200 cm and mean annual temperature above 22oC. Tropical evergreen forests
are well stratified, with layers closer to the ground and are covered with shrubs and creepers, with
short structured trees followed by tall variety of trees. In these forests, trees reach great heights up
to 60 m or above. There is no definite time for trees to shed their leaves, flowering and fruition. As
such these forests appear green all the year round.

6. Consider the following statements


1. British were first to clear Indian forests for cultivation.
2. British cleared forest mainly for cultivation of Indigo.

Select the correct answer from the following codes


a. Only 1 b. Only 2 c. Both 1 and 2 d. Neither 1 nor 2

Answer: D

Explanation: Indigenous tribes of India had followed practice of slash and burn/ Jhumm cultivation.
The British were aware of the economic value of the forests in India, hence, large scale exploitation
of these forests was started. The structure of forests was also changed. Forests were also cleared
for introducing plantations of tea, rubber and coffee.

7. Consider the following statements


1. Moist deciduous forest in India are found in North eastern states along foot hills of Himalayas.
2. Teak is one of the main species of moist deciduous forest in India.

Select the correct answer from the following codes


a. Only 1 b. Only 2 c. Both 1 and 2 d. Neither 1 nor 2

Answer: C
www.achieveias.co.in, YouTube Channel: http://youtube.com/c/AchieveIAS Telegram Channel: http://t.me/Achieve_Ias,
Mail: achieveias21@gmail.com, Contact Number: 8968920720

Explanation: The Moist deciduous forests are more pronounced in the regions which record rainfall
between 100 and 200 cm. These forests are found in the north-eastern states along the foothills of
Himalayas, eastern slopes of the Western Ghats and Orissa. Teak, sal, shisham, hurra, mahua,
amla, semul, kusum, and sandalwood etc. are the main species of these forests.

8. With reference to dry deciduous forest, which of the following is/are true?
1. Dry deciduous forest transcend into semi evergreen forest in wetter margins and on dryer margins
it transcends into Tropical thorny forest.
2. It covers areas with rainfall between 70 and 100 cm.
3. In India it is found in plains of Uttar Pradesh, Bihar and drier areas of peninsular India

Select the correct answer from the following codes


a. Only 1 b. Only 1 and 2 c. Only 2 and 3 d. 1, 2 and 3

Answer: C

Explanation: Dry deciduous forest covers vast areas of the country, where rainfall ranges
between 70 -100 cm. On the wetter margins, it has a transition to the moist deciduous, while on
the drier margins to thorn forests. These forests are found in rainier areas of the Peninsula and the
plains of Uttar Pradesh and Bihar.

9. Which of the following terms is used for plants which are at present part of natural
vegetation but have originally come to India from abroad?
a. Indigenous plants b. Endemic species
c. Endangered plants d. Exotic plants

Answer: D

Explanation: Native” means the organism evolved in the area where it is residing. But of course,
some areas are bigger than others. Giant sequoia trees and Tule elk are native to California, but not
to Claremont. Even within smaller areas like Claremont, the local conditions dictate what is “native”.
Buckwheat, sagebrush, and quail are native to the coastal sage scrub in the Bernard Field Station;
mulefat and rushes were native to formerly wetter areas such as where Pilgrim Place is now. A
native species can cover a very wide area or can be very restricted in its habitat—some insect
species may only occupy a few acres. Saying a species is “endemic” means it is restricted to a
particular area; bristlecone pines are endemic to high altitudes in the western US. “Exotic” plants
and animals are ones that haven’t evolved in an area. Hybrid tea roses, saguaro cactus, daffodils,
corn, tomatoes, those pesky fox squirrels, and our dogs and cats (and us) are all exotics in
Claremont. The coast redwoods you see around town are native to California but exotic to
Claremont. Many exotic species do just fine when introduced to areas that have conditions similar
to those in which they evolved or if the conditions they need are provided, and many don’t create
any serious problems in their new location. “Naturalized” species are exotic species that are well-
adapted to a new habitat, which means that they treat the new area as their own and will grow and
reproduce there without any outside help. Sweet alyssum, black mustard, tree tobacco, and Norway
rats all will flourish locally without any help from us. In our urban gardens that get some degree of
extra water, fertilizer, or shade, many other exotic plants will seed themselves around. In my garden,
honesty, heavenly bamboo, violets, and fortnight lily appear in random spots but are easily removed
where not wanted. Many species such as European honeybees were introduced intentionally and
www.achieveias.co.in, YouTube Channel: http://youtube.com/c/AchieveIAS Telegram Channel: http://t.me/Achieve_Ias,
Mail: achieveias21@gmail.com, Contact Number: 8968920720

are so well-adapted to a variety of habitats that they have spread throughout the country. “Invasive”
species, however, are another kettle of fish (so to speak). Without the natural checks and balances
that evolved with them, some exotic species naturalize so well that they start crowding out native
species and altering the local ecology. Argentine ants and zebra mussels came in accidentally, but
Kudzu in the south and tamarisk in the desert were both introduced intentionally.

10. Which one of the following animals lives in the swampy and marshy lands of Assam
and West Bengal?
a. Tiger b. Lion c. One-horned Rhino d. Elephants

Answer: C
www.achieveias.co.in, YouTube Channel: http://youtube.com/c/AchieveIAS Telegram Channel: http://t.me/Achieve_Ias,
Mail: achieveias21@gmail.com, Contact Number: 8968920720

ACHIEVE IAS GEOGRAPHY MCQ SERIES, DAY 17, SOLUTIONS

1. What is the definition of sex ratio?


(a) Number of deaths per 1,000 live births of children
(b) Number of women per 1000 men
(c) Number of women per 100000 men
(d) Number of men per 1000 women

Answer: B

Explanation: The sex ratio means the number of females per 1000 males. According to the 2011
census, India's sex ratio is 943 females per 1,000 males. The highest sex ratio in the whole India is
in Kerala.

2. What does infant mortality mean?


(a) The number of deaths per 1,000 live births of children under 1 year of age
(b) Death of children under the age of 5 years
(c) Death of children before the age of 6 months
(d) Death of children under the age of 3 years

Answer: B

Explanation: Infant Mortality Rate (IMR) is the number of deaths per 1,000 live births of children
under age of one year. The present level of IMR (33 infant deaths per thousand live births, for the
year 2017) is about one-fourth as compared to 1971 (129 infant deaths per thousand live
births). In the last ten years, IMR has witnessed a decline of about 36.7% in rural areas and about
36% in urban areas.

3. What is the Maternal Mortality Rate?


(a) Women's death occurred before 9th month of pregnancy
(b) The annual number of female deaths per 100,000 live births due to pregnancy.
(c) The death of women occurred within 2 year of childbirth
(d) None of the above

Answer: B

Explanation: The Maternal Mortality Rate (MMR) is the annual number of female deaths per
100,000 live births from any cause related to or aggravated by pregnancy or its management
(excluding accidental or incidental causes).For its calculation, the number of deaths due to infant
birth is divided by the total number of infant births. The Maternal Mortality Ratio (MMR) in
India has declined from 167 in 2011-2013 to 130 in 2014-2016, according to a special bulletin
issued by the Health Ministry today. MMR is defined as the proportion of maternal deaths per
1,00,000 live births.

4. According to Census 2011, how much was the Total Fertility Rate (TFR) in India?
(a) 2.1 (b) 2.3 (c) 2.4 (d) 2.0
www.achieveias.co.in, YouTube Channel: http://youtube.com/c/AchieveIAS Telegram Channel: http://t.me/Achieve_Ias,
Mail: achieveias21@gmail.com, Contact Number: 8968920720

Answer: C

Explanation: According to Census 2011, the Total Fertility Rate (TFR) in India was 2.4 which has
come down to 2.3 in the year 2016. The aim of the Government of India is to reduce it to 2.1.

5. Which states of India have the lowest and highest Infant Mortality Rate respectively?
(a) Kerala, Bihar (b) Odisha, Jharkhand
(c) Goa, Madhya Pradesh (d) Maharashtra, Uttar Pradesh

Answer: C

Explanation: The state with the lowest Infant Mortality Rate in India is Goa, where the Infant
Mortality Rate is only 8 and Madhya Pradesh has highest Infant Mortality Rate i.e.47/1000

6. Which are the three main factors that cause population change?
(a) Births, Deaths and marriage (b) Births, Deaths and migration
(c) Births, Deaths and life expectancy (d) Births. Deaths and Sex ratio

Answer: B

Explanation: The three main factors that cause population change: birth, migration, and death. Let's
talk about each of these in detail:
Birth Rate: Now, this might seem very obvious to you, but when babies are born, it results in a
population change! Social scientists who study this trend have some key terms, though. First,
the fertility rate in a country refers simply to the number of children born. This is different from
the fecundity rate, which is the number of children who could be born in a given society. In other
words, fecundity is the biological capacity of humans to produce children.
For a woman, this is generally the time frame between her early teens and her mid-forties. For men,
this window is longer, but factors like sperm count diminish as a man ages. The fecundity rate is
greater than the fertility rate in most cases. Birth rates vary by society for a number of reasons,
including personal choice and social and political factors. Factors such as poverty level, access to
contraception, maternal and infant care, and availability of nutrition all impact the birth rate in a
society.
Death Rate:
Just as when babies are born, when members of a society die it changes the population. This might
be a bit morbid to think about, but scientists refer to this as the mortality rate, which is simply the
number of deaths that occur in a society. This is usually calculated by looking at the number of
deaths per 1,000 individuals. In the United States, for example, the death rate is 8/1000 people. Just
like birth rate, the death rate in a country depends on things such as availability of quality medical
care, disease, war, or famine.
Migration Rate:
One important factor that accounts for a change in population is migration. At the broadest level,
migration refers to movement; it's the flow or circulation of people in societies. There are two kinds
of migration: emigration and immigration. Emigration means to leave one's own country for
another. Immigration means to enter into a country other than one's own. Both of these processes
impact population change.

7. What is the unit of population density measurement?


www.achieveias.co.in, YouTube Channel: http://youtube.com/c/AchieveIAS Telegram Channel: http://t.me/Achieve_Ias,
Mail: achieveias21@gmail.com, Contact Number: 8968920720

(a) Family / meter (b) Person/ sq. km (c) Family / cm (d) Person / Hector

Answer: B

Explanation: Population density is measured in person per km. It is measured that how many
people live in one km.
India's current yearly growth rate is 1.02%. The country as a whole has a population density of
416 people per square kilometer, which ranks 31st in the world. In Mumbai, the population
density is 21,000 people per square kilometer (54,000/square mile).

8. When was first census held in India?


(a) 1870 (b) 1871 (c) 1872 (d) 1874

Answer: C

9. Which state in India has the highest literacy rate?


(a) Tamil Nadu (b) Kerala (c .Mizoram (d) Lakshadweep

Answer: B

Explanation: Literacy rate of population is defined as the percentage of literates to the total
population age 7 years and above. Literacy rate is the number of persons who is 7 or above, who
has the ability to read, write and understand in any language. India's literacy rate is at 74.04%.
Kerala has achieved a literacy rate of 93.91%. Bihar is the least literate state in India, with
a literacy of 63.82%.

10. What is the correct descending order on the basis of the number of sex ratio in the states?
(a) Kerala <Chhattisgarh <Mizoram <Andhra Pradesh
(b) Arunachal Pradesh <Kerala <Manipur <Meghalaya
(c) Meghalaya <Kerala << Chhattisgarh <Tamilnadu
(d) Kerala <Tamil Nadu <Andhra Pradesh <Chhattisgarh

Answer: D

Explanation: Kerala <Tamil Nadu <Andhra Pradesh <Chhattisgarh. Sex ratio in Indian states is
given below;
Sex ratio in Kerala: 1084
Sex ratio in Tamil Nadu: 995
Sex ratio in Andhra Pradesh: 992
Sex ratio in Chhattisgarh: 991
www.achieveias.co.in, YouTube Channel: http://youtube.com/c/AchieveIAS Telegram Channel: http://t.me/Achieve_Ias,
Mail: achieveias21@gmail.com, Contact Number: 8968920720

ACHIEVE IAS GEOGRAPHY MCQ SERIES, DAY 18, SOLUTIONS

1. The diverse flora and fauna of the planet are under great threat mainly due to:
1. Global Warming
2. Lack of water availability
3. Insensitivity to our environment
4. Increasing pollution

Select correct answer:


a. Only 1 and 2 b. Only 1, 2 and 3 c. Only 1, 3 and 4 d. All the above

Answer: C

Explanation: India is a land of varied flora and fauna. It is one of the 12 mega biodiversity
hotspots of the world. Two of India's great mountain ranges, the Eastern Himalayas and the
Western Ghats, have been designated among the world's 18 biodiversity hotspots. But, in the last
few decades, we have seen a steady increase in the extinction rate of flora and fauna all over the
world including India. The conservation of biodiversity is of paramount importance to our survival.
We have about 20 species of plants and 410 species of animals, birds and fishes on the verge of
extinction. Many factors are responsible for the depletion of flora and fauna — population
explosion, hunting, forest fires, expansion of agriculture, extensive use of fossil fuels, destruction
of natural habitats, climatic changes, pollution, natural disasters, deforestation and many more.
Among the flora Sariba, Peritha, Llex Khasiana and Malabar Mahagony and among the fauna the
Asiatic Lion, Asiatic Black Bear, Bengal Tiger, Great Indian Rhinoceros, Flamingo, Lion-tailed
Macaque, Asian Elephant, Ganges River Dolphins (also called ‘Susu') are some of the
endangered species.

2. Consider the following pairs


1. Dampa Tiger Reserve : Mizoram
2. Gumti Wildlife Sanctuary : Sikkim
3. Saramati Peak : Nagaland

Which of the above pairs is /are correctly matched?


a. 1 only b. 2 and 3 only c. 1 and 3 only d. 1, 2 and 3

Answer: C

Explanation: The Dampa Tiger Reserve occupies an area of 500 sq. km. and lies in west
Mizoram in north-eastern India, along the border between India and Bangladesh. The hills and
forests in this 'Land of the highlanders' are considered by biologists to be "biogeographic
highways" connecting India to Malayan and Chinese regions. The rich, luxuriant rainforests of
Dampa harbour several species such as the swamp deer, tiger, leopard, elephant and hoolock
gibbon. The Dampa Tiger Reserve is probably one of the last few safe havens for the tiger and a
host of other animal and bird species. Low altitude galliformes are visible and the three species
of Chloropsis found in India are endemic to these fabulous forests.
Gumti Wildlife Sanctuary:It is a sanctuary in South Tripura district located in south east corner of
the state. It has an area of 389.54 square kilometers. Adjoining to the sanctuary there is a vast
water reservoir / sparkling water body covering approximately 300 square kilometers. The lake not
www.achieveias.co.in, YouTube Channel: http://youtube.com/c/AchieveIAS Telegram Channel: http://t.me/Achieve_Ias,
Mail: achieveias21@gmail.com, Contact Number: 8968920720

only presents a breathtaking spectacle that accentuates the virgin natural beauty of the
surroundings, it is also haunted by several species of domestic as well as migratory aquatic birds
that seek refuge in the waters during the chilly European winters. The sanctuary is also home to a
vast and diversified flora and fauna. Reptiles are also found aplenty in the reserve. The water
reservoir attracts many resident and migratory water birds. It has elephants, Bison, Sambhar,
barking Deer, Wild Goat or Sarow apart from many other animals and reptiles. The sanctuary can
be approached via Ambassa and Gandachara or via Ambasa – Jatanbari.
Saramati Peak: Standing at an altitude of 3841 m, Saramati is the highest peak in the state
of Nagaland and occupies a place of pride in the heart of its residents. This peak is located on the
Nagaland-Myanmar border and the closest village to the base of this peak is Thanamir in Kiphire
district of Nagaland, close to 35 km from the village of Pungro. The trek to the peak is a two-day
process and starts from the village of Thanamir, where a guide is provided along with the
registration fee. There is a regular Nagaland state transport bus from Dimapur to Kiphire which
runs once in 24 hours and is a long overnight journey of close to 12-14 hours. From Kiphire,
shared cabs are available to Pungro which is another 50 km away. State guest houses and circuit
houses are available for accommodation in these areas with a lot of military presence in and
around since the peak lies on the border between Nagaland and Myanmar (formerly Burma). The
peak remains snow-capped throughout the winter and gradually melts and flows into the Likimro
River.

3. Consider the following:


1. Black-necked crane
2. Cheetah
3. Flying squirrel
4. Snow leopard

Which of the above are naturally found in India?


(a) 1, 2 and 3 only (b) 1, 3 and 4 only (c) 2 and 4 only (d) 1, 2, 3 and 4

Answer: B

Explanation: Black-necked crane: Sacred to Tibetan Buddhists, the black-necked crane faces a
variety of threats in its only wintering retreat in India. The black-necked crane is a resident of Inner
Asia—the regions of Xinjiang, Tibet (and the Chinese provinces carved out of it, namely Qinghai
and Gansu) as well the Chinese Proper provinces of Sichuan, Yunnan and Guizhou. It also occurs
in Jammu and Kashmir (of which it is the state bird) and Arunachal Pradesh as well as a few sites
in Bhutan. Habitat loss and degradation related to climate change, changes in agriculture
practices, pollution and environmental contamination. Cheetah: Cheetah, (Acinonyx jubatus), one
of the world’s most-recognizable cats, known especially for its speed. Cheetahs’ sprints have been
measured at a maximum of 114 km (71 miles) per hour, and they routinely reach velocities of 80–
100 km per hour while pursuing prey. Nearly all the cheetahs remaining in the wild live in Africa.
Adult cheetahs’ weight averages between 75 and 125 pounds. They can measure from 40 to 60
inches in length, measured from the head to the hind quarters. The tail can add a further 24 to 32
inches bringing the total overall length up to 7.5 feet. On average, cheetahs stand 28 to 36 inches
tall at the shoulder.
Flying squirrel: Flying squirrel, (subfamily Pteromyinae), any of 43 species of gliding squirrels.
Two species are North American, two live in northern Eurasia, and all others are found in
the temperate and tropical forests of India and Asia. Although these rodents do not fly, glides of up
www.achieveias.co.in, YouTube Channel: http://youtube.com/c/AchieveIAS Telegram Channel: http://t.me/Achieve_Ias,
Mail: achieveias21@gmail.com, Contact Number: 8968920720

to 450 metres (almost 1,500 feet) have been recorded for Oriental giant flying squirrels
(Petaurista). Ample, loose skin and underlying muscle typically form a fur-covered membrane
between each forelimb and hind limb; some species have smaller membranes between the head
and wrists and between the hind limbs and tail. A cartilaginous rod that extends from the wrist
supports the front part of each membrane alongside the body. The squirrels are most active
between dusk and dawn. Omnivorous, it eats nuts, seeds, berries, insects, tree buds and
sometimes eggs or nestlings. Although the northern and southern flying squirrels are mainly found
in trees, they forage the forest ground for food. They run slowly and clumsily on the ground and, if
startled far from a tree, will try to hide. The northern species lives in mixed or coniferous forests,
while its southern counterpart occupies hardwood forests of oak, maple, beech and hickory. The
southern flying squirrel is found throughout eastern parts of North America and the south-eastern
parts of Canada. It is found mostly in the Carolinian deciduous forests of southern Ontario, but can
also be seen north to Muskoka and the Ottawa Valley, as well as in parts of Quebec and Nova
Scotia. Its range partly overlaps that of the northern flying squirrel, which can generally be found
from the U.S.-Canada border north to the tree line. The two species, however, do not interbreed.
The snow leopard (Panthera uncia), also known as the ounce, is a large cat native to the mountain
ranges of Central and South Asia. It is listed as Vulnerable on the IUCN Red List because the
global population is estimated to number less than 10,000 mature individuals and is expected to
decline about 10% by 2040. It is threatened by poaching and habitat destruction following
infrastructural developments. It inhabits alpine and subalpine zones at elevations from 3,000 to
4,500 m (9,800 to 14,800 ft), ranging from eastern Afghanistan to Mongolia and western China. In
the northern range countries, it also occurs at lower elevations.

4. Which one of the following groups of animals belongs to the category of endangered
species?
(a) Great Indian Bustard, Musk Deer, Red Panda and Asiatic Wild Ass
(b) Kashmir Stag, Cheetal, Blue Bull and Great Indian Bustard
(c) Snow Leopard, Swamp Deer, Rhesus Monkey and Saras (Crane)
(d) Lion-tailed Macaque, Blue Bull, Hanuman Langur and Cheetal

Answer: A

Explanation: An endangered species is an animal or plant that's considered at risk of extinction. A


species can be listed as endangered at the state, federal, and international level. On the federal
level, the endangered species list is managed under the Endangered Species Act. The
Endangered Species Act (ESA) was enacted by Congress in 1973. Under the ESA, the federal
government has the responsibility to protect endangered species (species that are likely to
become extinct throughout all or a large portion of their range), threatened species (species that
are likely to become endangered in the near future), and critical habitat (areas vital to the survival
of endangered or threatened species). The Endangered Species Act has lists of protected plant
and animal species both nationally and worldwide. When a species is given ESA protection, it is
said to be a "listed" species. Many additional species are evaluated for possible protection under
the ESA, and they are called “candidate” species.

5. Which of the following statement is correct about the Biosphere Reserve?


A. Multipurpose protected areas to preserve genetic diversity in representative ecosystem.
B. A reserves area meant for preserving its natural vegetation, wildlife and natural beauty.
C. A reserved area meant for preservation and development of endangered species.
www.achieveias.co.in, YouTube Channel: http://youtube.com/c/AchieveIAS Telegram Channel: http://t.me/Achieve_Ias,
Mail: achieveias21@gmail.com, Contact Number: 8968920720

D. All of the above

(a) 1, 2 and 3 only (b) 1, 3 and (c) 2 and 1 (d) All the above
Answer: D

Explanation: Biosphere Reserve is a notified area which covers a larger area of land which may
cover multiple National Parks, Sanctuaries and reserves as well. It is an area meant for
conservation of biodiversity of a specific area. Biosphere reserves are sites established by
countries and recognized under UNESCO's Man and the Biosphere (MAB) Programme to promote
sustainable development based on local community efforts and sound science. The programme of
Biosphere Reserve was initiated by UNESCO in 1971. The purpose of the formation of the
biosphere reserve is to conserve in situ all forms of life, along with its support system, in its totality,
so that it could serve as a referral system for monitoring and evaluating changes in natural
ecosystems. The first biosphere reserve of the world was established in 1979, since then the
network of biosphere reserves has increased to 631 in 119 countries across the world. In 1968,
UNESCO organized the Biosphere Conference, which saw the beginning of the concept of a
“Biosphere Reserve”. This was the first intergovernmental conference examining how to reconcile
conservation and use of natural resources, thus foreshadowing the present-day notion of
sustainable development. This conference resulted in the launching of the UNESCO "Man and the
Biosphere" (MAB) Programme in 1970.One of the original MAB projects consisted in establishing
a coordinated World Network of sites representing the main ecosystems of the planet in which
genetic resources would be protected, and where research on ecosystems as well as monitoring
and training work could be carried out. These sites were named as "Biosphere Reserves”.
The Man and Biosphere Program recognizes areas that:
1. Are typical of the world’s major terrestrial or coastal ecosystems;
2. Demonstrate innovative approaches to living and working in harmony with nature; and
3. Demonstrate how to achieve a sustainable balance between conserving natural ecosystems
and biodiversity and fostering sound economic development.

6. Which of the following statement is correct about the Sanctuary?


A. Multipurpose protected areas to preserve genetic diversity in representative ecosystem.
B. A reserves area meant for preserving its natural vegetation, wildlife and natural beauty.
C. A reserved area meant for preservation and development of endangered species.
D. All of the above

Which of the above statement correctly matched?


a. 1 only b. 2 and 3 only c. 3 only d. 1, 2 and 3

Answer: C

Explanation: Sanctuary is natural area which is reserve by a governmental or private agency for
the protection of particular species. A sanctuary is a protected area which is reserved for the
conservation of only animal and human activities like harvesting of timber, collecting minor forest
products and private ownership rights are allowed as long as they do not interfere with well-being
of animals.

7. When was the Indian Board for Wildlife constituted?


A. 1952 B. 1950 C. 1949 D. 1948
www.achieveias.co.in, YouTube Channel: http://youtube.com/c/AchieveIAS Telegram Channel: http://t.me/Achieve_Ias,
Mail: achieveias21@gmail.com, Contact Number: 8968920720

Answer: A
Explanation: The Indian Board for Wildlife was constituted in 1952. The main purpose of the
board was to advise the Government on the means of conservation and protection of wildlife,
construction of national parks, sanctuaries and zoological gardens as well as promoting public
awareness regarding conservation of wildlife.

8. Project Tiger was started in:-


(a) 1953 (b) 1963 (c) 1973 (d) 1983

Answer: C
Explanation: As the number of tigers is decreasing day by day, we need to take preventive
measures to save them from getting extinct. Numerous efforts are being carried out to save their
species, and Project Tiger is an important movement aimed at the conservation of tiger in India.
The habitat required by the Tigers should be made proper, and any cutting of trees in that area
should be avoided. Being the national animal of India, it is our duty that we safeguard the wildlife
properly. Many projects taken by India has led to a decrease in the depletion of tigers. Many
conservation areas were created to make sure that no human could enter the area and do any
harm to the tiger or its habitat. Project Tiger was first initiated in the year April 1, 1973, and is still
going on. This project was started to save tigers. The much-needed project was launched in Jim
Corbett National Park, Uttrakhand under the leadership of Indira Gandhi. The objectives of the
Project Tiger was clear- saving Royal Bengal Tigers from getting extinct. The major cause of their
depletion is humans, and so all the conservation areas are made human free. They made sure
that the place that tigers lived in was also safe and secure. Project Tiger has been successful in
increasing the population of the tigers. The number has increased from 1200 to around 5000. The
project team has been doing its work with full dedication, and all national parks are putting efforts
in doing the project. There are around fifty national parks and sanctuaries that are involved in this
project. Jim Corbett, Bandipur, Ranthambore, Nagarhole, Nazgira, Dudhwa, Gir, Kanha,
Sunderbans, Bandhavgarh, Manas, Panna, Melghat, Palamau, Similipal, Periyar, Sariska, Buxa,
Indravati, Namdapha, Mundanthurai, Valmiki, Pench, Tadoba Andhari, Dampa, Bhadra, Pench
(Maharashtra), Pakke, Nameri, Satpura, Anamalai, Udanti- Sitanadi, Satkosia, Kaziranga,
Achanakmar, Dandeli Anshi, Sanjay- Dubri, Mudumalai, Nagarhole (Karnataka), Parambikulam,
Sahyadri, Bilgiri, Kawal, Sathyamangalam, Mukandra, Srisailam, Amrabad, Pilibhit, Bor, Rajaji,
Orang and Kamlang are the national parks involved in the Project Tiger. Recent additions to this
project are are: Ratapani Tiger Reserve(Madhya Pradesh), Sunabeda Tiger Reserve (Odisha),
and Guru Ghasidas (Chhattisgarh).There were many obstacles in the project like poaching and the
Forest Rights Act, but all were handled well by the government, and the project is running in full
speed. Any successful project has to bear a lot of pressure and has to face many challenges.
Making Project Tiger into a successful piece of work, there are efforts and dedication of various
government officials. During ancient times, it was difficult to take off the lands that were used for
hunting. Many didn’t like it and raised objections. But the project happened nonetheless. Another
major challenge was poaching. Many individuals use to sell tiger bones and skin to international
markets. This was a major business for them and earned good money. After all the initiatives
taken by the project, they couldn’t stop illegal trading of animal skin. Individuals used to break the
law and sell them to international buyers. This triggered the depletion of tigers. The government
officials made strict law and grounded the problem. During the building of sanctuaries and
reserves, the human population living over there faced the problem and therefore raised their
voice against it. They passed out a Forest Rights Act in which they stated their difficulty. They
www.achieveias.co.in, YouTube Channel: http://youtube.com/c/AchieveIAS Telegram Channel: http://t.me/Achieve_Ias,
Mail: achieveias21@gmail.com, Contact Number: 8968920720

wanted to have space for them as well and did not want to move from their original area. In some
national parks, humans still reside on the outskirts of the park. They have come in peace with the
Project Tiger and understood its importance. Though some individuals are not very sure about the
decision, the project is happening in full speed.

9. What is the name given to the species which are not found after searches of known or
likely areas where they may occur:-
(a) Endemic species (b) Rare species (c) Endangered species (d) Extinct species

Answer: D

Explanation: Extinction of a particular animal or plant species occurs when there are no more
individuals of that species alive anywhere in the world - the species has died out. This is a natural
part of evolution. But sometimes extinctions happen at a much faster rate than usual. For
example, at the end of the Cretaceous period 65 million years ago, a mass extinction caused the
death of many different types of animals and plants, including the dinosaurs. Dinosaur. Today
human intervention is also causing rapid extinction. Hunting, habitat destruction and the over
exploitation of wildlife means that many different types of plants and animals are being pushed to
the edge of extinction. It is this type of extinction, caused by humans.

10. The percentage of forest cover in India is:-


(a) 14.69% (b) 15.39% (c) 19.39% (d) 25%

Answer: D

Explanation:
 The India State of Forest Report 2019 released recently shows an increase of 5,188 square
kilometres of forest and tree cover across the country compared to the ISFR 2017.
 However, the report highlights that northeast India continues to lose forests when compared
to ISFR 2017 and previous reports.
 The forest report also reveals that the forest area under the category “recorded forest area”
(land notified as forest by the government) in tribal districts, which are home to about 60
percent of India’s forests, is decreasing as well.
 The report for the first time did an assessment of biodiversity for all states and union
territories and found that Arunachal Pradesh has the maximum richness of species in terms
of trees, shrubs and herbs followed by Tamil Nadu and Karnataka.
Nearly 25 percent (one fourth) of India’s total land area is now under forest and tree cover.
However, there is still a long way to go – more than a decade, admits the government – before
India reaches its target of having 33 percent of its total area under forest and tree cover. The
latest ‘India State of Forest Report (ISFR 2019) released by the country’s environment minister
Prakash Javadekar on December 30, 2019, revealed that the total forest and tree cover of the
country is 807,276 square kilometres (which is 24.56 percent of the geographical area of the
country) compared to 802,088 sq km (24.39 percent) in ISFR 2017. The report marked an
increase of 5,188 sq. km. of forest and tree cover combined, at the national level, as compared to
the previous assessment. When the last assessment, ISFR 2017, was released, an increase of
8,021 sq. km. was recorded compared to the data in ISFR 2015. While the overall forest and tree
cover marked an increase on a national level, the report highlighted a decrease in the forest area
in the country’s northeast region. This decline in forest area in the northeast has been an ongoing
www.achieveias.co.in, YouTube Channel: http://youtube.com/c/AchieveIAS Telegram Channel: http://t.me/Achieve_Ias,
Mail: achieveias21@gmail.com, Contact Number: 8968920720

trend with the region witnessing a loss of about 3,199 sq. km. of forest area since 2009.ISFR 2019
is a biennial report published by the Forest Survey of India (FSI) and is the 16th such report
published. The calculations are largely based on satellite data. It includes information on forest
cover, tree cover, mangrove cover, growing stock inside and outside the forest areas, carbon
stock in India’s forests, forest types and biodiversity, forest fire monitoring and forest cover in
different slopes and altitudes.
www.achieveias.co.in, YouTube Channel: http://youtube.com/c/AchieveIAS Telegram Channel: http://t.me/Achieve_Ias,
Mail: achieveias21@gmail.com, Contact Number: 8968920720

ACHIEVE IAS GEOGRAPHY MCQ SERIES, DAY 19, SOLUTIONS

1. What is the importance of rainwater harvesting?


(a) To reduce the consumption of groundwater, many people around the world are using rainwater
harvesting systems.
(b) Process or technique of collecting, filtering, storing and using rainwater for irrigation and for
various other purposes.
(c) Both I and II
(d) Neither I nor I

Answer: A

Explanation: Rainwater harvesting is the storing of rainwater during the monsoon season for the
purpose of using it during periods of water scarcity. Generally speaking, it is a process used for
collecting and storing rainwater for human use. Rainwater harvesting is best described as the
technique by which rain water is accumulated and stored with the intention of reusing it during the
dry season or when there is a drought. With rapid climatic changes, increase in global temperature
and population growth, there is a scarcity of potable water in many countries across the world. The
gradual falling of water levels, are a cause of serious concern not only because it leads to shortage
of usable water but also because in coastal areas it causes imbalance in salinity of the area.
1. Rainwater harvesting or the collection of rainwater in a proper way, can be a permanent solution
to the problem of water crisis in different parts of the world. This simple method can put forward a
solution which will be workable in areas where there is sufficient rain but the groundwater supply is
not sufficient on the one hand and on the other surface water resource is insufficient. This is
particularly applicable in hilly areas where it can be utilized for human consumption, by animals and
also for farming. In remote areas, where surface pollution is comparatively low, rainwater harvesting
is ideal.
2. Although the earth is three-fourths water; very little of it is suitable for human consumption or
agriculture. Rainfall is unpredictable and there is a constant shortage of water in countries which are
agriculture dependent or generally drought prone.
3. A bad monsoon means low crop yield and shortage of food. Even animals suffer from scarcity of
water. Africa and the Indian subcontinent face acute water crisis during the summer months. The
farmers are the most affected because they do not get sufficient water for their fields. Rainwater
harvesting therefore is an ideal solution for farmers who depend on monsoon for consistent water
supply.
4. Unavailability of clean water compels the consumption of polluted water, giving rise to water-
borne diseases and high rate of infant mortality. In recent studies it has been observed that in Lima
(Peru) nearly 2 million people do not have access to any water supply and those who do have
access get water supply which has a high possibility of being contaminated.
It has been reported that the water crisis in some parts of Honduras is so severe that the municipal
corporation of those areas cannot supply enough water even to those residents who have municipal
water supply connection. This has been reported by Anna Kajumuto Tibaijuka, Under-secretary
General, UNED UN-Habitat.
5. If rain water, which comes for free, can be collected and stored, instead of letting it run off, it could
be an alternative to back up the main water supply especially during dry spells. Its importance will
not be limited to an individual family but can be used by a community as well.
www.achieveias.co.in, YouTube Channel: http://youtube.com/c/AchieveIAS Telegram Channel: http://t.me/Achieve_Ias,
Mail: achieveias21@gmail.com, Contact Number: 8968920720

2. In which part or state of India people build diversion channels like ‘guls’ or ‘kuls’
1. Arid region
2. Western Himalayas
3. Plains of West Bengal
4. Semi-arid region

Select correct answer:


a. 1 only b. 2 only c. 1 and3 only d. all the above

Answer: B

Explanation: Kuls and Guls are channels built by people in the hilly areas-
1. They are made and used for the purpose of irrigation.
2. These are diversion channels.
3. They are mostly used in the state of Himachal Pradesh.
4. They serve a very important purpose and special care should be taken while construction and
maintenance.

3. How can we increase the water efficient irrigation system?


1. By pouring unnecessary water to the land
2. By not supplying adequate water
3. By applying drip irrigation method
4. By using more manures

Select correct answer:


a. 1 only b. 1 and 2 only c. 3 only d. all the above

Answer: C

Explanation: Drip irrigation is a method of controlled irrigation in which water is slowly delivered to
the root system of multiple plants. Drip irrigation increases the water efficient irrigation system and
hence it is very helpful in water conservation.

4. What are the two main types of drip irrigation?


a) Surface and Subsurface drip irrigation
b) Under and Subsurface drip irrigation
c) Top and Surface drip irrigation
d) Top and Under drip irrigation

Select correct answer:


a. 1 only b. 1 and 2 only c. 3 only d. all the above

Answer: A

Explanation: Drip irrigation is the practice of applying small amounts of water and fertilizer
uniformly across a specific area. The water and fertilizer are delivered directly to the crop root zone,
eliminating runoff, evaporation, and drift. A properly designed and managed drip irrigation system
gives producers the best uniformity and application efficiency available, consequently saving them
www.achieveias.co.in, YouTube Channel: http://youtube.com/c/AchieveIAS Telegram Channel: http://t.me/Achieve_Ias,
Mail: achieveias21@gmail.com, Contact Number: 8968920720

time, energy, and water, all while maximizing yields. There are two main types of drip irrigation
systems – surface and sub-surface.
A surface drip irrigation system uses close emitter spacing (12”-18”) and a thin wall (8-10 mil)
dripline injected 1” to 6” below the surface. These systems are often referred to as “temporary”
because the dripline is retrieved and recycled yearly. The submains can be permanent or
temporary. These systems are typically used on high value crops due to the yearly expense of new
dripline and the labor for installation.
A subsurface drip irrigation (SDI) system uses 20” – 27” emitter spacing and a slightly thicker wall
(13-15mil) dripline injected 8” – 14” below the surface. These systems are permanent, making
design and installation critical to ensure longevity. SDI systems are mainly used in row crop
agriculture but are making their way into some high value crops. Eco-Drip has been committed to
SDI since installing our first system in 1980.

5. The total volume of the world’s water is estimated to exist as ocean:


(a) 75.5% (b) 85.5% (c) 96.5% (d) 65.5%

Answer: C

Explanation: The percentage of the total volume of world's water is estimated to exist as oceans is
96.5%.

6. On which river has Nagarjun Sager Dam been constructed?


(a) River Coyana (b) River Krishna (c) River Godavari (d) River Tapi

Answer: B

Explanation: Nagarjuna Sagar Dam was built on River Krishnna. The construction commenced in
1955 and took 12 years for its completion. This place is at a distance of about 160 Kms from
Hyderabad. Nagarjuna Sagar is located in Nalgonda district of Telangana, and one of the most
prominent Buddhist centres in India. This is one of the most popular tourist places to visit near
Hyderabad and one of the best weekend getaways from Hyderabad city.
The Nagarjuna Sagar dam is 490 feet tall from its foundation and 1.6 kms long with 26 flood gates
which are 42 feet distance apart. The dam provides irrigation water to mainly Guntur, Krishna,
Khammam, and Nalgonda districts along with hydro electricity generation

7. The rank of India in terms of water availability per person p.a in the world is:
(a) 130th (b) 131st (c) 132nd (d) 133rd

Answer: D

Explanation: The per capita water availability in the country is reducing due to the increase in
population. According to 2011 census, the average annual per capita availability of water in the
country is 1545 cubic meters.

8. Narmada Bachao Andolan is related to:


(a) Tehri (b) Bhakra Nangal (c) Sardar Sarovar (d) Rihand

Answer: C
www.achieveias.co.in, YouTube Channel: http://youtube.com/c/AchieveIAS Telegram Channel: http://t.me/Achieve_Ias,
Mail: achieveias21@gmail.com, Contact Number: 8968920720

Explanation: Narmada Bachao Andolan is the most powerful mass movement, started in 1985,
against the construction of huge dam on the Narmada river. Narmada is the India's largest west
flowing river, which supports a large variety of people with distinguished culture and tradition ranging
from the indigenous (tribal) people inhabited in the jungles here to the large number of rural
population. The proposed Sardar Sarovar Dam and Narmada Sagar will displace more than 250,000
people. The big fight is over the resettlement or the rehabilitation of these people. The two proposals
are already under construction, supported by US$550 million loan by the World Bank. There are
plans to build over 3000 big and small dams along the river. Narmada Bachao Andolan It is a multi
crore project that will generate a big revenue for the government. The Narmada Valley Development
plan is the the most promised and most challenging plan in the history of India. The proponents are
of the view that it will produce 1450 MW of electricity and pure drinking water to 40 million people
covering thousands of villages and towns. Some of the dams have been already been completed
such as Tawa and Bargi Dams. But the opponents says that this hydro project will devastate human
lives and bio diversity by destroying thousands of acres of forests and agricultural land. On the other
hand it will overall deprive thousands of people of their livelihood. They believe that the water and
energy could be provided to the people through alternative technological means, which would be
ecologically beneficial. Led by one of the prominent leader Medha Patkar, it has now been turned
into the International protest, gaining support from NGO'S all around the globe. Protestors are
agitating the issue through the mass media, hunger strikes, massive marches, rallies and the
through the on screen of several documentary films. Although they have been protesting peacefully,
but they been harassed, arrested and beaten up by the police several times. The Narmada Bachao
Andolan has been pressurizing the World Bank to withdraw its loan from the project through media.
The strong protests throughout the country not only made impact on the local people but has also
influenced the several famous celebrities like film star Aamir Khan, who has made open efforts to
support Narmada Bachao Andolan. He said he only want that those who have been rendered
homeless should be given a roof. He pleaded to the common people to take part in the moment and
come up with the best possible solutions.

9. The following is the only state where roof top rain water harvesting is made compulsory:
(a) Bihar
(b) Assam
(c) Tamil Nadu
(d) Maharashtra

Select correct answer:


a. 1 only b. 1 and 2 only c. 3 only d. All the above

Answer: C

10. Which one of the following is the benefit of rainwater harvesting?


(a) Flood Mitigation
(b) Provide a lot of water to play
(c) Create good aesthetic view
(d) Decrease the ground water level

a. 1 only b. 1 and 2 only c. 3 only d. all the above


www.achieveias.co.in, YouTube Channel: http://youtube.com/c/AchieveIAS Telegram Channel: http://t.me/Achieve_Ias,
Mail: achieveias21@gmail.com, Contact Number: 8968920720

Answer: A

Explanation: Rainwater harvesting helps for flood mitigation. Appropriate designed recharges in
open public spaces will help to keep the roads from flooding. When water is not allowed to leave the
premises there is less chance for choking up of the roads.
The following are 5 advantages of harvesting rainwater.
1. Reduces Flooding and Erosion Harvesting rainwater can help the environment in a number of
ways. For starters, it can reduce erosion around downspouts and in gardens. It can also control
storm water run-off. Rainwater doesn’t produce scale and corrosion as hard water does. The
collection of rainwater may reduce flooding in certain areas as well.
2. Reduces Water Bills Rainwater harvesting will not only help individuals save on their water bills
but can cut costs for entire communities. The cost to supply mains and overall water services can
be substantially reduced when many people in one community use rainwater. Having a source of
water can also reduce dependence on municipal sources in case the water becomes
contaminated. Rainwater can be used as the primary source of water or as a backup source when
needed.
3. Reduces Demand on Ground Water Sources of ground water are increasingly being strained in
many areas throughout the world. Digging deeper wells is not only expensive but can cause
environmental damage such as collapsing the soil where the water used to be. It only makes
sense to use sources of rainwater whenever possible. Harvested rainwater can be stored and then
used during times of drought and when the ground water supplies have been depleted.
4. Can Be Used for Non-drinking Purposes. The majority of the water we need is used for non-
drinking. Everything from washing clothes and dishes to bathing and flushing toilets require large
amounts of water. Rainwater can be used for all of these things. Rainwater is soft and can lessen
the need for detergents when washing clothes and dishes. Rainwater can also be used for
washing vehicles, bathing pets, and nearly all cleaning that uses water.
5. Can Improve Plant Growth Rainwater harvesting can also be used to improve plants and
gardens. Using harvested water can flush the salt build up from plants and soil. Harvested
rainwater is generally free from several types of pollutants and man-made contaminants. Rain is
also free from chlorination. Using water that is this clean and healthy for plants and trees can save
money on overall property maintenance and landscaping needs.
While regular maintenance is required, simple collection systems can be constructed that most
people can easily build and maintain. Rainwater harvesting and storage can be incorporated in both
rural and urban areas and provides many benefits to individuals, communities, and the environment.
www.achieveias.co.in, YouTube Channel: http://youtube.com/c/AchieveIAS Telegram Channel: http://t.me/Achieve_Ias,
Mail: achieveias21@gmail.com, Contact Number: 8968920720

ACHIEVE IAS GEOGRAPHY MCQ SERIES, DAY 20, SOLUTIONS

1. Which of these describes a system of agriculture where a single crop is grown on a large
scale?
1. Shifting agriculture
2. Plantation agriculture
3. Horticulture
4. Intensive agriculture

Select correct answer


a. Only 1 b. Only 1, 2 c. Only 2 d. Only 2, 3 and 4

Answer: C

Explanation: Plantation agriculture is a form of commercial farming where crops are grown for profit.
Large land areas are needed for this type of agriculture. Countries that have plantation agriculture
usually experience tropical climate with high annual temperatures and receive high annual rainfall.
Plantation agriculture has its roots in the past. In the early 18th and 19th centuries, a number of
plantations were set up by westerners and western companies. They employed either local or
foreign workers who were willing to work for a small wage. For example, the rubber plantations set
up by the British in Malaysia employed many workers from India. Today, most plantations are owned
by the local governments or big companies.
PURPOSE: Plantation agriculture is a form of commercial farming where crops are grown for sale.
Some crops are sold as raw materials to manufacturing industries or partially processed and value
added before selling the products to large companies for further processing.
INPUTS: Land: Plantations are huge and can extend from a few hectares to a few thousand
hectares. For example, in Malaysia, an oil palm plantation is usually at least 40 hectares or more in
size.
Capital: A large amount of capital is put into building roads, buying machinery and building factories
to process the crops harvested from the plantations. Plantation owners also invest large amounts of
their capital on fertilizers and pesticides. Fertilizers are applied to plantation crops as frequently as
these crops use up nutrients from the land quickly. Since plantations usually grow one type of crop
(mono-cropping), pest attacks can cause total destruction of the plantations. Pesticides are therefore
used in huge quantities to prevent crops from being entirely destroyed by pests.
Labor: Due to the large size of a plantation, a lot of labor is needed to tend to the crops and work
in the nearby processing factories. Most of them are permanent laborers housed near the
plantations and taken care of by the plantation owner. For example in Malaysia’s large rubber
plantations, many workers are hired to tap latex from rubber trees, and in India for plucking tea
leaves in tea plantations or coffee berries in coffee plantations
Crops: Tree crops: rubber, coconut, arecanut, oil palm, teak, avocado;
Perennial shrubs: coffee, tea, bananas, sugar cane, cocoa, cashew nut, almond, pistachia,
macadamia
Perennial wines: grapes, vanilla, black pepper, betel vine, etc.
Spices: cardamom, pepper, and other spices
Annual crops: sugar beets, cassava, sweet potato, potato, pepper, and tobacco.
PRODUCE: the total output of a plantation is usually high. However, as a plantation covers a wide
area of land, its output per unit are is usually low.Usually, only one type of crop is grown in a
www.achieveias.co.in, YouTube Channel: http://youtube.com/c/AchieveIAS Telegram Channel: http://t.me/Achieve_Ias,
Mail: achieveias21@gmail.com, Contact Number: 8968920720

plantation - monocropping. Intercropping (e.g. banana or cocoa in coconut plantations) and crop
rotations are practiced wherever possible to minimize pest and disease incidence.

2. Jhumming refers to:


1. Primitive subsitence farming in Brazil.
2. 'Slash and burn' agriculture in North eastern states of India
3. Primitive farming in Malaysia.
4. Commercial farming in Punjab.

Select correct answer


a. Only 1 b. Only 1, 2 c. Only 2 d. Only 2, 3 and 4

Answer: C

Explanation: Slash and burn farming is a form of shifting agriculture where the natural vegetation
is cut down and burned as a method of clearing the land for cultivation, and then, when the plot
becomes infertile, the farmer moves to a new fresh plat and does the same again. This process is
repeated over and over. Shifting Cultivation is known as Ladang in Indonesia, Caingin in Philippines,
Milpa in Central America & Mexico, Ray in Vietnam, Taungya In Myanmar, Tamrai in Thailand,
Chena in Sri Lanka, Conuco in Venezuela, Roca in Brazil, Masole in central Africa.

3. Which one of the following is announced by the government in support of a crop?


1. Maximum support price
2. Minimum support price
3. Moderate support price
4. Influential support price

Select correct answer


a. Only 1 b. Only 1, 2 c. Only 2 d. Only 2, 3 and 4

Answer: C

Explanation: Minimum Support Price (MSP) is a form of market intervention by the Government of
India to insure agricultural producers against any sharp fall in farm prices. The minimum support
prices are announced by the Government of India at the beginning of the sowing season for certain
crops on the basis of the recommendations of the Commission for Agricultural Costs and Prices
(CACP). MSP is price fixed by Government of India to protect the producer - farmers - against
excessive fall in price during bumper production years. The minimum support prices are a guarantee
price for their produce from the Government. The major objectives are to support the farmers from
distress sales and to procure food grains for public distribution. In case the market price for the
commodity falls below the announced minimum price due to bumper production and glut in the
market, government agencies purchase the entire quantity offered by the farmers at the announced
minimum price. The Price Support Policy of the Government is directed at providing insurance to
agricultural producers against any sharp fall in farm prices. The minimum guaranteed prices are
fixed to set a floor below which market prices cannot fall. Till the mid-1970s, Government announced
two types of administered prices:
 Minimum Support Prices (MSP)
 Procurement Prices
www.achieveias.co.in, YouTube Channel: http://youtube.com/c/AchieveIAS Telegram Channel: http://t.me/Achieve_Ias,
Mail: achieveias21@gmail.com, Contact Number: 8968920720

The MSPs served as the floor prices and were fixed by the Government in the nature of a long-term
guarantee for investment decisions of producers, with the assurance that prices of their commodities
would not be allowed to fall below the level fixed by the Government, even in the case of a bumper
crop. Procurement prices were the prices of kharif and rabi cereals at which the grain was to be
domestically procured by public agencies (like the FCI) for release through PDS. It was announced
soon after harvest began. Normally procurement price was lower than the open market price and
higher than the MSP. This policy of two official prices being announced continued with some
variation upto 1973-74, in the case of paddy. In the case of wheat it was discontinued in 1969 and
then revived in 1974-75 for one year only. Since there were too many demands for stepping up the
MSP, in 1975-76, the present system was evolved in which only one set of prices was announced
for paddy (and other kharif crops) and wheat being procured for buffer stock operations.

4. Bhoodan-Gramdan movement was initiated by:


A. Mahatma Gandhi B. Jawaharlal Nehru
C. Vinobha Bhave D. Bal Gangadhar Tilak

Answer: C

Explanation: The Bhoodan-Gramdan movement initiated inspired by Vinoba brought Vinoba to


the international scene. In 1951, the Third Annual Sarvodaya Conference was held at
Shivarampali, a village a few miles south of the city of Hyderabad in South India. Vinoba was
persuaded to leave his community center (Ashram) at Pavnar, near Nagpur & attend the meetings.
Vinoba decided to walk three hundred miles to Hyderabad. Telangana had been the scene of
violent communist rebellion which was still smouldering in April 1951. For Vinoba the future of
India was essentially a contest between the fundamental creeds of Gandhi & Marx. In coming to
Hyderabad, Vinoba & other Gandhians were confronting a challenge & testing their faith in non-
violence. On April 11th 1951, the final day of conference, Vinoba announced that on his walk
home to Pavanar he & a few companions would tour the Communist infested areas of Telangana
to spread the message of Peace i.e. Non-violence. Once in Telangana, Vinoba quickly showed his
sensitivity to the new situation. On April 17th, at his second stop, Vinoba learned at first hand that
village people were afraid of the police as well as the Communists & that the village was torn
along class-lines. On April 18th 1951, the historic day of the very genesis of the Bhoodan
movement, Vinoba entered Nalgonda district, the centre of Communist activity. The organizers
had arranged Vinoba’s stay at Pochampalli, a large village with about 700 families, of whom two-
thirds were landless. Pochampalli gave Vinoba a warm welcome. Vinoba went to visit the Harijan
(the Untouchables) colony. By early afternoon villagers began to gather around Vinoba at Vinoba’s
cottage. The Harijans asked for eighty acres of land, forty wet, forty dry for forty families that would
be enough. Then Vinoba asked,”If it is not possible to get land from the government, is there not
something villagers themselves could do?” To everyone’s surprise, Ram Chandra Reddy, the local
landlord, got up & said in a rather excited voice: “I will give you 100 acres for these people.” At his
evening prayer meeting, Ram Chandra Reddy got up & repeated his promise to offer 100 acres of
land to the Harijans. This incident neither planned nor imagined was the very genesis of the
Bhoodan movement & it made Vinoba think that therein lay the potentiality of solving the land
problem of India. This movement later on developed into a village gift or Gramdan movement. This
movement was a part of a comprehensive movement for the establishment of a Sarvodaya
Society (The Rise of All socio-economic-political order), both in India & outside India.
www.achieveias.co.in, YouTube Channel: http://youtube.com/c/AchieveIAS Telegram Channel: http://t.me/Achieve_Ias,
Mail: achieveias21@gmail.com, Contact Number: 8968920720

The movement passed through several stages in regard to both momentum & allied programmes.
In October 1951, Vinoba was led to demand fifty million acres of land for the landless from the whole
of India by 1957. Thus a personal initiative assumed the form of a mass movement, reminding the
people of Gandhi’s mass movements. This was indeed a very remarkable achievement for a
constructive work movement. The enthusiasm for the movement lasted till 1957 & thereafter it began
to wane. Meanwhile the Bhoodan Movement had been transformed from a land-gift movement to a
village-gift or Gramdan movement, in which the whole or a major part of a village land was to be
donated by not less than 75% of the villagers who were required to relinquish their right of owner-
ship over their lands in favour of the entire village, with power to equitably redistribute the total land
among village’s families with a proviso for revision after some intervals. The Programme of individual
land-gifts was still there, but henceforth became a neglected activity. The Gramdan idea did not
prove popular in the non-tribal areas & this partly accounted for the decline of the movement at the
end of the 1950s. All this continued till 1974. from the view-point of its ups & downs. But there was
another aspect as well & it related to allied programmes unfolded from time to time. Those
progammes were Sampattidan (Wealth-gift), Shramdan (Labour-gift), Jeevandan (Life-long
commitment to the movement by co-workers), Shanti-Sena (Peace-army), Sadhandan (gift of
implements for agricultural operations). As regards attitudinal transformation, the propagation of
ideas combined with the above material achievements, could not but affect the mind of the thinking
people. The movement directly influenced the life-style of the co-workers, especially the life-long co-
workers & through them many workers & associates or fellow-seekers. By adopting Gandhi’s ideas
to the solution of the basic economic problem of land collection & equitable redistribution among the
landless, the Movement kept Gandhi’s ideas of socioeconomic reconstruction alive at a period when
the tendency of the educated elite was to overlook, if not to reject Gandhi’s ideas as irrelevant. The
Movement kindled interest in the individuals to study Gandhi’s ideas & to assess their relevance.
Jayaprakash Narayan, a renowned Marxist, and a Socialist, & one of the fore-most leaders in
politics, before & after India’s Independence, came to be more & more intimately associated with
the movement & realized that it was a superb endeavor to bring about revolution in human relations
founded on on the Gandhian philosophy of non-violence. Ultimately Jayaprakash devoted his entire
life to the construction of a Sarvodaya society. The Movement spontaneously attracted the attention
of many fellow-seekers & thinkers from outside India. Louis Fischer, the famous American
correspondent said: "Gramdan is the most creative thought coming from the East in recent times”.
Hallam Tennyson, the grandson of the famous English poet, Alfred Tennyson, wrote a book, “The
Saint on the march”. He narrated his memorable experiences as he moved with Vinoba into rural
India. Chester Bowles, the American ambassador to India, observed in his book, ”The dimensions
of peace”: We experienced in 1955, the Bhoodan Movement is giving the message of Renaissance
in India. It offers a revolutionary alternative to communism, as it is founded on human dignity”. The
British Industrialist, Earnest Barder was deeply impressed by the Bhoodan movement &
implemented the Gandhian concept of Trusteeship by alloting 90% share in the company to his
industrial workers. The British quaker, Donald Groom, trekked with Bhoodan Sarvodaya co-workers
for six months in the central India covering a distance of 1400 miles. The American friend Rev.
Kaithan turned himself into a Sarvodaya co-worker & established a community centre in South India.
David Graham, an English journalist of Sunday Standard, included Vinoba as one of the creative
rebels. Arthur Koestler, in 1959 wrote in London Observer, that the Bhoodan Movement presented
an Indian alternative to the Nehruvian model of Western development. To conclude taking an overall
view it cannot be gainsaid that the Bhoodan-Gramdan Movement, despite all its real & apparent
limitations, it would ever be deemed as a glorious attempt for a peaceful & non-violent solution of
the basic land problem of Indian society & through it for a non-violent reconstruction of the
Sarvodaya socio-economic-politico order of universal relevance & significance.
www.achieveias.co.in, YouTube Channel: http://youtube.com/c/AchieveIAS Telegram Channel: http://t.me/Achieve_Ias,
Mail: achieveias21@gmail.com, Contact Number: 8968920720

5. Which of the following is the commercial crop in India?


1. Mustard
2. Tobacco
3. Jute
4. All of the above

Select correct answer


a. 1 Only b. 1 and 2 Only c. 3 Only d. 1 and 3 Only

Answer: D

Explanation: This type of farming is what contributes to the country’s economy with
huge volumes of yield. In fact, the crops grown commercially in India are used as an export item
across the world. In this farming method, the Indian farmer uses a high amount of fertilizers,
pesticides, and insecticides to enhance and maintain the growth of the crops. Depending on the
crop best suited to the respective weather and soil, commercial farming in India varies across
different regions. For example, Haryana, Punjab and West Bengal grow rice commercially, while it
is a subsistence crop in Orissa. Major crops grown commercially in India are wheat, pulses, millets,
maize and other grains, vegetables, and fruits. Another method of commercial farming is ‘plantation’.
Plantation farming is a blend of agriculture and industry, practiced across a vast area of land. It is a
labor-intensive farming method that also uses the latest technological support for sustaining,
cultivating and yielding. The produce yielded from plantations is treated as raw materials to be
subsequently used in their respective industries. Crops grown: Some of the significantly grown
crops in plantation farming are tea, coffee, rubber, sugarcane, banana, coconuts, etc.

6. Which of the following is not matched correctly?


A. Rabi Crop Mustard, Cucumber
B. Rabi Crop Mustard, Barley
C. Zaid Crop Moong, vegetables
D. Kharif Crop Cotton

Answer: A

Explanation: The agriculture in India is an important topic in Indian geography. 49% of the
population in India is dependent on agriculture. In the total geographical area in India, 141 million
hectares is the net sown area while 195 million hectare is the gross cropped area. Agriculture in
India contributes to 14% of the GDP and the distribution of income and wealth. It provides essential
amenities like food for the people and fodder for the animals. It also provides the major source of
raw materials to the agro-based industries in India.The vast relief of the country, varied climate and
soil conditions cause to the provision of a variety of crops. All tropical, subtropical and temperate
crops are grown in India but predominantly food crops are cultivated in 2/3rd of the total cropped
area. There are three chief cropping seasons in India namely Kharif, Rabi and Zaid. The Kharif
season spreads from the month of July to the month of October and the Rabi season is from October
to the month of March. The crops cultivated between March and June are called Zaid. In this article,
you can read details about the different seasons. Rice and wheat form the staple food in this country.
Good monsoons, especially the southwest monsoons are essential to sow and harvest crops in
www.achieveias.co.in, YouTube Channel: http://youtube.com/c/AchieveIAS Telegram Channel: http://t.me/Achieve_Ias,
Mail: achieveias21@gmail.com, Contact Number: 8968920720

India. In order to ensure the availability of food for the country’s growing population, it is imperative
that we have a reasonably good Rabi and Kharif season.

Types of crops in India


1. KHARIF SEASON IN INDIA (kharif means autumn in Arabic)
July – October
Harvest – September to October
A.k.a Monsoon Crops
Such crops require a lot of water
Example: rice, sorghum, maize, Tea, rubber, coffee, guar, Sesame, cereals such as Arhar Dhal,
pearl millet, soybeans, cotton, oilseeds, etc.
2. RABI CROPS IN INDIA
Sowing between October and November
Harvest – February to April
A.k.a Winter Season Crops
Need cold weather for growth
Need less water
Example: wheat, oats, barley, pulses, cereals, oilseeds, linseed, etc.
3. ZAID SEASON
Sowing between March and June (between Kharif and Rabi)
Requires warm & dry weather for growth and a longer day-length for flowering
Example: Seasonal fruits and vegetable

7. Which agency is responsible for procurement, distribution and storage of food grain
production in India?
A. Ministry of Agriculture
B. Food Corporation of India
C. NAFED
D. TRIFED

Select correct answer


a. Only 1 b. Only 1, 2 c. Only 2 d. Only 2, 3 and 4

Answer: C

Explanation: "Food Corporation of India” is sensible for “distribution and storage of the food grain
in India”. It was structured under the Food Corporation Act of 1964 with the following objectives:
1. Guarantee minimal prices for agricultural products.
2. Adhere to the minimal food stock levels to protect food safety (Buffer stock)
3. Effectively and judicial distribution of food across the country, especially in the lower section of
people.
4. Increase price stability and protect the interest of the farmers.

8. Which of the above statement (s) is/are correct about climatic conditions for Growing
Wheat?
I. Wheat is sown in mid-October-mid-November and harvested in March.
II. Wheat is grows well in cool, moist climate and ripens in a warm, dry climate.
www.achieveias.co.in, YouTube Channel: http://youtube.com/c/AchieveIAS Telegram Channel: http://t.me/Achieve_Ias,
Mail: achieveias21@gmail.com, Contact Number: 8968920720

A. 1 Only B. 2 Only C. Both 1 and 2 D. Neither 1 nor 2

Answer: C

Explanation: Wheat is a Rabi crop. It is sown in mid-October-mid-November and harvested in


March. It grows well in cool, moist climate and ripens in a warm, dry climate. The cool winters and
the hot summers are conducive to a good crop. A cloudless sky having bright sunshine during
ripening and harvesting periods will make better quality wheat. Winter rainfall is ideal.

9. Under which plan did the Government introduce an agricultural strategy which gave rise
to Green Revolution?
A. First five year plan B. Second five year plan
C. Third five year plan D. Fourth five year plan

a. Only 1 b. Only 1, 2 c. Only 3 d. Only 4

Answer: C
Explanation: The Third 5 year Plan focused on agriculture & enhancement in the production of
wheat, but the succinct Sino - Indian War in the year 1962 exposed flaws in the economy & shifted
the spotlight towards the Defence industry. On May 31 & June 1, 1961, the National Development
Council considered the Draft Report on the Third Plan and generally approved it The Third 5 Year
Plan symbolizes the 1st phase in a system of enduring expansion extending over next 15 years or
so. In the course of this period, India's economy must not only enlarge swiftly but must, in unison,
become independent and self - generating. This' long-standing approach is projected to offer a wide-
ranging design of expansion for the country's natural resources, industrial advance and agricultural
changes in the social structure & an incorporated scheme of regional & national development. The
Third 5 Year Plan intended to give a more specific content to the social objectives of Constitution &
symbolizes a large progress towards their realization. It takes report of the victories and failures in
the 1st two Plans & sets the tasks to be accomplished in the viewpoint of expansion over the next
15 years & more.
Approach to the 3rd five year plan: • To secure a boost in national income of over 5 % per annum,
the prototype of investment being intended also to uphold this rate of enlargement during
subsequent Plans
• To accomplish self-support in food grains & enhance agricultural production to convene the
requirements of exports and industry
• To enlarge basic industries like chemicals industries, steel, power and fuel and institute machine
building capability, so that the requirements of supplementary industrialization can be met within a
period of 10 years or so largely from the country's own resources
• To utilize to the fullest probable extent the manpower resources & to guarantee a considerable
expansion in services opportunities

10. What is the correct descending order of food grain producing states?
A. Uttar Pradesh, Punjab, Madhya Pradesh And West Bengal
B. Punjab, Uttar Pradesh, Madhya Pradesh And West Bengal
C. Uttar Pradesh, Punjab, West Bengal And Madhya Pradesh
D. Uttar Pradesh, Madhya Pradesh, Punjab, And West Bengal

Answer: B
www.achieveias.co.in, YouTube Channel: http://youtube.com/c/AchieveIAS Telegram Channel: http://t.me/Achieve_Ias,
Mail: achieveias21@gmail.com, Contact Number: 8968920720

ACHIEVE IAS GEOGRAPHY MCQ SERIES, DAY 21, SOLUTIONS

1. Consider the following statements regarding distribution of mineral and energy


resources in India:
1. Coal deposits are mostly associated with Gondwana system.
2. Dharwar and Cuddapah systems contain resources of major metallic minerals like copper, lead,
zinc etc.
3. Major non-metallic minerals like limestone,
dolomite, gypsum, calcium, sulphate etc are found in and upper Vindhyan system.

Which of the following statement(s) is/are correct?


a. 1 Only b. 1 and 2 Only c. 2 and 3 Only d. All of the above

Answer: D

Explanation: The distribution of mineral and energy resources is uneven. It’s because
occurrence of mineral resources are associated with certain types of geological
formation. Coal deposits are mostly associated with Gondwana system, Dharwar and
Cuddapah systems contain resources of major metallic minerals
like copper, lead, zinc etc and major non-metallic minerals like limestone,
dolomite, gypsum, calcium, sulphate etc are found in Cuddapah and upper
Vindhyan system.

2. With reference to the coal reserves in India which of the following statements is
incorrect?
a. Coal is used as raw material in chemical and fertiliser industries and in the production of
thousands of items of daily use.
b. Coals are mainly found in the Gondwana and Tertiary coal field.
c. The states of Jharkhand, West Bengal, Chhattisgarh, Andhra Pradesh and Orissa are the
leading producers of coal.
d. The bulk of lignite reserves are found in and around Kolar in Karnataka.

Answer: D

Explanation: Coal is used as raw material in chemical and fertiliser industries and in the
production of thousands of items of daily use. Coals are mainly found in the Gondwana and
Tertiary coal field. The states of Jharkhand, West Bengal, Chhattisgarh, Andhra Pradesh and
Orissa are the leading producers of coal.
The bulk of lignite reserves are found in and around Neyveli in Tamil Nadu.

3. Consider the following statements regarding distribution of coal fields in India:


1. Coal in India occurs in two important types of coal fields which are Gondwana coal fields and
Tertiary coal fields.
2. Out of the total coal reserves and production in India, Gondwana coal fields contribute 98% and
the rest 2% is produced by tertiary coal fields.
3. The Gondwana coal fields are located in the sedimentary rock systems of lower Gondwana
Age.
www.achieveias.co.in, YouTube Channel: http://youtube.com/c/AchieveIAS Telegram Channel: http://t.me/Achieve_Ias,
Mail: achieveias21@gmail.com, Contact Number: 8968920720

Which of the following statement(s) is/are correct?


A. 1 and 2 Only B. 2 and 3 Only C. 1 and 3 Only D. All are Correct

Answer: D

Explanation: Coal in India occurs in two important types of coal fields. They are the Gondwana
coal fields and Tertiary coal fields. Out of the total coal reserves and production in India,
Gondwana coal fields contribute 98% and the rest 2% is produced by tertiary coal fields. The
Gondwana coal fields are located in the sedimentary rock systems of lower Gondwana Age. They
are distributed chiefly in the river velleys of the Damodar (Jharkhand - West Bengal); the Son
(Madhya Pradesh–Chhatisgarh); the Mahanadi (Orissa), the Godavari (Andhra Pradesh) and the
Wardha (Maharashtra). Tertiary coalfields occur in the extra-peninsular areas which include
Assam, Meghalaya, Nagaland, Arunachal Pradesh, Jammu & Kashmir and Sikkim. Besides lignite
or brown coal are found in coastal areas of Tamil Nadu, Gujarat and in land basins of Rajasthan.

4. Which one of the following minerals is formed by decomposition of rocks, leaving a


residual mass of weathered material?
A. Coal B. Bauxite C. Gold D. Zinc

Answer: B

Explanation: Bauxite is a mineral which is formed by decomposition of rocks, leaving a residual


mass of weathered material. A naturally occurring homogeneous substance that has a definable
internal structure is called a mineral. Minerals usually occur in various forms i.e. the various kinds
of atmospheric conditions that play an important role in the formation of minerals.

5. Which one of the following is a ferrous metal?


A. Copper B. Manganese C. Coal D. Bauxite

Answer: B

Explanation: A metal with the descriptor “ferrous” means that it has iron in its composition. When
the term ferrous metal is used, it also usually implies that iron is a large percentage of the
elemental composition. If it’s not the most abundant element, it would probably be the second or
third most prolific. If a metal only contains trace amounts of iron, as many metals do, then that
small amount is not considered enough to declare the metal ferrous.

The following are some examples of ferrous metals:


1. Carbon Steel 2. Stainless Steel
3. Cast Iron 4. Alloy Steel

6. Mica is used in electric and electronic industries because


A. Of its insulating properties and resistance to high voltage
B. It is a good conductor of electricity
C. Of its great malleability
D. Of its sonorous nature

Answer: A
www.achieveias.co.in, YouTube Channel: http://youtube.com/c/AchieveIAS Telegram Channel: http://t.me/Achieve_Ias,
Mail: achieveias21@gmail.com, Contact Number: 8968920720

Explanation: Mica is a mineral name given to a group of minerals that are physically and
chemically similar. They are all silicate minerals, known as sheet silicates because they form in
distinct layers. Micas are fairly light and relatively soft, and the sheets and flakes of mica are
flexible. Mica is heat-resistant and does not conduct electricity. There are 37 different mica
minerals. The most common include: purple lepidolite, black biotite, brown phlogopite and clear
muscovite. The principal use of ground mica is in gypsum wallboard joint compound, where it acts
as a filler and extender, provides a smoother consistency, improves workability, and prevents
cracking. In the paint industry, ground mica is used as a pigment extender that also facilitates
suspension due to its light weight and platy morphology. The ground mica also reduces checking
and chalking, prevents shrinkage and shearing of the paint film, provides increased resistance to
water penetration and weathering, and brightens the tone of colored pigments. Ground mica also
is used in the well-drilling industry as an additive to drilling “muds. ”The plastic industry used
ground mica as an extender and filler and also as a reinforcing agent. The rubber industry uses
ground mica as an inert filler and as a mold lubricant in the manufacture of molded rubber
products, including tires. Sheet mica is used principally in the electronic and electrical industries.
The major uses of sheet and block mica are as electrical insulators in electronic equipment,
thermal insulation, gauge “glass”, windows in stove and kerosene heaters, dielectrics in
capacitors, decorative panels in lamps and windows, insulation in electric motors and generator
armatures, field coil insulation, and magnet and commutator core insulation.

7. Consider the following statements regarding the mineral resource of India:


1. India possesses more than 100 minerals, out of which only 30 minerals have economic
significance.
2. Reserves of petroleum and some non-ferrous metallic minerals are inadequate in India and in
order to fulfil the internal demands for these minerals, the country is dependent on the imports
from other countries.
3. After independence though export continues but also mineral production has picked up in
consonance with the increasing industrial demands in the country.

Which of the following statement(s) is/are correct?


a. Only 1 b. 1 and 2 c. 2 and 3 d. All of the above

Answer: D

Explanation: India is richly endowed with minerals. Our country possesses more than 100
minerals. Out of 100 minerals, there are 30 minerals which have economic significance.
Some of the examples are coal, iron ore, manganese, bauxite, mica etc. The situation is also
satisfactory in feldspar, florides, lime- stones, dolomite and gypsum etc. But the reserves of
petroleum and some non-ferrous metallic minerals especially copper, lead, zinc, tin, graphite are
inadequate. Non-ferrous minerals are those which do not contain iron. Country fulfills internal
demands for these minerals by importing them from other countries. India was least industrialised
and most of the minerals were exported during British period. After independence though export
continues but also mineral production has picked up in consonance with the increasing industrial
demands in the country.
www.achieveias.co.in, YouTube Channel: http://youtube.com/c/AchieveIAS Telegram Channel: http://t.me/Achieve_Ias,
Mail: achieveias21@gmail.com, Contact Number: 8968920720

8. The upper Brahmaputra valley is known for which of the following resources?
A. Zinc B. Petroleum C. Coal D. Silver

Answer: B

Explanation: Outside of these mineral belts, upper Brahmaputra valley is a significant petroleum
producing area whereas Kerala possesses enormous concentration of heavy mineral sands.
Outside these above mentioned areas minerals deposits are very poor, scattered and reserves are
inconsistent.

9. A tidal barrage is a barrier built over:


A. River bed B. River estuary C. River end D. River starting

Answer: B

Explanation: A tidal barrage is a barrier built over a river estuary to make use of the kinetic
energy in the moving water. Huge amounts of water move in and out of river mouths each day
because of the tides. The barrage contains electricity generators.

10. Which is the finest quality iron ore in terms of iron content?
A. Hematite B. Magnetite C. Siderite . Limonite

Answer: B

Explanation: India is rich in good quality iron ores. Magnetite is the finest iron ore with a very high
content of iron upto 70%. This iron ore is valuable for the electrical industry because of its
excellent magnetic properties. Hematite ore is the most important industrial iron ore; in terms of
usage. The iron content of hematite is 50-60%.
1. Orissa Jharkhand Belt: Badampahar mines in the Mayurbhanj and Kendujhar districts of Orissa
have high grade hematite ore. Additionally, hematite iron ore is mined in Gua and Noamundi in
Singhbhum district of Jharkhand.
2. Durg Bastar Chandrapur Belt: This belt lies in Chhattisgarh and Maharashtra. The Bailadila
range of hills in the Bastar district of Chhattisgarh have very high grade hematite ore. This hilly
range has 14 deposits of super high grade hematite ore. Iron from these mines is exported to
Japan and South Korea via Vishakapatnam port.
3. Bellary Chitradurga Chikmaglur Tumkur Belt: This belt lies in Karnataka. The Kudremukh mines
located in the Western Ghats are a 100 percent export unit. The ore from these mines is
transported as slurry through a pipeline to a port near Mangalore.
4. Maharashtra Goa Belt: This belt inculdes the state of Goa and Ratnagiri district of Maharashtra.
The ores in these mines are not of very high quality. They are exported through Marmagao port.
www.achieveias.co.in, YouTube Channel: http://youtube.com/c/AchieveIAS Telegram Channel: http://t.me/Achieve_Ias,
Mail: achieveias21@gmail.com, Contact Number: 8968920720

ACHIEVE IAS GEOGRAPHY MCQ SERIES, DAY 22, SOLUTIONS

1. Which of the following factors are not necessary to set up an iron and steel plant?
1. Proximity to iron-ore mines
2. Availability of coking coal
3. Good resources of limestone and manganese
4. Nearness to a water body to discharge wastes

Select correct answer:


A. 1 and 2 only B. 2 and 3 only C. 4 Only D. 1, 2 and 3

Answer: C

Explanation: Factors influencing the locations of Iron and steel industries:


1. Raw material: Mostly large integrated steel plants are located close to source of raw materials,
as they use large quantity of heavy and weight losing raw materials. E.g. Concentration of Iron and
steel industry in Chota Nagpur region-Presence of Iron ore in this region. TISCO at Jamshedpur.
2. Markets: As its heavy & bulky transportation cost is high. Therefore nearness to market is
important especially for mini steel plants access to nearby markets is most important in order to
minimize transportation cost. Also as mini steel plants rely on scrap metal they are mostly present
in Maharashtra. Visakhapatnam steel plant located near the coast has excellent import-export facility
3. Labour: Availability of cheap labour is also important. Eg: Rourkhela plant, Orisa; Bhilai steel plant
in Chattisgarh, mostly in Chota Nagpur region
4. Availability of electricity mostly hydro and water for cooling. Eg: Bokaro steel plant on banks of
river Damodar, Visheshwarya steel plant, Kar near river Bhadra.

2. Which one of the following is an agro-based industry?


1. Sewing machine
2. Iron and steel
3. Jute

Select correct answer:


A. 1 and 2 only B. 2 and 3 only C. 3 Only D. 1, 2 and 3

Answer: C

Explanation: The agro-based industry includes industries related to textiles, sugar, paper and
vegetable oil. These industries use agricultural products as their raw materials. Textile industry is
the largest industry in the organized sector. It comprises of (i) cotton textiles, (ii) woollen textiles, (iii)
silk textiles (iv) synthetic fibres and (v) jute textile industries. Textiles have been a major component
of the industrial sector. It accounts for nearly a fifth of the industrial output and a third of the export
earnings. Types of Agro-based Industry

1. Cotton textiles
2. Woollen textiles
3. Silk textiles
4. Synthetic fibres
5. Jute textile industries
www.achieveias.co.in, YouTube Channel: http://youtube.com/c/AchieveIAS Telegram Channel: http://t.me/Achieve_Ias,
Mail: achieveias21@gmail.com, Contact Number: 8968920720

1. Textile Industry: It occupies unique position in the Indian economy, because it contributes
significantly to industrial production (14 per cent), employment generation (35 million persons
directly – the second largest after agriculture) and foreign exchange earnings (about 24.6 per cent).
It contributes 4 per cent towards GDP. It is the only industry in the country, which is self-reliant and
complete in the value chain i.e., from raw material to the highest value added products.

2. Cotton Textiles: In ancient India, cotton textiles were produced with hand spinning and handloom
weaving techniques. After the 18th century, power-looms came into use. Our traditional industries
suffered a setback during the colonial period because they could not compete with the mill-made
cloth from England. Today, there are nearly 1600 cotton and human made fibre textile mills in the
country. About 80 per cent of these are in the private sector and the rest in the public and cooperative
sectors. Apart from these, there are several thousand small factories with four to ten looms. In the
early years, the cotton textile industry was concentrated in the cotton growing belt of Maharashtra
and Gujarat. Availability of raw cotton, market, transport including accessible port facilities, labour,
moist climate, etc. contributed towards its localisation. This industry has close links with agriculture
and provides a living to farmers, cotton boll pluckers and workers engaged in ginning, spinning,
weaving, dyeing, designing, packaging, tailoring and sewing. The industry by creating demands
supports many other industries, such as, chemicals and dyes, mill stores, packaging materials and
engineering works. While spinning continues to be centralised in Maharashtra, Gujarat and Tamil
Nadu, weaving is highly decentralised to provide scope for incorporating traditional skills and
designs of weaving in cotton, silk, zari, embroidery, etc. India has world class production in spinning,
but weaving supplies low quality of fabric as it cannot use much of the high quality yarn produced in
the country. Weaving is done by handloom, power loom and in mills. The hand spun khadi provides
large scale employment to weavers in their homes as a cottage industry. India exports yarn to Japan.
Other importers of cotton goods from India are U.S.A., U.K., Russia, France, East European
countries, Nepal, Singapore, Sri Lanka, and African countries. India has the second largest installed
capacity of spindles in the world, next to China, at around 34 million (2003-04). Since the mid-
eighties, the spinning sector has received a lot of attention. We have a large share in the world trade
of cotton yarn, accounting for one fourth of the total trade. However, our trade in garments is only 4
per cent of the world’s total. Our spinning mills are competitive at the global level and capable of
using all the fibres we produce. The weaving, knitting and processing units cannot use much of the
high quality yarn that is produced in the country. There are some large and modern factories in these
segments, but most of the production is in fragmented small units, which cater to the local market.
This mismatch is a major drawback for the industry. As a result, many of our spinners export cotton
yarn while apparel/garment manufactures have to import fabric.

3. Jute Textiles: India is the largest producer of raw jute and jute goods and stands at second place
as an exporter after Bangladesh. There are about 70 jute mills in India. Most of these are located in
West Bengal, mainly along the banks of the Hugli river, in a narrow belt (98 km long and 3 km wide).
Factors responsible for their location in the Hugli basin are: proximity of the jute producing areas,
inexpensive water transport, supported by a good network of railways, roadways and waterways to
facilitate movement of raw material to the mills, abundant water for processing raw jute, cheap labour
from West Bengal and adjoining states of Bihar, Orissa and Uttar Pradesh. Kolkata as a large urban
centre provides banking, insurance and port facilities for export of jute goods. The jute industry
supports 2.61 lakh workers directly and another 40 lakhs small and marginal farmers who are
engaged in cultivation of jute and Mesta. Many more people are associated indirectly. Challenges
faced by the industry include stiff competition in the international market from synthetic substitutes
www.achieveias.co.in, YouTube Channel: http://youtube.com/c/AchieveIAS Telegram Channel: http://t.me/Achieve_Ias,
Mail: achieveias21@gmail.com, Contact Number: 8968920720

and from other competitors like Bangladesh, Brazil, Philippines, Egypt and Thailand. However, the
internal demand has been on the increase due to the Government policy of mandatory use of jute
packaging. To stimulate demand, the products need to be diversified. In 2005, National Jute Policy
was formulated with the objective of increasing productivity, improving quality, ensuring good prices
to the jute farmers and enhancing the yield per hectare. The main markets are U.S.A., Canada,
Russia, United Arab Republic, U.K. and Australia. The growing global concern for environment
friendly, biodegradable materials has once again opened the opportunity for jute products.

4. Sugar Industry: India stands second as a world producer of sugar but occupies the first place in
the production of gur and khandsari. The raw material used in this industry is bulky, and in haulage
its sucrose content reduces. There are over 460 sugar mills in the country spread over Uttar
Pradesh, Bihar, Maharashtra, Karnataka, Tamil Nadu, Andhra Pradesh and Gujarat along with
Punjab, Haryana and Madhya Pradesh. Sixty per cent mills are in Uttar Pradesh and Bihar. This
industry is seasonal in nature so, it is ideally suited to the cooperative sector. In recent years, there
is a tendency for the mills to shift and concentrate in the southern and western states, especially in
Maharashtra; this is because the cane produced here has higher sucrose content. The cooler
climate also ensures a longer crushing season. Moreover, the cooperatives are more successful in
these states. Major challenges include the seasonal nature of the industry, old and inefficient
methods of production, transport delay in reaching cane to factories and the need to maximise the
use of baggage.

3. Which are the two prime factors for the location of aluminium industry?
(a) Market and labour
(b) Transport network and water supply
(c) Cheap and regular supply of electricity and bauxite

Select correct answer:


A. 1 and 2 only B. 2 and 3 only C. 3 Only D. 1, 2 and 3

Answer: C

Explanation: The prime factors in location of aluminium smelting industries are as follows :
(i) Assured source of raw material, bauxite, at minimum cost as it is a bulky material at 4 to 6
tonnes of bauxite are required to manufacture 1 tonne of aluminium.
(ii) 18600 kwh of electricity is required per ton of ore for smelting of aluminium. Hence, regular
supply of power is another important factor for location of the industry.

Orissa produces about 45 per cent of the India's bauxite. Hence, aluminium smelting plants are
located in Orissa. Also, the Hirakud dam provides cheap hydroelectricity for the development of the
aluminium industry in the state. West Bengal, Kerala, Uttar Pradesh, Chhattisgarh, Maharashtra,
and Tamil Nadu, are other states where aluminium smelting plants are located. INDAL, HINDALCO,
MALCO, NALCO and Aluminium Corporation of India are names of the major smelting plants.

4. Which plateau is known as the mineral heart land of India?


A. Bhander Plateau B. Chotta Nagpur Plateau C. Deccan Plateau D. Tibetan Plateau

Answer: B
www.achieveias.co.in, YouTube Channel: http://youtube.com/c/AchieveIAS Telegram Channel: http://t.me/Achieve_Ias,
Mail: achieveias21@gmail.com, Contact Number: 8968920720

Explanation: Chota Nagpur plateau is a store house of minerals like mica, bauxite, copper,
limestone, iron ore and coal. The Damodar valley is rich in coal and it is considered as the prime
centre of coking coal in the country.

5. Consider the following statements regarding foot loose industries.


1. Foot loose industries can be located in a wide variety of places.
2. These industries are generally non-polluting industries.
3. They largely depend on component parts which can be obtained anywhere.

Which of the above statements is/are correct?


a. 1, 2 b. 2, 3 c. 1, 3 v. 1, 2, 3

Answer: D

Explanation: Foot loose industries can be located in a wide variety of places. They are not
dependent on any specific raw material, weight losing or otherwise. They largely depend on
component parts which can be obtained anywhere. They produce in small quantity and also employ
a small labour force. These are generally not polluting industries. The important factor in their
location is accessibility by road network. Diamonds, computer chips, and mobile manufacturing are
some examples of footloose industries. Non-footloose industries generally require raw material
availability within a time limit to make products. Sugar industry, jute industry and tea industry are the
examples of non-footloose industries.

6. Sugarcane industry is gradually shifting from north Indian states to South Indian states.
Which of the following are the reasons behind it?
1. The production of sugarcane per hectare is higher in Peninsular India.
2. The crushing season in south India is shorter than in north India.
3. The sucrose contents is higher in the tropical variety of sugarcane grown in the southern states.

Select the correct answer code:


a. 2 only b. 1, 3 c. 1, 2, 3 d. 2, 3

Answer: B

Explanation: Reasons for shifting of sugar industry from North India to Peninsular India
1. The production of sugarcane per hectare is higher is Peninsular India. In fact, sugarcane crop
grows well in the tropical climate of south India.
2. The sucrose content is higher in the tropical variety of sugarcane grown in the south.
3. The crushing season in south India is longer than in north India.
4. In south India most of the mills have modern machinery.
5. Most of the mills in Peninsular India are in cooperative sector, where profit maximization is not
the sole objective.

7. When was cotton textile industry established in Mumbai?


A. 1854 B. 1855 C. 1948 D. 1956

Answer: A
www.achieveias.co.in, YouTube Channel: http://youtube.com/c/AchieveIAS Telegram Channel: http://t.me/Achieve_Ias,
Mail: achieveias21@gmail.com, Contact Number: 8968920720

Explanation: The first successful modem cotton textile mill was established by an entrepreneur
C.N. Dewar. Bombay Spinning and Weaving Company. Bombay Spinning and Weaving Company
was the first cotton mill to be established in Bombay, India on 7 July 1854 at Tardeo by Cowasjee
Nanabhoy Davar (1815-73) and his associates. The Company was designed by Sir William
Fairbaim.

8. Which of the following industry is known as sun rising industry?


A. Dairy industry B. Information Technology
C. Health and clinic D. None of these

Answer: B

Explanation: Sunrise industry is a colloquial term for a burgeoning sector or business in its infancy
stage showing promise of a rapid boom. Sunrise industries are typically characterized by high growth
rates, numerous start-ups, and an abundance of venture capital funding.
These industries generate a lot of "buzz" as investors' interest in its long-term growth prospect and
public awareness increases.
KEY TAKEAWAYS:
 A sunrise industry is a new business or business sector showing potential for substantial and
rapid growth.
 Notable characteristics of sunrise industries include high-growth rates and a lot of start-ups
and venture capital funding.
 As a sunrise industry develops, it may transition to the maturity stage and then to the sunset
stage.
 To remain relevant and on an upward trajectory, sunrise industries must prove their viability
and sustainability.

9. Which activity is termed as activity of tertiary sector?


A. Wheat production B. Mobile production
C. Construction of a dam D. Fishing

Answer: C

Explanation: Tertiary Industries enable consumers to obtain and use the finished goods. Workers
in the tertiary part of the economy provide services rather than goods. Sales, repair services,
banking, and insurance are all part of the tertiary industry. People who work in the tertiary sector
include workers in the tourism and hospitality industry, doctors, couriers, and business consultants.
Some tertiary industries have close ties with the primary and secondary industries. As an example
fishermen rely on the weather forecasters to determine what the weather conditions will be like.
Tertiary Industries prosper where there is large groups of people, that is where they get the most
business. Many people need tertiary industries for there every day life. In PEI, since the 1950's
tertiary industries have become extremely important and have been growing steadily. Tertiary
Industries are most of the time grouped with Quaternary Industries because of their similarities.
Some people may get them confused, although Quaternary Industries and Tertiary Industries both
don't produce goods but provide services, Tertiary Industries focus on broader subjects. Only the
Tertiary Industries provide services, other than goods, although sometimes the Quaternary sector
does it sometimes, they don't all the time. In 1993, 70.5% of labor forces were hired in the Tertiary
sector of the economy.
www.achieveias.co.in, YouTube Channel: http://youtube.com/c/AchieveIAS Telegram Channel: http://t.me/Achieve_Ias,
Mail: achieveias21@gmail.com, Contact Number: 8968920720

10. The processing of raw material into more valuable products falls under the category of:
A. Secondary activities B. Tertiary activities
C. Primary activities D. None of the above

Answer: A

Explanation: Manufacturing and Industry sector known as the secondary sector, sometimes as
the production sector, includes all branches of human activities that transform raw
materials into products or goods. The secondary sector includes secondary processing of raw
materials, food manufacturing, textile manufacturing and industry. The secondary sector is often
divided into heavy industry and light industry (see mechanical engineering). Each business,
organization or professional group, chamber or guild may also have their own classification.
The secondary sector forms a substantial part of GDP, it creates values (goods) and it is the engine
of economic growth and is crucial for all developed economies, although the trend, in most
developed countries, is the predominant tertiary sector. Basic list of Manufacturing and Industry
sector, the secondary sector is following:
 Electrical industry
 Chemical Industry
 Energy industry (according to some sources it is on the border of the tertiary sector)
 Metallurgical industry
 Construction Industry
 Food Industry
 Glass industry
 Textile and clothing industry
 Consumer goods industry (all consumables)
 Automotive
www.achieveias.co.in, YouTube Channel: http://youtube.com/c/AchieveIAS Telegram Channel: http://t.me/Achieve_Ias,
Mail: achieveias21@gmail.com, Contact Number: 8968920720

ACHIEVE IAS GEOGRAPHY MCQ SERIES, DAY 23, SOLUTIONS

1. Which of the following National Highway connects the four metro cities of India (Delhi,
Mumbai, Kolkata and Chennai)?
A. National Highway 44 B. National Highway 47A
C. Golden Quadrilateral (GQ) D. North-South Corridor of NHDP

Answer: C

Explanation: Golden Quadrilateral (GQ) stretch which connects the four metro cities of India (Delhi,
Mumbai, Kolkata and Chennai). One of achievements of the Indian government is the 5846-
km Golden Quadrilateral (GQ) highway. It is designated as one of the longest highways in the
world. It is basically a network of highways that connect the four major metropolitan cities of the
country in four directions - Delhi (North), Chennai (South), Kolkata (East) and Mumbai (West) -
thereby forming a quadrilateral, and hence the name Golden Quadrilateral. Launched in 2001, this
was the largest highway and a very ambitious project. The project, which was undertaken by the
National Highways Development Project (NHDP) and managed by National Highways Authority of
India (N.H.A.I), was launched by the then prime minister Atal Behari Vaijpayee. The planning for the
project was completed in 1999 but the construction work officially started in 2001. Though it was
estimated to be completed by 2006, it actually became operational in January 2012. The Golden
Quadrilateral project included construction of new express highways, including renovation and
extension of the existing highways to four or six lanes. Major cities covered under Golden
Quadrilateral highway: The Golden Quadrilateral provides efficient transportation links between
major cities of India, like New Delhi; Jaipur, Udaipur, Ajmer (Rajasthan); Ahmedabad, Gandhinagar
(Gujarat); Mathura, Varanasi, Agra, Kanpur (Uttar Pradesh); Mumbai and Pune (Maharashtra);
Bangalore (Karnataka); Visakhapatnam (Andhra Pradesh); Chennai (Tamil Nadu); Bhubaneswar
(Orissa) Kolkata (West Bengal) etc.

The four sections of the Golden Quadrilateral:


 Section I: This covers National Highway 2 (NH2) from Delhi to Kolkata. Total stretch is 1454
km. States covered are Delhi, Haryana, Uttar Pradesh, Bihar, Jharkhand and West Bengal.
Major cities include Delhi, Mathura, Faridabad, Agra, Allahabad, Firozabad, Kanpur and
Varanasi.
 Section II: This covers NH6 from Kolkata to Chennai, NH60 (Kharagpur to Balasore) and
NH5 (Balasore to Chennai). Total stretch is 1684km. States include West Bengal, Andhra
Pradesh, Orissa and Tamil Nadu.
 Section III: Total stretch is 1,290km. It covers parts of NH4 (Mumbai to Bangalore), NH7
(Bangalore to Krishnagiri, Tamil Nadu) and NH46 (Krishnagiri to nearby Chennai). States
include Maharashtra, Karnataka, Andhra Pradesh and Tamil Nadu.
 Section IV: Covering parts of NH 8 (Delhi to Kishangarh), NH 79A (Ajmer bypass), NH 79
(Nasirabad to Chittaurgarh) and NH 76 (Chittaurgarh to Udaipur), the stretch is 1,419km.
States include Maharashtra, Gujarat, Rajasthan, Haryana and New Delhi. Major cities
connected are Delhi, Ajmer, Udaipur, Gurgaon, Jaipur, Gandhinagar, Ahmedabad,
Vadodara, Surat and Mumbai.

Major highlights of the Golden Quadrilateral:


 It is the largest highway project completed in India.
 It is the fifth longest highway project in the world.
www.achieveias.co.in, YouTube Channel: http://youtube.com/c/AchieveIAS Telegram Channel: http://t.me/Achieve_Ias,
Mail: achieveias21@gmail.com, Contact Number: 8968920720

 The overall length of the Golden quadrilateral is 5,846km.


 The Golden Quadrilateral passes through 13 states of India.
 The Golden Quadrilateral constitutes only the national highways of the country and not state
highways and rural-urban roadways.
 The project was estimated to cost INR600bn but was one such project which was completed
at about half of the estimated costs at INR308.58bn.

Benefits for the country:


 Provides faster transport networks between major cities and ports
 Provides connectivity to major agricultural, industrial, and cultural centres of India
 Provides smoother movement of goods and people within the country
 Enables industrial development and job creation in smaller towns through access to varied
markets
 Farmers are able to transport their produce to major cities and towns for sale and export, and
there is less wastage and spoils.
 More economic growth through construction and indirect demand for steel, cement, and other
construction materials
 Giving an impetus to truck transport

2. Match the following:


Set I Set II
a. Railwheel Factory 1. Patiala
b. Rail Coach Factory 2. Chennai
c. Integral Coach Factory 3. Kapurthala
d. Diesel Loco modernisation works 4. Bengaluru

a b c d
A. 1 2 3 4
B. 4 3 2 1
C. 3 1 2 4
D. 4 1 2 3

Answer: B

Explanation: The correct match is given below: Railwheel Factory- Bengaluru


Rail Coach Factory- Kapurthala
Integral Coach Factory- Chennai
Diesel Loco modernisation works- Patiala

3. Which railway line runs along the Indian west coast parallel to Arabian Sea and Western
Ghats?
A. South Central Railway4. Bengaluru B. Konkan Railways
C. Southern Railway D. South Eastern Railway

Answer: B
www.achieveias.co.in, YouTube Channel: http://youtube.com/c/AchieveIAS Telegram Channel: http://t.me/Achieve_Ias,
Mail: achieveias21@gmail.com, Contact Number: 8968920720

Explanation: The Konkan Railways passes through 3 states of India – Goa, Maharashtra and
Karnataka. It runs along the Indian west coast parallel to Arabian Sea and Western Ghats. It is an
extremely beautiful and scenic route running from Maharashtra till Karnataka.

4. What is the world ranking of Indian Railway network?


A. 1st B. 2nd C. 3rd D. 4th

Answer: C

Explanation: Indian Railway is the 3rd largest rail network in the world after US and China. It is a
multi-gauge, multi-traction system.

5. Consider the following statements regarding the impact of Economic liberalisation


industrialisation in India:
1. The process of industrialization in India can be divided into two parts – before and after 1992.
2. In August 1992, Government of India took a bold step by changing its economic policies from
state control to market forces.
3. The immediate cause of these changes in economic policy was to tide over balance of payment
crises but having wide social, economic, political and geographical implications.

Which of the following statement(s) is/are correct?


a. Only 1 b. 1 and 2 c. 2 and 3 d. All of the above

Answer: D

Explanation: The process of industrialization in India can be divided into two parts – before and
after 1992. During first forty years after independence the Indian economy had diversified and
expanded very fast. But this growth was characterized by rigid controls and regulations. In August
1992, Government of India took a bold step by changing its economic policies from state control to
market forces.A need was felt to give more responsibility to private capital and enterprise, both
domestic as well as foreign. In response to this, the new industrial policy of liberalization,
privatisation and globalization was adopted in August 1992. The immediate cause of these changes
in economic policy was to tide over balance of payment crises but having wide social, economic,
political and geographical implications.
www.achieveias.co.in, YouTube Channel: http://youtube.com/c/AchieveIAS Telegram Channel: http://t.me/Achieve_Ias,
Mail: achieveias21@gmail.com, Contact Number: 8968920720

ACHIEVE IAS GEOGRAPHY MCQ SERIES, DAY 24, SOLUTIONS

1. Consider the following statements regarding the iron and steel industry in India:
1. Indian Petro-Chemical Corporation has set up a huge petro-chemical complex near vadodara
producing a wide range of products.
2. India is self-sufficient in the production of petro-chemicals.
3. The only private oil refineries belong to Reliance Industries Ltd. is located at Jamnagar (Gujarat).

Which of the following statement(s) is/are correct?


a. Only 1 b. 1 and 2 c. 2 and 3 d. All of the above

Answer: D

Explanation: Petro-chemicals industry is one of the fastest growing industries of India. This industry
has revolutionised the industrial scene by providing the products which are substituting the traditional
raw materials like wood, glass and metals. Its products meet various needs of the people at the low
cost. Petro-chemicals are derived from petroleum or natural gas. We use a variety of products from
morning till evening made from petrochemicals Toothbrushes, toothpaste, combs, hairpins, soap
cases, plastic mugs, garments, radiocaes, ball point pens, detergents, electric switches, lipstick,
insecticides, bags, bed covers, and foam are some of the goods made from petro-chemicals.
Indian Petro-Chemical Corporation has set up a huge petro-chemical complex near vadodara
producing a wide range of products. Besides Vadodara, Gandhar, and Hazira in Gujarat and
Nagathone in Maharashtra are other important centres of petro-chemical industry. India is self-
sufficient in the production of petro-chemicals. Crude oil has no value unless it is refined, while
refining crude oil; thousands of products like kerosene, diesel, lubricants and raw material for petro-
chemical industry are derived. India has at present 18 refineries. Industries can be classified into
different categories on the basis, such as of sources of raw material, ownership, functions, size of
industry and weight of raw material and finished products. Since India is still an agricultural country, it
has developed various agro-based industries such as cotton textile, woolen textile, jute textile and
sugar industry. So, an UPSC IAS aspirant must have such understanding of industrial development.

2. Arrange the following National Parks from West to East:


1. Velvadar
2. Gir
3. Nawegaon
4. Betla
5. Sirohi

Choose the correct option:


a. 2-3-4-1-5 c. 2-4-1-5-3
b. 2-1-3-4-5 d. 2-3-1-5-4

Answer: B
www.achieveias.co.in, YouTube Channel: http://youtube.com/c/AchieveIAS Telegram Channel: http://t.me/Achieve_Ias,
Mail: achieveias21@gmail.com, Contact Number: 8968920720

3. Consider the following statements regarding the distribution of cotton textile industry in
India:
1.These mills are located in more than 88 centres in different parts of the country.
2. Majority of cotton textile mills are still located in the cotton growing areas of the Great Plains and
peninsular India.

Which of the following statement(s) is/are correct?


a. Only 1 b. Only 2 c. Both 1 and 2 d. Neither 1 nor 2

Answer: C

Explanation: Cotton textile industry is one of the most widely distributed industries in our country.
These mills are located in more than 88 centres in different parts of the country But majority of cotton
textile mills are still located in the cotton growing areas of the Great Plains and peninsular India.
Maharashtra is the leading producer of cotton textile in the country. Mumbai is the major centre of
textile mills. About a half of the Cotton textile mills are located in Mumbai alone. It is, therefore, rightly
called as ‘Cottonpolis’ of India. Sholapur, Kohlapur, Nagpur, Pune, Aurangabad and Jalgaon are
other important centres in Maharashtra.

4. Which of the following railway is not recognised by UNESCO World heritage Site?
A. Darjeeling Himalayan Railway B. Nilgiri Mountain Railway
C. Kalka-Shimla Railway D. North-Eastern Railway

Answer: D

Explanation: There are two UNESCO World Heritage Sites on IR viz. the Chatrapati Shivaji Terminus
and the Mountain railways of India. Mountain railways of India is not contiguous, but consists of three
separate railway lines located in different parts of the country viz. the Darjeeling Himalayan Railway;
the Nilgiri Mountain Railway; and the Kalka-Shimla Railway.

5. India's first Post Office set up in which place?


A. Madras (Now Chennai) B. Calcutta (Now Kolkata)
C. Bombay (Now Mumbai) D. Delhi

Answer: C
Explanation: The first post office in India was established by British East India Company in Bombay in
1764. Postage stamps were first used in India in 1852 at district of scinde (also known as scinde dawK)
www.achieveias.co.in, YouTube Channel: http://youtube.com/c/AchieveIAS Telegram Channel: http://t.me/Achieve_Ias,
Mail: achieveias21@gmail.com, Contact Number: 8968920720

ACHIEVE IAS GEOGRAPHY MCQ SERIES, DAY 25, SOLUTIONS

1. Consider the following statements regarding layered structure of earth:


1. Inner Core is the densest layer of earth’s interior.
2. Continental crust is less dense than the oceanic crust.

Choose the correct statements?


a) Only 1 b) Only 2 c) Both 1 and 2 d) None of the above

Answer: C

Explanation: The Crust is the outermost solid part of the earth. It is brittle in nature. The thickness
of the crust varies under the oceanic and continental areas. Oceanic crust is thinner as compared
to the continental crust. The oceanic crust is basalt and the mean density of material in oceanic
crust is 2.7 g/cm3. The outer core is in liquid state while the inner core is in solid state. At the centre
of the earth at 6,300 km, the density value is around 13g/cm3. The core is made up of very heavy
material mostly constituted by nickel and iron. It is sometimes referred to as the nife layer.

2. Consider the following statements regarding the gravitation force of the earth:
1. Gravitation force is not same at different latitudes on the surface.
2. Gravitational force is greater near the poles and less at the equator.

Choose the incorrect statements:


a) Only 1 b) Only 2 c) Both 1 and 2 d) None of the above

Answer: D

Explanation: Gravity anomalies give us information about the distribution of mass of the material in
the crust of the earth. The gravity values differ according to the mass of material. The uneven
distribution of mass of material within the earth influences this value. The reading of the gravity at
different places differs from the expected values. Such a difference is called gravity anomaly. The
gravitation force is not the same at different latitudes on the surface. It is greater near the poles and
less at the equator. This is because of the distance from the centre at the equator being greater than
that at the poles.

3. Which of the following statements is correct regarding the composition of interior of the
Earth?
a) Earth as a whole is composed of mainly iron (Fe) but the earth’s crust consists of mostly oxygen.
b) The temperature and pressure increase with the increasing distance from the surface towards
the interior in deeper depths.
c) Continental crust is thicker than oceanic crust.
d) All of the above.

Answer: D

Explanation: Earth’s elemental composition reflects mostly heavier elements not blown away by
solar wind during formation of the solar system. Most abundant elements are Fe, O, Si, Mg and
most common minerals consist of silica (SiO2 ) mixed in varying proportions with other elements
www.achieveias.co.in, YouTube Channel: http://youtube.com/c/AchieveIAS Telegram Channel: http://t.me/Achieve_Ias,
Mail: achieveias21@gmail.com, Contact Number: 8968920720

such as Fe, Mg, Al, Ca, K, Na. The thickness of the crust varies under the oceanic and continental
areas. Oceanic crust is thinner as compared to the continental crust. The mean thickness of oceanic
crust is 5 km whereas that of the continental is around 30 km. The continental crust is thicker in the
areas of major mountain systems.

4. Which of the following statement is the most appropriate reason of earthquake waves
developing a shadow zone?
a) Composition of material in the shadow zone is impermeable.
b) The body waves (p and s waves) follow the Snell’s law when they pass from one medium to
another and hence deflect from the path.
c) Secondary waves do not pass through liquid medium (outer core).
d) Both (b) and (c).

Answer: D

Explanation: The layering of Earth has been inferred indirectly using the time of travel of refracted
and reflected seismic waves created by earthquakes. The changes in seismic velocity between
different layers causes refraction owing to Snell's law, like light bending as it passes through a prism.
Secondary waves (S-waves) are shear waves that are transverse in nature. S-waves can travel only
through solids, as fluids (liquids and gases) do not support shear stresses .S wave shadow zone
form the boundary of outer core as it is in liquid state.

5. Which of the following are correctly matched?


1. Lithosphere - the crust and the uppermost part of the mantle.
2. Shadow zone - the upper portion of the mantle.
3. Asthenosphere - NIFE layer

Choose the correct answer:


a) Only 1 b) Only 2 c) Both 1 and 2 d) All are correctly matched.

Answer: A

Explanation: The portion of the interior beyond the crust is called the mantle. The upper portion of
the mantle is called asthenosphere. The word astheno means weak. It is the main source of magma
that finds its way to the surface during volcanic eruptions. It has a density higher than the crusts.
The crust and the uppermost part of the mantle are called lithosphere. Its thickness ranges from 10-
200 km. Earthquake waves get recorded in seismographs located at far off locations. However,
there exist some specific areas where the waves are not reported. Such a zone is called the ‘shadow
zone’.

6. Which of the following are correctly matched?


a) The moon- The big Splat theory
b) Origin of universe- The Big Bang theory
c) Evolution of earth- The Nebula Hypothesis
d) All are correctly matched

Answer: D
www.achieveias.co.in, YouTube Channel: http://youtube.com/c/AchieveIAS Telegram Channel: http://t.me/Achieve_Ias,
Mail: achieveias21@gmail.com, Contact Number: 8968920720

Explanation: In 1950, Otto Schmidt and Carl Weizascar gave the ‘nebular hypothesis’, that the sun
was surrounded by solar nebula containing mostly the hydrogen and helium along with what may
be termed as dust. The friction and collision of particles led to formation of a disk-shaped cloud and
the planets were formed through the process of accretion. The most popular argument regarding
the origin of the universe is the Big Bang Theory. It is also called expanding universe hypothesis.
Edwin Hubble, in 1920, provided evidence that the universe is expanding. It is generally believed
that the formation of moon, as a satellite of the earth, is an outcome of ‘giant impact’ or what is
described as “the big splat”.

7. Consider the following statements regarding Nebular theory of Laplace?


1. The sun was surrounded by solar nebula containing mostly the hydrogen and silicon.
2. The collision of particle led to formation of a disk-shaped cloud and planets were formed
through the process of accretion.

Choose the correct statements?


a) Only 1 b) Only 2 c) Both 1 and 2 d) None of the above

Answer: B

Explanation: Nebular Hypothesis considered that the planets were formed out of a cloud of
material associated with a youthful sun, which was slowly rotating. Later in 1950, Otto Schmidt in
Russia and Carl Weizascar in Germany somewhat revised the ‘nebular hypothesis’, though
differing in details.
1. They considered that the sun was surrounded by solar nebula containing mostly the hydrogen
and helium along with what may be termed as dust.
2. The friction and collision of particles led to formation of a disk-shaped cloud and the planets
were formed through the process of accretion.

8. Choose the correct statements regarding inner planets?


a) Planets between the sun and the earth.
b) Planets between the sun and the belt of asteroids.
c) Planets in gaseous state.
d) Planets without satellite(s)

Answer: B

Explanation: Our Solar system consists of eight planets. Our solar system consists of the sun (the
star), 8 planets, 63 moons, millions of smaller bodies like asteroids and comets and huge quantity
of dust-grains and gases. Out of the eight planets, Mercury, Venus, Earth and Mars are called as
the inner planets as they lie between the sun and the belt of asteroids the other four planets are
called the outer planets. Alternatively, the first four are called Terrestrial, meaning earth-like as they
are made up of rock and metals, and have relatively high densities. The rest four are called Jovian
or Gas Giant planets. Jovian means Jupiter-like.

9. Consider the following statements regarding the characteristics of Terrestrial Planets?


1. The terrestrial planets were formed in the close vicinity of the parent star where it was too warm
for gases to condense to solid particles.
www.achieveias.co.in, YouTube Channel: http://youtube.com/c/AchieveIAS Telegram Channel: http://t.me/Achieve_Ias,
Mail: achieveias21@gmail.com, Contact Number: 8968920720

2. The terrestrial planets are larger than Jovian planets and their lower gravity could hold the
escaping gases.

Choose the correct statement?


a) Only 1 b) Only 2 c) Both1 and 2 d) None of the above

Answer: A

Explanation: The first four planets are called Terrestrial, meaning earth-like as they are made up
of rock and metals, and have relatively high densities. The rest four are called Jovian or Gas Giant
planets. Jovian means Jupiter-like. Most of them are much larger than the terrestrial planets and
have thick atmosphere, mostly of helium and hydrogen.
(i) The terrestrial planets were formed in the close vicinity of the parent star where it was too
warm for gases to condense to solid particles. Jovian planets were formed at quite a distant
location.
(ii) The solar wind was most intense nearer the sun; so, it blew off lots of gas and dust from the
terrestrial planets. The solar winds were not all that intense to cause similar removal of gases from
the Jovian planets.
(iii) The terrestrial planets are smaller and their lower gravity could not hold the escaping gases.

10. Seismology helps us understand the origin and intensity of earth’s interior. Consider the
following statements regarding Seismic waves?
1. All seismic waves are mechanical waves and require medium to propagate.
2. Their behavior change with the change in medium

Choose the correct option?


a) Only 1 b) Only 2 c) Both 1 and 2 d) None of the above

Answer: C

Explanation: Seismic waves are waves of energy that travel through the Earth's layers, and are a
result of earthquakes, volcanic eruptions, magma movement, large landslides and large man-made
explosions that give out low-frequency acoustic energy. Seismic wave fields are recorded by a
seismometer, hydrophone (in water), or accelerometer. The propagation velocity of the waves
depends on density and elasticity of the medium. Velocity tends to increase with depth and ranges
from approximately 2 to 8 km/s in the Earth's crust, up to 13 km/s in the deep mantle. The refraction
or reflection of seismic waves is used for research into the structure of the Earth's interior, and man-
made vibrations are often generated to investigate shallow, subsurface structures.
www.achieveias.co.in, YouTube Channel: http://youtube.com/c/AchieveIAS Telegram Channel: http://t.me/Achieve_Ias,
Mail: achieveias21@gmail.com, Contact Number: 8968920720

ACHIEVE IAS GEOGRAPHY MCQ SERIES, DAY 26, SOLUTIONS

1. Which of the following support as evidence for continental drift theory?


1. Sediments from India is known to have its counter parts in six different landmasses of the
Southern Hemisphere.
2. Placer deposits of gold in the Ghana coast have their source in Brazil
3. Identical species of plants and animals found on either side of the marine barriers

Select the correct answer using following codes.


a. Only 1 b. Only 1 and 2 c. 1, 2 and 3 d. None

Answer: C

Explanation: All statements are true. Also the radiometric dating methods suggest that the ocean
did not exist prior to 2,000 million years. Continental drift describes one of the earliest ways
geologists thought continents moved over time. This map displays an early "supercontinent,"
Gondwana, which eventually moved to form the continents we know today. Fossils of similar
organisms across widely disparate continents encouraged the revolutionary theory of continental
drift. The theory of continental drift is most associated with the scientist Alfred Wegener. In the early
20th century, Wegener published a paper explaining his theory that the continental landmasses
were “drifting” across the Earth, sometimes plowing through oceans and into each other. He called
this movement continental drift.

Evidence in support of Continental Drift:


1. Apparent Affinity of Physical Features
2. Causes of Drift
3. Polar wandering (Shifting of Poles)
4. Distribution of Fossils
5. Botanical Evidence
6. Rocks of Same Age across the Oceans
7. Tillite deposits
8. Placer Deposits

2. Which of the following are revealed by concept of sea floor spreading?


1. Volcanic eruptions are common along the mid oceanic ridges
2. The age of the rocks increases as one moves away from the mid oceanic ridges crest.
3. The continental rocks are much younger than the ocean crust rocks.

Select the correct answer using following codes.


a. 1 Only b. 1 and 2 Only c. 2 and 3 Only d. 1, 2 and 3

Answer: B

Explanation: The ocean crust rocks are much younger than the continental rocks. The deep
trenches have deep-seated earthquake occurrences. Sediments on the ocean floor are thin which
shows oceans are younger than continents.
 The idea that the seafloor itself moves (and carries the continents with it) as it expands from a
central axis was proposed by Harry Hess.
www.achieveias.co.in, YouTube Channel: http://youtube.com/c/AchieveIAS Telegram Channel: http://t.me/Achieve_Ias,
Mail: achieveias21@gmail.com, Contact Number: 8968920720

 According to this theory, the intense heat generated by radioactive substances in the mantle
(100-2900 km below the earth surface) seeks a path to escape, and gives rise to the formation
of convention currents in the mantle.
 Wherever rising limbs of these currents meet, oceanic ridges are formed on the sea floor and
wherever the failing limbs meet, trenches are formed.
 Seafloor spreading is a process that occurs at mid-ocean ridges, where new oceanic crust is
formed through volcanic activity and then gradually moves away from the ridge.
 Seafloor spreading helps explain continental drift in the theory of plate tectonics. When oceanic
plates diverge, tensional stress causes fractures to occur in the lithosphere.
 Basaltic magma rises up the fractures and cools on the ocean floor to form new sea floor.
 Older rocks will be found farther away from the spreading zone while younger rocks will be found
nearer to the spreading zone.

Evidences:
1. Volcanic eruptions are common all along the mid-oceanic ridges and they bring huge amounts of
lava to the surface in this area.
2. The rocks equidistant on either sides of the crest of mid-oceanic ridges show remarkable
similarities
3. Rocks closer to the mid-oceanic ridges are normal polarity and are the youngest.
4. The age of the rocks increases as one moves away from the crest.
5. The deep trenches have deep-seated earthquake occurrences while in the mid-oceanic ridge
areas, the quake foci have shallow depths.

3. Consider the following statements


1. Convergent boundaries of plates don’t happen between two oceanic plates.
2. The rotation of the earth has its effect on the separated blocks of the plate portions.

Choose correct statements from the following codes


a. Only 1 b. Only 2 c. Both 1 and 2 d. Neither 1 nor 2

Answer: B

Explanation: A convergent plate boundary is a location where two tectonic plates are moving
toward each other, often causing one plate to slide below the other (in a process known as
subduction). The collision of tectonic plates can result in earthquakes, volcanoes, the formation of
mountains, and other geological events
• When two tectonic plates move toward each other and collide, they form a convergent plate
boundary.
• There are three types of convergent plate boundaries: oceanic-oceanic boundaries, oceanic-
continental boundaries, and continental-continental boundaries. Each one is unique because of the
density of the plates involved.
• Convergent plate boundaries are often the sites of earthquakes, volcanoes, and other significant
geological activity.
Earth's surface is made up of two types of lithospheric plates: continental and oceanic. The crust
that makes up continental plates is thicker yet less dense than oceanic crust because of the lighter
rocks and minerals that compose it. Oceanic plates are made up of heavier basalt, the result of
magma flows from mid-ocean ridges. When plates converge, they do so in one of three settings:
oceanic plates collide with each other (forming oceanic-oceanic boundaries), oceanic plates collide
www.achieveias.co.in, YouTube Channel: http://youtube.com/c/AchieveIAS Telegram Channel: http://t.me/Achieve_Ias,
Mail: achieveias21@gmail.com, Contact Number: 8968920720

with continental plates (forming oceanic-continental boundaries), or continental plates collide with
each other (forming continental-continental boundaries).

Earthquakes are common any time large slabs of Earth come into contact with each other, and
convergent boundaries are no exception. In fact, most of the Earth's most powerful quakes have
occurred at or near these boundaries.
1. Oceanic-Oceanic Boundaries: When two oceanic plates collide, the denser plate sinks below the
lighter plate and eventually forms dark, heavy, basaltic volcanic islands. The western half of the
Pacific Ring of Fire is full of these volcanic island arcs, including the Aleutian, Japanese, Ryukyu,
Philippine, Mariana, Solomon, and Tonga-Kermadec. The Caribbean and South Sandwich island
arcs are found in the Atlantic, while the Indonesian archipelago is a collection of volcanic arcs in the
Indian Ocean. When oceanic plates are sub ducted, they often bend, resulting in the formation of
oceanic trenches. These often run parallel to volcanic arcs and extend deep beneath the
surrounding terrain. The deepest oceanic trench, the Mariana Trench, is more than 35,000 feet
below sea level. It is the result of the Pacific Plate moving beneath the Mariana Plate.
2. Oceanic-Continental Boundaries: When oceanic and continental plates collide, the oceanic plate
undergoes subduction and volcanic arcs arise on land. These volcanoes release lava with chemical
traces of the continental crust they rise through. The Cascade Mountains of western North America
and the Andes of western South America feature such active volcanoes. So do Italy, Greece,
Kamchatka, and New Guinea. Oceanic plates are denser than continental plates, which means they
have a higher subduction potential. They are constantly being pulled into the mantle, where they are
melted and recycled into new magma. The oldest oceanic plates are also the coldest, as they have
moved away from heat sources such as divergent boundaries and hot spots. This makes them
denser and more likely to subduct.
3. Continental-Continental Boundaries: Continental-continental convergent boundaries pit large
slabs of crust against each other. This results in very little subduction, as most of the rock is too light
to be carried very far down into the dense mantle. Instead, the continental crust at these convergent
boundaries gets folded, faulted, and thickened, forming great mountain chains of uplifted rock.
Magma cannot penetrate this thick crust; instead, it cools intrusively and forms granite. Highly
metamorphosed rock, like gneiss, is also common. The Himalayas and the Tibetan Plateau, the
result of 50 million years of collision between the Indian and Eurasian plates, are the most
spectacular manifestation of this type of boundary. The jagged peaks of the Himalayas are the
highest in the world, with Mount Everest reaching 29,029 feet and more than 35 other mountains
exceeding 25,000 feet. The Tibetan Plateau, which encompasses approximately 1,000 square miles
of land north of the Himalayas, averages around 15,000 feet in elevation.

4. With reference to Plate boundaries, which of the following is/are true?


1. New crust is generated in convergent boundaries.
2. Convergent boundaries happen only between two continental plates.
3. In divergent boundaries one crust is destroyed by the other.

Select the correct answer from the following codes


a. 1 Only b. 1 and 2 Only c. 2 and 3 Only d. None

Answer: D

Explanation: Where new crust is generated as the plates pull away from each other in divergent
boundaries. Where the crust is destroyed as one plate dived under another in convergent
www.achieveias.co.in, YouTube Channel: http://youtube.com/c/AchieveIAS Telegram Channel: http://t.me/Achieve_Ias,
Mail: achieveias21@gmail.com, Contact Number: 8968920720

boundaries. There are three ways in which convergence can occur. These are: (i) between an
oceanic and continental plate; (ii) between two oceanic plates; and (iii) between two continental
plates.

5. Consider the following statements regarding plate movements.


1. There is always unified movement of these plates away from the axis of earth.
2. The rotation of earth has impact on the plate movement.

Select the correct answer from the following codes


a. Only 1 b. Only 2 c. Both 1 and 2 d. Neither 1 nor 2

Answer: B

Explanation: As the eruptions do not take all along the entire crest of the plate at the same time,
there is a differential movement of a portion of the plate away from the axis of the earth. Also, the
rotation of the earth has its effect on the separated blocks of the plate portions.

6. Consider the following statements


1. In plate movement only the crust part of the lithosphere moves over the mantle.
2. In geological past it is noted that very few plates haven’t moved.

Select the correct answer from the following codes


a. 1 Only b. 2 Only c. Both 1 and 2 d. Neither 1 nor 2

Answer: D

Explanation: Continents are part of a plate and what moves is the plate. Moreover, it may be noted
that all the plates, without exception, have moved in the geological past, and shall continue to move
in the future period as well. A tectonic plate (also called lithospheric plate) is a massive, irregularly-
shaped slab of solid rock, generally composed of both continental and oceanic lithosphere. Plates
move horizontally over the asthenosphere as rigid units. The lithosphere includes the crust and top
mantle.

7. Which of the following is/are true about Indian Plate?


1. Indian plate lies towards the east of Makrana coast.
2. Eastern boundary of Indian plate runs from Rakinyoma Mountains of Myanmar to Java Trench.
3. There is divergent boundary between Indian plate and Antarctic plate.

Select the correct answer from the following codes


a. Only 1 b. Only 1 and 2 c. Only 2 and 3 d. 1, 2 and 3

Answer: D

Explanation: In the east, it extends through Rakinyoma Mountains of Myanmar towards the island
arc along the Java Trench. The Western margin follows Kirthar Mountain of Pakistan. It further
extends along the Makrana coast and joins the spreading site from the Red Sea rift. The boundary
between India and the Antarctic plate is also marked by oceanic ridge (divergent boundary).
www.achieveias.co.in, YouTube Channel: http://youtube.com/c/AchieveIAS Telegram Channel: http://t.me/Achieve_Ias,
Mail: achieveias21@gmail.com, Contact Number: 8968920720

8. Which of the following is/are properties of quartz?


1. It consists of silica
2. Insoluble in water.
3. Used in radio and radar

Choose correct statements from the following codes:


a. Only 1 b. Only 1 and 2 c. Only 2 and 3 d. 1, 2 and 3

Answer: D

Explanation: It is one of the most important components of sand and granite. It consists of silica. It
is a hard mineral virtually insoluble in water. It is white or colourless and used in radio and radar. It
is one of the most important components of granite.

9. Consider the following statements:


1. Mica is commonly found in sedimentary and metamorphic rocks.
2. Olivine often found in basaltic rocks is used in jewellery.

Select the correct answer from the following codes


a. Only 1 b. Only 2 c. Both 1 and 2 d. Neither 1 nor 2

Answer: B

Explanation: Mica forms 4 per cent of the earth’s crust. It is commonly found in igneous and
metamorphic rocks. Olivine is used in jewellery. It is usually a greenish crystal, often found in basaltic
rocks.

10. Which of the following is/are organically formed sedimentary rocks?


1. Chalk
2. Limestone
3. Shale

Select the correct answer from the following codes


a. 1 Only b. 1 and 2 Only c. 2 and 3 Only d. 1, 2 and 3

Answer: D

Explanation: Sedimentary rocks are formed from pre-existing rocks or pieces of once-living
organisms. They form from deposits that accumulate on the Earth's surface. Sedimentary rocks
often have distinctive layering or bedding. Many of the picturesque views of the desert southwest
show mesas and arches made of layered sedimentary rock.

1. Common Sedimentary Rocks: Common sedimentary rocks include sandstone, limestone, and
shale. These rocks often start as sediments carried in rivers and deposited in lakes and oceans.
When buried, the sediments lose water and become cemented to form rock. Tuffaceous
sandstones contain volcanic ash.
www.achieveias.co.in, YouTube Channel: http://youtube.com/c/AchieveIAS Telegram Channel: http://t.me/Achieve_Ias,
Mail: achieveias21@gmail.com, Contact Number: 8968920720

2. Clastic Sedimentary Rocks: Clastic sedimentary rocks are the group of rocks most people think
of when they think of sedimentary rocks. Clastic sedimentary rocks are made up of pieces (clasts)
of pre-existing rocks. Pieces of rock are loosened by weathering, then transported to some basin
or depression where sediment is trapped. If the sediment is buried deeply, it becomes compacted
and cemented, forming sedimentary rock. Clastic sedimentary rocks may have particles ranging in
size from microscopic clay to huge boulders. Their names are based on their clast or grain size.
The smallest grains are called clay, then silt, then sand. Grains larger than 2 millimeters are called
pebbles. Shale is a rock made mostly of clay, siltstone is made up of silt-sized grains, sandstone is
made of sand-sized clasts, and conglomerate is made of pebbles surrounded by a matrix of sand
or mud.

3. Biologic Sedimentary Rocks:


Biologic sedimentary rocks form when large numbers of living things die. Chert is an example for
this type of rock, and this is one of the ways limestone can form. Limestone can also form by
precipitating out of the water.
www.achieveias.co.in, YouTube Channel: http://youtube.com/c/AchieveIAS Telegram Channel: http://t.me/Achieve_Ias,
Mail: achieveias21@gmail.com, Contact Number: 8968920720

ACHIEVE IAS GEOGRAPHY MCQ SERIES, DAY 27, SOLUTIONS

1. Which of the following factors affect the soil formation?


1. Climate
2. Vegetation
3. Weathering
4. Relief
5. Parent Material

Select the correct answer using the codes given below:


a. 1, 3 and 4 b. 1, 2 and 3 c. 1, 2, 3 and 5 d. All of these

Answer: D

Explanation: Climate is the most important single factor in the soil formation. The amount of
rainfall and range of temperature are the most important climate factors affecting soil formation
Factors affecting soil formation: Soils form from the interplay of five main factors namely Parent
material, Time, Climate, Relief and Organisms.

1. Parent material: This refers to the mineral material or organic material from which the soil is
formed. Soils will carry the characteristics of its parent material such as color, texture, structure,
mineral composition and so on. For example, if soils are formed from an area with large rocks
(parent rocks) of red sandstone, the soils will also be red in color and have the same feel as its
parent material.

2. Time: Soils can take many years to form. Younger soils have some characteristics from their
parent material, but as they age, the addition of organic matter, exposure to moisture and other
environmental factors may change its features. With time, they settle and are buried deeper below
the surface, taking time to transform. Eventually, they may change from one soil type to another.

3. Climate: This is probably the most important factor that can shape the formation of soils. Two
important climatic components, temperature and precipitation are key. They determine how quickly
weathering will be, and what kind of organic materials may be available on and inside of the soils.
Moisture determines the chemical and biological reactions that will occur as the soils are formed.
A warmer climate with more rainfall means more vegetative cover and more animal action. It also
means more runoff, more percolation and more water erosion. They all help to determine the kind
of soils in an area.

4. Relief: This refers to the landscape position and the slopes it has. Steep, long slopes mean
water will run down faster and potentially erode the surfaces of slopes. The effect will be poor soils
on the slopes, and richer deposits at the foot of the slopes. Also, slopes may be exposed to more
direct sunlight, which may dry out soil moisture and render it less fertile.

5. Organisms: The source and richness of organic matter are down to the living things (plants and
animals) that live on and in the soils. Plants, in particular, provide lots of vegetative residues that
are added to soils. Their roots also hold the soils and protect them from wind and water erosion.
They shelter the soils from the sun and other environmental conditions, helping the soils to retain
the needed moisture for chemical and biological reactions. Fungi, bacteria, insects, earthworms,
www.achieveias.co.in, YouTube Channel: http://youtube.com/c/AchieveIAS Telegram Channel: http://t.me/Achieve_Ias,
Mail: achieveias21@gmail.com, Contact Number: 8968920720

and burrowing animals help with soil aeration. Worms help break down organic matter and aid
decomposition. Animal droppings, dead insects and animals result in additional decaying organic
matter. Microorganisms also help with mineral and nutrient cycling and chemical reactions.

2. Consider the following statements


1. Physical weathering is faster in crystalline rocks.
2. Physical weathering is always greater near the surface of rocks.
3. Exfoliation is chemical weathering.

Select the correct answer from the following codes


a. Only 1 b. Only 1 and 2 c. Only 2 and 3 d. 1, 2 and 3

Answer: B

Explanation: In Crystalline rocks such as granite the crystals of various minerals (quartz, mica,
feldspar) will expand and contract at different rates, enhancing the stresses and accelerating
disintegration. Exfoliation is physical weathering. Physical weathering is caused by the effects of
changing temperature on rocks, causing the rock to break apart. The process is sometimes
assisted by water. There are two main types of physical weathering:
1. Freeze-thaw occurs when water continually seeps into cracks, freezes and expands, eventually
breaking the rock apart.
2. Exfoliation occurs as cracks develop parallel to the land surface a consequence of the reduction
in pressure during uplift and erosion.

Physical weathering happens especially in places places where there is little soil and few plants
grow, such as in mountain regions and hot deserts. Either through repeated melting and freezing
of water (mountains and tundra) or through expansion and contraction of the surface layer of rocks
that are baked by the sun (hot deserts).

3. With reference to physical weathering, which of the following statements?


1. Chemical weathering catalyses the physical weathering.
2. During daytime outer layers of rock expand rapidly than interiors.
3. During night interior layers contract slowly than outer layers

Select the correct answer from the following codes


a. 1 Only b. 1 and 2 Only c. 2 and 3 Only d. 1, 2 and 3

Answer: D

Explanation: Processes of physical weathering work much more easily when surface of rock has
already been weakened by action of chemical weathering. Chemical weathering does not break
rocks into smaller fragments through wind, water, and ice (that's physical weathering). Nor does it
break rocks apart through the action of plants or animals (that's biological weathering). Instead, it
changes the chemical composition of the rock, usually through carbonation, hydration, hydrolysis
or oxidation. Chemical weathering alters the composition of the rock material toward surface
minerals, such as clays. It attacks minerals that are relatively unstable in surface conditions, such
as the primary minerals of igneous rocks like basalt, granite or peridotite. It can also occur
in sedimentary and metamorphic rocks and is an element of corrosion or chemical erosion.
www.achieveias.co.in, YouTube Channel: http://youtube.com/c/AchieveIAS Telegram Channel: http://t.me/Achieve_Ias,
Mail: achieveias21@gmail.com, Contact Number: 8968920720

Water is especially effective at introducing chemically active agents by way of fractures and
causing rocks to crumble piecemeal. Water may also loosen thin shells of material (in spheroidal
weathering). Chemical weathering may include shallow, low-temperature alteration.
Let's take a look at the four main types of chemical weathering that were mentioned earlier. It
should be noted that these are not the only forms, just the most common.

1. Carbonation: Carbonation occurs when rain, which is naturally slightly acidic due to atmospheric
carbon dioxide (CO2), combines with a calcium carbonate (CaCO3), such as limestone or chalk.
The interaction forms calcium bicarbonate, or Ca(HCO3)2. Rain has a normal pH level of 5.0-5.5,
which alone is acidic enough to cause a chemical reaction. Acid rain, which is unnaturally acidic
from atmospheric pollution, has a pH level of 4 (a lower number indicates greater acidity while a
higher number indicates greater basicity). Carbonation, sometimes referred to as dissolution, is
the driving force behind the sinkholes, caverns and underground rivers of karst topography.

2. Hydration: Hydration occurs when water reacts with an anhydrous mineral, creating a new
mineral. The water is added to the crystalline structure of a mineral, which forms a hydrate.
Anhydrite, which means "waterless stone," is a calcium sulfate (CaSO 4) that is usually found in
underground settings. When exposed to water near the surface, it quickly becomes gypsum, the
softest mineral on the Mohs hardness scale.

3. Hydrolysis: Hydrolysis is the opposite of hydration; in this case, water breaks down the chemical
bonds of a mineral instead of creating a new mineral. It is a decomposition reaction.
The name makes this one particularly easy to remember: The prefix "hydro-" means water, while
the suffix "-lysis" means decomposition, breakdown or separation.

4. Oxidation: Oxidation refers to the reaction of oxygen with metal elements in a rock,
forming oxides. An easily recognizable example of this is rust. Iron (steel) reacts easily with
oxygen, turning into reddish-brown iron oxides. This reaction is responsible for the red surface of
Mars and the red color of hematite and magnetite, two other common oxides.

4. Denudation process happens in which order?


a. Erosion, Weathering, Transportation, Deposition
b. Weathering, Erosion, Deposition, Transportation
c. Erosion, Transportation, Weathering, Deposition
d. Transportation, Weathering, Erosion, Deposition

Answer: B

Explanation: Weathering is gradual disintegration of rocks by atmospheric forces, erosion is


active wearing away of earth surfaces, Transportation is removal of eroded debris to new
positions; Deposition is dumping of debris in certain parts of the earth.

5. Which of the following is true about glacier?


1. Glaciers are narrow at source but broadest down the hill.
2. Glaciers move because it is in liquid state.
3. The rate of movement is greatest in the side and bottom of glacier than middle.

Select the correct answer from the following codes


www.achieveias.co.in, YouTube Channel: http://youtube.com/c/AchieveIAS Telegram Channel: http://t.me/Achieve_Ias,
Mail: achieveias21@gmail.com, Contact Number: 8968920720

a. 1 Only b. 2 and 3 Only c. 1, 2 and 3 d. None

Answer: D

Explanation: Though glaciers are not in liquid state, due continual pressure from accumulated
snow above, it moves. The rate of movement is greatest in middle due little obstruction. The sides
and bottom are held back by little friction in valley.

6. Consider the following statements


1. Gorges are formed where the rocks are resistant.
2. Canyons are formed in arid regions of valley

Select the correct answer from the following codes


a. 1 Only b. 2 Only c. Both 1 and 2 d. Neither 1 nor 2

Answer: C

Explanation:

1. A gorge is a narrow valley with steep, rocky walls located between hills or mountains. The term
comes from the French word gorge, which means throat or neck. A gorge is often smaller than
a canyon, although both words are used to describe deep, narrow valleys with a stream or river
running along their bottom. A number of natural forces form gorges. The most common
is erosion due to streams or rivers. Streams carve through hard layers of rock, breaking down or
eroding it. Sediment from the worn-away rock is then carried downstream. Over time, this erosion
will form the steep walls of a gorge. The flooding of streams or rivers increases the speed and
intensity of this erosion, creating deeper and wider gorges. The deep Talari Gorges in Mali, for
instance, were formed by the Sngal River that flows into the Atlantic Ocean on the western coast
of Africa. Geologic uplift also forms gorges. Geologic uplift is the upward movement of the Earth’s
surface. Geologic uplift is often associated with earthquakes and orogeny, the process of creating
mountains. During geologic uplift, rock layers beneath the Earth’s surface bump against the
surface layers. Softer layers of surface rock erode.Erosion and geologic uplift often work together
to create gorges. Parts of streams or rivers can be elevated, along with land, during the process of
geologic uplift. As rivers or streams flow across this uplifted surface, waterfalls form. Over time,
the power of the waterfall erodes the softer rock layers underneath, causing the original river
bed to collapse and create a gorge. Macocha Gorge in the Jihomoravsk region of the Czech
Republic was probably formed by the collapse of an underground cave that had been eroded by
the Punkva River. The movement and melting of glaciers can also produce gorges. Glaciers cut
deep valleys into the Earths surface. These rivers of ice can create huge canyons and sharp,
steep gorges. As glaciers melt, or retreat, these gorges and canyons are exposed. The Columbia
River Gorge, located in the U.S. states of Washington and Oregon, was partially created by glacial
retreat during the last Ice Age.

2. A canyon is a deep, narrow valley with steep sides. “Canyon” comes from the Spanish
word cañon, which means “tube” or “pipe.” The term “gorge” is often used to mean “canyon,” but a
gorge is almost always steeper and narrower than a canyon.
The movement of rivers, the processes of weathering and erosion, and tectonic activity create
www.achieveias.co.in, YouTube Channel: http://youtube.com/c/AchieveIAS Telegram Channel: http://t.me/Achieve_Ias,
Mail: achieveias21@gmail.com, Contact Number: 8968920720

canyons.

A. River Canyons: The most familiar type of canyon is probably the river canyon. The water
pressure of a river can cut deep into a river bed. Sediments from the river bed are
carried downstream, creating a deep, narrow channel. Rivers that lie at the bottom of deep
canyons are known as entrenched rivers. They are entrenched because, unlike rivers in wide,
flat flood plains, they do not meander and change their course. The Yarlung Zangbo Grand
Canyon in Tibet, a region of southwestern China, was formed over millions of years by the Yarlung
Zangbo River. This canyon is the deepest in the world—at some points extending more than 5,300
meters (17,490 feet) from top to bottom. Yarlung Zangbo Canyon is also one of the world’s longest
canyons, at about 500 kilometers (310 miles).

B. Weathering and Erosion: Weathering and erosion also contribute to the formation of canyons.
In winter, water seeps into cracks in the rock. This water freezes. As water freezes, it expands and
turns into ice. Ice forces the cracks to become larger and larger, eroding bits of stone in the
process. During brief, heavy rains, water rushes down the cracks, eroding even more rocks and
stone. As more rocks crumble and fall, the canyon grows wider at the top than at the bottom.
When this process happens in soft rock, such as sandstone, it can lead to the development of slot
canyons. Slot canyons are very narrow and deep. Sometimes, a slot canyon can be less than a
meter (3 feet) wide, but hundreds of meters deep. Slot canyons can be dangerous. Their sides are
usually very smooth and difficult to climb. Some canyons with hard, underlying rock may develop
cliffs and ledges after their softer, surface rock erodes. These ledges look like giant steps.
Sometimes, entire civilizations can develop on and around these canyon ledges. Native American
nations, such as the Hopi and Sinagua, made cliff dwellings. Cliff dwellings were apartment-style
shelters that housed hundreds of people. The shaded, elevated ledges in Walnut Canyon and
Canyon de Chelly, in Arizona, provided protection from hostile neighbors and the
burning desert sun. Hard-rock canyons that are open at one end are called box canyons. The Hopi
and Navajo people often used box canyons as natural corrals for sheep and cattle. They simply
built a gate on the open side of the box canyon, and closed it when the animals were inside.
Limestone is a type of hard rock often found in canyons. Sometimes, limestone erodes and forms
caves beneath the earth. As the ceilings of these caves collapse, canyons form. The
Yorkshire Dales, an area in northern England, is a collection of river valleys and canyons created
by limestone cave collapses.

C. Tectonic Uplift: Canyons are also formed by tectonic activity. As tectonic plates beneath the
Earth’s crust shift and collide, their movement can change the area’s landscape. Sometimes,
tectonic activity causes an area of the Earth’s crust to rise higher than the surrounding land. This
process is called tectonic uplift. Tectonic uplift can create plateaus and mountains. Rivers
and glaciers that cut through these elevated areas of land create deep canyons. The Grand
Canyon, in the U.S. state of Arizona, is a product of tectonic uplift. The Grand Canyon, up to 447
kilometers (277 miles) long, 29 kilometers (18 miles) wide, and 1.8 kilometers (6,000 feet) deep, is
the largest canyon in the United States. The Grand Canyon has been carved, over millions of
years, as the Colorado River cuts through the Colorado Plateau. The Colorado Plateau is a large
area that was elevated through tectonic uplift millions of years ago. Geologists debate the age of
the canyon itself—it may be between 5 million and 70 million years old.

7. Which of the following influence formation of deltas?


www.achieveias.co.in, YouTube Channel: http://youtube.com/c/AchieveIAS Telegram Channel: http://t.me/Achieve_Ias,
Mail: achieveias21@gmail.com, Contact Number: 8968920720

1. Tides
2. Currents
3. Waves
4. Rate of sedimentation

Select the correct answer from the following codes


a. Only 1 and 4 b. Only 1 and 3 c. Only 1, 3 and 4 d. 1, 2, 3 and 4

Answer: D

Explanation: Tides, Currents, Waves, Rate of sedimentation influence the formation of deltas.
Deltas are wetlands that form as rivers empty their water and sediment into another body of water,
such as an ocean, lake, or another river. Although very uncommon, deltas can also empty into
land. A river moves more slowly as it nears its mouth, or end. This causes sediment, solid material
carried downstream by currents, to fall to the river bottom. The slowing velocity of the river and the
build-up of sediment allows the river to break from its single channel as it nears its mouth. Under
the right conditions, a river forms a deltaic lobe. A mature deltaic lobe includes
a distributary network—a series of smaller, shallower channels, called distributaries, that branch
off from the mainstream of the river.In a deltaic lobe, heavier, coarser material settles first.
Smaller, finer sediment is carried farther downstream. The finest material is deposited beyond the
river's mouth. This material is called alluvium or silt. Silt is rich in nutrients that help microbes and
plants—the producers in the food web—grow. As silt builds up, new land is formed. This is
the delta. A delta extends a river's mouth into the body of water into which it is emptying.

A delta is sometimes divided into two parts: subaqueous and subaerial.


1. The subaqueous part of a delta is underwater. This is the most steeply sloping part of the delta,
and contains the finest silt. The newest part of the subaqueous delta, furthest from the mouth of
the river, is called the prodelta.
2. The subaerial part of a delta is above water. The subaerial region most influenced by waves
and tides is called the lower delta. The region most influenced by the river's flow is called
the upper delta.

This nutrient-rich wetland of the upper and lower delta can be an extension of the river bank, or a
series of narrow islands between the river's distributary network. Like most wetlands, deltas are
incredibly diverse and ecologically important ecosystems. Deltas absorb runoff from both floods
(from rivers) and storms (from lakes or the ocean). Deltas also filter water as it slowly makes its
way through the delta's distributary network. This can reduce the impact of pollution flowing from
upstream. Deltas are also important wetland habitats. Plants such as lilies and hibiscus grow in
deltas, as well as herbs such as wort, which are used in traditional medicines. Many, many
animals are indigenous to the shallow, shifting waters of a delta. Fish, crustaceans such as
oysters, birds, insects, and even apex predators such as tigers and bears can be part of a
delta's ecosystem. Not all rivers form deltas. For a delta to form, the flow of a river must be slow
and steady enough for silt to be deposited and build up. The Ok Tedi, in Papua New Guinea is one
of the fastest-flowing rivers in the world. This river becomes a tributary of the Fly River. (The Fly,
on the other hand, does form a rich delta as it empties into the Gulf of Papua, part of the Pacific
Ocean.)A river will also not form a delta if exposed to powerful waves. The Columbia River in
Canada and the United States, for instance, deposits enormous amounts of sediment into the
Pacific Ocean, but strong waves and currents sweep the material away as soon as it is
www.achieveias.co.in, YouTube Channel: http://youtube.com/c/AchieveIAS Telegram Channel: http://t.me/Achieve_Ias,
Mail: achieveias21@gmail.com, Contact Number: 8968920720

deposited. The tides of the Atlantic Ocean are too strong to allow silt to create a delta on the
Amazon.

Types of Deltas: There are two major ways of classifying deltas. One considers the influences that
create the landform, while the other considers its shape.

Influence: There are four main types of deltas classified by the processes that control the build-up
of silt: wave-dominated, tide-dominated, Gilbert deltas, and estuarine deltas.
In a wave-dominated delta, the movement of waves controls a delta's size and shape. The Nile
delta (shaped by waves from the Mediterranean Sea) and Senegal delta (shaped by waves from
the Atlantic Ocean) are both wave-dominated deltas. Tide-dominated deltas usually form in areas
with a large tidal range, or area between high tide and low tide. The massive Ganges-
Brahmaputra delta, in India and Bangladesh, is a tide-dominated delta, shaped by the rise and fall
of tides in the Bay of Bengal. Gilbert deltas are formed as rivers deposit large, coarse sediments.
Gilbert deltas are usually confined to rivers emptying into freshwater lakes. They are usually
steeper than the normal flat plain of a wave-dominated or tide-dominated delta. This type of delta
was first identified by the geologist Grove Karl Gilbert, who described mountain streams
feeding ancient Lake Bonneville. (Utah's Great Salt Lake is the only remnant of Lake Bonneville.)
Estuarine deltas form as a river does not empty directly into the ocean, but instead forms
an estuary. An estuary is a partly enclosed wetland that features a brackish water (part-saltwater,
part-freshwater) habitat. The Yellow River forms an estuary, for instance, as it reaches the Bohai
Sea off the coast of northern China.

Shape: The term delta comes from the upper-case Greek letter delta (Δ), which is shaped like a
triangle. Deltas with this triangular or fan shape are called arcuate (arc-like) deltas. The Nile River
forms an arcuate delta as it empties into the Mediterranean Sea.
Stronger waves form a cuspate delta, which is more pointed than the arcuate delta, and is tooth-
shaped. The Tiber River forms a cuspate delta as it empties into the Tyrrhenian Sea near Rome,
Italy. Not all deltas are triangle-shaped. A bird-foot delta has few, widely spaced distributaries,
making it look like a bird's foot. The Mississippi River forms a bird-foot delta as it empties into the
Gulf of Mexico delta is inland, while a single stream reaches the ocean or other body of water. The
delta of the Sacramento-San Joaquin River in northern California is an inverted delta. The rivers
and creeks of the Sacramento and San Joaquin distributary networks meet in Suisun Bay, before
flowing to the Pacific Ocean through a single gap in the Coast Range, the Carquinez Strait.
Inland deltas, which empty into a plain, are extremely rare. The Okavango delta in Botswana is
probably the most well-known—and so unusual it is recognized as one of the "Seven Natural
Wonders of Africa." Water from the Okavango River never reaches another body of water. The
delta spreads water and silt across a flat plain in the Kalahari Desert before being evaporated.

8. Consider the following statements


1. Estuaries are more suitable for large ports than deltas.
2. Rivers are used to export bulky good, import food stuffs, and raw materials.

Select the correct answer from the following codes


a. Only 1 b. Only 2 c. Both 1 and 2 d. Neither 1 nor 2

Answer: C
www.achieveias.co.in, YouTube Channel: http://youtube.com/c/AchieveIAS Telegram Channel: http://t.me/Achieve_Ias,
Mail: achieveias21@gmail.com, Contact Number: 8968920720

Explanation: An estuary is an area where a freshwater river or stream meets the ocean. In
estuaries, the salty ocean mixes with a freshwater river, resulting in brackish water. Brackish water
is somewhat salty, but not as salty as the ocean. An estuary may also be called a bay,
lagoon, sound, or slough. Water continually circulates into and out of an estuary. Tides create the
largest flow of saltwater, while river mouths create the largest flow of freshwater.
When dense, salty seawater flows into an estuary, it has an estuarine current. High tides can
create estuarine currents. Saltwater is heavier than freshwater, so estuarine currents sink and
move near the bottom of the estuary. When less-dense freshwater from a river flows into the
estuary, it has an anti-estuarine current. Anti-estuarine currents are strongest near the surface of
the water. Heated by the sun, anti-estuarine currents are much warmer than estuarine currents.
In estuaries, water level and salinity rise and fall with the tides. These features also rise and fall
with the seasons. During the rainy season, rivers may flood the estuary with freshwater. During the
dry season, the outflow from rivers may slow to a trickle. The estuary shrinks, and becomes much
more saline. During a storm season, storm surges and other ocean waves may flood the estuary
with saltwater. Most estuaries, however, are protected from the ocean's full force. Geographical
features such as reefs, islands, mud, and sand act as barriers from ocean waves and wind.

9. Landslides are caused by:


1. Earthquake or Volcanic disturbances
2. Manmade steepening.
3. Lubricating action of rain water.

Select the correct answer from the following codes


a. Only 1 b. Only 1 and 2 c. Only 2 and 3 d. 1, 2 and 3

Answer: D

Explanation: The terms landslide or mudslide refer to the downward movement of large masses
of rocks, soil, mud and organic debris. Areas with steep slopes, for example mountainous regions,
are particularly susceptible to landslide hazards. Most landslides are caused by multiple factors
that act together to destabilize the slope. The primary cause of a landslide is the influence of
gravity acting on weakened materials that make up a sloping area of land. While some landslides
occur slowly over time (e.g., land movement on the order of a few meters/yards per month), the
most destructive ones happen suddenly after a triggering event such as heavy rainfall or an
earthquake. Water can trigger landslides and mudslides because it alters the pressure within the
slope, which leads to slope instability. Consequently, the heavy water-laden slope materials (soil,
rock, etc.) will succumb to the forces of gravity. Excessive water is thought to be one of the most
common triggers for landslides. Other factors that weaken slope materials also contribute to the
occurrence of landslides. These factors include both natural events such as geological weathering
and erosion and human-related activities such as deforestation and changes made to the flow of
groundwater. Destruction of vegetation by droughts, fires, and logging has been associated with
increased risk for landslides.

10. Mass movement of weathered materials are due to


1. Gradient of slope
2. Weight of weathered debris
3. Moisture
www.achieveias.co.in, YouTube Channel: http://youtube.com/c/AchieveIAS Telegram Channel: http://t.me/Achieve_Ias,
Mail: achieveias21@gmail.com, Contact Number: 8968920720

Select the correct answer from the following codes


a. Only 1 b. Only 1 and 2 c. Only 2 and 3 d. 1, 2 and 3

Answer: D

Explanation: Mass movement, also called Mass Wasting, bulk movements of soil and rock
debris down slopes in response to the pull of gravity, or the rapid or gradual sinking of
the Earth’s ground surface in a predominantly vertical direction. Formerly, the term mass wasting
referred to a variety of processes by which large masses of crustal materials are moved by gravity
from one place to another. More recently, the term mass movement has been substituted to
include mass wasting processes and the sinking of confined areas of the Earth’s ground surface.
Mass movements on slopes and sinking mass movements are often aided by water and the
significance of both types is the part each plays in the alteration of landforms.The types of mass
movements caused by the above factors include: the abrupt movement and free fall of loosened
blocks of solid rock, known as rock falls; several types of almost imperceptible downslope
movement of surficial soil particles and rock debris, collectively called creep; the
subsurface creep of rock material, known as bulging: the multiplicity of downslope movements of
bedrock and other debris caused by the separation of a slope section along a plane of least
resistance or slip surface, collectively called landslides; the separation of a mass along a concave
head scarp, moving down a curved slip surface and accumulating at the slope’s foot, known as
a slump; the saturation of debris and weathered material by rainfall in the upper section of a slope
or valley, increasing the weight of the debris and causing a slow downslope movement, called
an earthflow; a rapidly moving earthflow possessing a higher water content, known as
a mudflow; a fast-moving earthflow in a mountainous region, called a debris flow
or avalanche; and the downslope movement of moisture-saturated surficial material, known
as solifluction, over frozen substratum material, occurring in sub-Arctic regions during seasonal
periods of surface thaw.
www.achieveias.co.in, YouTube Channel: http://youtube.com/c/AchieveIAS Telegram Channel: http://t.me/Achieve_Ias,
Mail: achieveias21@gmail.com, Contact Number: 8968920720

ACHIEVE IAS GEOGRAPHY MCQ SERIES, DAY 28, SOLUTIONS

1. Consider the following statements:


1. The incoming solar radiation on earth is known as insolation
2. The farthest distance of the earth from the Sun is 152 million Km on 4th July
3. The position of the earth when it is nearest to the Sun is called Aphelion

Select the correct answer using the code given below:


(a) 1 only (b) 1 and 2 only (c) 1, 2 and 3 (d) 2 and 3 only

Answer: B

Explanation: Insolation or Incoming Solar Radiation


1. As we all know, the sun is the primary source of energy for the earth. The sun radiates its energy
in all directions into space in short wavelengths, which is known as solar radiation.
2. The earth’s surface receives only a part of this radiated energy (2 units out of 1,00,00,00,000
units of energy radiated by the sun).
3. The energy received by the earth’s surface in the form of short waves is termed as Incoming Solar
Radiation or Insolation.
4. The amount of insolation received on the earth’s surface is far less than that is radiated from the
sun because of the small size of the earth and its distance from the sun.
5. Moreover, water vapour, dust particles, ozone and other gases present in the atmosphere absorb
a small amount of solar radiation.
6. The solar radiation received at the top of the atmosphere varies slightly in a year due to the
variations in the distance between the earth and the sun.
7. During the earth’s revolution around the sun, the earth is farthest from the sun on 4th July. This
position of the earth is called aphelion. On 3rd January, the earth is nearest to the sun. This position
is called perihelion.
8. Due to this variation in the distance between the earth and the sun, the annual insolation received
by the earth on 3rd January is slightly more than the amount received on 4 th July.
9. However, the effect of this variation is masked by some other factors like the distribution of land
and sea and the atmospheric circulation. Hence the variation does not have a greater effect on daily
weather changes on the surface of the earth.

The amount of insolation received on the earth’s surface is not uniform everywhere. It varies
according to the place and time. When the tropical regions receive maximum annual insolation, it
gradually decreases towards the poles. Insolation is more in summers and less in winters. The major
factors which influence the amount of insolation received are:
1. Rotation of the earth on its axis
2. The angle of incidence of the sun’s rays
3. Duration of the day
4. Transparency of the atmosphere

2. Consider the following statements:


1. The transfer of heat through horizontal movement of air is called Advection
2. The short wave radiation on earth are absorbed by the carbon dioxide and the other greenhouse
gases where as long wave radiation pass through them without any heating
www.achieveias.co.in, YouTube Channel: http://youtube.com/c/AchieveIAS Telegram Channel: http://t.me/Achieve_Ias,
Mail: achieveias21@gmail.com, Contact Number: 8968920720

3. The Earth as a whole doesn’t accumulate or loose heat and its temperature has remained
constant

Select the correct answer using the code given below


(a) 1 only (b) 1 and 2 only (c) 1, 2 and 3 (d) 1 and 3 only

Answer .D

Explanation: Earth's atmosphere is a dynamic sea of gases in constant motion and


Earth's oceans contain currents that move water across the globe. Advection is a lateral or horizontal
transfer of mass, heat, or other property. Accordingly, winds that blow across Earth's surface
represent advectional movements of air. Advection also takes place in the ocean in the form of
currents. Currently, geologists debate the presence and role of substantial advective processes in
Earth's mantle. Differential pressures and temperatures drive the mass movement of air seeking
equilibrium (the lowest energy state). Advective winds move from areas of
higher temperature toward areas of lower temperature. In contrast, convection, the vertical
movement of mass or transfer of heat, manifests itself as air currents. Accordingly, winds are a result
of advection, while air currents are a result of convection.

3. The amount of visible radiation reflected by the earth is called as:


(a) Albedo (b) Scattering (c) Refraction (d) Dispersion

Answer: A

Explanation: Albedo (al-bee-doh) is a measure of how much light that hits a surface is reflected
without being absorbed. Something that appears white reflects most of the light that hits it and has
a high albedo, while something that looks dark absorbs most of the light that hits it, indicating a
low albedo.

4. With respect to Heat budget of the Earth, consider the following statements:
1. The surplus heat energy from the tropics is redistributed towards the pole as tropics do not get
progressively heated up and high latitudes get permanently frozen.
2. Roughly 55 percent of insolation are reflected back to space even before reaching the earth’s
surface.

Which of the statements given above is/are correct?


(a) 1 only (b) 2 only (c) 1 and 2 (d) None of these

Answer: A

Explanation: Earth's heat-budget or Earth's radiation balance refers to the net flow of energy into
Earth in the form of shortwave radiation and the outgoing infrared long-wave radiation into space.
Incoming heat is absorbed by the Earth, and outgoing heat escapes the Earth in the form of radiation
and both are evenly balanced. In case of their being not balanced, Earth would become either
warmer or cooler with the passage of time. This balance between incoming and outgoing heat is
known as Earth’s heat budget. In other words, the heat budget at the top of the atmosphere must
be in balanced state which in turn, is called Radiative equilibrium. It is note –worthy that the 29
percent of the solar energy that reaches the top of the atmosphere is reflected back to space by
www.achieveias.co.in, YouTube Channel: http://youtube.com/c/AchieveIAS Telegram Channel: http://t.me/Achieve_Ias,
Mail: achieveias21@gmail.com, Contact Number: 8968920720

clouds, atmospheric particles, or earth surfaces like sea ice and snow. This heat or energy plays no
role in Earth’s system. Besides, about 23 percent of incoming solar energy is absorbed in the
atmosphere by water vapour, dust, and ozone, and 48 percent passes through the atmosphere and
is absorbed by the surface. Thus, in totality, 71 percent of the total incoming solar energy is absorbed
by the Earth. This process occurs at the three levels: 1) the surface of the Earth, 2) the edge of
Earth’s atmosphere and 3) the atmosphere in between. At each level, the amount of incoming and
outgoing energy must be equal and balanced. The atmosphere and the surface of the Earth together
absorb 71 percent of incoming solar radiation, so it is necessary that to maintain the balance of
energy, they must radiate that much energy back to space to keep the earth ’s average temperature
stable. The atmosphere radiates heat equivalent to 59 percent of incoming sunlight; the surface
radiates only 12 percent. In other words, most solar heating happens at the surface, while most
radiative cooling happens in the atmosphere.
For the energy budget at Earth’s surface to balance, processes on the ground must get rid of the 48
percent of incoming solar energy that the ocean and land surfaces absorb. Energy leaves the
surface through three processes: 1) evaporation, 2) convection, and 3) emission of thermal infrared
energy. The surface absorbs about 48% of incoming sunlight. Three processes of evaporation
(25%), convection (5%), and thermal infrared radiation, or heat (net 17%)] remove an equivalent
amount of energy from the Earth’s surface.

5. Which of the following can influence the temperature of a place?


1. Latitude of the place
2. Altitude of the place
3. Distance from the sea
4. Air-mass circulation

Select the correct answer using the code given below


(a) 1, 2 and 4 only (b) 1 and 2 only (c) 1, 2 and 3 only (d) 1, 2 , 3 and 4

Answer: D

Explanation: (a) Latitude Temperature decreases as we go pole wards from the equator. The rays
of the sun fall vertically in the lower latitudes and slantingly in the higher latitudes. Places located in
the low latitude (e.g., Nagpur) are hotter than places located in the higher latitudes (e.g., Moscow).
(b) Altitude Temperature decreases with increasing altitude at the rate of 1°C for every 165 m of
ascent as the atmosphere gets heated by terrestrial radiation. The lower layers of atmosphere are
denser and have water vapor and dust particles which absorb heat, which is not prevalent in the
higher altitude. Missouri is cooler than Delhi.
(c) Distance from the sea Places located on the coast have moderate temperature as they are
affected by land and sea breezes, hence, the temperature is low. Places away from the coast have
extremes of temperature. Madras on the coast has moderate temperature and Delhi has extremes
of temperature. Prevailing winds Places which are affected by prevailing on-shore winds have
moderate temperature as compared to places which are affected by off-shore (land) winds.
(e) Ocean currents Warm currents raise the temperature of the places, and places affected by cold
currents have lower temperature. Winds blowing over the warm current also bring heavy rainfall in
the region. Whereas winds blowing over cold current absorb less moisture hence the rainfall is less,
e.g., North eastern Canada is affected by the cold Labrador Current hence it remains frozen in
winters, whereas North-western Europe in the same latitude is affected by the warm North Atlantic
drift and has ice-free ports in winters.
www.achieveias.co.in, YouTube Channel: http://youtube.com/c/AchieveIAS Telegram Channel: http://t.me/Achieve_Ias,
Mail: achieveias21@gmail.com, Contact Number: 8968920720

(f) Other factors affecting the temperature of a place are: direction of mountains, slope of the land
and vegetation, soil.

6. Consider the following statements about humidity


1. Absolute humidity represents the actual amount of moisture present in the air. Absolute
humidity remains constant with change in temperature.
2. Relative humidity is the ratio between the amount of moisture, which the air can hold at a given
temperature, and the amount it actually holds. Relative humidity increases with increase in
temperature, while it decreases with decrease in temperature.

Which of the above statement(s) is/are true?


(a) 1 Only (b) 1 and 3 (c) 2 and 3 (d) 1, 2 and 3

Answer: D

Explanation: Absolute humidity is ratio between the weight of vapour per unit volume of air
expressed as gram per cubic metre of air. Relative humidity is the ratio between the amount of
moisture, which the air can hold at a given temperature, and the amount it actually holds.

7. Consider the following statements about troposphere


1. It contains approximately 99% of atmosphere’s water vapour and aerosols.
2. It is deeper in the tropics up to 12 miles and shallower near the Polar Regions, at 4.3 miles in
summer and indistinct in winter.
3. Tropopause is the region of temperature inversion.

Which of the above statement(s) is/are correct?


(a) 1 Only (b) 2 Only (c) 1 and 3 Only (d) All of these

Answer: D

Explanation: The troposphere is the lowest portion of Earth’s atmosphere. It contains approximately
80% of the atmosphere’s mass and 99% of its water vapour and aerosols. The border between the
troposphere and stratosphere, called the tropopause, is a temperature inversion region.

8. Which one of the following is correct descending order (% composition) of permanent


gases of Atmosphere?
(a) Argon > Carbon dioxide > Neon > Helium
(b) Carbon dioxide > Argon > Neon > Helium
(c) Carbon dioxide > Argon > Helium > Neon
(d) Argon > Carbon dioxide > Helium > Neon

Answer: A

Explanation: The atmosphere is a huge blanket of air that surrounds the earth. It provides us the
air we breathe and protects us from the harmful effects of the sun’s rays. Atmosphere is consists
of mainly nitrogen (78%), oxygen (21%), argon (0.93%), carbon dioxide (0.03%) and other
gases like helium, ozone, argon and hydrogen (0.04%).
www.achieveias.co.in, YouTube Channel: http://youtube.com/c/AchieveIAS Telegram Channel: http://t.me/Achieve_Ias,
Mail: achieveias21@gmail.com, Contact Number: 8968920720

9. Which one of the following is correct order of layers in atmosphere?


(a) Troposphere, Stratosphere, Mesosphere, Thermosphere, Exosphere
(b) Stratosphere, Troposphere, Mesosphere, Thermosphere, Exosphere
(c) Stratosphere, Troposphere, Mesosphere, Exosphere, Thermosphere
(d) Troposphere, Stratosphere, Mesosphere, Exosphere, Thermosphere

Answer: A

Explanation: The atmosphere is divided into five layers starting from the earth’s surface. These are
Troposphere, Stratosphere, Mesosphere, Thermosphere and Exosphere.
1. Troposphere: This layer is the most important layer of the atmosphere. Its average height is 13
km. The air we breathe exists here. Almost all the weather phenomena like rainfall, fog and hailstorm
occur in this layer.
2. Stratosphere: It lies above the troposphere which extends up to a height of 50 km. This layer is
almost free from clouds and associated weather phenomenon, making conditions most ideal for
flying aeroplanes. One important feature of stratosphere is that it contains a layer of ozone gas.
3. Mesosphere: This is the third layer of the atmosphere. It lies above the stratosphere. It extends
up to the height of 80 km. Meteorites burn up in this layer on entering from the space.
4. Thermosphere: In thermosphere temperature rises very rapidly with increasing height. Ionosphere
is a part of this layer. It extends between 80 and 400 km. This layer helps in radio transmission. In
fact, radio waves transmitted from the earth are reflected back to the earth by this layer.
5. Exosphere: The upper most layer of the atmosphere is known as exosphere. This layer has very
thin air. Light gases like helium and hydrogen float into the space from here.

10. At night, the winds generally die out. This is because:


(a) Convection during daytime causes mixing of air and therefore, spreads up the wind near the
surface.
(b) Temperature inversion prevents any exchange of air from the upper layer to the lower layer
(c) The coriolis deflection is greater during daytime
(d) The conditions are quite during night time

Answer: B

Explanation: Temperature inversion is an increase in temperature with height. During the


temperature inversion the normal vertical temperature gradient is inverted such that the air is colder
near the surface of the earth. The cold air is near the surface will be stable and prevents any upward
and lateral movement. It suppresses the convection by acting as a cap.
www.achieveias.co.in, YouTube Channel: http://youtube.com/c/AchieveIAS Telegram Channel: http://t.me/Achieve_Ias,
Mail: achieveias21@gmail.com, Contact Number: 8968920720

ACHIEVE IAS GEOGRAPHY MCQ SERIES, DAY 29, SOLUTIONS

1. Which of the following is/are favourable conditions for the formation of tropical
cyclones?
1. Absence of the Coriolis force
2. Large sea surface with temperature higher than 27° C
3. Pre-existing weak low- pressure area

Select the correct answer from the following codes


a. 1 Only b. 1 and 2 Only c. 2 and 3 Only d. 1, 2 and 3

Answer: C

Explanation: Tropical cyclones originate and intensify over warm tropical oceans. The conditions
favourable for the formation and intensification f tropical storms are:
(i) Large sea surface with temperature higher than 27° C
(ii) Presence of the Coriolis force
(iii) Small variations in the vertical wind speed
(iv) A pre-existing weak low- pressure area or low-level-cyclonic circulation
(v) Upper divergence above the sea level system.

The tropical cyclones do not have a clear frontal system. The tropical cyclones originate only over
the seas and on reaching the land they dissipate. The wind velocity in a tropical cyclone is much
higher and it is more destructive. Tropical cyclones, move from east to west.

2. Which of the following is/are true about Extra tropical cyclones?


1. They move from west to east.
2. Extra tropical cyclones affect large area compared to tropical cyclones.
3. Extra tropical cyclones originate over land only.

Select the correct answer from the following codes


a. 1 Only b. 1 and 2 Only c. 2 and 3 Only d. 1, 2 and 3

Answer: B

Explanation: The extra tropical cyclones have a clear frontal system. They cover a larger area and
can originate over the land and sea. The extra tropical cyclone affects a much larger area as
compared to the tropical cyclone. The extra tropical cyclones move from west to east but tropical
cyclones, move from east to west.

3. With reference to Inter Tropical Convergence Zone (ITCZ), which of the following is/are
true?
1. It is low pressure zone where winds from tropics converge.
2. It raises upto 14km in troposphere and reaches land surfaces near equator as Easterlies.
3. It is zone of convergence of easterlies from either side of the equator.

Select the correct answer from the following codes


a. 1 Only b. 1 and 2 Only c. 2 and 3 Only d. 1, 2 and 3
www.achieveias.co.in, YouTube Channel: http://youtube.com/c/AchieveIAS Telegram Channel: http://t.me/Achieve_Ias,
Mail: achieveias21@gmail.com, Contact Number: 8968920720

Answer: D

Explanation: The air at the Inter Tropical Convergence Zone (ITCZ) rises because of convection
caused by high insolation and a low pressure is created. The winds from the tropics converge at
this low pressure zone. It reaches the top of the troposphere up to an altitude of 14 km. and moves
towards the poles. This causes accumulation of air at about 30o N and S. Part of the accumulated
air sinks to the ground and forms a subtropical high. Another reason for sinking is the cooling of air
when it reaches 30o N and S latitudes. Down below near the land surface the air flows towards the
equator as the easterlies. The easterlies from either side of the equator converge in the Inter
Tropical Convergence Zone (ITCZ).

4. With reference to Coriolis force, which of the following is/are true?


1. In southern hemisphere it deflects the wind to the left.
2. Coriolis force is directly proportional to the velocity of wind.
3. The Coriolis force acts perpendicular to the pressure gradient force

Select the correct answer from the following codes


a. 1 Only b. 1 and 2 Only c. 2 and 3 Only d. 1, 2 and 3

Answer: D

Explanation: The rotation of the earth about its axis affects the direction of the wind. This force is
called the Coriolis force. It acts perpendicular to the pressure gradient. If the pressure gradient
force is higher than the velocity of the wind will be more and larger deflection in the direction of
wind. The Coriolis force is zero and the wind blows perpendicular to the isobars at the equator.
Wind moves from higher pressure to low pressure and it is deflected to the right and left by
Coriolis force It deflects the wind to the right direction in the northern hemisphere and to the left in
the southern hemisphere. The deflection is more when the wind velocity is high. The pressure
gradient force is perpendicular to an isobar. The higher the pressure gradient force, the more is
the velocity of the wind and the larger is the deflection in the direction of wind.

5. Convectional rainfall is characterised by the presence of which clouds?


A. Cumulonimbus B. Cumulostratus C. Cirrocumulus D. Altocumulus

Answer: A

Explanation:
1. The air on getting heated becomes light and rises up in convection currents.
2. As the air rises, it expands and drops the temperature and subsequently, condensation takes
place and cumulus clouds are formed.
3. Heavy rainfall with lightning and thunder takes place which does not last long.
4. Such rain is usual in the summer or in the hotter part of the day.
5. This type of rainfall generally takes place in the equatorial regions and internal parts of the
continents, predominantly in the northern hemisphere.

6. What type of clouds are known as fair weather clouds?


A. Stratus B. Nimbus C. Cumulus D. Cirrus
www.achieveias.co.in, YouTube Channel: http://youtube.com/c/AchieveIAS Telegram Channel: http://t.me/Achieve_Ias,
Mail: achieveias21@gmail.com, Contact Number: 8968920720

Answer: C

Explanation: Cloud is a mass of minuscule water drops or minute crystals of ice formed by the
condensation of water vapour in free air at significant altitudes. According to their altitude, stretch,
density, and transparency or opaqueness clouds are classified under four types:
1. Cirrus
2. Cumulus
3. Stratus
4. Nimbus

1. Cirrus
a. Cirrus clouds are formed at high altitudes of 8,000 – 12,000m.
b. They are detached thin clouds.
c. They have a feathery appearance.
d. They are always white in colour.

2. Cumulus
a. Cumulus clouds are generally formed at a height of 4,000 – 7,000 m.
b. They look like cotton wool.
c. They exist in patches and can be seen dispersed here and there.
d. They have a flat base.

3. Stratus
a. Stratus clouds are layered clouds covering big portions of the sky.
b. These clouds are usually formed due to the mixing of air masses with various temperatures or
due to loss of heat.

3. Nimbus
a. Nimbus clouds form at middle levels or very near to the surface of the earth.
b. They are usually found in black or dark gray colour.
c. These are very dense and opaque to the rays of the sun.
d. Occasionally, the clouds are so low that they seem to touch the ground.
e. These clouds are shapeless masses of thick vapour.
A combination of these four types can give rise to the following types of clouds:

A. High clouds
1. Cirrus
2. Cirrostratus
3. Cirrocumulus

B. Middle clouds
1. Altostratus
2. Altocumulus

C. Low clouds
1. Stratocumulus
2. Nimbostratus
www.achieveias.co.in, YouTube Channel: http://youtube.com/c/AchieveIAS Telegram Channel: http://t.me/Achieve_Ias,
Mail: achieveias21@gmail.com, Contact Number: 8968920720

D. Clouds with extensive vertical development


1. Cumulus
2. Cumulonimbus

7. Consider the following statements regarding the atmospheric pressure:


1. The weight of the column of air at a given place and time is called air pressure or atmospheric
pressure.
2. Atmospheric pressure is measured by an instrument called Thermometer.
3. Atmospheric pressure is measured as force per unit area and the unit used for measuring
pressure is called millibar.

Which of the above statements is/are correct?


a. 2 only b. 1 and 3 c. 2 and 3 d. 1, 2 and 3

Answer: B

Explanation: The atmosphere is held on the earth by the gravitational pull of the earth. A column
of air exerts weight in terms of pressure on the surface of the earth. The weight of the column of air
at a given place and time is called air pressure or atmospheric pressure. Atmospheric pressure is
measured by an instrument called barometer. Nowadays, Fortin’s barometer and Aneroid barometer
I are commonly used for measuring air pressure. Atmospheric pressure is measured as force per
unit area. The unit used for measuring pressure is called millibar. Its abbreviation is ‘mb’. One
millibar is equal to the force of one gram per square centimetre approximately.

8. Which of the following weather conditions are associated with cold front?
1. The cold air mass is advancing and undercutting the warm air mass.
2. The warm air mass is advancing and over-riding the cold air mass.
3. Rapid ascent of warm moist air at the front causes rapid cooling and condensation.
4. Development of nimbus clouds.

Select the correct answer from the codes given below:


a. 1 and 3 b. 2 and 4 c. 1, 3 and 4 d. 2, 3 and 4

Answer: A

Explanation: In a cold front, the cold air mass will be advancing and under cutting the warm air.
The gradient of the cold front is usually much steeper than that of the warm front and the rapid
ascent of warm moist air at the front causes rapid cooling, condensation and the development of
cumulonimbus clouds. In a warm front, the warm air mass will be advancing and overriding the cold
air.

9. Which of the following is/are true about thunder storms?


1. Thunderstorms are of short duration.
2. Thunderstorms occur over short area and are violent.
3. A thunderstorm is a well-grown cumulonimbus cloud producing thunder and lightning.
www.achieveias.co.in, YouTube Channel: http://youtube.com/c/AchieveIAS Telegram Channel: http://t.me/Achieve_Ias,
Mail: achieveias21@gmail.com, Contact Number: 8968920720

Select the correct answer from the following codes


a. 1 Only b. 1 and 2 Only c. 2 and 3 Only d. 1, 2 and 3

Answer: D

Explanation: Other severe local storms are thunderstorms and tornadoes. They are of short
duration, occurring over a small area but are violent. Thunderstorms are caused by intense
convection on moist hot days. A thunderstorm is a well-grown cumulonimbus cloud producing
thunder and lightning.

10. Less dew is formed on cloudy nights because:


A. Clouds absorb the falling dew
B. Clouds scatter moisture
C. On cloudy nights the radiation takes place very slowly
D. On cloudy nights the radiation takes place very quickly

Answer: C

Explanation:
1. Dew: When the moisture is deposited in the form of water droplets on cooler surfaces of solid
objects (rather than nuclei in air above the surface) such as stones, grass blades and plant leaves,
it is known as dew. The ideal conditions for its formation are clear sky, calm air, high relative
humidity, and cold and long nights. For the formation of dew, it is necessary that the dew point is
above the freezing point.
2. White Frost: Frost forms on cold surfaces when condensation takes place below freezing point
(0° C), i.e. the dew point is at or below the freezing point. The excess moisture is deposited in the
form of minute ice crystals instead of water droplets. The ideal conditions for the formation of white
frost are the same as those for the formation of dew, except that the air temperature must be at or
below the freezing point.
3. Fog: When the temperature of an air mass containing a large quantity of water vapour falls all of
a sudden, condensation takes place within itself on fine dust particles.So, the fog is a cloud with its
base at or very near to the ground. Because of the fog and mist, the visibility becomes poor to zero.In
urban and industrial centers smoke provides plenty of nuclei which help the formation of fog and
mist. Such a condition when fog is mixed with smoke, is described as smog (will be discussed in
detail in next post). [Related Question Asked in Mains 2015: Mumbai, Delhi and Kolkata are the
three mega cities of the country but the air pollution is much more serious problem in Delhi as
compared to the other two. Radiation fog results from radiation, cooling of the ground and adjacent
air. These fogs are not very thick. Usual in winters. Fogs formed by condensation of warm air when
it moves horizontally over a cold surface, are known as advectional fog. These fogs are thick and
persistent. Occurs over warm and cold water mixing zones in oceans. Frontal or precipitation fog is
produced due to convergence of warm and cold air masses where warm air mass is pushed under
by the heavier cold air mass. Precipitation in the warm air mass condenses to produce fog at the
boundary of the two air masses. These are called frontal or precipitation fog. In fog visibility is less
than one kilometre.
www.achieveias.co.in, YouTube Channel: http://youtube.com/c/AchieveIAS Telegram Channel: http://t.me/Achieve_Ias,
Mail: achieveias21@gmail.com, Contact Number: 8968920720

ACHIEVE IAS GEOGRAPHY MCQ SERIES, DAY 30, SOLUTIONS


1. Which of the following statement is true regarding the ebb and flow of tides?

a. The time between the high tide and low tide, when the water level is falling, is called the flow.
b. The time between the low tide and high tide, when the tide is rising, is called the ebb.
c. Both (a) and (b)
d. None of the above

Answer: D

Explanation: Ebb and flow are two phases of the tide or any similar movement of water. The ebb
is the outgoing phase when the tide drains away from the shore, and the flow is the incoming phase
when the water rises again. They are also called as ebb and flood and flood drain of the tide or
similar movement of the water. Tidal currents occur in conjunction with the rise and fall of the tide.
The vertical motion of the tides near the shore causes the water to move horizontally, creating
currents. When a tidal current moves toward the land and away from the sea, it floods. When it
moves toward the sea away from the land, it ebbs. These tidal currents that ebb and flood in opposite
directions are called rectilinear or reversing currents.

2. Which of the following names of the positions moon, sun and earth are correctly matched?
1. Perihelion- When the earth is closest to the sun
2. Perigee- when the moon is closest to the earth
3. Apogee- When the earth is farthest from the sun

Which of the above statements is true?


a. 1 and 2 b. 2 and 3 c. 1 and 3 d. 1, 2 and 3

Answer: A

Explanation: Apogee and perigee refer to the distance from the Earth to the moon. Apogee is the
farthest point from the earth. Perigee is the closest point to the earth and it is in this stage that the
moon appears larger. Looking at the moon in the sky without anything to compare it to, you wouldn't
notice any size difference. But the difference in size can in fact be quite significant.
The perihelion is the point in the orbit of a celestial body where it is nearest to its orbital focus,
generally a star. It is the opposite of aphelion, which is the point in the orbit where the celestial body
is farthest from its focus.

3. Which of the following factors influence the movement of ocean water?


1. Temperature
2. Salinity of Ocean
3. Density of Ocean Water
4. Wind System
5. Coriolis Force

Choose the correct codes:


a. 1, 2 and 3 b. 2 and 3 c. 1, 2, 3 and 5 d. All of the above

Answer: D
www.achieveias.co.in, YouTube Channel: http://youtube.com/c/AchieveIAS Telegram Channel: http://t.me/Achieve_Ias,
Mail: achieveias21@gmail.com, Contact Number: 8968920720

Explanation: The ocean water is dynamic. Its physical characteristics like temperature, salinity,
density and the external forces like of the sun, the moon and the winds influence the movement of
ocean water. The horizontal and vertical motions are common in ocean water bodies. The horizontal
motion refers to the ocean currents and waves. The vertical motion refers to tides.

4. Consider the following statements


1. Nitrogen cannot be utilised by any species directly in gaseous form.
2. Apart from certain soil microorganism, certain marine animals can also do nitrogen fixing.

Select the correct answer from the following codes


a. 1 Only b. 2 Only c. Both 1 and 2 d. Neither 1 nor 2

Answer: B

Explanation: Only a few types of organisms like certain species of soil bacteria and blue green
algae are capable of utilising it directly in its gaseous form. Nitrogen can also be fixed in the
atmosphere by lightning and cosmic radiation. In the oceans, some marine animals can fix it.

5. Which of the following is/are not principal geochemical component of biosphere?


1. Hydrogen
2. Nitrogen
3. Carbon

Select the correct answer from the following codes


a. 1 Only b. 1 and 2 Only c. 2 and 3 Only d. 1, 2 and 3

Answer: D

Explanation: The biosphere, (from Greek bios = life, sphaira, sphere) is the layer of the planet Earth
where life exists. This layer ranges from heights of up to ten kilometres above sea level, used by
some birds in flight, to depths of the ocean such as the Puerto Rico trench, at more than 8 kilometres
deep. These are the extremes; however, in general the layer of the Earth containing life is thin: the
upper atmosphere has little oxygen and very low temperatures, while ocean depths greater than
1000 m are dark and cold. In fact, it has been said that the biosphere is like the peel in relation to
the size of an apple. The development of the term is attributed to the English geologist Eduard Suess
(1831-1914) and the Russian physicist Vladimir I. Vernadsky (1863-1945). The biosphere is one of
the four layers that surround the Earth along with the lithosphere (rock), hydrosphere (water) and
atmosphere (air) and it is the sum of all the ecosystems.

The biosphere is unique. So far there has been no existence of life elsewhere in the universe. Life
on Earth depends on the sun. Energy, provided as sun light, is captured by plants, some bacteria
and protists, in the marvellous phenomenon of photosynthesis. The captured energy transforms
carbon dioxide into organic compounds such as sugars and produces oxygen. The vast majority of
species of animals, fungi, parasitic plants and many bacteria depend directly or indirectly on
photosynthesis. Other than carbon, oxygen, nitrogen and hydrogen being the principal geochemical
www.achieveias.co.in, YouTube Channel: http://youtube.com/c/AchieveIAS Telegram Channel: http://t.me/Achieve_Ias,
Mail: achieveias21@gmail.com, Contact Number: 8968920720

components of the biosphere, many other minerals also occur as critical nutrients for plant and
animal life.

5. Which of the following is/are types of biomes?


1. Grassland
2. Aquatic
3. Altitudinal

Select the correct answer from the following codes


a. 1 Only b. 1 and 2 Only c. 2 and 3 Only d. 1, 2 and 3

Answer: D

Explanation: Biomes are large areas on Earth with similar conditions, such as similar climates and
similar living organisms. There are two main categories of biomes. Terrestrial biomes are usually
defined by the type of vegetation that is present. The major climatic factors contributing to the
vegetation types in these biomes are temperature and precipitation. Aquatic biomes are defined by
the type of water they contain. There are many different classification systems used to determine
biomes, each resulting in different numbers of biomes. There are five major biomes — forest, desert,
grassland, aquatic and altitudinal biomes. There are many different types of biomes on the Earth's
surface. Each biome is unique in that it has its own weather and temperature patterns, plant species,
and animal species. It is important to understand the uniqueness of each biome in order to
understand why certain animals and plants thrive in one area yet would not be able to survive in
another. Biomes are also very sensitive and each has special needs in order to maintain itself.
Although there are many types of biomes, they each fit into two categories: aquatic biomes and
terrestrial biomes.
1. Aquatic Biomes: Aquatic biomes are those that occur under water. This can be saltwater or
freshwater. There is saltwater in the ocean and this is where one would find the ocean biome and
the coral reef biome. The shallow part of the ocean that contains coral is a part of the coral reef
biome.
2. The freshwater biome: Includes areas of land covered in water that contains less than 1% of salt
water. This includes rivers, lakes, ponds, and wetlands such as swamps and marshes.
3. Terrestrial Biomes: Terrestrial biomes are those that occur on dry land. These biomes differ in
their climate, vegetation, and animal life. There are many types of terrestrial biomes but the main
biomes include tundra biome, desert biome, forest biome, and grassland biome.
4. The tundra biome is one of extreme weather conditions. Temperatures often remain very cold
and harsh. Only a few plants and animals can survive there.
5. The desert biome also has extreme weather conditions. There are two types of deserts, hot
deserts and cold deserts. Both desert types are very dry and receive very little rainfall.
6. The forest biome is characterized by its humidity and ample rainfall. The rain allows very tall trees
to grow there. There are two types of forest, tropical rainforest and temperate deciduous forest.
These forests are distinguishable by the types of plants and trees that grow there.
7. Grasslands have a lot of variety among its plants and animals. The plants that grow there include
many types of flowers. The soil in the grassland is very rich in nutrients so it is easy for many plants
to survive there. Even though the temperatures in the grassland biome remain pleasant for most of
the year, it does experience a dry season. This dry season sometimes results in a drought which
makes water scarce for the plants and animals there.
www.achieveias.co.in, YouTube Channel: http://youtube.com/c/AchieveIAS Telegram Channel: http://t.me/Achieve_Ias,
Mail: achieveias21@gmail.com, Contact Number: 8968920720

ACHIEVE IAS GEOGRAPHY MCQ SERIES, DAY 31, SOLUTIONS

1. Which of the following is/are true about ecology?


1. Study of interaction between living organisms
2. Study of interaction between living organisms and physical environment
3. Study of development and evolution of organisms

Select the correct answer from the following codes


a. 1 Only b. 1 and 2 Only c. 2 and 3 Only d. 1, 2 and 3

Answer: B

Explanation: The study of interactions between life forms (biotic) and the physical environment
(abiotic) is the science of ecology. Hence, ecology can be defined as a scientific study of the
interactions of organisms with their physical environment and with each other.

2. Equatorial counter-currents are unique because:


a. They flow in a direction opposite to that of the surface winds
b. They circulate from equator the poles uninterrupted.
c. Their travel speed is not affected by the ocean depth.
d. They are the only current to be sandwiched between two eastward-flowing ocean currents.

Answer: A

Explanation: Equatorial counter-currents are major surface flows that carry water eastward in the
Atlantic, Indian, and Pacific Oceans. They are located near the equator and are sandwiched
between two westward-flowing currents, the North Equatorial Current and the South Equatorial
Current. Equatorial counter-currents are unique, in that they flow in the opposite direction of the
surface winds. The other major surface currents in the tropics flow in the same direction as the
prevailing winds.

3. Sea waves have a great erosive force. Consider the following statements regarding the
functions of sea waves:
1. When the sea water loaded with rock fragments and sand attack the coastal rocks it is called
abrasion.
2. The rock particles present in the water hit against each other and break into progressively
smaller particles, the process is called attrition.
3. The rocks made up of limestone are subjected to solution action by the sea waves.

Which of the following above statements is true?


a. 1 and 2 b. 2 and 3 c. 1 and 3 d. 1, 2 and 3

Answer: D

Explanation: Sea waves have a great erosive force. In their role of an erosional agent, they perform
four functions. When the sea water loaded with rock fragments and sand attack the coastal rocks it
is called abrasion. The rock particles present in the water hit against each other and break into
progressively smaller particles. This process is called attrition. Thirdly the broadening of cracks and
www.achieveias.co.in, YouTube Channel: http://youtube.com/c/AchieveIAS Telegram Channel: http://t.me/Achieve_Ias,
Mail: achieveias21@gmail.com, Contact Number: 8968920720

crevices in the cliffs along the coast due to the attack of the sea waves is called the hydraulic action.
The rocks made up of limestone are subjected to solution action by the sea waves. All these
processes help in the formation of new features on the coastal margins.

4. Consider the following types of tides based on their frequency of occurrence and their
height:
1. In Diurnal tide, there is only one high tide and one low tide during each day.
2. Mixed tides generally occur along the west coast of South-East Asia.
3. In Semi-diurnal tide, successive high or low tides of the same height occur.

Which of the above statements is true?


a. 1 Only b. 1 and 3 c. 2 Only d. 1, 2 and 3

Answer: B

Explanation: Tides vary in their frequency, direction and movement from place to place and also
from time to time. Tides may be grouped into various types based on their frequency of occurrence
in one day or 24 hours or based on their height.
1. Tides based on Frequency Semidiurnal tide: The most common tidal pattern, featuring two high
tides and two low tides each day. The successive high or low tides are approximate of the same
height.
2. Diurnal tide: There is only one high tide and one low tide during each day. The successive high
and low tides are approximate of the same height.
3. Mixed tide: Tides having variations in height are known as mixed tides. These tides generally
occur along the west coast of North America and on many islands of the Pacific Ocean.
4. Tides based on the Sun, Moon and the Earth Positions: The height of rising water (high tide)
varies appreciably depending upon the position of sun and moon with respect to the earth.

5. In the atmosphere much of the oxygen is produced by, which of the following?
1. Transpiration
2. Respiration
3. Photosynthesis

Select the correct answer from the following codes


a. 1 Only b. 1 and 2 Only c. 2 and 3 Only d. 1, 2 and 3

Answer: B

Explanation: Much of oxygen is produced from the decomposition of water molecules by sunlight
during photosynthesis and is released in the atmosphere through transpiration and respiration
processes of plants.
www.achieveias.co.in, YouTube Channel: http://youtube.com/c/AchieveIAS Telegram Channel: http://t.me/Achieve_Ias,
Mail: achieveias21@gmail.com, Contact Number: 8968920720

ACHIEVE IAS GEOGRAPHY NCERT MCQ SERIES, DAY 32, SOLUTIONS

1. What do you mean by conservation of Biodiversity?


A. It envisions preservation, maintenance, sustainable use (conservation), recovery and
enhancement of the components of biological diversity.
B. It is the process of protecting an endangered plant or animal species in its natural habitat, either
by protecting or cleaning up the habitat itself, or by defending the species from predators.
C. It is the proper management of the biosphere by human beings in such a way that it gives
maximum benefits for the present generation and also develops its potential so as to meet the needs
of the future generations.
D. All of the above

Answer: D

Explanation: The dictionary defines biodiversity as ‘1. The number and variety of organisms found
within a specified geographic region. 2. The variability among living organisms on the earth,
including the variability within and between species and within and between ecosystems.’
Biological diversity – or biodiversity – is the term given to the variety of life on Earth. It is the variety
within and between all species of plants, animals and micro-organisms and the ecosystems within
which they live and interact. Biodiversity comprises all the millions of different species that live on
our planet, as well as the genetic differences within species. It also refers to the multitude of different
ecosystems in which species form unique communities, interacting with one another and the air,
water and soil. The biodiversity we see today is a result of 3.5 billion years of evolution.
Unfortunately, due to humanity’s over-exploitation of natural resources, our unsustainable
development and the resulting disturbances to the environment, we are undergoing the sixth
extinction crisis on this planet and degrading natural ecosystems at an unprecedented rate. It is
estimated that the current species extinction rate is between 1,000 and 10,000 times higher than it
would naturally be. Biodiversity conservation is about saving life on Earth in all its forms and keeping
natural ecosystems functioning and healthy. Conservation biology as a scientific discipline has
grown enormously over the past few decades and has increased our awareness and understanding
of the great extent to which humans depend on natural ecosystems and biodiversity. Conserving
biodiversity means ensuring that natural landscapes, with their array of ecosystems, are maintained,
and that species, populations, genes, and the complex interactions between them, persist into the
future. Benefits of conserving biodiversity:
 health of water, soil and air
 stock shelter
 prevention of salinity and soil acidification
 products such as timber, honey and medicine
 scenic amenity
 tourism
 a link to the past – preservation of the natural world
 support for living things including humans
 pest control
 prevention of weed invasion
 habitat for native species
 filtering of pollution.
 Resilience of the ecosystem in the face of climate change and other pressures
www.achieveias.co.in, YouTube Channel: http://youtube.com/c/AchieveIAS Telegram Channel: http://t.me/Achieve_Ias,
Mail: achieveias21@gmail.com, Contact Number: 8968920720

 Resilience against the marketplace


 Maintenance of natural processes
 Agriculture is totally dependent on ecosystem processes and functions provided by biodiversity
soil formation, nutrient cycling, pollination of cropsmaintaining water cycles
 protection from soil water loss and erosion
 Bacteria and fungi in a healthy understorey cause the constant breaking down and recycling of
nutrients
 Increased agricultural production
 Land value

2. Which of the following statement is related to the Ex situ conservation?


A. Established protected area network, with appropriate management practices, corridors to link
fragments restore degraded habitats within and outside.
B. Established botanical and zoological gardens conservation stands, bank of germ plasma pollen,
seed, seedlings, tissue culture, gene and DNA etc.
C. Agriculture is totally dependent on ecosystem processes and functions provided by biodiversity
soil formation, nutrient cycling, and pollination of crops maintaining water cycles
D. None of the above

Answer: B

Explanation: Ex situ conservation is a technique of conservation of biological diversity outside its


natural habitats, targeting all levels of biodiversity such as genetic, species, and ecosystems. Its
concept was developed earlier before its official adoption under the Convention on Biological
Diversity signed in 1992 in Rio de Janeiro. In general, ex situ conservation is applied as an additional
measure to supplement in situ conservation, which refers to conservation of biological diversity in
its natural habitats In some cases, ex situ management will be central to a conservation strategy
and in others it will be of secondary importance. Broadly, ex situ conservation includes a variety of
activities, from managing captive populations, education and raising awareness, supporting
research initiatives and collaborating with in situ efforts. It is used as valuable tools in studying and
conserving biological resources (plants, animals, and microorganisms) for different purposes
through different techniques such as zoos, captive breeding, aquarium, botanical gardens, and gene
banks.

3. Which of the following plant species are in the list of an endangered species in India?
A. Rauvolfia serpentine B. Sandal wood C. Cycas beddonei D. All of the above

Answer: D

Explanation: Endangered species, any species that is at risk of extinction because of a sudden
rapid decrease in its population or a loss of its critical habitat. Previously, any species of plant
or animal that was threatened with extinction could be called an endangered species.

4. Which of the following agency published the Red data book?


A. IUCN B. NEERI C. NWAP D. CITES

Answer: A
www.achieveias.co.in, YouTube Channel: http://youtube.com/c/AchieveIAS Telegram Channel: http://t.me/Achieve_Ias,
Mail: achieveias21@gmail.com, Contact Number: 8968920720

Explanation: What is The IUCN Red List?


Established in 1964, The International Union for Conservation of Nature’s Red List of Threatened
Species has evolved to become the world’s most comprehensive information source on the global
conservation status of animal, fungi and plant species. The IUCN Red List is a critical indicator of
the health of the world’s biodiversity. Far more than a list of species and their status, it is a powerful
tool to inform and catalyze action for biodiversity conservation and policy change, critical to
protecting the natural resources we need to survive. It provides information about range, population
size, habitat and ecology, use and/or trade, threats, and conservation actions that will help inform
necessary conservation decisions.

5. In which of the following places Royal Bengal Tiger found?


A. Gir forest B. Sunderban delta C. Godawari Delta D. Mahanadi Delta

Answer: B

Explanation: The Bengal tiger, also known as the Royal Bengal Tiger or the Indian tiger, is the
subspecies with the largest population. It is the national animal of India, place where its image is
part of the traditions and the culture. In Bangladesh, it is also the national animal present even in
the bills. The largest populations of Bengal tigers are in India, but there are some smaller groups in
Bangladesh, Nepal, and Bhutan. It may also be present in areas of China and Burma. An estimate
of the World Tiger Recovery Program indicates that there are about 440 individuals in Bangladesh,
about 155 in Nepal and about 75 in Bhutan. The approximate number of specimens is less than
2,500.However, the latest census of 2016 indicates that there are 106 tigers in Bangladesh, 103 in
Bhutan, 198 in Nepal and 2,226 in India. This information is from government sources and is not
confirmed by independent organizations. Bengal tiger habitats usually are tropical rainforests,
marshes, and tall grasses. The Bengal tiger is in danger of extinction, according to the International
Union for Conservation of Nature (IUCN). Although it is the most populated species, its numbers
keep decreasing rather than increasing, which has worried environmental organizations. The Bengal
tiger is in danger of extinction. The main threats to this species are two: poaching and conflicts with
humans over the territories. Poaching aim is to illegally trade the products obtained from tigers, such
as decorative objects or the active ingredient of “drugs” to cure various diseases, but which have no
proven efficacy. Their habitat, severely degraded by logging, and the invasion of humans in their
territories keeps reducing. When tigers attack domestic animals or even humans, they unleash the
wrath of people who in retaliation kill them. Fortunately, since the 1970s India began to establish
reserves through the Tiger Project that helped stabilize the Number of tigers. Also, the Indian Wildlife
Protection Act of 1972 empowers the government to take conservation measures. The Wildlife
Protection Society of India continues watching all allegations of tiger poaching. They have
confiscated large amounts of tiger parts and pelts, but it is often hard for them to get to the source
of who is responsible for killing and shipping them. Even though they arrest those caught with them,
there are more people involved in the process than they can catch.To get away with poaching though
many of these hunters have shifted to poisoning the Tigers. Then they are found already dead, and
they can’t be held responsible for killing them which is also the same practice that many farmers do
to protect their livestock.

6. Which one of the following is not included under In-situ conservation?


A. National Park B. Botanical Gardens
C. Wild Life Sanctuary D. Biosphere Reserve
www.achieveias.co.in, YouTube Channel: http://youtube.com/c/AchieveIAS Telegram Channel: http://t.me/Achieve_Ias,
Mail: achieveias21@gmail.com, Contact Number: 8968920720

Answer: B

Explanation: In-situ conservation is on site conservation or the conservation of genetic resources


in natural populations of plant or animal species, such as forest genetic resources in natural
populations of tree species. It is the process of protecting an endangered plant or animal species in
its natural habitat, either by protecting or cleaning up the habitat itself, or by defending the species
from predators. It is applied to conservation of agricultural biodiversity in agro forestry by farmers,
especially those using unconventional farming practices. In-situ conservation is being done by
declaring area as protected area. In India following types of natural habitats are being maintained:
1. National parks
2. Wildlife sanctuaries
3. Biosphere reserves
INDIA has over 600 protected areas, which includes over 90 national parks, over 500 animal
sanctuaries and 15 biosphere reserves.

7. The process of converting unconsolidated sediments into sedimentary rocks requires


which of the following
1. Compaction
2. Re-crystallisation
3. Cementation
4. Weathering

Select the correct answer using the codes given below


a. 1 and 2 b.2 and 3 c. 1, 2 and 3 d. All of these

Answer: C

Explanation: The process, by which loose sediment is hardened to rock are collectively called
Lithification. Once this happens, continued erosion and re-transport of the sediments become much
more difficult. The process, which involves in it are compaction, re-crystallisation and cementation.

8. What is Karewas?
A. Glacial deposits in Eastern Himalayas
B. Glacial deposits in Kashmir Himalayas
C. Alluvial deposits of western Himalayas
D. It denotes the general alluvial deposits

Answer: B

Explanation: Karewas are the glacial deposits in Kashmir valley. They are fertile plains they are
used for saffron cultivation, especially in pampore area of Kashmir valley.

9. Which of the following seismic waves is correctly matched with its respective property?
1. Secondary Waves surface seismic waves that cause horizontal shifting of the Earth during an
earthquake.
2. Love Wave can pass through all the mediums.
3. Rayleigh Waveà a type of surface acoustic wave that travel on solids
www.achieveias.co.in, YouTube Channel: http://youtube.com/c/AchieveIAS Telegram Channel: http://t.me/Achieve_Ias,
Mail: achieveias21@gmail.com, Contact Number: 8968920720

Choose the correct answer?


a. Only 1 b. Only 2 c. Only 3 d. All are correctly matched.

Answer: C

Explanation: Secondary waves (S-waves) are shear waves that are transverse in nature. S-waves
can travel only through solids, as fluids (liquids and gases) do not support shear stresses. S-waves
are slower than P-waves, and speeds are typically around 60% of that of P-waves in any given
material. Love waves are surface seismic waves that cause horizontal shifting of the Earth during
an earthquake. They form a distinct class, different from other types of seismic waves, such as P-
waves and S-waves (both body waves), or Rayleigh waves (another type of surface wave). Love
waves travel with a lower velocity than P- or S- waves, but faster than Rayleigh waves. Rayleigh
waves are part of the seismic waves that are produced on the Earth by earthquakes. They are a
type of surface acoustic wave that travel on solids. They can be produced in materials in many ways,
such as by a localized impact or by piezo-electric transduction, and are frequently used in non-
destructive testing for detecting defects.

10. Consider the following statements regarding the gravitation force of the earth:
1. Gravitation force is not same at different latitudes on the surface.
2. Gravitational force is greater near the poles and less at the equator.

Choose the incorrect statements?


a. Only 1 b. Only 2 c. Both 1 and 2 d. None of the above

Answer: D

Explanation: Gravity anomalies give us information about the distribution of mass of the material in
the crust of the earth. The gravity values differ according to the mass of material. The uneven
distribution of mass of material within the earth influences this value. The reading of the gravity at
different places differs from the expected values. Such a difference is called gravity anomaly.The
gravitation force is not the same at different latitudes on the surface. It is greater near the poles and
less at the equator. This is because of the distance from the centre at the equator being greater than
that at the poles.
www.achieveias.co.in, YouTube Channel: http://youtube.com/c/AchieveIAS Telegram Channel: http://t.me/Achieve_Ias,
Mail: achieveias21@gmail.com, Contact Number: 8968920720

ACHIEVE IAS GEOGRAPHY NCERT MCQ SERIES, DAY 33, SOLUTIONS

1. Which of the following is correct about India?


1. India has total area of 3.28 million sq. Km.
2. India has a coastline of 6100 km in mainland.
3. India has a total coastline of 7600 km including islands.

A. 1 and 2 only B. 2 and 3 only C. 1 and 3 only D. All of the above

Answer: D
Explanation: It is the seventh-largest country in the world, with a total area of 3,287,263 square
kilometres (1,269,219 sq mi). India measures 3,214 km (1,997 mi) from north to south and 2,933
km (1,822 mi) from east to west. It has a land frontier of 15,200 km (9,445 mi) and a coastline of
7,516.6 km (4,671 mi). India measures 3,214 km (1,997 mi) from north to south and 2,933 km
(1,822 mi) from east to west. It has a land frontier of 15,200 km (9,445 mi) and a coastline of
7,516.6 km (4,671 mi). India has a total coastline of 7516.6 km, out of which
mainland coastline consists of 5422.6 km and islands' coastline consists of 2094 km.

2. Which of the following cities has maximum northern latitude?


A. Ranchi B. Patna C. Jamshedpur D. Barddhaman

Answer: B

3. Which one of the following pairs of islands is separated from each other by the ‘Ten
Degree Channel’?
A. Andaman and Nicobar B. Nicobar and Sumatra
C. Maldives and Lakshadweep D. Sumatra and Java

Answer: A
Explanation: The Ten Degree Channel is a channel that separates the Andaman
Islands and Nicobar Islands from each other in the Bay of Bengal. The two sets of islands together
form the Indian Union Territory (UT) of Andaman and Nicobar Islands.

4. Which of the following state is not crossed by the Tropic of Cancer?


A. Rajasthan B. Chhattisgarh
C. Odisha D. Tripura

Answer: C

Explanation: The tropic of cancer passes through 8 Indian states. They areGujarat, Rajasthan,
Madhya Pradesh, Chhattisgarh, West Bengal, Jharkand, Tripura and Mizoram.

5. Which is the easternmost longitude of India?


A. 97° 25' E B. 77° 6' E
C. 68° 7' E D. 82° 32' E

Answer: C
www.achieveias.co.in, YouTube Channel: http://youtube.com/c/AchieveIAS Telegram Channel: http://t.me/Achieve_Ias,
Mail: achieveias21@gmail.com, Contact Number: 8968920720

Explanation: "97° 25' East" is the "eastern most longitude of India".


It is located in "Arunachal Pradesh". It is this longitude beyond which the territory of India doesn’t
extend. Longitudes are the imaginary lines that run from the "North Pole" to the "south pole".
Longitudes help in finding the time at a place. The "Prime meridian" which passes through
"Greenwich" is used for this purpose. Longitudes along with latitudes help in "determining the
position" of a place.

6. Which of the following country have common frontier with the Indian State like
Uttarakhand, Uttar Pradesh, Bihar, West Bengal and Sikkim?
A. China B. Nepal C. Bhutan D. Myanmar

Answer: B
Explanation: India is an ancient civilization but now it is a developing nation. It has shown
remarkable progress in various fields. India has an important position in South Asia and has 28
States and 9Union Territories. India shares its boundaries with Pakistan, Afghanistan, China,
Nepal, Bangladesh, Myanmar and Bhutan. The southern neighbors across the sea consist of the
two island countries, namely Maldives and Sri Lanka. Sri Lanka is separated from India by Pak
Strait, a narrow strip of sea. India always have and had strong geographical and historical links
with her neighbors. India stands apart from the rest of Asia. Uttarakhand, Uttar Pradesh, Bihar,
West Bengal and Sikkim have common frontiers with Nepal.
1) Nepal is one of the friendly nations of India and has no rivalry with India.
2) It is a peace-loving country like India and has no terrorist activities
3) It is a beautiful country with lots of tourism destinations

7. Which of the following the country does not share land boundary with India?
A. Bhutan B. Bangladesh C. Tajikistan D. Nepal

Answer: C
Explanation: Pakistan, China, Nepal, Bhutan, Bangladesh and Myanmar share a land border with
India. It might also share a border with Sri Lanka (the boundary runs over Adam’s Bridge and may
or may not run over one of its islands or shoals) and Afghanistan (Afghanistan shares a border
with Kashmir, which is claimed by both India and Pakistan, but it only borders the part that is
occupied by Pakistan, so India considers it sharing a border with it, but other countries do not). All
other countries (over 180) do not share a land border with India.

8. Which state of India shares the smallest border with China?


A. Uttarakhand B. Sikkim C. Himachal Pradesh D. Arunachal Pradesh

Answer: C
Explanation: Himachal Pradesh shares about 200 km border with China which is the smallest
among other sharing border states of India. China and India are the two neighbouring countries in
Asia. India shares 3488 km of border with China. Jammu and Kashmir, Himachal Pradesh, Sikkim,
Uttarakhand and Arunachal Pradesh Indian states shares border with China. Also, the Tibet
Autonomous region of China touched border with India. Jammu and Kashmir border shares 1597
km length with China, which is the longest.

9. Which state of India share longest border with Pakistan?


www.achieveias.co.in, YouTube Channel: http://youtube.com/c/AchieveIAS Telegram Channel: http://t.me/Achieve_Ias,
Mail: achieveias21@gmail.com, Contact Number: 8968920720

A. Jammu and Kashmir B. Rajasthan C. Gujarat D. Punjab

Answer: A
Explanation: Jammu and Kashmir, the state of India share longest border with Pakistan having a
length of 1222 km, followed by Rajasthan with 1170 km. Gujarat has 506 km, while Punjab has
425 km. The borders of Pakistan were drawn just after two days of partition that is on 17 August,
1947. Let us tell you that the Radcliffe Line was officially announced on August 17, 1947, a few
days after the independence of India and Pakistan. Indian states that share boundaries with
Pakistan are: Jammu and Kashmir, Rajasthan, Gujarat and Punjab.

10. What is the total length of India-Bangladesh border?


A. 4000 km B. 4018.50 km C. 4068 km D. 4096.70 km

Answer: D
Explanation: The longest border is shared by Bangladesh which is 4096.70 Km (2,545-mile).
India-Bangladesh border is the fifth longest border in the World. Indian states that share borders
with Bangladesh are: Assam, West Bengal, Mizoram, Meghalaya and Tripura. West Bengal
Indian state shares longest border between India and Bangladesh. Its length is 2216.70
km. Assam Indian state shares smallest border between India and Bangladesh. Its length is 263
km.
www.achieveias.co.in, YouTube Channel: http://youtube.com/c/AchieveIAS Telegram Channel: http://t.me/Achieve_Ias,
Mail: achieveias21@gmail.com, Contact Number: 8968920720

ACHIEVE IAS GEOGRAPHY NCERT MCQ SERIES, DAY 34, Solutions

1. Consider the following statements


1. Chotanagpur plateau extends into North eastern parts of India.
2. Karbi Anglong and the Meghalaya Plateau in the northeast are extensions of Peninsular block.

Select the correct answer from the following codes


a. 1 Only b. 2 Only c. Both 1 and 2 d. Neither 1 nor 2

Answer: B

Explanation: The Karbi Anglong and the Meghalaya Plateau in the northeast and Rajasthan in the
west are also extensions of this block. The north eastern parts are separated by the Malda fault in
West Bengal from the Chotanagpur plateau. Chota Nagpur, plateau in eastern India, in north-
western Chhattisgarh and central Jharkhand states. The plateau is composed of Precambrian rocks
(i.e., rocks more than about 540 million years old). Chota Nagpur is the collective name for
the Ranchi, Hazaribagh, and Kodarma plateaus, which collectively have an area of 25,293 square
miles (65,509 square km). Its largest division is the Ranchi Plateau, which has an average elevation
of about 2,300 feet (700 metres). The Chota Nagpur plateau in its entirety lies between the basins
of the Ganges (Ganga) and Son rivers to the north and the Mahanadi River to the south. Through
its centre, from west to east, runs the coal-bearing, faulted Damodar River valley. Numerous streams
have dissected the uplands into a peneplain (an area reduced almost to a plain by erosion) with
isolated hills. It is believed that due to the force exerted by the northeastward movement of the
Indian plate at the time of the Himalayan origin, a huge fault was created between the Rajmahal
hills and the Karbi-Meghalaya plateau. Later, this depression was filled up by the depositional
activity of numerous rivers. Today the Maghalaya and Karbi Anglong plateau remains detached from
the main Peninsular block. This area receives maximum rainfall from the South-West monsoon. It
is located in the north eastern plateau of India

2. Which of the following is/are features of Himalayas?


1. Weak and flexible geological structure
2. Interplay of exogenic and endogenic forces
3. Tectonic in origin

Select the correct answer from the following codes


a. Only 1 b. Only 1 and 2 c. Only 2 and 3 d. 1, 2 and 3

Answer: D

Explanation: The Himalayas along with other Peninsular mountains are young, weak and flexible
in their geological structure unlike the rigid and stable Peninsular Block. Consequently, they are
still subjected to the interplay of exogenic and endogenic forces, resulting in the development of
faults, folds and thrust plains. These mountains are tectonic in origin, dissected by fast-flowing
rivers which are in their youthful stage. The most characteristic features of the Himalayas are their
soaring heights, steep-sided jagged peaks, valley and alpine glaciers often of stupendous
size, topography deeply cut by erosion, seemingly unfathomable river gorges, complex geologic
structure, and series of elevational belts (or zones) that display different ecological associations of
flora, fauna, and climate. Viewed from the south, the Himalayas appear as a gigantic crescent with
www.achieveias.co.in, YouTube Channel: http://youtube.com/c/AchieveIAS Telegram Channel: http://t.me/Achieve_Ias,
Mail: achieveias21@gmail.com, Contact Number: 8968920720

the main axis rising above the snow line, where snowfields, alpine glaciers, and avalanches all
feed lower-valley glaciers that in turn constitute the sources of most of the Himalayan rivers. The
greater part of the Himalayas, however, lies below the snow line. The mountain-building process
that created the range is still active. As the bedrock is lifted, considerable stream erosion and
gigantic landslides occur. The Himalayan ranges can be grouped into four parallel longitudinal
mountain belts of varying width, each having distinct physiographic features and its own geologic
history. They are designated, from south to north, as the Outer, or Sub-, Himalayas (also called
the Siwalik Range); the Lesser, or Lower, Himalayas; the Great Himalaya Range (Great
Himalayas); and the Tethys, or Tibetan, Himalayas. Farther north lie the Trans-
Himalayas in Tibet proper. From west to east the Himalayas are divided broadly into three
mountainous regions: western, central, and eastern.

3. Match the following passes and mountain ranges.


1. Zoji La ----------- A.Pir Panjal
2. Photu la --------- B. Ladhakh
3. Banihal --------- C.Great Himalayas
4. Khardung la ------ D.Zaskar

a. 1.A,2.B,3.C,4.D b. 1.C,2.D,3.A,4.B
c. 1.B,2.C,3.D,4.A d. 1.D,2.A,3.B,4.C

Answer: B

Explanation:
1. Zoji La: Nestled on National highway 1 between Srinagar and Leh, Zoji La is an important
mountain pass.Around 3528 m (11, 649 ft) above the sea level, the pass separates the Kashmir
Valley from Drass Valley. It lies 9 km away from Sonamarg and is an important link between Ladakh
and Kashmir. This pass is considered to be the second highest pass after Fotu La on the Srinagar-
Leh National Highway. The pass mostly remains closed during the winter due to heavy snowfall. But
the Border Roads Organization (BRO) works day and night for smooth flow traffic throughout the
year. The Beacon Force is that unit of the BRO, which takes care of clearing and maintaining the
road during the winter season.At the time of the Indo-Pakistani War in 1947, this pass was seized
by the Pakistani raiders in 1948 while they were in the wake of capturing Ladakh. Later on
1st November, Zoji pass was captured by Indian forces under Operation Bison.
Once you reach here, you are going to savor an amazing view of the mighty Himalayan mountains,
which makes the ride worth it. The pass remains disconnected from the rest of the world for almost
six months in a year. This 434 km long highway stretch is the lifeline of Ladakh. After the Kargil war
in 1999, the government paced-up the construction of the road due to its strategic importance. In
2018, the Government of India approved the Zoji La tunnel project. When completed, it will the
longest bi-directional in Asia. It will further reduce the time to cross the pass from more than 3 hours
to just 15 minutes.
2. Photu la: Fotu La pass on the Srinagar-Leh highway of the Himalayan Zanskar Range stands at
a height of of 13,478ft (4,108m) above the sea level. This mountain pass is referred to as the highest
point on the highway, going beyond the well-known Zoji La. The drive through the pass is a beautiful
journey where you can stop to get an amazing view of the snow-clad mountain ranges.
Along with Namika La, the Fotu La is one of two high mountain passes between the regions of Leh
and Kargil. When one moves eastwards, the highway starts to slide down towards the town of
www.achieveias.co.in, YouTube Channel: http://youtube.com/c/AchieveIAS Telegram Channel: http://t.me/Achieve_Ias,
Mail: achieveias21@gmail.com, Contact Number: 8968920720

Lamayuru, after one crosses the Fotu La. On the pass is located the relay station of Prasar Bharati
television that serves Lamayuru.
3. Banihal Pass: Banihāl Pass, pass in the Pīr Panjāl Range in the Indian-held sector of the state
of Jammu and Kashmir in the northern part of the Indian subcontinent. Banihāl—a name that in
Kashmīrī means “blizzard”—lies at an altitude of 9,290 ft (2,832 m) in the Doda district. It forms the
main gateway to the Vale of Kashmir from the Indian plains. The Jammu–Srīnagar road enters the
pass through the Jawahar Tunnel, which is occasionally blocked by snow in winter. Goods formerly
crossed the pass on the backs of porters, who made the journey in a day.
4. Khardung La: Situated at an altitude of 18,379 ft. above the sea level, Khardung La makes one
feel on the top of the world. It was once believed to be the highest motorable pass in the world, but
facts prove it is among the highest motorable passes in India. This mountain pass in the Ladakh
region of Jammu and Kashmir was prepared for motors in 1976 and opened in 1988 for the public.
Khardung Pass (as ‘La’ implies ‘pass’ in Tibetan Language) is the gateway to the
remote Nubra and Shyok valleys. The mountain pass holds a special significance as it is the way to
carry supplies to the Siachen Glacier. It offers astonishing views of the twisting roads up the
mountain and the awesome valley. Alternatively known as the ‘Pass of Lower Castle’, Khardung La
is the inspiration of motorbike, automobile and mountain biking expeditions. This makes it adventure
aficionados’ paradise indeed.

4. Arrange the following Himalayan sub-divisions from west to east


1. Kashmir Himalayas
2. Himachal Himalayas
3. Darjeeling Himalayas
4. Arunachal Himalayas

a. 1,2,3,4 b. 4,3,2,1 c. 1,2,4,3 d. 2,3,4,1

Answer: A

5. Which of the following coastal plain is located between the Sahyadri and the Arabian Sea?
A. Gujarat Coastal Plain B. West Coastal Plain
C. Karnataka Coastal Plain D. East Coastal Plain

Answer: B

Explanation: West Coastal Plain is located between the Sahyadri and the Arabian Sea. It is mainly
characterised by sandy beaches, coastal sand dunes, mud flats, lagoons, alluvial tracts along rivers,
estuary, laterite platforms and residual hills. The Western Ghats Coastal Plain extends from Surat
to Kanyakumari which is divided into four parts:The Gujarat Plain- Coastal area of Gujarat; The
Konkan Plain- Between Daman and Goa; The Kannad Plain- Between Goa and Mangalore; The
Malabar Plain- Between Mangalore and Kanyakumari. The Western Ghats Coastal Plain extends
from Surat to Kanyakumari which is divided into four parts:
• The Gujarat Plain- Coastal area of Gujarat
• The Konkan Plain- Between Daman and Goa.
• The Kannad Plain- Between Goa and Mangalore.
• The Malabar Plain- Between Mangalore and Kanyakumari
They are actually submerged coastal plain. It is a narrow belt and provides natural conditions for the
development of ports and harbours because of this submergence. Kandla, Mazagaon, JLN port
www.achieveias.co.in, YouTube Channel: http://youtube.com/c/AchieveIAS Telegram Channel: http://t.me/Achieve_Ias,
Mail: achieveias21@gmail.com, Contact Number: 8968920720

Navha Sheva, Marmagao, Mangalore, Cochin, etc. are the important natural ports located along the
west coast. Extending from the Gujarat coast in the north to the Kerala coast in the south, the
western coast may be divided into following divisions – the Kachchh and Kathiawar coast in Gujarat,
Konkan coast in Maharashtra, Goan coast and Malabar coast in Karnataka and Kerala respectively.
The western coastal plains are narrow in the middle and get broader towards north and south. The
rivers flowing through this coastal plain do not form any delta. The Malabar coast has got certain
distinguishing features in the form of ‘Kayals’ (backwaters), which are used for fishing, inland
navigation and also due to its special attraction for tourists. Every year the famous Nehru Trophy
Vallamkali (boat race) is held in Punnamada Kayal in Kerala.

6. The 9 degree channel is located on which of the following Islands?


A. Arabian Sea Islands B. Bay of Bengal Islands
C. Offshore Islands D. None of the above

Answer: A

Explanation: The Arabian Sea Islands comprise 36 islands of Lakshadweep Group. The
southernmost island, i.e. Minicoy is separated from the rest of the group by the 9 degree channel.
Nine Degree Channel separates Lakshadweep and Minicoy. The Nine Degree Channel is a channel
in the Indian Ocean between the Laccadive Islands of Kalpeni and Suheli Par, and Maliku Atoll.
These two subgroups of islands, together with the Amindivi Subgroup, form the Indian Union
Territory (UT) of Lakshadweep.

7. Consider the following statement(s) is/are related to the Thar Desert


1. The desert continues into Pakistan as the Cholistan Desert.
2. Most of the Thar Desert is situated in Rajasthan, covering 61% of its geographic area.

Which of the above statement(s) is/are correct?


A. Only 1 B. Only 2 C. Both 1 and 2 D. Neither 1 nor 2

Answer: C

Explanation: Thar Desert, also called Great Indian Desert, arid region of rolling sand hills on the
Indian subcontinent. It is located partly in Rajasthan state, northwestern India, and partly
in Punjab and Sindh (Sind) provinces, eastern Pakistan. The Thar Desert covers some 77,000
square miles (200,000 square km) of territory. It is bordered by the irrigated Indus River plain to the
west, the Punjab Plain to the north and northeast, the Aravalli Range to the southeast, and the Rann
of Kachchh to the south. The subtropical desert climate there results from persistent high
pressure and subsidence at that latitude. The prevailing southwest monsoon winds that bring rain
to much of the subcontinent in summer tend to bypass the Thar to the east. The name Thar is
derived from thul, the general term for the region’s sand ridges.
TheThar’desert sands overlie Archean (early Precambrian) gneiss (metamorphic rocks formed
between 4 billion and 2.5 billion years ago), Proterozoic (later Precambrian) sedimentary
rocks (formed about 2.5 billion to 541 million years ago), and more-recent alluvium (material
deposited by rivers). The surface consists of aeolian (wind-deposited) sand that has accumulated
over the past 1.8 million years.

8. Consider the following statement(s) is/are related to the Deccan Plateau:


www.achieveias.co.in, YouTube Channel: http://youtube.com/c/AchieveIAS Telegram Channel: http://t.me/Achieve_Ias,
Mail: achieveias21@gmail.com, Contact Number: 8968920720

1. It is a large triangular plateau, bounded by the Vindhyas to the north and flanked by the Eastern
and Western Ghats.
2. It slopes gently from west to east and gives rise to several peninsular rivers such as the Godavari,
the Krishna, the Kaveri and the Mahanadi which drain into the Bay of Bengal.

Which of the above statement(s) is/are correct?


A. Only 1 B. Only 2 C. Both 1 and 2 D. Neither 1 nor 2

Answer: C

Explanation: Deccan, the entire southern peninsula of India south of the Narmada River, marked
centrally by a high triangular tableland. The name derives from the Sanskrit daksina (“south”).
The plateau is bounded on the east and west by the Ghats, escarpments that meet at the plateau’s
southern tip. Its northern extremity is the Satpura Range. The Deccan’s average elevation is about
2,000 feet (600 metres), sloping generally eastward. Its principal rivers—the Godavari, Krishna,
and Kaveri (Cauvery)—flow from the Western Ghats eastward to the Bay of Bengal. The plateau’s
climate is drier than that on the coasts and is arid in places.

9. Consider the following statement(s) is/are related to the Karakoram mountain ranges
1.The world's second highest mountain peak K2 (Godwin Austen), which has a height of 8611
meters, belongs to this chain of mountains.
2. It rises from the Pamir Knot in the north-west and stretch towards southeast up to the Indus gorge
in Jammu and Kashmir.

Which of the above statement(s) is/are correct?


A. Only 1 B. Only 2 C. Both 1 and 2 D. Neither 1 nor 2

Answer: C

Explanation: Karakoram Range, Chinese (Pinyin) Karakorum Shan or (Wade-Giles


romanization) K’a-la-k’un-lun Shan, great mountain system extending some 300 miles (500 km) from
the easternmost extension of Afghanistan in a south easterly direction along the watershed between
Central and South Asia. Found there are the greatest concentration of high mountains in the world
and the longest glaciers outside the high latitudes. The Karakorams are part of a complex of
mountain ranges at the centre of Asia, including the Hindu Kush to the west, the Pamirs to the
northwest, the Kunlun Mountains to the northeast, and the Himalayas to the southeast. The borders
of Tajikistan, China, Pakistan, Afghanistan, and India all converge within the Karakoram system,
giving this remote region great geopolitical significance. The name “Kurra-koorrum,” a rendering of
the Turkic term for “Black Rock” or “Black Mountain,” appeared in early 19th-century English
writings. The Karakorams consist of a group of parallel ranges with several spurs. Only the central
part is a monolithic range. The width of the system is about 150 miles (240 km); the length is
increased from 300 miles (500 km) to 500 miles (800 km) if its easternmost extension—the Chang
Chenmo (Chinese: Qiangchenmo) and Pangong ranges of the Plateau of Tibet—is included. The
system occupies about 80,000 square miles (207,000 square km). The average elevation of
mountains in the Karakorams is about 20,000 feet (6,100 metres), and four peaks exceed 26,000
feet (7,900 metres); the highest, K2 (Mount Godwin Austen), at 28,251 feet (8,611 metres), is the
second highest peak in the world. Because of their great height, the Karakorams exhibit heavy
glaciation, particularly on the southern, more humid slopes. Glaciers of the central, highest
www.achieveias.co.in, YouTube Channel: http://youtube.com/c/AchieveIAS Telegram Channel: http://t.me/Achieve_Ias,
Mail: achieveias21@gmail.com, Contact Number: 8968920720

mountains include Hispar, Chogo Lungma, Braldu, Biafo, Baltoro with its famous Concordia junction,
and Siachen (which is some 45 miles [70 km] long). The snow line on the southern slopes of the
Karakorams lies at an elevation of 15,400 feet (4,700 metres); glaciers extend down to 9,500 feet
(2,900 metres). On the northern slopes the corresponding elevations are 19,400 feet (5,900 metres)
and 11,600 feet (3,500 metres), respectively. Often, glaciers combine to form complex glacial
systems occupying not just valleys but entire watersheds. Seasonal thawing of the glaciers gives
rise to serious floods on the southern slopes. Traces of ancient glaciation are evident at elevations
as low as 8,500 feet (2,600 metres) and 2,800 feet (850 metres) in the Indus River valley. The
Karakorams serve as a watershed for the basins of the Indus and Yarkand rivers. The formation of
river channels, for the most part, occurs in the high-elevation zone, where the melted waters of
seasonal and perpetual snows and glaciers feed the rivers. Suspended pulverized stone, or rock
flour, makes glacial meltwater opaque. Rock flour and eroded material from the mountain channels
give the Indus the highest suspended sediment load of any major river. Groundwater accumulates
in the rocky talus and contributes to the flow throughout the year.

10. Consider the following statement(s) is/are related to the Ladakh range
1. It is situated to the north of the Indus Tsangpo Suture Zone (ITSZ) and south of Karakoram,
between River Indus and Shyok.
2. The highest peak is Mt. Rakaposhi (steepest peak in the world).

Which of the above statement(s) is/are correct?


A. 1 Only B. 2 Only C. Both 1 and 2 D. Neither 1 nor 2

Answer: C

Explanation: Ladakh Range, southeastern extension of the Karakoram Range, south-central Asia.
The range extends southeastward for some 230 miles (370 km) from the mouth of the Shyok River in
the Northern Areas (the sector of the Kashmir region administered by Pakistan), through Jammu
and Kashmir state (the portion of Kashmir administered by India), to the border with
the Tibet Autonomous Region of China. With a crest line of about 20,000 feet (6,100 metres), the
range parallels the northeast bank of the Indus River. The Ladakh Range is composed of granite
rocks of the Ladakh batholith, which is bounded by the Shyok suture zone to the north and the Indus
suture zone to the south. The climate is semiarid, with maximum summer temperatures exceeding
86 °F (30 °C) along the lower slopes and mean monthly temperatures in winter well below 32 °F (0
°C) at elevations above about 13,000 feet (4,000 metres). Vegetation is sparse, limited mainly to
short grasses and scrub. The Deosai Mountains, located southwest of the Indus River in Pakistani-
administered Kashmir, are sometimes considered part of the range. The Shyok River rises near the
Karakoram Pass. The region comprising the valley of Shyok and Nubra Rivers is known as Nubra.
The Karakoram Range in Ladakh is not as mighty as in Baltistan. The massifs to the north and east
of the Nubra-Siachen valley include the Apsarasas group (highest point 7245 m), the Rimo group
(highest point 7385 m) and the Teram Kangri group (highest point 7464 m), together with
Mamostong Kangri (7526m) and Singhi Kangri (7202 m.) North of the Karakoram lie the Kun Lun
Mountains. Thus, between Leh and eastern Central Asia, there is a triple barrier: the Ladakh Range,
the Karakoram range, and the Kun Lun. Nevertheless, a major trade route was established between
Leh and Yarkand.Sedimentary rock formations in Ladakh. The enormous mass of the Himalayas
creates a rain shadow, denying entry to the moisture-laden clouds of the Indian monsoon. Ladakh
is thus, a high altitude desert. The main source of water is the winter snowfall on the mountains. The
regions on the north flank of the Himalayas—Dras, the Suru valley and Zanskar—experience heavy
www.achieveias.co.in, YouTube Channel: http://youtube.com/c/AchieveIAS Telegram Channel: http://t.me/Achieve_Ias,
Mail: achieveias21@gmail.com, Contact Number: 8968920720

snowfall and remain virtually cut off from the rest of the country for several months in the year.
Summers are short, although long enough to grow crops. The proportion of oxygen is less than in
many other places at a comparable altitude because of lack of vegetation. There is little moisture to
temper the effects of rarefied air.Naked barley (Urdu: grim), normal barley and wheat are the staple
crops all over Ladakh, along with mustard (for oil), lentils and other pulses, and vegetables. The
extreme limit of cultivation is at Korzok near Tso Moriri (lake), at 4560 m, said to be among the
highest fields in the world.
www.achieveias.co.in, YouTube Channel: http://youtube.com/c/AchieveIAS Telegram Channel: http://t.me/Achieve_Ias,
Mail: achieveias21@gmail.com, Contact Number: 8968920720

ACHIEVE IAS GEOGRAPHY MCQ SERIES, DAY 35, SOLUTIONS

1. Consider the following statement(s) is/are related to the Indus River System
1. It is known as Singi Khamban or the Lion’s mouth in Tibet.
2. It originates from a glacier near Bokhar Chu in the Kailash Mountain range.

Which of the above statement(s) is/are correct?


A. Only 1 B. Only 2 C. Both 1 and 2 D. Neither 1 nor 2

Answer: C

Explanation: The Indus System is one of the largest river basins of the world. It is also known as
the Sindhu, is the westernmost of the Himalayan rivers in India. It originates from a glacier near
Bokhar Chu (31°15' N latitude and 81°40' E longitude) in the Tibetan region at an altitude of 4,164
m in the Kailash Mountain range. It receives a number of Himalayan tributaries such as the Shyok,
the Gilgit, the Zaskar, the Hunza, the Nubra, the Shigar, the Gasting and the Dras. The Indus System
is one of the largest river basins of the world, covering an area of 11, 65,000 sq. km (in India it is
321, 289 sq. km and a total length of 2,880 km (in India 1,114 km). The Indus also known as the
Sindhu. It originates from a glacier near Bokhar Chu (31°15' N latitude and 81°40' E longitude) in
the Tibetan region at an altitude of 4,164 m in the Kailash Mountain range. In Tibet, it is known
as ‘Singi Khamban; or Lion’s mouth. After flowing in the northwest direction between the Ladakh
and Zaskar ranges, it passes through Ladakh and Baltistan. It cuts across the Ladakh range, forming
a spectacular gorge near Gilgit in Jammu and Kashmir. It enters into Pakistan near Chillar in the
Dardistan region.

Tributaries of Indus River: The Indus receives a number of Himalayan tributaries such as
the Shyok, the Gilgit, the Zaskar, the Hunza, the Nubra, the Shigar, the Gasting and the Dras. It
finally emerges out of the hills near Attock where it receives the Kabul River on its right bank. The
other important tributaries joining the right bank of the Indus are the Khurram, the Tochi, the Gomal,
the Viboa and the Sangar. They all originate in the Sulaiman ranges. The river flows southward and
receives ‘Panjnad’ a little above Mithankot. The Panjnad is the name given to the five rivers of
Punjab, namely the Satluj, the Beas, the Ravi, the Chenab and the Jhelum. It finally discharges into
the Arabian Sea, east of Karachi.

2. Consider the following statement(s) is/are related to the drainage pattern


1. Radial drainage patterns form when rivers originate from a hill and flow in all directions. For
example, the rivers originating from the Amarkantak.
2. Centripetal drainage pattern is formed when rivers discharge their waters from all directions into
a lake or a depression. For example, Loktak lake in Manipur.
3. Trellis drainage pattern is formed when the primary tributaries of main rivers flow parallel to
each other and secondary tributaries join them at right angles. For example, rivers in the upper
part of the Himalayan region.

Which of the above statement(s) is/are correct?


A. 1 Only B. 2 Only C. Both 1 and 2 D. 1, 2 and 3

Answer: D
www.achieveias.co.in, YouTube Channel: http://youtube.com/c/AchieveIAS Telegram Channel: http://t.me/Achieve_Ias,
Mail: achieveias21@gmail.com, Contact Number: 8968920720

Explanation: 1. Radial Pattern: It is a pattern characterised by out flowing rivers, away from a
central point, analogous with the spokes of a wheel. It tends to develop on the flanks of a dome or
a volcanic cone. A good example of a radial drainage pattern is provided by the rivers originating
from the Amarkantak Mountain. Rivers like Narmada, Son and Mahanadi originating from
Amarkantak Hills flow in different directions and are good examples of radial pattern. This pattern is
also found in the Girnar Hills (Kathiwar, Gujarat), and Mikir Hills of Assam.
2. Trellis Drainage: It is a rectangular pattern formed where two sets of structural controls occurs
at right angles. In a trellis pattern, the river forms a net like system and the tributaries flow roughly
parallel to each other. The old folded mountains of the Singhbhum (Chotanagpur Plateau) have
drainage of trellis pattern.
3. Centripetal or inland drainage pattern: is opposite to the radial drainage pattern because it is
characterized by the streams which converge at a point which is generally a depression or a basin.
This pattern is formed by a series of streams which after emerging from surrounding uplands
converge in a central low land which may be a depression, or a basin or a crater lake.The
Kathmandu valley of Nepal presents an ideal example of centripetal drainage pattern wherein the
tributary streams of the Baghmati converge in the tectonically formed circular basin. The depression
formed at the top of Raigarh Dome in the Lower Chambal Basin has given birth to centripetal
drainage pattern.

3. The correct sequence of the west flowing rivers in the peninsular India from north to
south is.
A. Sabarmati, Mahi, Kalinadi, Periyar, Bharathapuzha
B. Mahi, Sabarmati, Kalinadi, Periyar, Bharathapuzha
C. Sabarmati, Mahi, Kalinadi, Bharathapuzha, Periyar
D. Mahi, Sabarmati, Kalinadi, Bharathapuzha, Periyar

Answer: C

4. Match the following items in List-I with the items in List-II with the help of codes given
below:
List-I (River) List-II (Meeting Place)
A. Bhagirathi-Alaknanda 1. Joshimath
B. Dhauli-Vishnuganga 2. Devprayag
C. Alaknanda-Mandakini 3. Rudraprayag
D. Alaknanda-Pinder 4. Karnaprayag

Code:
A B C D
a) 2 3 4 1
b) 3 1 2 4
c) 1 3 2 4
d) 2 1 4 3

Answer: D

5. Consider the following statement(s) related to India's Peninsular River that discharge their
water in the Arabian Sea.
1. The rivers flowing towards the Arabian Sea have short courses.
www.achieveias.co.in, YouTube Channel: http://youtube.com/c/AchieveIAS Telegram Channel: http://t.me/Achieve_Ias,
Mail: achieveias21@gmail.com, Contact Number: 8968920720

2. The important peninsular rivers flowing towards the west are Shetrunji, Bhadra (Bhadar),
Vaitarna, Kalindi, Bedti, Sharavati, Bharathpuzha, Periyar and Pamba.
Which of the above statement(s) is/are correct?
A. Only 1 B. Only 2 C. Both 1 and 2 D. Neither 1 nor 2

Answer: C

Explanation: The Peninsular drainage system is older than the Himalayan one. This is evident from
the broad, largely-graded shallow valleys, and the maturity of the rivers. The Western Ghats running
close to the western coast act as the water divide between the major Peninsular Rivers, discharging
their water in the Bay of Bengal and as small rivulets joining the Arabian Sea. Most of the major
Peninsular Rivers except Narmada and Tapi flow from west to east. The Peninsular drainage system
is older than the Himalayan one. This is evident from the broad, largely-graded shallow valleys, and
the maturity of the rivers. The Western Ghats running close to the western coast act as the water
divide between the major Peninsular Rivers, discharging their water in the Bay of Bengal and as
small rivulets joining the Arabian Sea. Most of the major Peninsular Rivers except Narmada and
Tapi flow from west to east. The Chambal, the Sind, the Betwa, the Ken, the Son, originating in the
northern part of the Peninsula belong to the Ganga river system. The other major river systems of
the peninsular drainage are – the Mahanadi the Godavari, the Krishna and the Kaveri. Peninsular
rivers are characterised by fixed course, absence of meanders and non-perennial flow of water. The
Narmada and the Tapi which flow through the rift valley are, however, exceptions. Evolution of
Peninsular Drainage System: Three major geological events in the distant past have shaped the
present drainage systems of Peninsular India: Subsidence of the western flank of the Peninsula
leading to its submergence below the sea during the early tertiary period. Generally, it has disturbed
the symmetrical plan of the river on either side of the original watershed.
• Upheaval of the Himalayas when the northern flank of the peninsular block was subjected to
subsidence and the consequent trough faulting. The Narmada and The Tapi flow in trough faults
and fill the original cracks with their detritus materials. Hence, there is a lack of alluvial and deltaic
deposits in these rivers.
• Slight tilting of the peninsular block from northwest to the south-eastern direction gave orientation
to the entire drainage system towards the Bay of Bengal during the same period.

Rivers of the peninsular India:


The major river systems of the peninsular drainage are – the Mahanadi the Godavari, the Krishna
and the Kaveri, the Narmada, the Tapi and the Luni which is discussed below:
1. The Godavari: It is the largest peninsular river system due to this it is also called the Dakshin
Ganga. It rises in the Nasik district of Maharashtra and discharges its water into the Bay of Bengal.
Its tributaries run through the states of Maharashtra, Madhya Pradesh, Chhattisgarh, Orissa and
Andhra Pradesh. The Penganga, the Indravati, the Pranhita, and the Manjra are its principal
tributaries. The Godavari is subjected to heavy floods in its lower reaches to the south of Polavaram,
where it forms a picturesque gorge. It is navigable only in the deltaic stretch. The river after
Rajamundri splits into several branches forming a large delta.
2. The Krishna: It is the second largest east flowing Peninsular River which rises near
Mahabaleshwar in Sahyadri. Its total length is 1,401 km. The Koyna, the Tungbhadra and the Bhima
are its major tributaries.
3. The Mahanadi: It rises near Sihawa in Raipur district of Chhattisgarh and runs through Orissa to
discharge its water into the Bay of Bengal. It is 851 km long and its catchment area spreads over
1.42 lakh sq. km. Some navigation is carried on in the lower course of this river. Fifty three per cent
www.achieveias.co.in, YouTube Channel: http://youtube.com/c/AchieveIAS Telegram Channel: http://t.me/Achieve_Ias,
Mail: achieveias21@gmail.com, Contact Number: 8968920720

of the drainage basin of this river lies in Madhya Pradesh and Chhattisgarh, while 47 per cent lies
in Orissa.
4. The Kaveri: It rises in Brahmagiri hills (1,341m) of Kogadu district in Karnataka. Its length is 800
km and it drains an area of 81,155 sq. km. Since the upper catchment area receives rainfall during
the southwest monsoon season (summer) and the lower part during the northeast monsoon season
(winter), the river carries water throughout the year with comparatively less fluctuation than the other
Peninsular Rivers. It’s important tributaries are the Kabini, the Bhavani and the Amravati.
5. The Tapi: It is the other important westward flowing river. It originates from Multai in the Betul
district of Madhya Pradesh. It is 724 km long and drains an area of 65,145 sq. km. Nearly 79 per
cent of its basin lies in Maharashtra, 15 per cent in Madhya Pradesh and the remaining 6 per cent
in Gujarat.
6. The Luni: It is the largest river system of Rajasthan, west of Aravali. It originates near Pushkar
in two branches, i.e. the Saraswati and the Sabarmati, which join with each other at Govindgarh.
From here, the river comes out of Aravali and is known as Luni. It flows towards the west till Telwara
and then takes a southwest direction to join the Rann of Kuchchh. The entire river system is
ephemeral. The important peninsular rivers flowing towards the west are Shetrunji, Bhadra (Bhadar),
Vaitarna, Kalindi, Bedti, Sharavati, Bharsthpuzha, Periyar and Pamba. The rivers flowing towards
the Arabian Sea have short courses. These rivers cover in the states of Gujarat, Maharashtra,
Karnataka and Kerala. Peninsular Rivers flowing towards the west are discussed below:
1. The Shetruniji is one such river which rises near Dalkahwa in Amreli district.
2. The Bhadra originates near Aniali village in Rajkot district. The Dhadhar rises near Ghantar village
in Panchmahal district. Sabarmati and Mahi are the two famous rivers of Gujarat.
3. The Vaitarna originates from the Trimbak hills in Nasik district at an elevation of 670 m. The
Kalinadi rises from Belgaum district and falls in the Karwar Bay. The source of Bedti River lies in
Hubli Dharwar and traverses a course of 161 km.
4. The Sharavati is another important river in Karnataka flowing towards the west. The Sharavati
originates in Shimoga district of Karnataka and drains a catchment area of 2,209 sq. km.
5. The Mandovi and the Juari is two important rivers of Goa.
www.achieveias.co.in, YouTube Channel: http://youtube.com/c/AchieveIAS Telegram Channel: http://t.me/Achieve_Ias,
Mail: achieveias21@gmail.com, Contact Number: 8968920720

ACHIEVE IAS GEOGRAPHY MCQ SERIES, DAY 36, SOLUTIONS

1. Which tributary of Ganga River popularly known as 'Sorrow of Bihar'?


A. Gandak B. Ghaghara C. Kosi D. Kali

Answer: C

Explanation: The Kosi River, one of the oldest river in Indian history, it is also known as Sapta Kosi
means seven rivers. is known as the "Sorrow of Bihar" as the annual floods affect about 21,000 km
sq (8,100 sq mi) of fertile agricultural lands thereby disturbing the rural economy.

2. Which of the following tributary of Ganga originates from Nepal-Sikkim border and joins
Ganga in Bangladesh?
A. Mahananda River B. Kosi River C. Gandak River D. Gomti

Answer: A

Explanation: The Mahananda River is a trans-boundary river that flows through the Indian states
of West Bengal, Bihar, and Bangladesh.It originates in the Himalayas: Paglajhora Falls on
Mahaldiram Hill near Chimli, east of Kurseong in Darjeeling district.

3. The Brahmaputra, Irrawady and Mekong rivers originate in Tibet and flow through narrow
and parallel mountain ranges in their upper reaches. Of these rivers, Brahmaputra makes a
“U” turn in its course to flow into India. This “U” turn is due to :
A. Uplift of folded Himalayan series
B. Syntaxial bending of geologically young Himalayas
C. Geo-tectonic disturbance in the tertiary folded mountain chains
D. Both (a) and (b) above

Answer: B

4. Consider the following rivers:


1. Barak
2. Lohit
3. Subansiri

Which of the above flows / flow through Arunachal Pradesh?


A. 1 only B. 2 and 3 only C. 1 and 3 only D. 1, 2 and 3

Answer: B

Explanation: Arunachal Pradesh criss-crossed by numerous rivers, riverines and streams in true
sense is a land of rivers. The prominent rivers flowing through this Himalayan state are:
The 2,900-km long Brahmaputra, major river of the state, is known for transport, irrigation and
potable water source.
1. Siang River, a major tributary of Brahmaputra, provides enough opportunities for adventure
sports, like tubing, fishing and trekking.
2. Subansiri river emerging from the Himalayas is the biggest tributary of the Brahmaputra river.
www.achieveias.co.in, YouTube Channel: http://youtube.com/c/AchieveIAS Telegram Channel: http://t.me/Achieve_Ias,
Mail: achieveias21@gmail.com, Contact Number: 8968920720

3. Lohit river rises from Zayal Chu range of eastern Tibet and flows 200-km in Arunachal Pradesh
criss-crossing Lohit and Namsi districts
4. Dibang river flowing in the same Valley, a holy source for Idu-Mishmi tribe, on which and a
Dibang Multi-Purpose Project, a major hydropower project has been proposed.
5. Drangme Chhu river rises in western part of the state and joins with southern region Mangde
Chhu River.
6. Papumpare is a deep river in same but beautiful river valley. Papumpare river valleys known for
green lush vegetations throughout the year.

The rivers of India play an important role in the lives of the Indians. They provide potable water,
cheap transportation, electricity, and the livelihood for a large number of people all over the country.
This easily explains why nearly all the major cities of India are located by the banks of rivers. Seven
major rivers along with their numerous tributaries make up the river system of India. The largest
basin system of the rivers pour their waters into the Bay of Bengal; however, some of the rivers
whose courses take them through the western part of the country and towards the east of the state
of Himachal Pradesh empty into the Arabian Sea. Parts of Ladak, northern parts of the Aravalli range
and the arid parts of the Thar Desert have inland drainage. Rivers flow on every continent and on
all but the smallest island. They occur with an almost bewildering variety, ranging from a mere
trickle to a mighty surge. As a source of water, rivers have always been objects of wonder and
practical concern for people everywhere. They have acted as cradles of civilization and agents of
disaster. A river may be a barrier or a highway. It can bear trade and sediment; culture and conflict.
A river may inspire or it may terrify. Rivers have played an important role in development of human
being since time immemorial. It is well known that many towns and cities are distinguished by the
rivers that flow through them. Some of them are London (Thames), Rome (Tevere), Paris (Seine),
Banaras (Ganga), Delhi (Yamuna). Besides, one cannot deny the interconnecting relationship that
exist between people, river and the state. The evolutionary history of men suggest that transport
and communication were possible mostly through water ways. Moreover, waters helped to
permanently settle people in one place. Settled cultivation was possible where continued supply of
water was assured. Historically, populations have mostly settled close to sources of water. Major
civilizations of the world developed and flourished on the banks of river, for example, the Indus valley
civilization, Egyptian, Tigris, Euphrates, Hwang-Ho etc. The Aryan Civilization flourished in Ganga
Valley and continues to exist. Such civilizations have seen rivers being worshipped as Gods and
Goddesses. They held the belief that Earth originated in water. We can find traces of such belief
even in Vedas, Puranas and Upanishads etc. Gradually, apart from revering rivers as Gods and
Goddesses, they have been the primary source of drinking water, food, for domestic usages,
irrigational and industrial purposes and livelihood and as a means of transport and communication.
Rivers have also been site of development and exercise of state power. Harnessing rivers for
irrigation and power has been one of the many ways through which post independent India decided
to steer development.

5. The Narmada river flows to the west, while most other large peninsular rivers flow to the
east. Why?
1. It occupies a linear rift valley.
2. It flows between the Vindhyas and the Satpuras.
3. The land slopes to the west from Central India.

Select the correct answer using the codes given below.


A. 1 only B. 2 and 3 C. 1 and 3 D. None
www.achieveias.co.in, YouTube Channel: http://youtube.com/c/AchieveIAS Telegram Channel: http://t.me/Achieve_Ias,
Mail: achieveias21@gmail.com, Contact Number: 8968920720

Answer: A

Explanation: The Narmada: It originates on the western flank of the Amarkantak plateau at a
height of about 1,057 m. flowing in a rift valley between the Satpura in the south and the Vindhyan
range in the north; it forms a picturesque gorge in marble rocks and Dhuandhar waterfall near
Jabalpur. After flowing a distance of about 1,312 km, it meets the Arabian sea south of Bharuch,
forming a broad 27 km long estuary. Its catchment area is about 98,796 sq. km. The Sardar Sarovar
Project has been constructed on this river.
www.achieveias.co.in, YouTube Channel: http://youtube.com/c/AchieveIAS Telegram Channel: http://t.me/Achieve_Ias,
Mail: achieveias21@gmail.com, Contact Number: 8968920720

ACHIEVE IAS GEOGRAPHY NCERT MCQ SERIES, DAY 37, SOLUTIONS

1. Consider the following statement (s) is/are related to the natural vegetation.
1. Natural Vegetation follow thE climatic variables and grow naturally.
2. Natural vegetation vary according to the climate, soil and altitude.

Which of the following statement(s) is/are correct?


A. 1 only B. 2 only C. Both 1 & 2 D. Neither 1 nor 2

Answer: C

Explanation: Natural vegetation means the plants that have not been grown by humans. It doesn’t
need help from humans and gets whatever it needs from its natural environment. There is a close
relationship between height of land and the character of vegetations. With the change in height, the
climate changes and that changes natural vegetation. The growth of vegetation depends on
temperature and moisture. It also depends on factors like slope and thickness of soil. It is
categorized into three broad categories: Forest, grassland and shrubs. Natural vegetation means
the plants that have not been grown by humans. It doesn’t need help from humans and gets
whatever it needs from its natural environment. There is a close relationship between height of land
and the character of vegetations. With the change in height, the climate changes and that changes
natural vegetation. The growth of vegetation depends on temperature and moisture. It also depends
on factors like slope and thickness of soil. It is categorized into three broad categories: Forest,
grassland and shrubs.
1. Tropical Evergreen Forests: It is also called tropical rainforests and occurs in the regions near
the equator and close to the tropics. These regions are hot and receive heavy rainfall throughout
the year. These forests are called evergreen because they do not shed their leaves. Hardwood
trees like rosewood, ebony, and mahogany are common here. The tropical evergreen forest in
Brazil is so enormous that it is like the lungs of the earth. Its distribution in India are- North-Eastern
India, Western Slopes of the Western Ghats, Andaman & Nicobar Islands.
2. Tropical Deciduous Forests: They are the monsoon forests found in the large part of India-
Eastern Slopes of the Western Ghats, the Tarai regions of the Himalayas, Bihar, Uttar Pradesh,
Odisha, West Bengal, Maharashtra, Andhra Pradesh, Karnataka, and Madhya Pradesh. Trees shed
their leaves in the dry season to conserve water. Sal, teak, neem and shisham are hardwood trees
found in this forest. Tigers, lions, elephants, langoors and monkeys are the common animals of
these regions.
3. Tropical Dry Deciduous Forests: The vegetation is found in those areas where the annual rainfall
is between 50 and 100 cm. It is found in Eastern Rajasthan, Northern Gujarat, Western Madhya
Pradesh, South-Western Uttar Pradesh, South Punjab, Haryana and the rain-shadow area of the
Western Ghats.
4. Desert and Semi- Arid Vegetation: This kind of vegetation is found in the areas having less than
50 cm rainfall. Here the trees are in the forms of small shrubs. Generally their maximum height is
up to 6 cm. The trees have deeps roots, and thick and thorny leaves. It is found in Western
Rajasthan, Northern Gujarat, and the rain shadow region of the Western Ghats.
5. Mangrove Vegetation: This is found in the sea-coast and the lower deltaic regions. In these areas,
the saline water spreads because of high tide. Here, the soil is marshy. The Ganga- Brahmaputra
delta, deltaic regions of the rivers Mahanadi, Krishna, Godavari, Kaveri etc, and some parts of the
eastern and western coast comes under this vegetation.
www.achieveias.co.in, YouTube Channel: http://youtube.com/c/AchieveIAS Telegram Channel: http://t.me/Achieve_Ias,
Mail: achieveias21@gmail.com, Contact Number: 8968920720

6. Moist Sub-tropical Montane Vegetation: This vegetation is found at the heights 1070-1500 m in
the Peninsular India. This Vegetation is evergreen. The tree woods are almost soft. It is found in the
area like- the Western Ghats, the Eastern Ghats, the Nilgiri, the Cardamom hills and the Annamalai
hills.
7. Moist Temperate Montane Vegetation: This vegetation is found at the heights of 1500 m. It is
found mostly in the Peninsular India. This forest is not very dense. There are shrubs on the surface.
It is found in the hills of Annamalai, Nilgiri and Palni. The main trees of this forest are- Magnolia,
Eucalyptus, and Elm.
8. The Himalayan Vegetation: A wide range of species is found in the mountains according to the
variation in height. With increase in height, the temperature falls. At a height between 1500metres
and 2500 metres most of the trees are conical in shape. Chir, Pine and Deodar are important
coniferous trees found in these forests.

2. Which of the following forest that are found in the Western Ghats, hills of the north eastern
region and the Andaman and Nicobar Islands.
A. Mangroove Forest B. Dry deciduous Forest
C. Tropical Evergreen Forests D. Semi-desert and desert vegetation

Answer: C

Explanation: Tropical Evergreen forests are a dominant part of the Natural vegetation in India.The
evergreen forests are essential in not only promoting greenery on the planet, but they are also useful
in the continual survival of animals and plants in the forest ecosystem. The trees are evergreen as
there is no period of drought. They are mostly tall and hardwood.
Tropical Evergreen Forest – Characteristic Features:
1. The tropical wet evergreen forest in India is usually found in areas receiving more than 200 cm of
rainfall and having a temperature of 15-30 degrees Celsius.
2. They occupy about 7% of the earth’s surface.
3. They are found mostly near the equator.
4. They have sparse undergrowth interspersed with clearings
5. They have a scarce presence of litter (organic matter settling on the ground)
These forests are dense and multi-layered. They harbour many types of plants and animals. The
forests constitute an important part of the environment and ecology. These trees are an important
component of the Forest Biology and ecosystem, that helps to promote life in the ecosystem. This
allows the plants and animal life to harmonize and live with one another in absolute peace. Variety
of Plant/Animal Species in Tropical Evergreen Forest: The different types of animals that live in
the evergreen forests are usually of a certain type such as indigenous birds like owls, hawks,
cardinals and even some mammals such as deers, possums and raccoons.In India, evergreen
forests are primarily located in states such as Karnataka and even Kerala. The Western Ghats are
the primary location of the evergreen forest. The forests mainly contain trees such as rosewood,
mahogany and ebony.

3. Consider the following statement (s) is/are regarding deciduous forest.


1. It is found in that regions where rainfall is between 70 and 200 cm.
2. Forests are divided into moist and dry deciduous on the basis of the availability of water.

Which of the following statement(s) is/are correct?


A. 1 only B. 2 only C. Both 1 & 2 D. Neither 1 nor 2
www.achieveias.co.in, YouTube Channel: http://youtube.com/c/AchieveIAS Telegram Channel: http://t.me/Achieve_Ias,
Mail: achieveias21@gmail.com, Contact Number: 8968920720

Answer: B

Explanation: A deciduous forest is a biome dominated by deciduous trees which lose their leaves
seasonally. The Earth has temperate deciduous forests, and tropical and subtropical deciduous
forests, also known as dry forests. Another name for these forests is broad-leaf forests because of
the wide, flat leaves on the trees. Trees in tropical deciduous forests lose their leaves in the dry
season and regrow them in the rainy season. In temperate deciduous forests, trees lose their leaves
in the fall and regrow them in the spring. Animals inhabiting deciduous forests include insects,
spiders, reptiles and birds. Mice, rabbits, foxes, deer, otters, bears and humans are just some
examples of mammals that live in deciduous forests. Tropical and subtropical deciduous forests are
also home to mammals such as elephants, monkeys, tigers, and giraffes. Deciduous forests are
home to trees such as oak, birch, beech, aspen, elm and maple. Tropical and subtropical forests
also have teak trees, palm trees and bamboo. Plants found in these forests include flowers, ferns,
mosses and herbs. In addition, tropical and subtropical forests feature flowers such as orchids and
numerous vines called lianas. The average temperature of deciduous forests is 50°F and annual
rainfall averages 30 to 60 inches. Temperate deciduous forests also have precipitation in the form
of snow. Deciduous forests must have at least 120 days without frost. This period can extend to 250
days in some tropical and subtropical deciduous forests. Tropical and subtropical deciduous forests
have a very tight temperature range between 68°F to 77°F. This is in stark contrast to temperate
deciduous forests, which have a far wider range of -22°F to 86°F. Precipitation in temperate
deciduous forests ranges 30 to 60 inches per year, while annual rainfall can be over 80 inches in
tropical and subtropical deciduous forests. Recent research shows climate change is altering the
way in which deciduous forests respond to temperature, precipitation and drought. Another
distinction between these two types of forests is canopy cover. Tropical and subtropical forest
canopies are dense and have several layers. This prevents most of the sun’s light from reaching the
forest floor. Temperate forest canopies let more light reach the forest floor, creating more plant and
animal diversity.

4. Consider the following statement (s) is/ are about Tropical Thorn Forests.
1. They occur in the areas which receive rainfall more than 60 cm.
2. These consist of variety of herbs, thorny grasses and shrubs.

Which of the following statement(s) is/are correct?


A. 1 only B. 2 only C. Both1 & 2 D. Neither 1 nor 2

Answer: D

Explanation: A thorny forest is a dense, scrub-like vegetation characteristic of dry subtropical and
warm temperate areas with a seasonal rainfall averaging 250 to 500 mm (9.8 to 19.7 in). This
vegetation covers a large part of southwestern North America and southwestern Africa and smaller
areas in Africa, South America, and Australia. In South America, thorn forest is sometimes
called Caatinga, and consists primarily of small, thorny trees that shed their leaves seasonally. Trees
typically do not exceed 10 metres (33 ft) in height, usually averaging between 7 and 8 metres (23
and 26 ft) tall. Thorn forest grades into savanna woodland as the rainfall increases and into desert
as the climate becomes dryer.

5. Which of the following state has got largest forest in the country?
www.achieveias.co.in, YouTube Channel: http://youtube.com/c/AchieveIAS Telegram Channel: http://t.me/Achieve_Ias,
Mail: achieveias21@gmail.com, Contact Number: 8968920720

A. Madhya Pradesh B. Arunachal Pradesh


C. Chhattisgarh D. Maharashtra

Answer: A

Explanation: In terms of area, Madhya Pradesh has the largest forest cover of 77,414 square km
in the country, followed by Arunachal Pradesh with 66,964 square km and Chhattisgarh with 55,547
square km.

6. Which of the following statement is correct about the Biosphere Reserve?


A. Multipurpose protected areas to preserve genetic diversity in representative ecosystem.
B. A reserves area meant for preserving its natural vegetation, wildlife and natural beauty.
C. A reserved area meant for preservation and development of endangered species.
D. All of the above

Answer: D

Explanation: Biosphere Reserve is a notified area which covers a larger area of land which may
cover multiple National Parks, Sanctuaries and reserves as well. It is an area meant for conservation
of biodiversity of a specific area.

7. Consider the following statements


1. Shedding leaves and appearing as vast grassland in dry seasons is feature of dry deciduous
forest.
2. Tendu trees are commonly found in Dry deciduous forest.

Select the correct answer from the following codes


a. 1 Only b. 2 Only c. Both 1 and 2 d. Neither 1 nor 2

Answer: C

Explanation: As the dry season begins, the trees shed their leaves completely and the forest
appears like a vast grassland with naked trees all around. Tendu, palas, amaltas, bel, khair,
axlewood, etc. are the common trees of dry deciduous forests.

8. Which of the following is true regarding natural vegetation of India?


1. Himalayan heights are marked with tropical evergreen vegetation
2. The Andaman Nicobar Islands have tropical rain forests
3. The deltaic regions have tropical deciduous forests and mangroves

Select the correct statements using following codes


a. 2 Only b. 1 and 2 Only c. 2 and 3 Only d. 1, 2 and 3

Answer: A

Explanation: India is a land of great variety of natural vegetation. Himalayan heights are marked
with temperate vegetation; the Western Ghats and the Andaman Nicobar Islands have tropical rain
www.achieveias.co.in, YouTube Channel: http://youtube.com/c/AchieveIAS Telegram Channel: http://t.me/Achieve_Ias,
Mail: achieveias21@gmail.com, Contact Number: 8968920720

forests, the deltaic regions have tropical forests and mangroves; the desert and semi desert areas
of Rajasthan are known for cactii, a wide variety of bushes and thorny vegetation.

9. The Forest Act was enacted in which of the following year?


a. 1927 b. 1972 c. 1980 d. 1982

Answer: A

Explanation: First Forest Act was enacted in 1927. This is one of the many surviving colonial
legislations. It was enacted to consolidate the law related to forest, the transit of forest produce and
the duty livable on timber and other forest produce. Subsequently, the Forest (Conservation). The
act was promulgated in 1980 to make certain reforms over the preceding Act of 1927.The 1927 Act
deals with the four categories of the forests, namely reserved forests, village forests, protected
forests and private forests. A state may declare forestlands or wastelands as reserved forest and
may sell the produce from these forests. Any unauthorized felling of trees quarrying, grazing and
hunting in reserved forests is punishable with a fine or imprisonment, or both reserved forests
assigned to a village community are called village forests.

10. Which of the following statement (s) is/are correct about Indian forest reserve?
1. Indian mangroves cover about 4975 km2 area in the coastal regions.
2. Sunderban is the largest mangrove forests in India and it have been added to the list of biosphere
reserves of UNESCO.

A. Only 1 B. Only 2 C. Both 1 & 2 D. Neither 1 nor 2

Answer: C

Explanation: The Mangrove ecosystems are unique & rich in biodiversity and they provide
numerous ecological services. Mangrove cover has been separately reported in the ISFR 2019
and the total mangrove cover in the country is 4,975 sq km. An increase of 54 sq Km in
mangrove cover has been observed as compared to the previous assessment of 2017. Top three
states showing mangrove cover increase are Gujarat (37 sq km) followed by Maharashtra (16
sq km) and Odisha (8 sq km). . The deltas of the Ganges, Mahanadi, Krishna, Godavari, and
Kaveri rivers contain mangrove forests. The Sundarbans mangrove forest, one of the largest such
forests in the world (140,000 ha), lies on the delta of the Ganges, Brahmaputra and Meghna rivers
on the Bay of Bengal. It is adjacent to the border of India’s Sundarbans World Heritage site inscribed
in 1987. The site is intersected by a complex network of tidal waterways, mudflats and small islands
of salt-tolerant mangrove forests, and presents an excellent example of ongoing ecological
processes. The area is known for its wide range of fauna, including 260 bird species, the Bengal
tiger and other threatened species such as the estuarine crocodile and the Indian python.
www.achieveias.co.in, YouTube Channel: http://youtube.com/c/AchieveIAS Telegram Channel: http://t.me/Achieve_Ias,
Mail: achieveias21@gmail.com, Contact Number: 8968920720

ACHIEVE IAS GEOGRAPHY NCERT MCQ SERIES, DAY 38, SOLUTIONS

1. Consider the following statement (s) related to black soil.


1. Black soilis rich in metals such as Iron, Magnesium and Aluminum.
2. Typical characteristics of this black soil are swelling (during wet period) and shrinkage (dry
period).

Which of the following statement(s) is correct?


A. 1 only B. 2 only C. Both 1 & 2 D. Neither 1 nor 2

Answer: C

Explanation: Black soil in India is rich in metals such as Iron, Magnesium and Aluminium. However
it is deficient in Nitrogen, Potassium, Phosphorous and Humus. Black soil is of red colour mainly
due to its iron oxide content. This soil shares 15 % of all types of soil in India. These soils are made
up of volcanic rocks and lava-flow. It is concentrated over Deccan Lava Tract which includes parts
of Maharashtra, Chhattisgarh, Madhya Pradesh, Gujarat, Andhra Pradesh and Tamil Nadu. Typical
characteristics of this black soil are swelling (during wet period) and shrinkage (dry period). While
dry, it forms very deep cracks of more than 30-45 cm. In Kovilpatti (Tamil Nadu) areas the cracks
may extend to 2 to 3 m with a width of 1 to 6 cm. Field preparations takes longer time compared to
other soil. Only after secondary tillage, the soil is suited for crop production. The soils are fine
grained contain high proportion of Calcium and Magnesium carbonates. Black soil holds more
moisture and available for a long time.In Tamil Nadu Black soils have high pH (8.5 to 9) and are rich
in lime (5-7%), have low permeability. The soils are with more cation exchange capacity (40-60
m.e./100 g). Crops cultivated from this type of soil are Rice, Ragi, Sugarcane and Cashew nuts etc.
This type of soil formed as a result of high leaching and found in the areas of high rainfall.
Important features of black soil:
• Regur means cotton – best soil for cotton cultivation.
• Most of the Deccan is occupied by Black soil.
• Mature soil.
• High water retaining capacity.
• Swells and will become sticky when wet and shrink when dried.
• Self-ploughing is a characteristic of the black soil as it develops wide cracks when dried.
• Rich in: Iron, lime, calcium, potassium, aluminum and magnesium.
• Deficient in: Nitrogen, Phosphorous and organic matter.
• Colour: Deep black to light black.
• Texture: Clayey.

2. Consider the following statements(s) related to desertification.


1. It is a forms of land degradation, for example soil erosion in the humid tropics.
2. It is a form of land degradation in aria, semi-arid and dry subhumid areas resulting from adverse
human impact.

Which of the following statement(s) is correct?


A. 1 only B. 2 only C. Both 1 & 2 D. Neither 1 nor 2

Answer: C
www.achieveias.co.in, YouTube Channel: http://youtube.com/c/AchieveIAS Telegram Channel: http://t.me/Achieve_Ias,
Mail: achieveias21@gmail.com, Contact Number: 8968920720

Explanation: Desertification, also called desertization, the process by which natural or human
causes reduce the biological productivity of drylands (arid and semiarid lands). Declines in
productivity may be the result of climate change, deforestation, overgrazing, poverty, political
instability, unsustainable irrigation practices, or combinations of these factors. The concept does not
refer to the physical expansion of existing deserts but rather to the various processes that threaten
all dryland ecosystems, including deserts as well as grasslands and scrublands. Slightly less than
half of Earth’s ice-free land surface—approximately 52 million square km (about 20 million square
miles)—is drylands, and these drylands cover some of the world’s poorest countries. The United
Nations Environment Programme (UNEP) notes that desertification has affected 36 million square
km (14 million square miles) of land and is a major international concern. According to the United
Nations Convention to Combat Desertification, the lives of 250 million people are affected by
desertification, and as many as 135 million people may be displaced by desertification by 2045,
making it one of the most severe environmental challenges facing humanity. In general,
desertification is caused by variations in climate and by unsustainable land-management practices
in dryland environments. By their very nature, arid and semiarid ecosystems are characterized by
sparse or variable rainfall. Thus, climatic changes such as those that result in extended droughts can
rapidly reduce the biological productivity of those ecosystems. Such changes may be temporary,
lasting only a season, or they may persist over many years and decades. On the other
hand, plants and animals are quick to take advantage of wetter periods, and productivity can rapidly
increase during these times. Four Areas Affected By Desertification: To better understand how
climatic changes and human activities contribute to the process of desertification, the consequences
listed above can be grouped into four broad areas:
1. Irrigated croplands, whose soils are often degraded by the accumulation of salts.
2. Rain-fed croplands, which experience unreliable rainfall and wind-driven soil erosion.
3. Grazing lands, which are harmed by overgrazing, soil compaction, and erosion.
4. Dry woodlands, which are plagued by the overconsumption of fuelwood.

Solutions To Desertification: The struggle against desertification can occur at several levels. Since
regional variations in climate are the main causes of the loss of dryland productivity, it is important
to understand the influence of global warming in specific dryland regions. According to some models
of climate change, many grasslands in western North America, for example, are predicted to be at
greater risk of drought due to projected increases in summer temperatures and changes to existing
rainfall patterns. Many authorities argue that since desertification and global warming are so closely
related, one of the main solutions to the former may be the implementation of effective economic
policies (such as carbon trading) and technical measures (such as carbon sequestration) that
reduce the production of greenhouse gases. At local scales, however, desertification is often the
result of unsustainable land and soil management. To maintain the biological productivity of the
land, soil conservation is often the priority. A number of innovative solutions have been devised that
range from relatively simple changes in how people grow crops to labour-intensive landscape
engineering projects. Some of the techniques that may help ameliorate the consequences of
desertification in irrigated croplands, rain-fed croplands, grazing lands, and dry woodlands include:
1. Salt traps, which involve the creation of so-called void layers of gravel and sand at certain depths
in the soil. Salt traps prevent salts from reaching the surface of the soil and also help to inhibit water
loss.
2. Irrigation improvements, which can inhibit water loss from evaporation and prevent salt
accumulation. This technique involves changes in the design of irrigation systems to prevent water
from pooling or evaporating easily from the soil.
www.achieveias.co.in, YouTube Channel: http://youtube.com/c/AchieveIAS Telegram Channel: http://t.me/Achieve_Ias,
Mail: achieveias21@gmail.com, Contact Number: 8968920720

3. Cover crops, which prevent soil erosion from wind and water. They can also reduce the local
effects of drought. On larger scales, plant cover can help maintain normal rainfall patterns. Cover
crops may be perennials or fast-growing annuals.
4. Crop rotation, which involves the alternation of different crops on the same plot of land over
different growing seasons. This technique can help maintain the productivity of the soil by
replenishing critical nutrients removed during harvesting.
5. Rotational grazing, which is the process of limiting the grazing pressure of livestock in a given
area. Livestock are frequently moved to new grazing areas before they cause permanent damage
to the plants and soil of any one area.
6.Terracing, which involves the creation of multiple levels of flat ground that appear as long steps
cut into hillsides. The technique slows the pace of runoff, which reduces soil erosion and retards
overall water loss.
7. Contour bunding (or contour bundling), which involves the placement of lines of stones along the
natural rises of a landscape, and contour farming. These techniques help to capture and hold rainfall
before it can become runoff. They also inhibit wind erosion by keeping the soil heavy and moist.
8. Windbreaks, which involve the establishment of lines of fast-growing trees planted at right angles
to the prevailing surface winds. They are primarily used to slow wind-driven soil erosion but may be
used to inhibit the encroachment of sand dunes.
9. Dune stabilization, which involves the conservation of the plant community living along the sides
of dunes. The upper parts of plants help protect the soil from surface winds, whereas the root
network below keeps the soil together.
10. Charcoal conversion improvements, which include the use of steel or mud kilns or high-pressure
compacting equipment to press the wood and other plant residues into briquettes. Conversion
improvements retain a greater fraction of the heating potential of fuelwood.

3. Consider the following statements(s) related to soil.


1. It is the earth’s fragile skin that anchors all life on Earth.
2. It is comprised of countless species that create a dynamic and complex ecosystem and is among
the most precious resources to humans.

Which of the following statement(s) is correct?


A. 1 only B. 2 only C. Both 1 & 2 D. Neither 1 nor 2

Answer: C

Explanation: Soil is a natural body comprised of solids (minerals and organic matter), liquid, and
gases that occurs on the land surface, occupies space, and is characterized by one or both of the
following: horizons, or layers, that are distinguishable from the initial material as a result of additions,
losses, transfers, and transformations of energy and matter or the ability to support rooted plants in
a natural environment. The upper limit of soil is the boundary between soil and air, shallow water,
live plants, or plant materials that have not begun to decompose. Areas are not considered to have
soil if the surface is permanently covered by water too deep (typically more than 2.5 meters) for the
growth of rooted plants. The lower boundary that separates soil from the nonsoil underneath is most
difficult to define. Soil consists of horizons near the Earth's surface that, in contrast to the underlying
parent material, have been altered by the interactions of climate, relief, and living organisms over
time. Commonly, soil grades at its lower boundary to hard rock or to earthy materials virtually devoid
of animals, roots, or other marks of biological activity. For purposes of classification, the lower
boundary of soil is arbitrarily set at 200 cm. The soil contains numerous organisms ranging from
www.achieveias.co.in, YouTube Channel: http://youtube.com/c/AchieveIAS Telegram Channel: http://t.me/Achieve_Ias,
Mail: achieveias21@gmail.com, Contact Number: 8968920720

microscopic bacteria to large soil animals such as earthworms. The soil micro-organisms include
bacteria, fungi, actinomycetes, algae, protozoa and nematodes. The diversity of soil organisms can
both assist and hinder plant growth. Beneficial activities include organic matter decomposition,
nitrogen fixation, transformation of essential elements from one form to another, improvement in soil
structure through soil aggregation, and improved drainage and aeration. Under some circumstances
soil organisms compete with plants for nutrients. Bacteria are the smallest and most numerous
micro-organisms in the soil. They make an important contribution to organic matter decomposition,
nitrogen fixation and the transformation of nitrogen and sulphur. The fungi and actinomycetes
contribute beneficially to organic matter decomposition. The group of large soil animals includes
earthworms, which incorporate organic matter into the soil as well as improving aeration and
drainage by means of their channels. Some soil fungi, nematodes, and insects feed on roots and
lateral shoots to the detriment of plants.

4. Consider the following statements(s) related to Soil retrogression and degradation


1. Retrogression is primarily due to soil erosion and corresponds to a phenomenon where
succession reverts the land to its natural physical state.
2. Degradation is an evolution, different from natural evolution, related to the local climate and
vegetation.

Which of the following statement(s) is correct?


A. 1 only B. 2 only C. Both 1 & 2 D. Neither 1 nor 2

Answer: C

Explanation: Soil retrogression and degradation are two important regressive evolution processes
which take place due to loss of equilibrium of a stable soil. Retrogression primarily occurs due to
soil erosion. Soil Degradation is an evolution process which is different from natural evolution and
is largely related to the local climate and vegetation. It is due to the replacement of primary plant
communities (known as climax) by the secondary communities. This replacement in turn modifies
the humus composition and amount, which will ultimately affect the formation of the soil. To bear in
mind is that, degradation is largely associated with human activity. In simple word, soil degradation
is any sort change or disturbance in the soil which is undesirable. The consequences of both the
processes are-
1. Impacts the yield bearing capacity of the soil.
2. A deterioration in water quality.
3. Economic loss- this is major negative externality.

5. Consider the following statements(s) related to Soil degradation.


1. It is the decline in soil quality caused by its improper use, usually for agricultural, pastural,
industrial or urban purposes.
2. It is a serious global environmental problem and may be exacerbated by climate change.

Which of the following statement(s) is correct?


A. 1 only B. 2 only C. Both 1 & 2 D. Neither 1 nor 2

Answer: C
www.achieveias.co.in, YouTube Channel: http://youtube.com/c/AchieveIAS Telegram Channel: http://t.me/Achieve_Ias,
Mail: achieveias21@gmail.com, Contact Number: 8968920720

Explanation: Soil degradation is the decline in soil condition caused by its improper use or poor
management, usually for agricultural, industrial or urban purposes. It is a serious environmental
problem. Soils are a fundamental natural resource, and are the basis for all terrestrial life. Avoiding
soil degradation is crucial to our well-being.

6. Which of the following is/ are the important agents of soil erosion?
1. Water
2. Wind
3. Human Activities
4. Disasters

Which of the following statement(s) is correct?


A. Both 1 & 2 B. Both 3 & 4 C. Both 2 & 4 D. All of these

Answer: A

Explanation: Soil Erosion is a common term that is often confused with soil degradation as a
whole, but in fact refers only to absolute soil losses in terms of topsoil and nutrients. This is indeed
the most visible effect of soil degradation, but does not cover all of its aspects. Soil erosion is a
natural process in mountainous areas, but is often made much worse by poor management
practices. Soil erosion i.e., the displacement of upper fertile layer of soil, is a natural process that
occurs at a very slow pace due to -
1. Moving Water (splash erosion, sheet erosion, rill erosion, and gully erosion)
2. Wind (surface creep, saltation and suspension)
3. Glaciers (due to heavy weight/ huge mass)
4. Gravity (causes mass movement)

7. Consider the following statement(s) related to alluvial soils.


1. The soil covers 40 percent part of total geographical area of India.
2. Alluvial soils are formed due to depositional work done by rivers in plains .valleys, flood plain and
deltas

Which of the above statement(s) is/are correct?


A. Only 1 B. Only 2 C. Both 1and 2 D. Neither 1 nor 2

Answer: C

Explanation: Alluvial soils are formed mainly due to silt deposited by Indo-Gangetic-Brahmaputra
Rivers. In coastal regions some alluvial deposits are formed due to wave action. Rocks of the
Himalayas form the parent material. Thus the parent material of these soils is of transported origin.
They are the largest soil group covering about 15 lakh sq km or about 46 per cent of the total area.
They support more than 40% of the India’s population by providing the most productive agricultural
lands.

A. Characteristics of Alluvial Soils:


1. They are immature and have weak profiles due to their recent origin.
2. Most of the soil is Sandy and clayey soils are not uncommon.
www.achieveias.co.in, YouTube Channel: http://youtube.com/c/AchieveIAS Telegram Channel: http://t.me/Achieve_Ias,
Mail: achieveias21@gmail.com, Contact Number: 8968920720

3. Pebbly and gravelly soils are rare. Kankar (calcareous concretions) beds are present in some
regions along the river terraces.
4. The soil is porous because of its loamy (equal proportion of sand and clay) nature.
5. Porosity and texture provide good drainage and other conditions favorable for agriculture.
6. These soils are constantly replenished by the recurrent floods.

B. Chemical properties of Alluvial Soils:


1. The proportion of nitrogen is generally low.
2. The proportion of Potash, phosphoric acid and alkalies are adequate
3. The proportion of Iron oxide and lime vary within a wide range.

C. Distribution of Alluvial Soils in India:


1. They occur all along the Indo-Gangetic-Brahmaputra plains except in few places where the top
layer is covered by desert sand.
2. They also occur in deltas of the Mahanadi, the Godavari, the Krishna and the Cauvery, where
they are called deltaic alluvium (coastal alluvium)
3. Some alluvial soils are found in the Narmada, Tapi valleys and Northern parts of Gujarat.

D. Crops in Alluvial Soils:


1. They are mostly flat and regular soils and are best suited for agriculture.
2. They are best suited to irrigation and respond well to canal and well/tube-well irrigation.
3. They yield splendid crops of rice, wheat, sugarcane, tobacco, cotton, jute, maize, oilseeds,
vegetables and fruits.

E. Geological divisions of alluvial soils: Geologically, the alluvium of the Great plain of India is divided
into newer or younger khadar and older bhangar soils.

Bhabar: The bhabar belt is about 8-16 km wide running along the Shiwalik foothills. It is a porous,
northern most stretch of Indo-Gangetic plain.Rivers descending from the Himalayas deposit their
load along the foothills in the form of alluvial fans. These alluvial fans (often pebbly soils) have
merged together to build up the bhabar belt.The porosity of bhabar is the most unique feature. The
porosity is due to deposition of huge number of pebbles and rock debris across the alluvial fans.The
streams disappear once they reach the bhabar region because of this porosity. Therefore, the area
is marked by dry river courses except in the rainy season.The area is not suitable for agriculture and
only big trees with large roots thrive in this belt.

Terai: Terai is an ill-drained, damp (marshy) and thickly forested narrow tract (15-30 km wide) to
the south of Bhabar running parallel to it.The underground streams of the Bhabar belt re-emerge in
this belt. It is a swampy lowland with silty soils.The terai soils are rich in nitrogen and organic matter
but are deficient in phosphate.These soils are generally covered by tall grasses and forests but are
suitable for a number of crops such as wheat, rice, sugarcane, jute etc..This thickly forested region
provides shelter to a variety of wild life.

Bhangar: The Bhangar is the older alluvium along the river beds forming terraces higher than the
flood plain (about 30 metres above the flood level).It is of a more clayey composition and is generally
dark colored.A few metres below the terrace of the bhangar are beds of lime nodules known as
“Kankar”.
www.achieveias.co.in, YouTube Channel: http://youtube.com/c/AchieveIAS Telegram Channel: http://t.me/Achieve_Ias,
Mail: achieveias21@gmail.com, Contact Number: 8968920720

Khadar: The Khadar is composed of newer alluvium and forms the flood plains along the river
banks.The banks are flooded almost every year and a new layer of alluvium is deposited with every
flood. This makes them the most fertile soils of Ganges.They are sandy clays and loams, more dry
and leached, less calcareous and carbonaceous (less kankary). A new layer of alluvium is deposited
by river flood almost every year.

8. Which of the following comes under the phenomena of mass movement?


1. Soil creep
2. Weathering
3. Landslide

Which of the statements given above is/are correct?


A. Both 1 & 2 B. Both 1 & 3 C. Both 2 & 3 D. 1, 2 and 3

Answer: B

Explanation: Mass movement, also called Mass Wasting, bulk movements of soil and rock debris
down slopes in response to the pull of gravity, or the rapid or gradual sinking of the Earth’s ground
surface in a predominantly vertical direction. Formerly, the term mass wasting referred to a variety
of processes by which large masses of crustal materials are moved by gravity from one place to
another. More recently, the term mass movement has been substituted to include mass wasting
processes and the sinking of confined areas of the Earth’s ground surface. Mass movements on
slopes and sinking mass movements are often aided by water and the significance of both types is
the part each plays in the alteration of landforms. The variety of downslope mass movements reflects
the diversity of factors that are responsible for their origin. Such factors include: weathering or
erosional debris cover on slopes, which is usually liable to mass movement; the character and
structure of rocks, such as resistant permeable beds prone to sliding because of underlying
impermeable rocks; the removal of the vegetation cover, which increases the slope’s susceptibility
to mass movement by reducing its stability; artificial or natural increases in the slope’s steepness,
which will usually induce mass movement; earthquake tremors, which affect the
slope equilibrium and increase the likelihood of mass movement; and flowing ground water, which
exerts pressure on soil particles and impairs slope stability. These factors affecting slope conditions
will often combine with climatic factors such as precipitation and frost activity to produce downslope
mass movement. The types of mass movements caused by the above factors include: the abrupt
movement and free fall of loosened blocks of solid rock, known as rockfalls; several types of almost
imperceptible downslope movement of surficial soil particles and rock debris, collectively called
creep; the subsurface creep of rock material, known as bulging: the multiplicity of downslope
movements of bedrock and other debris caused by the separation of a slope section along a plane
of least resistance or slip surface, collectively called landslides; the separation of a mass along a
concave head scarp, moving down a curved slip surface and accumulating at the slope’s foot, known
as a slump; the saturation of debris and weathered material by rainfall in the upper section of a slope
or valley, increasing the weight of the debris and causing a slow downslope movement, called
an earthflow; a rapidly moving earthflow possessing a higher water content, known as a mudflow; a
fast-moving earthflow in a mountainous region, called a debris flow or avalanche; and the downslope
movement of moisture-saturated surficial material, known as solifluction, over frozen substratum
material, occurring in sub-Arctic regions during seasonal periods of surface thaw. Sinking mass
movements occur in relatively rapid fashion, known as subsidence, and in a gradual manner,
called settlement. Subsidence involves a roof collapse or breakdown of a subsurface cavity such as
www.achieveias.co.in, YouTube Channel: http://youtube.com/c/AchieveIAS Telegram Channel: http://t.me/Achieve_Ias,
Mail: achieveias21@gmail.com, Contact Number: 8968920720

a cave. Extensive subsidence is evident in areas where coal, salt, and metalliferous ores are mined.
Marine erosion sometimes causes the roof collapse of sea caves. Regions of karst topography will
exhibit widespread subsidence in the form of sinkholes caused by underground drainage. Other
types of subsidence caused by underground solutions have been found in chalk, gypsum, anhydrite,
halite (salt), and loess terrains. The melting of ground ice also contributes to subsidence such as
the formation of glacial kettles and depressions following the seasonal surface thaw of perennially
frozen land. The chemical decomposition of subsurface rocks and ores is also a cause of
subsidence. Another form of subsidence is the steep-walled depression, known as a volcanic sink,
formed following the withdrawal of magma from below the ground surface.

9. What is the reason for the red colour of the red soil?
A. Phosphoric Acid B. Humus C. Nitrogen D. Iron

Answer: D

Explanation: Soil color is produced by the minerals present and by the organic matter content.
Yellow or red soil indicates the presence of oxidized ferric iron oxides. Dark brown or black color in
soil indicates that the soil has a high organic matter content. Wet soil will appear darker than dry
soil.

Four main factors influence the colour of a soil:


1. Mineral matter derived from the constituents of the parent material.
2. Organic matter.
3. The nature and abundance of iron.
4. Moisture content.

Their colour is mainly due to ferric oxides occurring as thin coatings on the soil particles while the
iron oxide occurs as haematite or as hydrous ferric oxide, the colour isred and when it occurs in the
hydrate form as limonite the soilgets a yellow colour.

10. Consider the following statement (s) related to the laterite soil:
1. It is formed due to intense leaching away of siliceous matter.
2. They are fully equipped with organic matter.

Which of the statements given above is/are correct?


A. 1 Only B. 2 Only C. Both 1 and 2 D. Neither 1 nor 2

Answer: C

Explanation: The word laterite has been derived from the Latin word ‘Later’ which means brick.
These soils when wet are as soft as butter but become hard and cloddy on drying. Therefore, these
are widely cut as bricks for use in house construction.
Formation: The lateritic soils are particularly found on high flat erosion surfaces in areas of
high(>200cm) and seasonal rainfall. The alternating wet and dry seasons lead to the leaching away
of the siliceous matter of the rocks leaving behind the compounds of iron and aluminium. These are
zonal soils.
www.achieveias.co.in, YouTube Channel: http://youtube.com/c/AchieveIAS Telegram Channel: http://t.me/Achieve_Ias,
Mail: achieveias21@gmail.com, Contact Number: 8968920720

Areas: These soils have mainly developed in the higher areas of the Peninsular plateau. The laterite
soils are commonly found in Karnataka, Kerala, Tamil Nadu, Madhya Pradesh and the hilly areas of
Orissa and Assam.
Soil Colour: Reddish brown in colour due to the presence of iron oxide.

Other characteristic features:


1. With rain, lime and silica are leached away, and soils rich in iron oxide and aluminium compound
are left behind (thus the reddish brown colour). Also, humus content of the soil is removed fast by
bacteria that thrives well in high temperature.
2. These soils represent the end product of decomposition and are generally low in fertility.
3. The pebbly crust is the important feature of laterites which is formed due to alteration of wet and
dry periods.
4. These soils are acidic in character due to leaching. Application of manures and fertilisers is
required for making these soils fertile for cultivation.
5. These soils are poor in organic matter, nitrogen, phosphate and calcium, while iron oxide and
potash are in excess.

Suitable crops: Red laterite soils in Tamil Nadu, Andhra Pradesh and Kerala are more suitable for
tree crops like cashewnut. These soils are also suitable for tea plantations.
www.achieveias.co.in, YouTube Channel: http://youtube.com/c/AchieveIAS Telegram Channel: http://t.me/Achieve_Ias,
Mail: achieveias21@gmail.com, Contact Number: 8968920720

ACHIEVE IAS GEOGRAPHY MCQ SERIES, DAY 39, SOLUTIONS

1. Consider the following statements:


1. Natural Hazards are elements of circumstances in the Natural environment that have the potential
to cause harm to people or property or both.
2. Natural disasters are relatively sudden and cause large scale, widespread death, loss of property
and disturbance to social systems and life over which people have a little or no control.

Which of the statements given above is/are correct?


A. 1 only B. 2 only C. Both 1 and 2 D. None of these

Answer: C

Explanation: Natural disasters can be categorized as "acute" or "slow" in their onset. They are
predictable because they cluster in geographic areas. Natural hazards are unpreventable and, for
the most part, uncontrollable. Even if quick recovery occurs, natural disasters can have long term
effects. Natural disasters with acute onsets include events such as earthquake, flood, hurricane or
typhoon, tornado, fire, tsunami or storm surge, avalanche, volcanic eruption, extreme cold or
blizzard, and heat wave. Natural hazards with a slow or gradual onset include drought, famine,
desertification, deforestation, and pest infestation.

Natural disasters – such as hurricanes, cyclones, earthquakes, mudslides, floods, wildfires, volcanic
eruptions and weather events like extreme droughts and monsoons – are likely increasing in
frequency due to climate change. These events bring with them a host of issues, including
humanitarian, public health, environmental and infrastructural problems.

1. Humanitarian Crises: Climate change and accompanying natural disasters have created a large
migrant population, called climate refugees or environmental migrants. These people can be been
forced out of their homes by an abrupt natural disaster, like a tsunami, or a slower-moving natural
disaster, like a relentless drought. In any case, the area where they formerly lived is no longer
habitable for one reason or another, or the standard of living has dropped so drastically that the
uncertain future of migration looks more promising. It is predicted that by the end of the century
there will be 2 billion climate refugees and environmental migrants. Out of a projected population of
11 billion by 2100, that is almost 1/5 of the people on earth. Most of these people will have lived
along the coastlines

2. Public Health Issues: Health issues are one of the most pressing problems after any natural
disaster. It is often the case that facilities for water and toilet hygiene are damaged or inoperable:
meaning that the safe disposal of human waste quickly becomes a public health hazard. Further,
without running water, hand washing and food hygiene rapidly deteriorate. During and after events
like hurricanes and floods, standing water can be a breeding ground for pathogenic bacteria and
disease vectors like mosquitoes. In cases where transportation capabilities and infrastructure are
damaged, survivors of natural disasters can be cut off from life-saving medications for both acute
and chronic conditions, and be isolated from rescue and emergency healthcare services. After a
natural disaster event, survivors can experience mental health consequences, including post-
traumatic stress disorder, or PTSD.
www.achieveias.co.in, YouTube Channel: http://youtube.com/c/AchieveIAS Telegram Channel: http://t.me/Achieve_Ias,
Mail: achieveias21@gmail.com, Contact Number: 8968920720

3. Environmental Problems: In March 2011, a tsunami following the 9.0-magnitude Tōhoku


earthquake in Japan caused what came to be known as the Fukushima Daiichi nuclear disaster,
where radioactive material was released in Japan and into the Pacific Ocean. This was the largest
nuclear disaster since Chernobyl, and it caused a cascade of issues in the ecosystem and
surrounding waters, spreading radioactive material through far-ranging ocean currents.Natural
disasters, from tsunamis to wildfires, can cause wide-ranging and long-term consequences for
ecosystems: releasing pollution and waste, or simply demolishing habitats.

4. Infrastructural Damage: One of the most immediate and economically devastating concerns with
natural disasters is the damage to both public and private infrastructure. These events can cause
billions of dollars in damages, and not all governments are equipped to fund the process of post-
disaster cleanup and rebuilding.Natural disasters can have long-term negative consequences
beyond the immediate loss of life and demolition of infrastructure. Often, an area impacted by a
natural disaster will show scars of the event for years to come.

2. Tropical Cyclones are intense low pressure areas confined to the area lying between
A. 30 degree north and 30 degree south.
B. 50 degree north and 50 degree south.
C. 50 degree north and 30 degree south.
D. 5 degree north and 5 degree south.

Answer: A

Explanation: Tropical cyclones are violent storms that originate over oceans in tropical areas and
move over to the coastal areas bringing about large scale destruction due to violent winds (squalls),
very heavy rainfall (torrential rainfall) and storm surge. They are irregular wind movements
involving closed circulation of air around a low pressure center. This closed air circulation (whirling
motion) is a result of rapid upward movement of hot air which is subjected to Coriolis force. The low
pressure at the center is responsible for the wind speeds. The cyclonic wind movements are anti-
clockwise in the northern hemisphere and clockwise in the southern hemisphere (This is due
to Coriolis force).The cyclones are often characterized by existence of an anticyclone between two
cyclones.

Conditions Favourable for Tropical Cyclone Formation:


1. Large sea surface with temperature higher than 27° C,
2. Presence of the Coriolis force enough to create a cyclonic vortex,
3. Small variations in the vertical wind speed,
4. A pre-existing weak low-pressure area or low-level-cyclonic circulation,
5. Upper divergence above the sea level system.

3. The Indian Tsunami Early Warning Centre (ITEWC) established at Indian National Centre
for Ocean Information Sciences is located in:
A. Chennai B. Goa C. Kochi D. Hyderabad

Answer: D

Explanation: The Indian Tsunami Early Warning Centre (ITEWC) established at Indian National
Centre for Ocean Information Sciences, (INCOIS - ESSO) Hyderabad, autonomous body under
www.achieveias.co.in, YouTube Channel: http://youtube.com/c/AchieveIAS Telegram Channel: http://t.me/Achieve_Ias,
Mail: achieveias21@gmail.com, Contact Number: 8968920720

Ministry of Earth Sciences, is being upgraded continuously to provide tsunami advisories for the
events occurring in the global oceans, though it has been recognized as one of the best systems in
the world. The major upgradation work would be Standardization of the Operating Procedures,
bulletin formats and terminologies with warning centres operating in other global basins. To achieve
this, the Intergovernmental Oceanographic Commission (IOC of UNESCO) has set up a task team
comprising of experts from tsunami warning centres of all ocean basins, with India as the Chair. The
ITEWC encompasses a real-time seismic monitoring network of 17 broadband seismic stations to
detect tsunamigenic earthquakes, a network of real-time sea-level sensors with 4 Bottom Pressure
Recorders (BPR) in the open ocean and 25 tide gauge stations at different coastal locations monitor
tsunamis and a 24 X 7 operational tsunami warning centre to provide timely advisories to vulnerable
community. It also receives earthquake data from all other global networks to detect earthquakes of
M>6.5. The state-of-the-art early warning centre at INCOIS - ESSO is operational since October 15,
2007 with all the necessary computational and communication infrastructure that enables reception
of real-time data from seismic & sea-level sensors, analysis of the data, tsunami modeling, and
dissemination of tsunami advisories guided by a comprehensive Standard Operating Procedure
(SOP). A host of all available communication technology options have been employed for timely
dissemination of advisories to various designated authorities to deal with effective emergency
response actions as appropriate. The centre is capable of detecting tsunami genic earthquakes
occurring in the whole of Indian Ocean region as well as in the Global Oceans within 10 minutes of
their occurrence and disseminates the advisories to the concerned authorities within 20 minutes
through various modes of communication like email, fax, SMS, GTS and website. Since its inception
in October 2007 to till date, ITEWC has monitored 339 earthquakes of M > 6.5 out of which 63 are
in the Indian Ocean region. ITEWC also acts as one of the Regional Tsunami advisory Service
Provider (RTSP) along with Australia & Indonesia for the Indian Ocean region. As the oceans on
the earth are interconnected, the tsunami waves generated due to any great earthquakes in the
global oceans can affect the Indian Coasts. In order to protect our coasts from tsunamis up-grading
the present system is very essential. Up-gradation of ITEWC will also enhance its capability to
provide tsunami advisories to the other needy countries in the world. The basic infra-structure and
the necessary computational facilities are established while setting up the Indian Tsunami Early
Warning Centre and hence no major hardware upgradations are proposed. The maintenance of the
entire early warning system is carried out with a budget allocation of Rs. 17.00 Crores per annum.
Model simulations required for global operations would be run as part of this itself. Additional data
required for the enhancement of ITEWC for global operations can be obtained by collaborations with
centres operating in other countries. Currently, there are 1800 coastal forecast points covering
coastal areas of the entire Indian Ocean region. The necessary actions have been initiated to
upgrade the same for covering other the coastal regions.

4. With reference to flood consider the following statements:


1. National Flood Management Programme was launched in 1954.
2. Disturbances along the natural drainage channels and colonisation of flood-plains and river-beds
are some of the human activities that play an important role in increasing the intensity, magnitude
and gravity of floods.

Which of the statements given above is/are correct?


A. 1 only B. 2 only C. Both 1 and 2 D. Neither 1 nor 2

Answer: C
www.achieveias.co.in, YouTube Channel: http://youtube.com/c/AchieveIAS Telegram Channel: http://t.me/Achieve_Ias,
Mail: achieveias21@gmail.com, Contact Number: 8968920720

Explanation: Floods are natural occurrences where an area or land that is normally dry abruptly
becomes submerged in water. In simple terms, flood can be defined as an overflow of large
quantities of water onto a normally dry land. Flooding happens in many ways due to overflow of
streams, rivers, lakes or oceans or as a result of excessive rain. Whenever flooding takes place,
there is the possibility of loss of life, hardship to people, and extensive damage to property. This is
because flooding can carry bridges, cars, houses, and even people. Flooding also destroys crops
and can wipe away trees and other important structures on land. Some floods occur abruptly and
recede quickly whereas others take several days or even months to form and to recede because of
variation in size, duration, and the area affected. “A flood is an overflow of water that submerges
land which is usually dry. The European Union (EU) Floods Directive defines a flood as a covering
by water of land not normally covered by water. In the sense of “flowing water”, the word may also
be applied to the inflow of the tide.” Causes of Flooding: Many conditions result in flooding.
Hurricanes, clogged drainages, and rainfall are some of the conditions that have led to flooding in
various regions across the globe. Here are the leading causes of flooding.
1. Rain : Rain is the leading contributor to most of the flooding cases witnessed across the world.
Too much rain causes water to flow overland contributing to flooding. In particular, it is due to high
rainfall intensity over a prolonged period. Depending on the rainfall distribution, the amount of rain,
and soil moisture content, short rainfall period can also result in flooding. Light rains for longer
periods – several days or weeks, can also result in floods. The rain water erosive force can weaken
the foundations of buildings, causing tumbles and cracks.
2. River Overflows: Rivers or streams can overflow their banks. This happens when the river or
stream holds more water upstream than usual, and it flows downstream to the neighboring low-lying
areas, typically referred to as the floodplains. As a consequence, this creates a sudden discharge
of water into the adjacent lands leading to flooding. Dams in rivers may also at times overwhelm
rivers when the carriage capacity is exceeded, causing the water to burst and get into the
floodplains. Flood caused by river overflow has the potential of sweeping everything in its path
downstream.
3. Lakes and Coastal Flooding: Lake and Coastal flooding occurs when large storms or tsunamis
causes the water body to surge inland. These overflows have destructive power since they can
destroy ill-equipped structures to withstand water’s strength such as bridges, houses, and cars.
In the coastal areas, strong and massive winds and hurricanes drive water onto the dry coastal lands
and give rise to flooding. The situation is even worsened when the winds blowing from the ocean
carry rains in them. Sea waters from the tsunami or hurricane can cause widespread damage.
4. Dam Breakage: Dams are man-made structures used to hold water from flowing down from a
raised ground. The potential energy stored in the dam water is used to generate electricity. At times,
the walls can become weak and break because of overwhelming carriage capacity. Due to this
reason, breakage of the dam can cause extensive flooding in the adjacent areas.
Flooding occurs when the embankments built along the sides of the river to stop high water from
flowing onto the land breaks. Sometimes, the excess water from the dam is deliberately released
from the dam to prevent it from breaking thereby causing floods.
5. Melting of the Glaciers and Mountain Tops: In the cold regions, ice and snows build up during
the winters. When the temperature rises in summer, the accumulated snows and ice are subjected
to melting resulting in vast movements of water into lands that are normally dry. Regions with
mountains that have ice on top of them also experience the same outcome when the atmospheric
temperature rises. This type of flooding is usually termed as snowmelt flood.
6. Clogged Drainages: Flooding also takes place when snowmelt or rainfall runoff cannot be
channeled appropriately into the drainage systems forcing the water to flow overland. Clogged or
lack of proper drainage system is usually the cause of this type of flooding.
www.achieveias.co.in, YouTube Channel: http://youtube.com/c/AchieveIAS Telegram Channel: http://t.me/Achieve_Ias,
Mail: achieveias21@gmail.com, Contact Number: 8968920720

The areas remain flooded until the stormwater systems or waterways are rectified. Instances where
the systems or water ways are not rectified, the areas remain flooded until the excess water
evaporates or is transpired into the atmosphere by plants.
National Flood Management Programme:1. Launched in 1954, different methods of flood
protection structural as well as non-structural have been adopted in different states depending upon
the nature of the problem and local conditions.
2. Structural measures include storage reservoirs, flood embankments, drainage channels, anti-
erosion works, channel improvement works, detention basins etc. and non-structural measures
include flood forecasting, flood plain zoning, flood proofing, disaster preparedness etc.
3. Since then more than 35,000 km of embankments has been constructed and more than 39,000
km of drainage channels improved which could absorb and regulate peak floods when necessary.

5. Which of these is/are flood prevention and mitigation strategy?


1. Construction of flood protection embankments
2. Depopulating the flood plains
3. Afforestation
4. Decongesting river channels

Select the correct option using the codes given below


A. 1 and 2 only . 1, 2 and 3 only C. 2, 3 and 4 only D. 1, 2, 3 and 4

Answer: D

Explanation: With the advancement of technology such as satellite and remote-sensing


equipment, flood waves can be tracked as the water level rises. Evacuation is possible with suitable
monitoring and warning. A warning is issued by the Central Water Commission (CWC), Irrigation
and Flood Control Department and Water Resources Department.
1. Mapping of the Flood Prone Areas: Flood hazard mapping will give the proper indication of water
flow during floods. Land use Control It will reduce danger of life and property, when the waters
inundate the flood plains and the coastal areas. No major development should be permitted in the
areas which are subjected to high flooding.
2. Flood Control: It aims to reduce flood damage. This can be done by decreasing the amount of
run-off with the help of reforestation. Flood diversion includes levees, embankments, dams and
channel improvement. Dams can store water and can release water at a manageable rate. But the
failure of dams in earthquakes and operation of releasing the water can cause floods in the lower
areas.
3. Flood Proofing: It reduces the risk of damage. Measures include use of sand bags to keep flood
water away, blocking or sealing of doors and windows of houses, etc. Restoration of Original
Drainage System Drainage system is generally choked by the construction of roads, canals railway
tracks etc. Floods could be checked if the original form of drainage system is restored.

6. Consider the following statement (s) is/are related to the strategy to combat earthquake
disasters.
1. Establishing earthquake monitoring centres for regular monitoring and fast dissemination of
information among the people in the vulnerable areas.
2. Modifying the house types and building designs in the vulnerable areas and discouraging
contribution of high rise buildings, large industrial establishments and big urban centres in such
areas.
www.achieveias.co.in, YouTube Channel: http://youtube.com/c/AchieveIAS Telegram Channel: http://t.me/Achieve_Ias,
Mail: achieveias21@gmail.com, Contact Number: 8968920720

A. Only 1 B. Only 2 C. Both 1& 2 D. Neither 1 nor 2


Answer: C

Explanation: Earthquake is one of the major natural disasters. Every year it causes a huge loss of
life and property all around the world. Proper strategies and awareness about disaster
management is the need of the hour. This will protect us from the losses and help to cope up with
these hazards. However, in order to reduce the destruction caused by these occurrences the
disaster management committee has laid some safety measures. An earthquake is the shaking of
the surface of Earth due to the sudden release of energy in the Earth’s crust, as a result, seismic
waves (also known as S waves)are created. The seismic activities in an area determine the type
and intensity of the earthquake. Earthquakes are caused due to sudden tectonic movements in the
earth’s crust. When the tectonic plates slide over one another, there is a cause of orogeny which
results in earthquakes and volcanoes. These disturbances cause vibrations which spread in all the
directions. As there is a relative motion of these plates, there is stress built up, which breaks by
releasing the stored energy known as shock waves.

A. Develop a Culture of Preparedness:


1. Every household, government agency, and business must know the seismic risks of the buildings
they occupy, the transportation systems they use, and the utilities that serve them, as well as the
actions they can take to protect themselves.
2. Every household, government agency, and business needs to be prepared to be self-sufficient
for at least three days (72 hours) following a disaster.
3. Citizens and governments need to take steps to ensure adequate response care for special needs
and vulnerable populations.
4. Government agencies, the region’s major industries, and earthquake professionals have to work
together to prepare the region to respond to and recover from major earthquakes. This can be done
through region-wide, multi-organizational plans, training, exercises and coordination assessments,
as well as continuing improvements in our collective understanding of seismic risks.

B. Invest in Reducing Losses:


1. Building owners, governments, and the earth science and engineering professions must target
potential collapse-hazard buildings for seismic mitigation, through retrofit, reduced occupancy, or
reconstruction.
2. Governments and other relevant agencies must retrofit or replace all facilities essential for
emergency response to ensure that they function following earthquakes. These facilities include fire
and police stations, emergency communications centers, medical facilities, schools, shelters, and
other community- serving facilities.
3. Governments and other relevant agencies must set priorities and retrofit or replace vulnerable
response and community-serving infrastructure, including cellular communications, airports, ports,
roads and bridges, transportation, water, dams and levees, sewage and energy supplies, to ensure
that functions can be resumed rapidly after earthquakes.

C. Ensure Resiliency in Recovery:


1. Government agencies, the region’s major industries, and earthquake professionals have to plan
collaboratively for the housing, both short- and long-term, of residents displaced by potential fires,
large numbers of uninhabitable buildings, and widespread economic and infrastructure disruption
following a major earthquake.
www.achieveias.co.in, YouTube Channel: http://youtube.com/c/AchieveIAS Telegram Channel: http://t.me/Achieve_Ias,
Mail: achieveias21@gmail.com, Contact Number: 8968920720

2. Every household, government agency, and business has to assess and plan for financing the
likely repair and recovery costs following a major earthquake.
3. Federal, state and local governments, the insurance industry, and the region’s major industries
have to collaborate to ensure adequate post-event funding to provide economic relief to individuals
and communities after a major earthquake, when resources are most scarce yet crucial for recovery
and reconstruction.

7. Consider the following statement (s) is/are related to the characteristics of tsunami waves.
1. Tsunamis are high energy sea waves caused mainly by the deep focus earthquakes of high
magnitude.
2. Preparing a vulnerability map of the country and dissemination of vulnerability risk information
among the people.

A. Only 1 B. Only 2 C. Both 1 & 2 D. Neither 1 nor 2

Answer: C

Explanation: Tsunamis are giant waves caused by earthquakes or volcanic eruptions under the
sea. Out in the depths of the ocean, tsunami waves do not dramatically increase in height. But as
the waves travel inland, they build up to higher and higher heights as the depth of the ocean
decreases. The speed of tsunami waves depends on ocean depth rather than the distance from the
source of the wave. Tsunami waves may travel as fast as jet planes over deep waters, only slowing
down when reaching shallow waters. While tsunamis are often referred to as tidal waves, this name
is discouraged by oceanographers because tides have little to do with these giant waves.

MITIGATION MEASURES:
1. Effective Planning
2. Building of walls as done by Japan.
3. Planting trees as don in Tamil Nadu by a village
4. Proper relief and rehabilitation preparedness
5. Awareness among the masses.
Tsunamis are characterized as shallow-water waves. Shallow-water waves are different from wind-
generated waves, the waves many of us have observed at the beach. Wind-generated waves
usually have period (time between two successional waves) of five to twenty seconds and a
wavelength (distance between two successional waves) of about 300 to 600 ft. (100 to 200 meters).
A tsunami can have a period in the range of ten minutes to two hours and a wavelength in excess
of 300 miles (500 km). It is because of their long wavelengths that tsunamis behave as shallow-
water waves. A wave is characterized as a shallow-water wave when the ratio between the water
depth and its wavelength gets very small. The speed of a shallow-water wave is equal to the square
root of the product of the acceleration of gravity (32ft/sec/sec or 980cm/sec/sec) and the depth of
the water. The rate at which a wave loses its energy is inversely related to its wavelength. Since a
tsunami has a very large wavelength, it will lose little energy as it propagates. Hence in very deep
water, a tsunami will travel at high speeds and travel great transoceanic distances with limited
energy loss. For example, when the ocean is 20,000 ft. (6100 m) deep, unnoticed tsunamis travel
about 550 miles per hour (890 km/hr), the speed of a jet airplane. And they can move from one side
of the Pacific Ocean to the other side in less than one day. As a tsunami leaves the deep water of
the open sea and propagates into the more shallow waters near the coast, it undergoes a
transformation. Since the speed of the tsunami is related to the water depth, as the depth of the
www.achieveias.co.in, YouTube Channel: http://youtube.com/c/AchieveIAS Telegram Channel: http://t.me/Achieve_Ias,
Mail: achieveias21@gmail.com, Contact Number: 8968920720

water decreases, the speed of the tsunami diminishes. The change of total energy of the tsunami
remains constant. Therefore, the speed of the tsunami decreases as it enters shallower water, and
the height of the wave grows. Because of this “shoaling” effect, a tsunami that was imperceptible in
deep water may grow to be several feet or more in height. When a tsunami finally reaches the
shore, it may appear as a rapidly rising or falling tide, a series of breaking waves, or even a bore.
Reefs, bays, entrances to rivers, undersea features and the slope of the beach all help to modify
the tsunami as it approaches the shore. Tsunamis rarely become great, towering breaking waves.
Sometimes the tsunami may break far offshore. Or it may form into a bore: a step-like wave with a
steep breaking front. A bore can happen if the tsunami moves from deep water into a shallow bay
or river. The water level on shore can rise many feet. In extreme cases, water level can rise to more
than 50 ft. (15 m) for tsunamis of distant origin and over 100 ft. (30 m) for tsunamis generated near
the earthquake’s epicenter. The first wave may not be the largest in the series of waves. One coastal
area may see no damaging wave activity while in another area destructive waves can be large and
violent. The flooding of an area can extend inland by 1000 ft. (305 m) or more, covering large
expanses of land with water and debris. Flooding tsunami waves tend to carry loose objects and
people out to sea when they retreat. Tsunamis may reach a maximum vertical height onshore above
sea level, called a runup height, of 98 ft. (30 meters). A notable exception is the landslide-generated
tsunami in Lituya Bay, Alaska in 1958, which produced a 1722 ft. wave (525 m).

8. Consider the following statement (s) is/are related to the remedial steps to reduce the risk
of life and properties by landslides.
1. Mapping of landslide prone areas a7 construction of houses, felling of trees and grazing in
landslide prone areas should be prohibited or restricted.
2. Afforestation in the vulnerable areas is an effective way of durable landslides control.

A. Only 1 B. Only 2 C. Both 1 & 2 D. Neither 1 nor 2

Answer: C

Explanation: There are wide varieties of the names given to the denudation process whereby soil
or rocks are displaced along the slope by mainly gravitational forces and landslides are one
them. Landslides refer to the movement of mass of rock, debris or earth down the slope, when the
shear stress exceeds the shear strength of the material. It occurs when the consequence of a
complex field of forces (stress is a force per unit area) active on a mass of rock or soil on the slope.
It happens due to geological causes, morphological causes, physical causes and human causes.
There are two parameters that determine the landslides are as follows:
1. Increase of shear stress: It happen due to the removal of lateral and underlying support; increase
of lateral forces as well as load; transitory stresses like blasting, earthquakes etc.; and geological
movement.
2. Decrease of material strength: It happens due to the weathering, pore water pressure and
changes in structure.

The incidents of landslides are increasing day by day due to the over urbanisation, massive
deforestation, construction and development work in landslide prone areas.

How landslides can mitigate?


1. By restricting or even removing population from landslides prone areas.
2. By restricting certain types of land use where slopes are vulnerable.
www.achieveias.co.in, YouTube Channel: http://youtube.com/c/AchieveIAS Telegram Channel: http://t.me/Achieve_Ias,
Mail: achieveias21@gmail.com, Contact Number: 8968920720

3. By installing early warning systems based on the monitoring of ground conditions such as strain
in rocks and soils, slope displacement, and groundwater levels.

Remedial steps for landslides:


1. Modification of Slope Geometry: In order to improve the stability of the unstable or potentially
unstable slopes, the profile of the slope is sometimes changed by excavation or by filling at the toe
of the slope.
2. Drainage Control: The presence of water in joints or in soil slope has a fundamental influence on
the slope stability.
3. Internal Slope Reinforcement Systems: The aim of rock slope stabilization with structural
elements is to help the rock mass to support itself by applying external structures which are not part
of the rock mass but support it externally.
4. Retaining Walls: Construction of wall along the problematic slopes area.

9. Which of the following are immediate hazardous effects of earthquake?


1. Soil erosion
2. Ground lurching
3. Tsunami
4. Floods from dams

Select the correct answer using following codes:


1. 3 Only 2. 1, 3 and 4 only 3. 2, 3 and 4 only 4. 1, 2, 3 and 4

Answer: C

Explanation: The following are the immediate hazardous effects of earthquake: Ground Shaking,
Differential ground settlement, Land and mud slides, Soil liquefaction, Ground lurching, Avalanches,
Ground displacement, Floods from dam and levee failures, Tsunami.

10. Consider the following statements


1. The point where energy is released is called epicentre of earth.
2. The point on the surface nearest to focus is called hypocentre.

Select the correct answer using following codes.


a. 1 Only b. Only 2 c. Both 1 and 2 d. Neither 1 nor 2

Answer: D

Explanation: The point where the energy is released is called the focus of an earthquake,
alternatively, it is called the hypocentre. The energy waves travelling in different directions reach the
surface. The point on the surface, nearest to the focus, is called epicentre.
www.achieveias.co.in, YouTube Channel: http://youtube.com/c/AchieveIAS Telegram Channel: http://t.me/Achieve_Ias,
Mail: achieveias21@gmail.com, Contact Number: 8968920720

ACHIEVE IAS GEOGRAPHY MCQ SERIES, DAY 40, SOLUTIONS

1. Consider the following statements about the population density/growth in India. Which are
Correct?
1. Assam and most of the peninsular states have moderate population densities
2. The Northern Plains and Kerala in the south have high to very high population densities
3. Since 1981 the rate of population growth started declining gradually in India
4. Meghalaya is having the highest decadal growth in the 2011 census

A. 1 & 2 Only B. 1, 2 & 4 Only C. 1, 2 & 3 Only D. All the above

Answer: D

Explanation: Areas of Moderate Density: Gujarat (258), Karnataka (276), Andhra Pradesh (277),
Tripura (305), Maharashtra (315), Jharkhand (338), Assam (340), Goa (364), Dadra and Nagar
Haveli (449), Haryana (478), Tamil Nadu (480) and Punjab (484) are included in this category.
Since 1981, the rate of growth started declining gradually, because of popularized.
(a) Family planning measures were adopted, leading to decline in the birth rate.
(b) Awareness about advantages of small family came to be recognized.
(c) There was a growth of nuclear families occured which adopted the small family norm.
(d) Promotion of family planning programme by the government.

The population of Meghalaya is only 0.25 percent of the total population of India. The decadal
growth rate of India between 2001 and 2011 census is found to be 17.68, whereas decadal
growth rate of Meghalaya in the decade is 27.95. ... population between 2001-2011 is 28.25 which
is higher than the State growth rate.

2. Consider the following statement (s) related to Human resources.


1. The environmental factors such as high altitude, extreme cold, aridity, relief, climate, soil,
vegetation types, mineral, and energy resources influences the population distribution
2. Technological and economic advancements influences the population distribution

Which is / are correct option?


A. 1 Only B. 2 Only C. Both 1 and 2 D. Neither 1 nor 2

Answer: C

Explanation: We as a human beings are social animals. We need to interact and co-operate with
other people for various purposes, such as requiring their help, their assistance in our day to day
work and making them understand the intent and purpose of our actions. Now what we saw here is
the need of other people in our day to day life but when you take the perspective of a whole nation,
you need to understand the need of people on a macro level. Without question, people of a nation
are its greatest asset. It is their everyday skills and abilities that turn them into a resource for the
nation. Healthy, educated and motivated people develop resources as per their requirement.
Distribution of Population: Population means how many individuals reside in an area, state,
country, continent or the world. Now they main aspect of these individuals is that every individual
differs either in age, sex, and the location of his/her residence. Now, the distribution of humans in
the world is extremely uneven. The reason population changes over time is that of births, deaths
www.achieveias.co.in, YouTube Channel: http://youtube.com/c/AchieveIAS Telegram Channel: http://t.me/Achieve_Ias,
Mail: achieveias21@gmail.com, Contact Number: 8968920720

and migration of the people. Studies help the government to plan areas such as health, education,
housing, social security and employment. In short, distribution of the population is in which people
are spread across the Earth’s surface. As it is displayed that the population of the world is extremely
uneven. Areas in Asia and some parts of Europe are crowded while some are sparsely populated.
90 percent of the world’s population live in 10 percent of the world’s area. Features such as
topography of the region are one of the reasons for the area being highly populated or not. High
mountainous areas, tropical deserts and areas of equatorial forests are sparsely populated. Areas
which are North to the equator are more populated than the areas South of the equator because the
climate and topography are warmer for humans to habitat and life to grow. Three-quarter of the
world’s people live in Asia and Africa. Almost 60 percent of the world’s inhabitants live in just 10
countries.

Factors Affecting the Distribution of Population:

1. Geographical Factors: A. Topography– Highly dense areas where people live are in plains rather
than plateau and mountains. Because plains are suitable for farming, transportation and many other
factors such as water and electricity. The Ganges plains are the most densely populated area of the
world.
B. Climate– The reason most people live north of the equator is that of the presence of moderate
climate and to avoid extreme climates. Therefore, less number of people live in Sahara desert and
in polar regions of Russia and Canada.
C. Soil– Fertile plains of the Ganges and Brahmaputra (India), Hwang-he, Chang Jiang (China) and
the Nile (Egypt) are the areas with a high population density.
D. Water and Minerals– Areas with Water and Mineral pull people because of the potential of
economic development. Therefore, fewer people live in a desert than in river valleys.

2. Social, Cultural and Economic factors: A. Social– Regions were there is better housing, education
and health facilities are more densely populated. Example, Pune and Bangalore in India.
B. Cultural– Cities with cultural and religious significance attract people. Example, Varanasi,
Jerusalem and Vatican City are densely populated.
C. Economic– Where there is the emergence of potential industry, people come flooding in there.
Mumbai’s population is growing because of this reason alone that it is home to multiple industries.

3. Which of the following state's population growth rate is around or equal to India's
population growth rate?
(A) Chandigarh, Uttarakhand, Assam
(B) Andhra Pradesh, Himachal Pradesh, West Bengal
(C) Maharashtra, Bihar, Uttar Pradesh
(D) Madhya Pradesh, Karnataka, Arunachal Pradesh

Answer: A

Explanation: The population growth rate of Chandigarh, Uttarakhand and Assam is around or equal
to India's population growth rate (17%)

4. Consider the following statement (s) related to rural settlements.


1. On the basis of setting: The main types are plain villages, plateau villages, coastal villages, forest
villages and desert villages.
www.achieveias.co.in, YouTube Channel: http://youtube.com/c/AchieveIAS Telegram Channel: http://t.me/Achieve_Ias,
Mail: achieveias21@gmail.com, Contact Number: 8968920720

2. On the basis of functions: There may be farming villages, fishermen’s villages, lumberjack
villages, pastoral villages etc.
Which is / are correct option?
A. 1 Only B. 2 Only C. Both 1 and 2 D. Neither 1 nor 2

Answer: C

Explanation: The rural settlements are concerned with the degree of dispersion of the dwellings
and the life is supported by land based primary economic activities. Rural people are less mobile
and therefore, social relations among them are intimate. In India, the rural settlement varies with the
diversity of climatic condition in India that is compact or clustered village of a few hundred houses
is a rather universal feature, particularly in the northern plains. The rural settlements are concerned
with the degree of dispersion of the dwellings and the life is supported by land based primary
economic activities. Rural people are less mobile and therefore, social relations among them are
intimate. In India, the rural settlement varies with the diversity of climatic condition in India that is
compact or clustered village of a few hundred houses is a rather universal feature, particularly in the
northern plains. However, there are several areas, which have other forms of rural settlements.
There are various factors and conditions responsible for having different types of rural settlements
in India which is given below:
1. Physical features – nature of terrain, altitude, climate and availability of water
2. Cultural and ethnic factors – social structure, caste and religion
3. Security factors – defence against thefts and robberies.

Rural settlements in India:


A. Clustered, agglomerated or nucleated: It is a compact or closely built up area of houses. In this
type of village the general living area is distinct and separated from the surrounding farms, barns
and pastures.
B. Semi-clustered or fragmented: These types of settlements may result from tendency of
clustering in a restricted area of dispersed settlement. More often such a pattern may also result
from segregation or fragmentation of a large compact village.
C. Hamleted: Sometimes settlement is fragmented into several units physically separated from
each other bearing a common name. These units are locally called panna, para, palli, nagla, dhani,
etc. in various parts of the country. This segmentation of a large village is often motivated by social
and ethnic factors.
D. Dispersed or isolated: This pattern of settlement appears in the form of isolated huts or hamlets
of few huts in remote jungles, or on small hills with farms or pasture on the slopes. Extreme
dispersion of settlement is often caused by extremely fragmented nature of the terrain and land
resource base of habitable areas.

5. Consider the following statement (s) related to composition of population.


1. The population is usually divided into three age groups- children (0-14yrs), adults (15-59yrs) and
aged (60 and over).
2. Population of males and females, children, young and old comprises the population of a country.

Which is / are correct option?


A. 1 Only B. 2 Only C. Both 1 and 2 D. Neither 1 nor 2

Answer: C
www.achieveias.co.in, YouTube Channel: http://youtube.com/c/AchieveIAS Telegram Channel: http://t.me/Achieve_Ias,
Mail: achieveias21@gmail.com, Contact Number: 8968920720

Explanation: Population of males and females, children, young and old comprises the population
of a country. The population is usually divided into three age groups- children (0-14yrs), adults (15-
59yrs) and aged (60 and over). This is called age-group of population. The proportion of adult
population is the least variable in the three groups. The main difference is found in the population of
children and old people.

6. Consider the following statement (s) related to pillars of human development.


1. Equity means making equal access to opportunities available to everybody that opportunities
available to people must be equal irrespective of their gender, race, income and in the Indian case,
caste.
2. Sustainability to the human labour productivity or productivity in terms of human work that must
be constantly enriched by building capabilities in people.

Which is / are correct option?


A. 1 Only B. 2 Only C. Both 1 and 2 D. Neither 1 nor 2

Answer: A

Explanation: The term ‘human development’ may be defined as an expansion of human


capabilities, a widening of choices, ‘an enhancement of freedom, and a fulfilment of human rights.
At the beginning, the notion of human development incorporates the need for income expansion.
However, income growth should consider expansion of human capabilities. Hence development
cannot be equated solely to income expansion. The first UNDP Human Development Report
published in 1990 stated that: “The basic objective of development is to create an enabling environ-
ment for people to enjoy long, healthy and creative lives.” It also defined human development as “a
process of enlarging people’s choices”, “and strengthen human capabilities” in a way which enables
them to lead longer, healthier and fuller lives.

Components of Human Development:


1. Equality,
2. Sustainability,
3. Productivity, and
4. Empowerment.

1. Equality: If development is viewed in terms of enhancing people’s basic capabilities, people must
enjoy equitable access to opportunities. Such may be called equality-related capabilities. To ensure
equality-related capabilities or access to opportunities what is essential is that the societal
institutional structure needs to be more favourable or progressive. In other words, the unfavourable
initial asset distribution, like land, can be made more farmer-friendly through land reform and other
redistributive measures. In addition, uneven income distribution may be addressed through various
tax-expenditure policies. Economic or legislative- measures that interferes with market exchange
may enable people to enlarge their capabilities and, hence, well-being. Further, to ensure basic
equality, political opportunities need to be more equal. In the absence of effective political
organisation, disadvantaged groups are exploited by the ‘rich’ to further their own interests rather
than social goals. However, participatory politics gets a beating by the inequality in opportunities in
having basic education. It is to be added here that basic education serves as a catalyst of social
change. Once the access to such opportunity is opened up in an equitable way, women or religious
www.achieveias.co.in, YouTube Channel: http://youtube.com/c/AchieveIAS Telegram Channel: http://t.me/Achieve_Ias,
Mail: achieveias21@gmail.com, Contact Number: 8968920720

minorities or ethnic minorities would be able to remove socio-economic obstacles of development.


This then surely brings about a change in power relations and makes society more equitable.

2. Sustainability: Another important facet of human development is that development should ‘keep
going’, should ‘last long’. The concept of sustainable development focuses on the need to maintain
the long term protective capacity of the biosphere. This then suggests that growth cannot go on
indefinitely; there are, of course, ‘limits to growth. Here we assume that environment is an essential
factor of production. In 1987, the Bruntland Commission Report (named after the then Prime Minister
Go Harlem Bruntland of Norway) defined sustainable development as ‘… development that meets
the needs of the present without compromising the ability of future generations to meet their basic
needs.’ This means that the term sustainability focuses on the desired balance between future
economic growth and environ-mental quality. To attain the goal of sustainable development, what
is of great impor-tance is the attainment of the goal of both intra- generation and inter-generation
equality. This kind of inequality includes the term ‘social well-being’ not only for the present
generation but also for the people who will be on the earth in the future. Any kind of environmental
decline is tantamount to violation of distributive justice of the disadvan-taged peoples. Social well-
being thus, then, depends on environ-mental equality.

3. Productivity: Another component of human development is productivity which requires


investment in people. This is commonly called investment in human capital. Investment in human
capital—in addition to physical capital—can add more productivity. The improvement in the quality
of human resources raises the productivity of existing resources. Theodore W. Schultz—the Nobel
Prize-winning economist—articulated its importance: “The decisive factors of production in
improving the welfare of poor people are not space, energy, and crop land; the decisive factor is the
improvement in population quality.” Empirical evidence from many East Asian countries corroborate
this view.

4. Empowerment: The empowerment of people—particularly women—is another com-ponent of


human development. In other words, genuine human development requires empower-ment in all
aspects of life. Empowerment implies a political democracy in which people themselves make the
decisions about their lives. Under it, people enjoy greater political and civil liberties and remain free
from excessive controls and regu-lations. Empowerment refers to decentralisation of power so that
the benefits of governance are reaped by all peoples. It focuses on grassroots participation which
promotes democracy by enfranchising the disadvantaged groups. Unfortu-nately, benefits are
cornered by the elites because of lack of empowerment of people. Participation as a goal is a feature
of ‘bottom-up’ development strategy rather than ‘top-down’. Further, develop­ment policies and
strategies male-dominated. But the benefits of development are to be made ‘gender-sensitive’
.Discrimination against women in health and education is very costly from the viewpoint of achieving
development goals. Education of women can lead to a virtuous circle of lower fertility, better care of
children, more educational oppor-tunity, and higher productivity. Above all, as women’s education
rises, women’s independence in making their own choices also increase. Anyway, decentralization
and participation empower people, specially the women and the poor. It then breaks the ‘deprivation
trap’. Mahbub ul Haq asserts: “If people can exercise their choices in the political, social and
economic spheres, there is a good prospect that growth will be strong, democratic, participatory and
durable.”

7. Which one of the following streams is dominated by male migrants in India?


A. Rural-rural B. Urban-rural C. Rural-urban D. Urban-Urban
www.achieveias.co.in, YouTube Channel: http://youtube.com/c/AchieveIAS Telegram Channel: http://t.me/Achieve_Ias,
Mail: achieveias21@gmail.com, Contact Number: 8968920720

Answer: C
Explanation: Rural-urban migration is a form of so-called internal migration which means a
movement within a country and which stays in contrast to international or intercontinental migration.
It refers to the movement of people from the countryside respectively the rural areas into the cities,
often the metropolitan cities of a country. This change of residence is often connected with the
migration of labor and a career change from primary to second or third sector - not necessarily,
though, as it can refer to the migration of people who are not working in agriculture or farming as
well. It is obvious that these developments always show their two sides: one side or the area of
destination gains population whereas the other side respectively the area of origin losses people.
One always has to keep that in mind because interaction, networking and communication between
both sides are absolutely necessary in trying to achieve a balanced development of both rural and
urban areas .There are different reasons that cause rural-urban migration in developing countries -
they are the so-called push and pull factors. They can be seen as a simultaneous analysis of factors
that force migrants out of rural areas (push-factors) and factors that attract migrants to urban areas.
Factors and determinants of migration are rather diverse and they can be split up in economic and
non-economic reasons:

1. Economic push factors include unemployment or underemployment in rural areas, low wages
and no assets as well as lack of land, which is sometimes due to inheritance systems that split the
land among a large number of people, making it less productive.

2. Non-economic push factors play an additional role and mainly include a poor rural infrastructure
in general, i.e. poor living conditions referring to housing, education possibilities and healthcare;
furthermore aspects of agricultural change like modernization of farming, new techniques and
machines as well as economic reforms in general cause less demand of labor in the agricultural
sector and therefore it causes people to search for new job opportunities; additional factors that
act as push factors are natural disasters, drought or famine, war and conflicts (esp. in Africa) that
in most of the times affect rural areas by destroying most of their belongings and farm land and
there with their livelihood.

3. Economic pull factors include factors that attract people to move into the cities which are mainly
the counterparts of the pus factors: rural migrants hope for employment and higher wages in the
cities caused by a higher demand of labor there in general, due to economic growth - non-economic
pull factors include social factors such as the hope for better healthcare and education provision or
the urban facilities and way of life in general as well as factors like protection from conflicts, family
reunion and family networks .

8. Which of the following statements is/are true?


1. The sex ratio in India is 940.
2. The sex ratio has seen a decline in the past decade.
3. India had the worst sex ratio in 1991 when it was 927.

A. 1 and 2 only B. 1 and 3 only C. 2 and 3 only D. All three

Answer: B
www.achieveias.co.in, YouTube Channel: http://youtube.com/c/AchieveIAS Telegram Channel: http://t.me/Achieve_Ias,
Mail: achieveias21@gmail.com, Contact Number: 8968920720

Explanation: In the Population Census of 2011 it was revealed that the population ratio in
India 2011 is 940 females per 1000 of males. The Sex Ratio 2011 shows an upward trend from the
census 2001 data. Census 2001 revealed that there were 933 females to that of 1000 males
Sex ratio is used to describe the number of females per 1000 of males. Sex ratio is a valuable source
for finding the population of women in India and what is the ratio of women to that of men in India.
In the Population Census of 2011 it was revealed that the population ratio in India 2011 is 940
females per 1000 of males. The Sex Ratio 2011 shows an upward trend from the census 2001 data.
Census 2001 revealed that there were 933 females to that of 1000 males. Since decades India has
seen a decrease in the sex ratio 2011, but since the last two of the decades there has been in slight
increase in the sex ratio. Since the last five decades the sex ratio has been moving around 930 of
females to that of 1000 of males. The major cause of the decrease of the female birth ratio in India
is considered to be the violent treatments meted out to the girl child at the time of the birth. The Sex
Ratio in India was almost normal during the phase of the years of independence, but thereafter it
started showing gradual signs of decrease. Though the Sex Ratio in India has gone through
commendable signs of improvement in the past 10 years, there are still some states where the sex
ratio is still low and is a cause of concern for the NGO organizations. One of the states which is
showing a decreasing trend in the population of women 2011 and is a cause of concern is Haryana.
The state of Haryana has the lowest rate of sex ratio in India and the figure shows a number of 877
of females to that of 1000 of males. There are also states such as Puducherry and Kerala where
the number of women is more than the number of men. Kerala houses a number of 1084 females
to that of 1000 males. While Puducherry and Kerala are the only two states where the number of
female is more than the number of men, there are also states in India like that of Karnataka, Andhra
Pradesh and Maharashtra where the sex ratio 2011 is showing considerable signs of improvement.
Some facts related to the Sex Ratio in India follows, the main cause of the decline of the sex ration
in India is due to the biased attitude which is meted out to the women. The main cause of this gender
bias is inadequate education. Pondicherry and Kerala houses the maximum number of female while
the regions of Daman and Diu and Haryana have the lowest density of female population. The
decennial censuses conducted in India suggest that there has been an almost monotonic decline in
the sex ratio in India. In 1901, the sex ratio as 972 females for every 1000 males;
by 1991 it had reached the lowest point at 927. In Census 2001, the sex ratio increased by six points
to 933.

9. Which of the following statements is/are true?


1. From 1901 to 1911 India’s population registered a four-fold growth.
2. From 1901 to 1951, India’s population grew only one and half times.

A. 1 Only B. 2 Only C. Both 1 and 2 D. Neither 1 nor 2

Answer: C

Explanation: India’s population has grown steadily from 1901, except for a decrease in 1921. It
increased from 23.8 crore in 1901 to 36.1 crore in 1951 and further to 102.7 crore in 2001, as shown
in Table 31.1. It reveals that during the first 50 years from 1901 to 1951, population grew by 12.3
crore while during the next 50 years from 1951 to 2001, it increased by more than 5 times, that is,
by 66.6 crore.
The growth of population in India can be divided into four phases as under:
www.achieveias.co.in, YouTube Channel: http://youtube.com/c/AchieveIAS Telegram Channel: http://t.me/Achieve_Ias,
Mail: achieveias21@gmail.com, Contact Number: 8968920720

1. Phase of Stagnant Population: 1901-21: During this period, population increased by 1.29 crore
only. If we were to break this period in accordance with the two censuses, we find that during 1901-
11, the decadal growth rate was only 5.75 per cent, as the natural growth rate of population was low
(6.6).But in the next decade (1910-21), the growth rate of population declined to (-) 0.3 per cent.
This was due to high death rate (47.2) on account of famines and epidemics like plague, cholera
and malaria which took a heavy toll of human lives. The year 1921 is, therefore, known as the year
of Great Divide.

2. Phase of Steady Population Growth: 1921-51: From 1921 onwards till 1951 there was a steady
growth of population. During these years, population increased by 2.8 crore in 1931, to 4 crore in
1941 and to 5 crore in 1951. But the decadal growth rate in 1951 census was 0.9 percentage less
than in the 1941 census due to the Partition of India in 1947.The main reason for the steady increase
in population was the decline in death rate from 36.3 to 27.4 and of birth rate from 46.4 to 39.9 over
the period. This resulted in an average natural growth rate of 12.2 during this phase of 30 years.

3. Phase of Rapid Population Growth or of Population Explosion: 1951-81: This was the period of
rapid population growth when population increased by 32.5 crore as against about 12 crore during
the last 50 years from 1901 to 1951. The average annual growth rate of population increased from
1.25 per cent to 2.2 per cent in 1981. The main reason was a large decline in the death rate from
22.8 to 15 due to better medical facilities while the birth rate fell slowly from 41.7 to 37.2 over the
period. This led to population explosion in the country.

4. Phase of High Growth with Declining Trend: 1981-2001: During this phase, a definite declining
trend of population growth is visible. Total population increased by 34.37 crore during 20 years. But
the average annual growth rate had definitely declined to 1.93 per cent in 2001 as against 2.2 per
cent in 1981. However, the trend of population explosion still persists even now.

10. During which decade did India see a negative population growth?
A.1901-11 B.1931-41 C.1911-21 D.1951-61

Answer: C
www.achieveias.co.in, YouTube Channel: http://youtube.com/c/AchieveIAS Telegram Channel: http://t.me/Achieve_Ias,
Mail: achieveias21@gmail.com, Contact Number: 8968920720

ACHIEVE IAS GEOGRAPHY NCERT MCQ SERIES, DAY 41, SOLUTIONS

1. Consider the following statement (s) regarding Economic geography.


1. It is the study of the location, distribution and spatial organization of economic activities across
the world.
2. It represents a traditional subfield of the discipline of geography.

Which of the above statement(s) is/are correct?


A. 1 Only B. 2 Only C. Both 1 and 2 D. Neither 1 nor 2

Answer: C

Explanation: Economic geography, which is a sub-field of Geography, is the study of the location,
distribution and spatial organization of economic activities across the globe. Over the past few
decades, economist have approached subject in ways more typical of a sub-discipline of
economics. Economic geography today looks at how such forces as migration, trade costs, and the
location of firms and sectors interact. Today there are several different working definitions of
economic geography, because every economic geographer looks at his or her discipline in a
subjective, personal and slightly diverging way. In Economic Geography, we study the (locational,
organizational and behavioral) principles and processes associated with the spatial allocation of
scarce (human, man-made and natural) resources (which are also distributed spatially) and the
spatial patterns and (direct and indirect, social, environmental and economic) consequences
resulting from such allocations.

2. Consider the following statements


1. In India, Semi-Clustered Settlements is a rather universal feature and particularly in the northern
plains
2. Dominant community and lower strata of society is clearly visible in Semi-Clustered Settlements

Which is/are CORRECT?


A. 1 Only B. 2 Only C. Both 1 & 2 D. None of the above

Answer: B

Explanation: Semi-Clustered Settlements Semi-clustered or fragmented settlements may result


from tendency of clustering in a restricted area of dispersed settlement. More often such a pattern
may also result from segregation or fragmentation of a large compact village. In this case, one or
more sections of the village society choose or is forced to live a little away from the main cluster or
village. In such cases, generally, the land-owning and dominant community occupies the central
part of the main village, whereas people of lower strata of society and menial workers settle on the
outer flanks of the village. Such settlements are widespread in the Gujarat plain and some parts of
Rajasthan.

3. Consider the following statement (s) related to shapes of the settlements.


1. Linear pattern: Such patterns of rural settlements are found in plain areas or wide inter montane
valleys.
2. Rectangular pattern: In such settlements houses are located along a road, railway line, and river,
canal edge of a valley or along a levee.
www.achieveias.co.in, YouTube Channel: http://youtube.com/c/AchieveIAS Telegram Channel: http://t.me/Achieve_Ias,
Mail: achieveias21@gmail.com, Contact Number: 8968920720

Which is / are correct option?


A. 1 Only B. 2 Only C. Both 1 and 2 D. Neither 1 nor 2

Answer: D

Explanation: Linear settlements are settlements where the buildings are constructed in lines, often
next to a geographical feature like a lake shore, a river or following a road. Where linear settlements
follow a road, the road often predates the settlement.
Rectangular pattern: Such patterns of rural settlements are found in plain areas or wide inter
montane valleys. The roads are rectangular and cut each other at right angles.

4. Consider the following statement (s) related to dispersed or isolated rural settlement.
1. Pattern of settlement appears in the form of isolated huts or hamlets of few huts in remote jungles,
or on small hills with farms or pasture on the slopes.
2. Extreme dispersion of settlement is often caused by extremely fragmented nature of the terrain
and land resource base of habitable areas.

Which is / are correct option?


A. 1 Only B. 2 Only C. Both 1 and 2 D. Neither 1 nor 2

Answer: C

Explanation: A dispersed settlement, also known as a scattered settlement, is one of the main
types of settlement patterns used by landscape historians to classify rural settlements found in
India and other parts of the world. Typically, there are a number of separate farmsteads scattered
throughout the area. In these settlements, houses are spaced far apart and often interspersed with
fields. A cultural feature such as a place of worship or a market, binds the settlement together. Such
settlements are found in the western Malwa Plateau, where they occur in association with dispersed
clusters. These are also found in areas of Western Ghats stretching from Satara towards Kerala
high lands and also in some portions of Himalayan ranges in Kashmir and Uttar Pradesh. These
isolated homesteads are also found unoccupied due to frequent floods in eastern Uttar Pradesh and
Bihar districts. Thus, isolated homesteads seem to grow according to the exigencies of local
circumstances. This pattern of settlement appears in the form of isolated huts or hamlets of few huts
in remote jungles, or on small hills with farms or pasture on the slopes. Extreme dispersion of
settlement is often caused by extremely fragmented nature of the terrain and land resource base of
habitable areas.

5. Consider the following statement (s) related to Hamleted rural settlement.


1. Units are locally called panna, para, palli, nagla, dhani, etc. in various parts of the country.
2. A pattern may also result from segregation or fragmentation of a large compact village.

Which is / are correct option?


A. Only 1 B. Only 2 C. Both 1 and 2 D. Neither 1nor 2

Answer: A
www.achieveias.co.in, YouTube Channel: http://youtube.com/c/AchieveIAS Telegram Channel: http://t.me/Achieve_Ias,
Mail: achieveias21@gmail.com, Contact Number: 8968920720

Explanation: Sometimes settlement is fragmented into several units physically separated from each
other bearing a common name. These units are locally called panna, para, palli, nagla, dhani, etc.
in various parts of the country. This segmentation of a large village is often motivated by social and
ethnic factors. A hamlet is a small settlement, smaller than a village. Usually, all settlers in a hamlet
are centered around a single economic activity. A hamlet may consist of a farm, a mill, a mine or
a harbor. All the people living there would be workers on that farm, mill, mine or harbour. Hamlets,
especially those with a medieval church may have resulted from a medieval village, that was
abandoned for some reason. Because of the small size of the settlement, there are usually no
buildings which have a central or admninistrative function.5-6 families may live there usually within
a 2km radius area.

6. Consider the following statements regarding Population Growth in India:


1. The rural decadal growth rate of population is more than that of urban areas.
2. The contribution of rural and urban areas to the total increase in population is nearly equal.

Which of the statements given above is/are correct?


A. 1 only B. 2 only C. Both 1 and 2 D. Neither 1 nor 2

Answer: B

Explanation: The urban decadal growth rate of population is more than that of rural areas. This is
because urbanization is increasing. More and more areas are coming under the definition of “urban
areas”. There is no point in memorizing how much is urban decadal growth rate and how much is
rural one. As I said data/facts are not important, but trends are. The second statement is also true.
The contribution of rural and urban areas to the total increase in population is nearly equal – 91
million.

7. Which one of the following is NOT one of the indicators for the Human Development Index
dimensions?
A. Real GNP per capita (PPP$) B. Birth rates
C. Life expectancy at birth D. Educational attainment

Answer: B

Explanation: UNDP has released the annual HDI 2019 report. The focus of the 2019 Report is on
‘Inequality in Human Development’. India’s position: India’s rank- 129. Last year’s rank- 130.
Despite lifting 271 million people out of poverty between 2005-15, India still remains home to 28%
(364 million) of the world’s poor. Between 1990 and 2018, India’s HDI value increased by 50 per
cent (from 0.431 to 0.647), which places it above the average for countries in the medium human
development group (0.634) and above the average for other South Asian countries (0.642).
This means that in the last three decades, life expectancy at birth in India increased by 11.6 years,
whereas the average number of schooling years increased by 3.5 years. Per capita incomes
increased 250 times.India is only marginally better than the South Asian average on the Gender
Development Index (0.829 vs 0.828), and ranks at a low 122 (of 162) countries on the 2018 Gender
Inequality Index.

India’s neighbours: Sri Lanka (71) and China (85),Bhutan (134), Bangladesh (135), Myanmar
(145), Nepal (147), Pakistan (152) and Afghanistan (170).
www.achieveias.co.in, YouTube Channel: http://youtube.com/c/AchieveIAS Telegram Channel: http://t.me/Achieve_Ias,
Mail: achieveias21@gmail.com, Contact Number: 8968920720

Global scenario: Norway, Switzerland, Ireland occupied the top three positions in that order.Globally,
there are 1.3 billion poor people.Around 661 million of these poor people live in Asia and the
Pacific.South Asia constitutes 41% of the world’s poor.

Changing nature of inequality: As the number of people coming out of poverty is increasing, the
world is veering towards another type of poverty. The old inequalities were based on access to
health services and education whereas the next generation of poverty is based on technology,
education and climate.

What is HDI: Published by the United Nations Development Programme (UNDP), it is a statistical
tool used to measure a country’s overall achievement in its social and economic dimensions. The
social and economic dimensions of a country are based on the health of people, their level of
education attainment and their standard of living.It is part of the Human Development Report.
The other indices that form the part of the 2019 Report are:
1. Inequality-adjusted Human Development Index (IHDI),
2. Gender Development Index (GDI),
3. Gender Inequality Index (GII) and
4. Multidimensional Poverty Index (MPI).

HDI measures average achievement of a country in three basic dimensions of human development:
1. A long and healthy life.
2. Access to knowledge.
3. A decent standard of living.

8. Which of the following statements is correct regarding Economic Development and Human
Development?
A. The basic difference between Economic development and Human development is that Human
development entirely focuses on the increase of income whereas the economic development
believes in expanding and widening of all aspects of human life be it economic, social, political,
cultural, etc.
B. In economic aspect human development is one of the essential elements.
C. The basic idea behind Human development is that it is the use of income and not the income
itself that decides the human choices.
D. The real wealth of a nation is its Economy; therefore, the goal of development should be the
enrichment of the whole economy.

Answer: B

Explanation: The basic difference between economic development and human development is that
economic development entirely focuses on the increase of income whereas the human development
believes in expanding and widening of all aspects of human life be it economic, social, political,
cultural, etc. In economic aspect human development is one of the essential elements. The basic
idea behind this is that it is the use of income and not the income itself that decides the human
choices. Since, the real wealth of a nation is its people; therefore, the goal of development should
be the enrichment of human life.

9. Consider the following reasons which are responsible to keep India at the bottom of the
Human Development:
www.achieveias.co.in, YouTube Channel: http://youtube.com/c/AchieveIAS Telegram Channel: http://t.me/Achieve_Ias,
Mail: achieveias21@gmail.com, Contact Number: 8968920720

1. Rapid increase in population


2. Large number of adult illiterates and low gross enrolment ratio
3. Inadequate government expenditure on education and health

Which of the following statement(s) is/are correct?


A. 1 Only B. 1 and 2 Only C. 1, 2 and 3 D. 1 and 3 Only

Answer: C

Explanation: The following are the reasons to keep India at the bottom of human development (a)
rapid increase in population (b) large number of adult illiterates and low gross enrolment ratio (c)
high drop-out rates (d) inadequate government expenditure on education and health, (e) large
proportion of underweight children as well as under nourished people (f) very poor sanitation
facilities and low access to essential lifesaving medicines.

10. Consider the following statement (s) related to pillars of human development.
1. Equity means making equal access to opportunities available to everybody that opportunities
available to people must be equal irrespective of their gender, race, income and in the Indian case,
caste.
2. Sustainability to the human labour productivity or productivity in terms of human work that must
be constantly enriched by building capabilities in people.

Which is / are correct option?


A. 1 Only B. 2 Only C. Both 1 and 2 D. Neither 1 nor 2

Answer: A

Explanation: The term ‘human development’ may be defined as an expansion of human


capabilities, a widening of choices, ‘an enhancement of freedom, and a fulfilment of human rights.
At the beginning, the notion of human development incorporates the need for income expansion.
However, income growth should consider expansion of human capabilities. Hence development
cannot be equated solely to income expansion. The first UNDP Human Development Report
published in 1990 stated that: “The basic objective of development is to create an enabling environ-
ment for people to enjoy long, healthy and creative lives.” It also defined human development as “a
process of enlarging people’s choices”, “and strengthen human capabilities” in a way which enables
them to lead longer, healthier and fuller lives.

Components of Human Development:


1. Equality,
2. Sustainability,
3. Productivity, and
4. Empowerment.

1. Equality: If development is viewed in terms of enhancing people’s basic capabilities, people must
enjoy equitable access to opportunities. Such may be called equality-related capabilities. To ensure
equality-related capabilities or access to opportunities what is essential is that the societal
institutional structure needs to be more favourable or progressive. In other words, the unfavourable
www.achieveias.co.in, YouTube Channel: http://youtube.com/c/AchieveIAS Telegram Channel: http://t.me/Achieve_Ias,
Mail: achieveias21@gmail.com, Contact Number: 8968920720

initial asset distribution, like land, can be made more farmer-friendly through land reform and other
redistributive measures. In addition, uneven income distribution may be addressed through various
tax-expenditure policies. Economic or legislative- measures that interferes with market exchange
may enable people to enlarge their capabilities and, hence, well-being. Further, to ensure basic
equality, political opportunities need to be more equal. In the absence of effective political
organisation, disadvantaged groups are exploited by the ‘rich’ to further their own interests rather
than social goals. However, participatory politics gets a beating by the inequality in opportunities in
having basic education. It is to be added here that basic education serves as a catalyst of social
change. Once the access to such opportunity is opened up in an equitable way, women or religious
minorities or ethnic minorities would be able to remove socio-economic obstacles of development.
This then surely brings about a change in power relations and makes society more equitable.

2. Sustainability: Another important facet of human development is that development should ‘keep
going’, should ‘last long’. The concept of sustainable development focuses on the need to maintain
the long term protective capacity of the biosphere. This then suggests that growth cannot go on
indefinitely; there are, of course, ‘limits to growth. Here we assume that environment is an essential
factor of production. In 1987, the Bruntland Commission Report (named after the then Prime Minister
Go Harlem Bruntland of Norway) defined sustainable development as ‘… development that meets
the needs of the present without compromising the ability of future generations to meet their basic
needs.’ This means that the term sustainability focuses on the desired balance between future
economic growth and environ-mental quality. To attain the goal of sustainable development, what
is of great impor-tance is the attainment of the goal of both intra- generation and inter-generation
equality. This kind of inequality includes the term ‘social well-being’ not only for the present
generation but also for the people who will be on the earth in the future. Any kind of environmental
decline is tantamount to violation of distributive justice of the disadvan-taged peoples. Social well-
being thus, then, depends on environ-mental equality.

3. Productivity: Another component of human development is productivity which requires investment


in people. This is commonly called investment in human capital. Investment in human capital—in
addition to physical capital—can add more productivity. The improvement in the quality of human
resources raises the productivity of existing resources. Theodore W. Schultz—the Nobel Prize-
winning economist—articulated its importance: “The decisive factors of production in improving the
welfare of poor people are not space, energy, and crop land; the decisive factor is the improvement
in population quality.” Empirical evidence from many East Asian countries corroborate this view.

4. Empowerment: The empowerment of people—particularly women—is another com-ponent of


human development. In other words, genuine human development requires empower-ment in all
aspects of life. Empowerment implies a political democracy in which people themselves make the
decisions about their lives. Under it, people enjoy greater political and civil liberties and remain free
from excessive controls and regu-lations. Empowerment refers to decentralisation of power so that
the benefits of governance are reaped by all peoples. It focuses on grassroots participation which
promotes democracy by enfranchising the disadvantaged groups. Unfortu-nately, benefits are
cornered by the elites because of lack of empowerment of people. Participation as a goal is a feature
of ‘bottom-up’ development strategy rather than ‘top-down’. Further, develop­ment policies and
strategies male-dominated. But the benefits of development are to be made ‘gender-sensitive’.
Discrimination against women in health and education is very costly from the viewpoint of achieving
development goals. Education of women can lead to a virtuous circle of lower fertility, better care of
www.achieveias.co.in, YouTube Channel: http://youtube.com/c/AchieveIAS Telegram Channel: http://t.me/Achieve_Ias,
Mail: achieveias21@gmail.com, Contact Number: 8968920720

children, more educational oppor-tunity, and higher productivity. Above all, as women’s education
rises, women’s independence in making their own choices also increase

Anyway, decentralization and participation empower people, specially the women and the poor. It
then breaks the ‘deprivation trap’. Mahbub ul Haq asserts: “If people can exercise their choices in
the political, social and economic spheres, there is a good prospect that growth will be strong,
democratic, participatory and durable.”
www.achieveias.co.in, YouTube Channel: http://youtube.com/c/AchieveIAS Telegram Channel: http://t.me/Achieve_Ias,
Mail: achieveias21@gmail.com, Contact Number: 8968920720

ACHIEVE IAS GEOGRAPHY MCQ SERIES, DAY 42, SOLUTIONS

1. Which of the following types of farming is more suitable for the production of crops like
sugarcane, wheat and rice?
A. Shifting farming B. Irrigated farming
C. Subsistence farming D. Terrace farming

Answer: B

Explanation: Irrigated farming- This type of farming is practiced in the areas where average
rainfall is between 80 and 200 cms which is insufficient for certain crops. This system of farming
can be practised only in those areas where availability of water from underground or surface water
bodies like rivers, tanks, and lakes is sufficient throughout the year. The other condition for this
farming is the availability of levelled agricultural land. The main areas were much farming is
practised are in Punjab, Haryana, Uttar Pradesh, north western Tamil Nadu and the deltas of
peninsular rivers. The other important pockets of irrigated farming are found in the Deccan
Plateau region particularly in Maharashtra, Karnataka and Andhra Pradesh. Wheat, Rice and
Sugarcane are important crops of this farming.

2. With reference to various types of farming, which of the following is correctly matched?
Types of farming Description
a Dry Farming This type of farming is practised in the areas of
alluvial soils where annual average rainfall is more
than 200cm.
b Wet Farming This type of farming is practised in the areas where
the amount of annual rainfall is generally less than
80 cms.
c Irrigated Farming This type of farming is practiced in the areas where
average rainfall is between 80 to 200 cms which is
insufficient for certain crops.
d Subsistence In this type of cultivation, land is cleared by cutting
Farming and burning of forests for raising crops.

Answer: C

Explanation: The bases for the classification of different types of agriculture in India are rainfall,
irrigational facilities, purpose of production, ownership and size of holding and technology used.
On the basis of these factors a number of farming can be identified. Some of the main types of
farming in India are:

1. Dry Farming: This type of farming is practised in the areas where the amount of annual rainfall
is generally less than 80 cms. In such regions, the farmers are generally dependent upon rainfall.
Here, moisture content in the soil is less. Hence, only one crop can be grown in a year.
www.achieveias.co.in, YouTube Channel: http://youtube.com/c/AchieveIAS Telegram Channel: http://t.me/Achieve_Ias,
Mail: achieveias21@gmail.com, Contact Number: 8968920720

2. Wet Farming: This type of farming is practised in the areas of alluvial soils where annual
average rainfall is more than 200cm. Here, more than one crop is grown in a year because
enough amount of moisture in the soil is available. Rice and jute are the main crops of this types of
farming. West Bengal, Assam, Nagaland, Meghalaya, Tripura, Manipur, Mizoram and Malabar
Coast fall under this category of farming.

3. Irrigated Farming: This type of farming is practiced in the areas where average rainfall is
between 80 and 200 cms which is insufficient for certain crops. This system of farming can be
practised only in those areas where availability of water from underground or surface water bodies
like rivers, tanks, and lakes is sufficient throughout the year.

4. Subsistence Farming: These types of farming are practised primarily to fulfil self-requirements
of the people of the area. The main objective of this farming is to provide subsistence to the
largest number of people of a given area. Size of holdings is small, use of manual labour and
simple farm implements are common features of this type of farming.

5. Shifting Cultivation: In this type of cultivation land is cleared by cutting and burning of forests for
raising crops. The crops are grown for a few years (2-3 years). As fertility of land declines,
farmers move to new areas, clear the forests and grow crops there for next few years.

3. Consider the following statements regarding the Agricultural Land Use in India:
1. The net sown area, current fallows and land under tree crops and groves are included in
agricultural land use.
2. The agricultural land in India is little more than 50 per cent of the total geographical area in the
country.
3. The per capita agricultural land in some select countries is lesser than India.

Which of the following statement(s) is/are correct?


a. 1 Only b. 1 and 2 c. 2 and 3 d. All of the above

Answer: D

Explanation: The net sown area, current fallows and land under tree crops and groves are
included in agricultural land use. The agricultural land in India is little more than 50 per cent of the
total geographical area in the country. This is the highest share of land in any country in the world.
But due to large size of population in India, per capita arable land is available only 0.17 hectares,
which is lower than the world average (0.24 hec). The per capita agricultural land in some select
countries is much higher than India.

4. Which of the following types of land locally known as reh?


A. The land under the settlements, roads, mines and quarries along with barren lands are included
in this category.
B. The total land area on which crops are grown in a region.
C. It is the land in which crops were raised for some period of time but has not been cultivated for
the last five years due to certain deficiencies such as alkalinity and salinity in the soils.
D. When lands are left unused to regain their lost fertility in a natural way.

Answer: C
www.achieveias.co.in, YouTube Channel: http://youtube.com/c/AchieveIAS Telegram Channel: http://t.me/Achieve_Ias,
Mail: achieveias21@gmail.com, Contact Number: 8968920720

Explanation: Cultivable Waste is the land in which crops were raised for some period of time but
has not been cultivated for the last five years due to certain deficiencies such as alkalinity and
salinity in the soils. Such cultivable wastes are locally known as reh, bhur, usar, and khola in the
some part of North India. Meghalaya, Himachal Pradesh and Rajasthan have a very high share of
cultivable waste land in total land use in respective states.

5. With reference to the Fallow Lands which of the following statements is incorrect?
A. When lands are left unused to erode away their fertility in a natural way is called fallow land.
B. On the basis of usability criteria follow lands can be divided into two groups current and old.
C. Current fallow is the land in which no crop is raised during the current year.
D. Old fallow land remains unused for a period of one or more years but not exceeding 5 years.

Answer: A

Explanation: When lands are left unused to regain their lost fertility in a natural way is called
fallow land. On the basis of usability criteria follow lands can be divided into two groups current
and old. Current fallow is the land in which no crop is raised during the current year. Old fallow
land remains unused for a period of one or more years but not exceeding 5 years. This is due to
low investment capacity of numerous small and marginal farmers in advanced technology, lack of
awareness, loss of fertility of soil, inadequacy of rainfall, lacking in irrigational facility etc. The
fallow land occupies about 7.5 per cent of the total reported area.
The states of Mizoram, Tamil Nadu, Meghalaya, Bihar, Andhra Pradesh and Rajasthan have a
high percentage of area under fallow land. It is to be noted here that old fallow land may not be
economically important but from ecological point of view fallow land is important category of land.

6. Consider the following


1. Wheat is sown in mid-October-mid-November and harvested in March.
2. Wheat is grows well in cool, moist climate and ripens in a warm, dry climate.

Which of the above statement (s) is/are correct about climatic conditions for Growing Wheat?
A. Only 1 B. Only 2 C. Both 1 and 2 D. Neither 1 nor 2

Answer: C

Explanation: Wheat is a Rabi crop. It is sown in mid-October-mid-November and harvested in


March. It grows well in cool, moist climate and ripens in a warm, dry climate. The cool winters and
the hot summers are conducive to a good crop. A cloudless sky having bright sunshine during
ripening and harvesting periods will make better quality wheat. Winter rainfall is ideal.

7. Which of the following statements is incorrect regarding the treatment of waste water?
A. Proper treatment of domestic and municipal waste water which is rich in organic matter and
pathogens material also helps to conserve water.
B. Treatment ensures the removal of pollutants but not germs and toxic elements.
C. Growing algae or floating masses of water hyacinth also helps in cleaning the water polluted by
absorbing phosphates, nitrates and other nutrients.
D. None of the above
www.achieveias.co.in, YouTube Channel: http://youtube.com/c/AchieveIAS Telegram Channel: http://t.me/Achieve_Ias,
Mail: achieveias21@gmail.com, Contact Number: 8968920720

Answer: B

Explanation: Removal of pollutants to transform polluted water into usable form:


1. Proper treatment of domestic and municipal waste water which is rich in organic matter and
pathogens material also helps to conserve water. Treatment ensures the removal of pollutants,
germs and toxic elements.
2. Growing algae or floating masses of water hyacinth also helps in cleaning the water polluted by
absorbing phosphates, nitrates and other nutrients. These aquatic plants can be harvested for
producing biogas.

8. Which of the following method is suitable for conservation of water?


a. Reforestation b. Ground water recharge
c. Water harvesting d. All of the above

Answer: D

Explanation: We can do conservation of water by using various methods such as reforestation,


re-use, recycle, increase water use efficiency, water harvesting and ground water recharge.

1. Reforestation: There is a continuous interchange of moisture between earth and atmosphere,


constituting the hydrological cycle. The movement of water through the hydrological cycle has a
major influence on rainfall distribution and temperature modification. Plants play an important role
in hydrological cycle through the process of transpiration. In tropical forests, 75% of the annual
precipitation is returned to the atmosphere by the plants. Thus forests play a very important role in
maintaining water balance of the soil and atmosphere. Forests play a protective function and
conserve soil and water.

2. Recycling of water: The wastewater from industrial or domestic sources can be used after
proper treatment for irrigation, recharging ground water and even for industrial and municipal use.
Agricultural lands close to cities can be irrigated with municipal wastewater. Household level
recycled ‘grey water’ (untreated household water which does not come in contact with toilet waste)
can be used for various purposes. Grey water from shower , bath, washbasins, laundry troughs
and washing machines can be put to various uses to reduce pure water requirement. Pure water
after used in bath and shower can be used for watering plants.

3. Reuse of wastewater: Wastewater containing lots of nutrients. These nutrients may help the
growth of plants when such water is used in irrigation. Reuse of wastewater occurs most
effectively with on-site or small treatment system. The reuse option in the local context at the
community level must be seriously considered.

4. Water harvesting: Water harvesting is collecting rainwater that falls on a house or on and
around any building and then putting it to use later on or simply replenishing the ground water by
allowing the water to reach underground.

5. Recharging of groundwater: The available groundwater is about 13-20 times as much as water
available on surface. The groundwater consists of water contained in the soil or aquifers
(underground natural water reservoir). Rain water is the major source of water for irrigation of
agricultural fields in the country and for recharging the ground water. Rain fall is restricted to three
www.achieveias.co.in, YouTube Channel: http://youtube.com/c/AchieveIAS Telegram Channel: http://t.me/Achieve_Ias,
Mail: achieveias21@gmail.com, Contact Number: 8968920720

months in a year therefore rain water should be stored and use of ground water well planned as
ground water is a renewable water source and gets replenished by natural process of recharge.
Loss of water through seepage and evaporation, water wasted on weeds, cost of bringing water
from ponds to place of use should be minimized.

9. Which of the following factors are responsible for increasing water demand?
A. Expansion of irrigation B. Increasing demand by industry
C. Rising demand due to growing population D. All of the above

Answer: D
Explanation:
1. Expansion of irrigation: India is an agricultural country hence plenty of water is needed for
irrigation. 5.36 billion Cubic meter water was used for irrigation in the year 2000. It is 81% of the
total water used. The remaining percentage (19%) of water was used for domestic, industrial and
other purposes mentioned above. There has been a rapid increase in the irrigated area in India
since independence. Thus the demand for irrigation in India has been increasing continuously.
2. Industrial use of water: Most industries require water at various stages of production of goods
and products. Water is used in industries in both consumptive and non-consumptive ways. Be it
agro-based industries (cotton, textile, jute, sugar and paper) or mineral based industries (iron,
steel, chemical and cement). Water is needed in large amounts during the production process or
as heat exchanger for cooling various machine parts which get heated up during the production
process.
3. Rising demand for growing population: Population of India has been increasing continuously
and it has increased three times since independence. Due to this increase in population, the
demand for water has increased. We need water for drinking, for flushing or draining sewage or
human waste, domestic use, irrigation, industries.
4. Changing life style: Industrial development led to economic development. Purchasing capacity
of individuals has increased. Thus the life style of people changed and the standard of living has
gone up. Large number of attractive appliances, gadgets and fittings for kitchen and bathroom are
available in the market and people are generally tempted to use them, for example taps and
showers are designed in such a way that large amounts of water come out when they are turned
on. Washing machines and dishwashers use large amounts of water but are convenient and suit
the present day life style.

10. Consider the following statements regarding the Net Sown Area (NSA) of any region:
1. The total land area on which crops are grown in a region is called net sown area.
2. The net sown area and the area sown more than once together are called gross cultivated area.
3. In India, about 47 per cent of total reporting area is under the net sown area.

Which of the following statement(s) is/are correct?


a. 1 Only b. 1 and 2 c. 2 and 3 d. All of the above

Answer: D

Explanation: The total land area on which crops are grown in a region is called net sown area.
The net sown area and the area sown more than once together are called gross cultivated area. In
India, about 47 per cent of total reporting area is under the net sown area. States namely Punjab,
Haryana, West Bengal, Uttar Pradesh, have the high proportional share of NSA than the national
www.achieveias.co.in, YouTube Channel: http://youtube.com/c/AchieveIAS Telegram Channel: http://t.me/Achieve_Ias,
Mail: achieveias21@gmail.com, Contact Number: 8968920720

average. Against this, the share of NSA is less than one half of the national average in states of
Himachal Pradesh, Uttarakhand, Meghalaya, Manipur, Nagaland, Mizoram, Sikkim and Arunachal
Pradesh.

11. Consider the following statements regarding reforestation, one of the methods for
conservation of water:
1. Reforestation is the process of planting of trees on the deforested area or of increased tree
cover from re-growth and planting of trees (plantations).
2. There is a continuous interchange of moisture between earth and atmosphere, constituting the
hydrological cycle which has a major influence on rainfall distribution and temperature
modification.
3. Plants play an important role in hydrological cycle through the process of transpiration.

Which of the following statement(s) is/are correct?


a. 1 Only b. 1 and 2 c. 1 and 3 d. All of the above

Answer: D

Explanation: There is a continuous interchange of moisture between earth and atmosphere,


constituting the hydrological cycle. The movement of water through the hydrological cycle has a
major influence on rainfall distribution and temperature modification. Plants play an important role
in hydrological cycle through the process of transpiration. In tropical forests, 75% of the annual
precipitation is returned to the atmosphere by the plants. Thus forests play a very important role in
maintaining water balance of the soil and atmosphere. Forests play a protective function and
conserve soil and water. Reforestation is the process of planting of trees on the deforested area or
of increased tree cover from re-growth and planting of trees (plantations). Reforestation increases
soil fertility by preventing soil erosion, prevents run-off of eroded soil into aquatic systems and
prevents flooding. Thus reforestation plays a major role in conservation of water.

12. Consider the following statements regarding the water as a natural resource:
1. Water is an indispensable, natural resource as no life can exist without water and it is also
renewable and reusable.
2. It is estimated that nearly three fourths of earth is covered with water of the ocean, seas, rivers,
lakes, snow, glaciers and ground water.
3. Only less than 1% of this water is fresh water and usable for living organisms including humans.

Which of the following statement(s) is/are correct?


a. 1 Only b. 1 and 2 c. 1 and 3 d. All of the above

Answer: D

Explanation: Water is an indispensable, natural resource as no life can exist without water. It is
also renewable and reusable. Scientists estimate that nearly three fourths of earth is covered with
water of the ocean, seas, rivers, lakes, snow, glaciers and ground water. However, only less than
1% of this water is fresh water and usable for living organisms including humans. Though water
cycle throughout the year, everywhere, and it is being overused and some of it is even wasted.
Hence, water conservation has become very necessary. At present, one-third of the global
population is facing acute water shortage. At present, one-third of the global population is facing
www.achieveias.co.in, YouTube Channel: http://youtube.com/c/AchieveIAS Telegram Channel: http://t.me/Achieve_Ias,
Mail: achieveias21@gmail.com, Contact Number: 8968920720

acute water shortage. In the villages, women have to walk long distances to fetch water. In certain
hilly areas, women may have to walk as much as ten kilometres uphill to reach a well. According
to UN estimates, by 2025, almost two-third of the world will face shortage of potable (fit for
drinking) water. We have to be cautious about using water and devise methods of conserving it.
www.achieveias.co.in, YouTube Channel: http://youtube.com/c/AchieveIAS Telegram Channel: http://t.me/Achieve_Ias,
Mail: achieveias21@gmail.com, Contact Number: 8968920720

ACHIEVE IAS GEOGRAPHY MCQ SERIES, DAY 43, SOLUTIONS

1. A naturally occurring substance that has a definite chemical composition is known as :


A. Ore B. Mineral C. Soil D. Land

Answer: B

Explanation: Minerals are solid substances that are present in nature and can be made of one
element or more elements combined together (chemical compounds). Gold, Silver and carbon are
elements that form minerals on their own. They are called native elements. Instead, ordinary kitchen
salt is a chemical compound that is called rock salt, which is a mineral formed of sodium and chlorine
ions. Atoms, ions and molecules that form a mineral are present in the space in a tidy way and
according to well-defined geometrical shapes, which are called crystal lattices. The structure of the
crystal lattice defines the shape of the crystal as we see it. For example, rock salt or kitchen salt is
a mineral formed of cubic-shaped crystals. Its crystal lattice has the same shape and consists of
sodium and chlorine ions that are present in the space in alternate order. The order of atoms in the
space and the way they combine with each other determine the way a mineral can laminate or
exfoliate. Lamination is the property that some materials have to break according to their geometrical
shape. Its chemical composition also determines the colour of the crystal, such as the yellow colour
for the topaz, red for ruby, purple for amethyst quartz. Another characteristic of minerals is their
hardness, which is their resistance to scratches. Hardness is classified by numbers (from 1 to 10),
according to the Moh’s scale. At the beginning of the scale there are very soft minerals that can be
scratched with a nail, such as talc, chalk and calcite. At the end of the scale there is the diamond,
which is the hardest mineral in nature. "A mineral is an element or chemical compound that is
normally crystalline and that has been formed as a result of geological processes" .

2. Assertion (A): In India, 98% of the coal is found in Gondwana rocks of Moran region.
Reason (R): The main regions of Gondwana rocks are found in West Bengal, Jharkhand and
Odisha.

A. Both A and R are true and R is the correct explanation of A


B. Both A and R are true but R is not a correct explanation of A
C. A is true but R is false
D. Both A & R is not true

Answer: A

Explanation: Around 67 % of total commercial energy produced in India and 98% of India's total
account is found in Gondwana rocks of Moran region. The main regions of Gondwana rocks
are found in West Bengal, Jharkhand and Odisha.

3. Assertion (A): India has its fifth largest Bauxite reserves (5%) of the world.
Reason (R): Odisha, Andhra Pradesh and Gujarat stand first and third respectively.

A. Both A and R are true and R is the correct explanation of A


B. Both A and R are true but R is not a correct explanation of A
C. A is true but R is false
D. Both A & R is not true
www.achieveias.co.in, YouTube Channel: http://youtube.com/c/AchieveIAS Telegram Channel: http://t.me/Achieve_Ias,
Mail: achieveias21@gmail.com, Contact Number: 8968920720

Answer: B

Explanation: Bauxite is an important ore which is used for making aluminium. It is an oxide of
aluminium (name derived after Le Beaux in France). India’s bauxite ore production has remained
reasonably in line with its aluminium output and is estimated to rise this year buoyed by increasing
domestic demand. As per UNFC system, resources of bauxite in India are placed at about 3,480Mt
which is about 5% of the world total. These resources include 593Mt reserves and 2,887Mt
remaining resources. A number of projects have been lined up which include development of the
Pottangi bauxite mines for Nalco. Analysts say bauxite production in India should grow at 17.7%
though 2020, with an estimated 8.2% contribution to the global output, this year alone. The year
2016 has essentially been a year of transformation for the global bauxite industry. While Indonesia
has been seen as still striving for value-added investment, Malaysia started off well as the new
temporary Indonesia but soon succumbed to the ill effects of unregulated mining that resulted in
growing environmental pressure and an extended mining moratorium. Bauxite production from the
country, since then, has declined steadily only to be covered up by countries like Guinea, Australia,
Brazil, China, and India. As per the Indian Minerals Yearbook 2018 (latest) of GoI, Odisha is the
largest producer of bauxite in India. Bauxite is the principal ore of aluminium. India produces around
22.3 thousand tonnes of bauxite every year. Following states are major producers of bauxite in
India:-
1. Odisha (51.3%)
2. Gujarat (14%)
3. Chhattisgarh (11.4%)
4. Maharashtra (9.1%)
There are 152 active bauxite mines in India at present. Panchpatmali bauxite mines in Koraput
district of Odisha is the highest producer of bauxite in India. It produces 32% of total bauxite in India.
Some other famous bauxite mines are: Baphlimali mines (Rayagada district, Odisha), Asota
Mewasa and Ghangura mines (Devbhoomi Dwarka district, Gujarat), Bodai Daldali mines
(Kabirdham district, Chhattisgarh). Districts which are principal producers of bauxite are:-
 Koraput and Rayagada in Odisha
 Surguja and Kabirdham in Chhattisgarh
 Gumla, Latehar and Lohardaga in Jharkhand
 Kutch and Devbhoomi Dwarka in Gujarat
 Kolhapur and Ratnagiri in Maharashtra

In terms of reserves, following is the rank:-


1. Odisha (51%)
2. Andhra Pradesh (16%)
3. Gujarat (9%)
4. Jharkhand (6%)
Reserves are concentrated in the east coast bauxite deposits in Odisha and Andhra Pradesh.
World bauxite reserves are estimated at 30 billion tonnes. Among countries, Guinea has the largest
reserves of bauxite at 25%. Indian is at rank 10 in terms of bauxite reserves. Australia is the largest
producer of bauxite in the world at 29%. India at rank 5 in terms of bauxite production. Largest
importer of bauxite is China at 83%. Largest exporter is Guinea at 40%.

4. Consider the following statements regarding the mineral deposits found in North-
western region of India:
www.achieveias.co.in, YouTube Channel: http://youtube.com/c/AchieveIAS Telegram Channel: http://t.me/Achieve_Ias,
Mail: achieveias21@gmail.com, Contact Number: 8968920720

1. This belt extends from gulf of Khambhat in Gujarat to the Aravalli range in Rajasthan.
2. Petroleum and natural gas are principal resources of this belt.
3. The region is known for reserves and production of several non-ferrous metals particularly
copper, silver, lead, and Zinc.

Which of the following statement(s) is/are correct?


A. 1 Only B. 1 and 2 Only C. 1, 2 and 3 D. 1 and 3 Only

Answer: C

Explanation: This belt extends from gulf of Khambhat in Gujarat to the Aravalli range in Rajasthan.
Petroleum and natural gas are principal resources of this belt. Deposits of other minerals are small
and scattered. However, it is known for reserves and production of several non-ferrous metals
particularly copper, silver, lead, and Zinc.

5. Which two of the following extreme locations are connected by the east west corridor?
A. Mumbai and Nagpur B. Mumbai and Kolkata
C. Silcher and Porbandar D. Nagpur and Siligudi

Answer: C

Explanation: In the North East, the East-West Corridor starts at Srirampur and ends at Silchar.
This project aims to improve the connectivity of the North Eastern Region with the rest of India
through a 670 kms long four-lane divided highway between Srirampur and Silchar. The Corridor
does not go beyond Assam. Entire East West Corridor is targeted for completion by Dec, 2011 to
Dec, 2012 except the five packages of NC Hills district re-awarded for the balance work targeted for
completion March, 2014. Reasons for delay, inter-alia, problems inland acquisition, forest clearance
for cutting trees, transfer of electric poles, etc.

6. Which of the following are correct with reference to economic reforms (LPG reforms) and
public policies?
1. Economic reforms have placed limits on the growth of public expenditure especially in social
sectors.
2. The reform policies involving tariff reduction have curtailed the scope for raising revenue
through customs duties.
3. In order to attract foreign investment, tax incentives were provided to foreign investors which
further reduced the scope for raising tax revenues.

Select the correct statements using the codes given below:


a) 1 and 2 only b) 2 and 3 only c) 1 and 3 only d) All are correct

Answer: D

Explanation: The economy of India had undergone significant policy shifts in the beginning of the
1990s. This new model of economic reforms is commonly known as the LPG or Liberalisation,
Privatisation and Globalisation model. The primary objective of this model was to make the
economy of India the fastest developing economy in the globe with capabilities that help it match up
with the biggest economies of the world.
www.achieveias.co.in, YouTube Channel: http://youtube.com/c/AchieveIAS Telegram Channel: http://t.me/Achieve_Ias,
Mail: achieveias21@gmail.com, Contact Number: 8968920720

7. Consider the following statements:


1. Natural gas occurs in the Gondwana beds.
2. Mica occurs in abundance in Kodarma.
3. Dharwars are famous for petroleum.

Which of the statements given above is/are correct?


A. 1 and 2 B. 2 only C. 2 and 3 D. None

Answer: B

Explanation: Dharwar rocks are non fossilliferous rather they are metalliferous. They bear out gold,
iron ore, manganese mica, cobalt, chromium, copper, tungsten, lead, nickel, precious stones and
budding stones. Kodarma is a store house of mica and Gondwana beds has coal and not natural
gas.

8. With reference to two non-conventional energy sources called ‘coal bed methane’ and
‘shale gas’, consider the following ‘statements:
1. Coal bed methane is the pure methane gas extracted from coal seams, while shale gas is a
mixture of propane and butane only that can be extracted from fine-grained sedimentary rocks.
2. In India abundant coal bed methane sources exist, but so far no shale gas sources have been
found.

Which of the statements given above is/are correct?


A. 1 only B. 2 only C. Both 1 and 2 D. Neither 1 nor 2

Answer: D

Explanation: Coal Bed Methane (CBM) is an unconventional form of natural gas found
in coal deposits or coal seams. CMB is formed during the process of coalification, the transformation
of plant material into coal.It is considered a valuable energy resource with reserves and production
having grown nearly every year since 1989. Varied methods of recovery make CBM a stable source
of energy. CBM can be recovered from underground coal before, during, or after mining operations.
It can also be extracted from “unminable” coal seams that are relatively deep, thin or of poor or
inconsistent quality. Vertical and horizontal wells are used to develop CBM resources. Extraction
requires drilling wells into the coal seams and removing water contained in the seam to reduce
hydrostatic pressure and release absorbed (and free) gas out of the coal.
Coalbed Methane (CBM), an unconventional source of natural gas is now considered as an
alternative source for augmenting India’s energy resource. India has the fifth largest proven coal
reserves in the world and thus holds significant prospects for exploration and exploitation of CBM.
The Gondwana sediments of eastern India host the bulk of India’s coal reserves and all the current
CBM producing blocks. The vast majority of the best prospective areas for CBM development are
in eastern India, situated in Damodar Koel valley and Son valley. CBM projects exist in Raniganj
South, Raniganj East and Raniganj North areas in the Raniganj coalfield, the Parbatpur block in
Jharia coalfield and the East and west Bokaro coalfields. Son valley includes the Sonhat North and
Sohagpur East and West blocks. Currently, commercial production has commenced from Raniganj
South CBM block operated by M/s. GEECL since July 2007. Shale gas has abundant reserves
around the world, which may be sufficient to meet the demand of clean energy for many years to
www.achieveias.co.in, YouTube Channel: http://youtube.com/c/AchieveIAS Telegram Channel: http://t.me/Achieve_Ias,
Mail: achieveias21@gmail.com, Contact Number: 8968920720

come. Shale gas is found in unconventional reservoirs typically trapped in shale rock, having low
permeability, originally deposited as clay and silt. This makes it more difficult and more expensive
to extract because of high upfront costs. Shale gas is generally found at 2000 to 5000 meters below
the earth’s surface unlike conventional natural gas – trapped in sandstone rock having high
permeability and can be easily assessed to be produced by traditional vertical drilling, found at 1500
meters. The technique used for shale gas production requires first drilling a vertical well to the
targeted rock followed by horizontal drilling exposing the well to more of producing shale. As per the
US EIA 2015 report, India has got technically recoverable shale gas of 96 trillion cubic feet. The
recoverable reserves are identified in Cambay, Krishna – Godavari, Cauvery, Damodar Valley,
Upper Assam, Pranahita – Godavari, Rajasthan and Vindhya Basins. The Oil and Natural Gas
Corporation has drilled the first exploratory shale gas well in Jambusar near Vadodara, Gujarat, in
Cambay basin during October 2013. It is estimated that from this basin around 20 TCF shale gas is
recoverable. In addition to shale gas, India has got reserves of other unconventional natural gases
like Coal Bed Methane, Coal Mine Methane, and Tight Gas which are in the different stages of
development/production. The country has 9.9 trillion cubic feet of recoverable Coal Bed Methane
(CBM). The CBM is extracted from virgin coal mines. At present CBM is produced from four blocks
- Jharia in Jharkhand, Raniganj East and South in West Bengal and Sohagpur West in Madhya
Pradesh. It is estimated that India may produce about 5.5 million standard cubic meters of CBM by
the end of this year which could be about 5 percent of the total natural gas production in the country.
At present Coal Mine Methane is not trapped and is blown out of coal mines. Tight Gas reservoirs
in Eocene formations in Gujarat and Northeast India have been discovered. As per the study by the
US, Oil Field Services Company, total reserves of tight gas at Cambay amount to 0.55 TCF,
production of which at present is not economically viable but with the new technologies in place
these reserves may be trapped in future.

9. Which of the following agencies markets steel for the public sector plants?
A. HAIL B. SAIL C. TATA Steel D. MNCC

Answer: B

Explanation: Steel Authority of India Limited (SAIL) is an Indian state-owned steel making company
based in New Delhi, India. It is a public sector undertaking, owned and operated by the Government
of India with an annual turnover of INR 66,267 Crore (US$9.32 Billion) for fiscal year 2018-19.
Incorporated on 24 January 1974, SAIL has 71,021 employees (as of 01-Oct-2019). With an annual
production of 16.30 million metric tons, SAIL is the 20th largest steel producer in the world and the
3rd largest in India. The Hot Metal production capacity of the company will further increase and is
expected to reach a level of 50 million tonnes per annum by 2025. Sri Anil Kumar Chaudhary is the
current Chairman of SAIL. SAIL operates and owns 5 integrated steel plants
at Bhilai, Rourkela, Durgapur, Bokaro and Burnpur (Asansol) and 3 special steel plants
at Salem, Durgapur and Bhadravathi. It also owns a Ferro Alloy plant at Chandrapur. As a part of
its global ambition, the company is undergoing a massive expansion and modernization programme
involving upgrading and building new facilities with emphasis on state of the art green technology.
According to a recent survey, SAIL is one of India's fastest growing Public Sector Units. Besides, it
has R&D centre for Iron & Steel (RDCIS), centre for Engineering in Ranchi, Jharkhand.

10. Which of the following statements is/are correct about Smelting?


A. Smelting is the process in which metals are extracted from their ores by cooling below the
boiling point.
www.achieveias.co.in, YouTube Channel: http://youtube.com/c/AchieveIAS Telegram Channel: http://t.me/Achieve_Ias,
Mail: achieveias21@gmail.com, Contact Number: 8968920720

B. Smelting is the process in which metals are extracted from their ores by heating beyond the
melting point.
C. Smelting is the process of producing refined goods.
D. Smelting is the major process used in Nuclear Reactors.

Answer: B

Explanation: Smelting, process by which a metal is obtained, either as the element or as a


simple compound, from its ore by heating beyond the melting point, ordinarily in the presence of
oxidizing agents, such as air, or reducing agents, such as coke. The first metal to be smelted in
the ancient Middle East was probably copper (by 5000 BCE), followed by tin, lead, and silver. To
achieve the high temperatures required for smelting, furnaces with forced-air draft were developed;
for iron, temperatures even higher were required. Smelting thus represented a major technological
achievement. In modern ore treatment, various preliminary steps are usually carried out before
smelting in order to concentrate the metal ore as much as possible. In the smelting process a metal
that is combined with oxygen—for example, iron oxide—is heated to a high temperature, and
the oxide is caused to combine with the carbon in the fuel, escaping as carbon monoxide or carbon
dioxide. Other impurities, collectively called gangue, are removed by adding a flux with which they
combine to form a slag.In modern copper smelting, a reverberatory furnace is used. Concentrated
ore and a flux, commonly limestone, are charged into the top, and molten matte—a compound of
copper, iron, and sulphur—and slag are drawn out at the bottom.

11. Following pair represent the sites of Nuclear power plants and corresponding states in
India.
1. Kalpakkam – Tamil Nadu
2. Tarapur – Karnataka
3. Narora – Maharashtra
4. Kaiga – Uttar Pradesh

Which of the above pair is/are not correctly matched?


A. 2 and 3 only B. 1 and 4 only C. 1, 2 and 3 only D. 2, 3 and 4 only

Answer: D
www.achieveias.co.in, YouTube Channel: http://youtube.com/c/AchieveIAS Telegram Channel: http://t.me/Achieve_Ias,
Mail: achieveias21@gmail.com, Contact Number: 8968920720

ACHIEVE IAS GEOGRAPHY MCQ SERIES, DAY 44, SOLUTIONS

1. What are the Primary Goals of Sustainability?


1. The end of poverty and hunger
2. Better standards of education and healthcare - particularly as it pertains to water quality and better
sanitation
3. To achieve gender equality
4. Sustainable economic growth while promoting jobs and stronger economies

A. 1, 2 & 3 B. 1, 3 & 4 C. 1 & 3 D. I, 2, 3 & 4

Answer: D

Explanation: The Sustainable Development Goals are the blueprint to achieve a better and more
sustainable future for all. They address the global challenges we face, including those related to
poverty, inequality, climate change, environmental degradation, peace and justice. The 17 Goals are
all interconnected, and in order to leave no one behind, it is important that we achieve them all by
2030. The Sustainable Development Objectives cover different aspects of social development,
environmental protection and economic growth, and these are the main ones:
1. The eradication of poverty and hunger so as to ensure a healthy life.
2. Achieve universal access to basic services such as water, sanitation and sustainable energy.
3. Support the generation of development opportunities through inclusive education and decent
work.
4. Foster innovation and resilient infrastructures by creating communities and cities capable of
producing and consuming in a sustainable way.
5. Reduce inequalities in the world, especially gender inequalities.
6. Caring for the environment by combating climate change and protecting oceans and terrestrial
ecosystems.
7. Promote collaboration between different social actors to create an environment of peace and
sustainable development.

2. Which of the following railway is not recognised by UNESCO World heritage Site?
A. Darjeeling Himalayan Railway B. Nilgiri Mountain Railway
C. Kalka-Shimla Railway D. North-Eastern Railway

Answer: D

Explanation: There are two UNESCO World Heritage Sites on IR viz. the Chatrapati Shivaji
Terminus and the Mountain railways of India. Mountain railways of India is not contiguous, but
consists of three separate railway lines located in different parts of the country viz. the Darjeeling
Himalayan Railway; the Nilgiri Mountain Railway; and the Kalka-Shimla Railway.

3. Petroleum refinery is/are located in which of the following cities?


1. Jamnagar
2. Bathinda
3. Panipat

Select the correct answer using the code given below.


www.achieveias.co.in, YouTube Channel: http://youtube.com/c/AchieveIAS Telegram Channel: http://t.me/Achieve_Ias,
Mail: achieveias21@gmail.com, Contact Number: 8968920720

A. 1 and 2 only B. 2 and 3 only C. 1 and 3 only D. 1, 2 and 3

Answer: D

Explanation: Petroleum refineries are located at the following places in India: Barauni, Koyali,
Haldia, Mathura, Panipat, Digboi, Bongaigaon, Guwahati, Paradip, Mumbai, Visakhapatnam,
Bathinda, Kochi, Chennai, Nagapattinam, Numaligarh, Tatipaka, Mangalore, Jamnagar.

4. Consider the following statements with regard to density of roads in India:


1. Major determinants of density of roads in India are Nature of terrain, Population, Level of
economic development.
2. The density of road is high in most of the northern states and major southern states

Which of the statements given above is/are correct?


A. 1 only B. 2 only C. Both 1 and 2 D. Neither 1 nor 2

Answer: B

Explanation: Nature of terrain and the level of economic development are the main determinants
of density of roads. Construction of roads is easy and cheaper in the plain areas while it is difficult
and costly in hilly and plateau areas. Therefore, not only the density but also the quality of roads is
relatively better in plains as compared to roads in high altitude areas, rainy and forested regions.
Density of roads (length of roads per 100 square km of area) varies from only 12.14 km in Jammu
and Kashmir to 517.77 km in Kerala with a national average of 142.68 km in 2011) The density of
road is high in most of the northern states and major southern states. It is low in the Himalayan
region, north-eastern region. Madhya Pradesh and Rajasthan despite having high population have
low road density. Thus, population is not a major determinant.

5. Consider the following statement:


1. About 80 percent of the total road length in India are categorised as district roads.
2. District Roads are the connecting link between District Headquarters and the other important
nodes in the district.

Select the correct answer using the code given below.


A. 1 only B. 2 only C. Both 1 and 2 D. Neither 1 nor 2

Answer: B

Explanation: About 80 per cent of the total road length in India are categorised as rural roads.
District Roads are the connecting link between District Headquarters and the other important nodes
in the district.

6. On which river National Waterway 1 ( NW1) is located?


A. Ganga B. Mahanadi C. Barak D. Narmada

Answer: A
www.achieveias.co.in, YouTube Channel: http://youtube.com/c/AchieveIAS Telegram Channel: http://t.me/Achieve_Ias,
Mail: achieveias21@gmail.com, Contact Number: 8968920720

Explanation: The National Waterway 1 or NW-1 or Ganga-Bhagirathi-Hooghly river system is


located in India and runs from Haldia (Sagar) to Prayagraj across
the Ganges, Bhagirathi and Hooghly river systems.[1] It is 1,620 km (1,010 mi) long,[2] making it the
longest waterway in India.[3] It is of prime importance amongst all the national waterways
considering its locational advantages. The NW-1 passes through West
Bengal, Jharkhand, Bihar and Uttar Pradesh and serves major cities and their industrial hinterlands.
Transportation plays an important role in the development of a country and it is of great significance
for a developing country like India. The country is bestowed with a plethora of diverse topography
which enables different kinds of transportation. India has about 14500 km of navigable waterways.
This includes rivers, backwaters, canals, creeks and so on. National Waterways Act came into effect
in 2016. It proposed 106 additional National Waterways and merges 5 existing Acts which were
declared the 5 National Waterways. In 1986, the Government of India created Inland Waterways
Authority of India (IWAI) for regulation and development of Inland Waterways for navigation and
shipping. Out of the 111 National Waterways declared under the National Waterways Act, 2016, 13
are operational for shipping and navigation and cargo/passenger vessels are moving on them. About
the IWAI Inland Waterways Authority of India: This body was created by the government of India in
1986 for regulating and developing inland waterways for shipping and navigation. The body chiefly
undertakes development and maintenance projects of IWT infrastructure on national waterways. It
undertakes these projects through grants from the Shipping Ministry. Its headquarters is in Noida. It
also has regional offices in various other cities and towns across the country.

7. Consider the following statements about Pradhan Mantri Gram Sadak Yojana (PMGSY).
1. Pradhan Mantri Gram Sadak Yojana is a nationwide plan to provide good all-weather road
connectivity to all unconnected villages.
2. PMGSY also involves Road Connectivity for Left Wing Extremism affected Areas.
3. PMGSY phase three involves consolidation of Through Routes and Major Rural Links connecting
habitations to Gramin Agricultural Markets (GrAMs), Higher Secondary Schools and Hospitals.

Which of the above statements is/are correct?


A. 1, 2 only B. 1, 3 only C. 2, 3 only D. 1, 2, 3

Answer: C

Explanation: Cabinet Committee on Economic Affairs has given its approval for the launch of
Pradhan Mantri Gram Sadak Yojana-lll (PMGSY-III). Under the PMGSY-III Scheme, it is proposed
to consolidate 1,25,000 Km road length in the States.It involves consolidation of Through Routes
and Major Rural Links connecting habitations to Gramin Agricultural Markets (GrAMs), Higher
Secondary Schools and Hospitals. The funds would be shared in the ratio of 60:40 between the
Centre and State for all States except for 8 North Eastern and 3 Himalayan States (Jammu &
Kashmir, Himachal Pradesh & Uttarakhand) for which it is 90:10. PMGSY was launched in
December, 2000 with an objective to provide single all-weather road connectivity to eligible
unconnected habitation of designated population size (500+ in plain areas and 250+ in North-East,
hill, tribal and desert areas as per Census, 2001) for overall socio-economic development of the
areas.

8. Which of following statements is/are correct?


1. The first railway line was opened between Mumbai and Thane in 1853 and was extended to
Kalyan in 1854 and Khopoli in 1856.
www.achieveias.co.in, YouTube Channel: http://youtube.com/c/AchieveIAS Telegram Channel: http://t.me/Achieve_Ias,
Mail: achieveias21@gmail.com, Contact Number: 8968920720

2. The general trend of growth of railways has been from ports to hinterlands.
3. Indian railways comprise the largest public sector undertaking in India.

A. Only 1 B. 1 and 2 only C. 1, 2 and 3 D. None

Answer: C

9. Which one of the following sets of States stands to benefit from the Konkan railways:
A. Goa, Karnataka, Maharashtra B. Madhya Pradesh, Maharashtra, Tamil Nadu
C. Tamil Nadu, Kerata, Goa D. Gujarat, Maharashtra, Goa

Answer: A

Explanation: The Konkan Railway is a railway operated by the Konkan Railway Corporation, with
its headquarters at CBD Belapur in Navi Mumbai, Maharashtra, India. The first passenger train ran
on Konkan railway tracks on 20 March 1993, between Udupi and Mangalore. During its initial years
of operation in the mountainous Konkan region, a number of accidents prompted Konkan Railway
to implement new technology. Anti-collision devices, the Sky Bus and roll-on/roll-off (RORO) are
several of the railway's innovations. The 741-km (461-mi) line connects the states
of Maharashtra, Goa and Karnataka. The first train on the completed track was sent off on January
26, 1998.

10. Consider the following statements.


1. National Highways constitute less than 2% of the total road length but carry about 40% of the
traffic.
2. National Highway No. 7 is the longest highway in India.

A. Only 1 B. Only 2 C. Both 1 and 2 D. Neither 1 nor 2

Answer: C

Explanation: A major part of NH 7 covers the North-South Corridor of NHDP and it is officially listed
as running over 2,369 km (1,472 mi) from Varanasi to Kanyakumari. It is the longest national
highway in India. The National Highways constitute only two per cent of the total road length but
carry 40 per cent of the road traffic. The National Highways Authority of India (NHAI) is the apex
body to improve the quality of the National Highways. It was constituted by an act of Parliament,
the National Highways Authority of India Act, 1988. It is responsible for the development,
maintenance and management of National Highways entrusted to it and for matters connected or
incidental thereto.
www.achieveias.co.in, YouTube Channel: http://youtube.com/c/AchieveIAS Telegram Channel: http://t.me/Achieve_Ias,
Mail: achieveias21@gmail.com, Contact Number: 8968920720

ACHIEVE IAS GEOGRAPHY MCQ SERIES, DAY 45, SOLUTIONS

1. Depletion of ozone layer causes, Which of the following?


1. Cataract in eyes leading to blindness
2. Reduced productivity of forests
3. Lung infection

Select the correct answer from the following codes


A. 1 Only B. 1 and 2 Only C. 2 and 3 Only D. 1, 2 and 3

Answer: B

Explanation: Depletion of ozone layer permits passage of UV radiation on earth’s atmosphere


which causes sunburn, cataract in eyes leading to blindness, skin cancer, reduced productivity of
forests. Respiratory problems occur due to air pollution.

Serious Effects of Ozone Depletion:


1. Damage to human health: If the ozone layer is depleted, it means humans will be overly exposed
to strong UV light. Overexposure to strong UV light causes skin cancer, cataracts, sunburns,
weakening of immune system and quick aging.
2. Devastation to environment: Many crops species are vulnerable to strong UV light and
overexposure may well lead to minimal growth, photosynthesis and flowering. Some of the crop
species vulnerable to UV light include barley, wheat, corn, oats, rice, broccoli, tomatoes, cauliflower
just to name a few. Forests equally bear the brunt of ozone depletion.
3. Threat to marine life: Certain marine life, especially planktons, is greatly impacted by exposure to
strong ultraviolet rays. In the aquatic food chain, planktons appear high up. If planktons decrease in
number due to ozone layer destruction, the marine food chain would be disrupted in many ways.
Also, overexposure of sun rays could reduce the fortunes of fishers. On top of that, certain species
of marine life have been greatly affected by overexposure to ultraviolet radiation at their early stage.
4. Effect on animals: In domesticated animals, too much Ultraviolet radiation could also lead to skin
and eye cancer.
5. Impacts certain materials: Materials like plastics, wood, fabrics, rubber are massively degraded
by too much ultraviolet radiation

2. Which of the following is/are not Green House Gases?


1. Nitrous oxide
2. Ozone
3. Sulphur dioxide

Select the correct answer from the following codes


A. 3 Only B. 1 and 2 Only C. 2 and 3 Only D. 1, 2 and 3

Answer: A

Explanation: Atmospheric gases like carbon dioxide, methane, nitrous oxide, water vapour, and
Chlorofluorocarbons are capable of trapping the out-going infrared radiation from the earth.
www.achieveias.co.in, YouTube Channel: http://youtube.com/c/AchieveIAS Telegram Channel: http://t.me/Achieve_Ias,
Mail: achieveias21@gmail.com, Contact Number: 8968920720

3. Which of the following chemicals cause water pollution?


1. Arsenic
2. Copper
3. Zinc

Select the correct answer from the following codes


A. 1 Only B. 1 and 2 Only C. 2 and 3 Only D. 1, 2 and 3

Answer: D

Explanation: Metals like lead, zinc, arsenic, copper, mercury and cadmium in industrial waste
waters adversely affect humans and other animals. arsenic polluted water leads to accumulation of
arsenic in the body parts like blood, nails and hairs causing skin lesions, rough skin, dry and
thickening of skin and ultimately skin cancer.

4. With reference to Eutrophication, Which of the following is/are true?


1. It is phenomenon of nutrient enrichment of a water body
2. It depletes the water of dissolved oxygen (DO).
3. Human activities are mainly responsible for the eutrophication

Select the correct answer from the following codes


a. 1 Only b. 1 and 2 Only c. 2 and 3 Only d. 1, 2 and 3

Answer: D

Explanation: Discharge of domestic waste, agricultural surface runoff, land drainage and industrial
effluents in a water body leads to rapid nutrients enrichment in a water body. The excessive nutrient
enrichment in a water body encourages the growth of algae, water hyacinth, Phytoplankton and
other aquatic plants. The biological demand for oxygen (BOD) increases with the increase in aquatic
organisms.

5. Consider the following statement (s) related to the Mumbai Port


1. It is fully integrated multi-purpose Port handling container, dry, liquid bulk and break bulk cargo.
2. Prince, Victoria and Indira Dock are three wet enclosed docks.

A. 1 Only B. 2 Only C. Both 1 & 2 D. Neither 1 nor 2

Answer: C

Explanation: Since its institution in the latter part of the 19 th century, the Mumbai Port has played
an integral role in providing marine infrastructure in India. Comprising of a natural harbor extending
up to 400 square kilometres, the port is spread over a very vast area – 4, 63,000 square metres –
in totality. With a pier length touching nearly 8,000 km, the Mumbai Port has excellent road
transportation network of over 125 km and rail connectivity with both the city’s rail networks. This
makes the port exceedingly viable in terms of transportation connectivity with the whole of the city,
which in-turn minimises the time spent in transit. Comprising of three wet wharfing facilities and two
dry wharfing facilities, the port is equipped with facilities for handling both dry as well as frozen
cargo. Vessels can avail of re-fuelling facilities at about all berthing areas, thus further adding to the
www.achieveias.co.in, YouTube Channel: http://youtube.com/c/AchieveIAS Telegram Channel: http://t.me/Achieve_Ias,
Mail: achieveias21@gmail.com, Contact Number: 8968920720

port’s functionality. In the order of their establishment, the oldest wet wharfing is the Prince’s Dock
(1880) followed by Victoria Dock (1888) and the Indira Dock (1914). Of the three, the Indira Dock
has the largest berthing capacitance at 15, followed by Victoria at 14 and Prince at eight berths. All
the berthing facilities offer poly-utility. The two dry wharfing areas used for the purposes of upkeep
and furbishing of the vessels are Merewether and Hughes, located within the ambit of Prince Dock
and Indira Dock respectively.

6. Which of the following is the tidal port?


A. Kandla Port B. Kochi Port C. Tuticorin Port D. Ennore Port

Answer: A

Explanation: The Kandla port of India forms a vital marine network for vessels entering India
through its Western waters – the Arabian Sea. Situated in the Gulf of Kutch, in the province of
Gujarat Kutch, the port of Kandla has been in existence since the mid-1900s, specifically constructed
to fulfil the dearth of marine harbours in the country’s western water route. A sheltered natural cove,
the port of Kandla was first considered to take up the mantle of being India’s port of choice in its
western side, in the mid-19th century. At the time, at the peak of the British colonisation in India,
even though the port accounted for an appropriate marine interface, the construction and
consequentially the presence of the Karachi port made it the primary marine harbour in the whole
of western, undivided India.
1 . First port developed after independence.
2. it was made to ease the volume of trade on Mumbai port.
3. It is a tidal port.
4. it caters to the convenient handling of exports and imports of highly productive granary and
industrial belts across the states of Jammu and Kashmir, Himachal Pradesh, Punjab, Haryana,
Rajasthan and Gujarat.
5. it is a too beautiful place. It helps in agro transport.

7. Which is the artificial port and second busiest port in India after Mumbai?
A. New Mangalore B. Chennai Port C. Visakhapattanam Port D. Paradip

Answer: B

Explanation: Chennai is an important port of south India with its hinterland extending over Tamil
Nadu, south Andhra Pradesh and eastern Karnataka. It is an artificial port enclosing a sea-area of
81 hectares (0.91 km x 0.76 km) and depth of 15 m. The harbour has an entrance from the north
quay with a sheltering arm.It can accommodate up to 21 vessels inside the harbour which has 18
quay berths, 3 oil mooring berths, mechanical ore handling docks and 16 warehouses and 10 transit
sheds. Plans are under way to construct container terminal and extend quay area and berths. The
port mostly imports petroleum, coal, edible oils, fertilizers, chemicals, machinery, cotton, iron and
steel and metals etc. Its main exports are iron ore, food grains. Hides and skins, turmeric, tobacco,
mica, sugar, oil cake and timber etc. The port is the largest one in the Bay of Bengal. It is the third-
oldest port among the 13 major ports of India with official port operations beginning in 1881, although
maritime trade started much earlier in 1639 on the undeveloped shore. It is an artificial and all-
weather port with wet docks.

8. Which of the following is world-class international container handling port?


www.achieveias.co.in, YouTube Channel: http://youtube.com/c/AchieveIAS Telegram Channel: http://t.me/Achieve_Ias,
Mail: achieveias21@gmail.com, Contact Number: 8968920720

A. Jawaharlal Nehru Port B. Mumbai Port C. Cochin Port D. New Mangalore Port

Answer: A

Explanation: Jawaharlal Nehru Port, also known as Nhava Sheva port, is the largest container port
in India. East of Mumbai, in the state of Maharashtra, JNPT handles the majority of India’s
containerized trade, and is the focal point of long-haul calls to and from the emerging market
economy. Jawaharlal Nehru Port, abbreviated as JNPT, also known as Nhava Sheva, is the largest
container port in India. Located east of Mumbai (previously known as 'Bombay') in Navi
Mumbai, Maharashtra, this port on the Arabian Sea is accessed via Thane Creek. Its common name
derives from the names of Nhava and Sheva villages that were situated here. This port is also the
terminal of Western Dedicated Freight Corridor . The port was established on 26 May 1989. JN Port
is the biggest container handling port in India handling around 55% of the country’s containerized
cargo, crossing the historic landmark of 4 million TEUs (Twenty-foot equivalent units) in container
throughput consecutively for the last five years. In its coveted role as the Hub Port on the West
Coast of India, JNP is ranked 28th among the top 100 Container Ports in the world. Having set for
itself a long-term goal of achieving 10 million TEUs by the year 2020-21, through addition of two
more Terminals, viz. the 330M Stand-alone Container Terminals (DP World) and the 4th Container
Terminal (Port of Singapore Authority) and a Satellite Port at Vadhvan Point, JNP throws open an
array of worthwhile opportunities for the maritime trade, including the shipping lines and shippers,
to ferry their cargo to various sectors across the globe. JNP consist of a full-fledged Custom House,
30 Container Freight Stations and connectivity to 52 Inland Container Depots across the country.
The hinterland connectivity both by rail and road is being further strengthened by ongoing projects
like the Dedicated Freight Corridor (DFC), which will increase the existing train capacity of 27 to 100
trains per day; Multi-Modal Logistics Park (MMLP) and widening of the Port road connectivity. Its
proximity to the Cities of Mumbai, Navi Mumbai and Pune; airports; hotels, exhibition centres, etc.
gives the Port an extra edge to address the shippers’ needs, efficiently and promptly.

9. What kind of unemployment is found in the agricultural sector of India?


A. Situational B. Voluntary C. Frictional D. Disguise

Answer: D

Explanation: Disguised unemployment exists where part of the labor force is either left without work
or is working in a redundant manner where worker productivity is essentially zero. It is
unemployment that does not affect aggregate output. An economy demonstrates disguised
unemployment when productivity is low and too many workers are filling too few jobs. Disguised
unemployment exists frequently in developing countries whose large populations create a surplus
in the labor force. It can be characterized by low productivity and frequently accompaniesinformal
labor markets and agricultural labor markets, which can absorb substantial quantities of labor.
Disguised, or hidden, unemployment can refer to any segment of the population not employed at
full capacity, but it is often not counted in official unemployment statistics within the national
economy. This can include those working well below their capabilities, those whose positions
provide little overall value in terms of productivity, or any group that is not currently looking for work
but is able to perform work of value. Another way to think about disguised unemployment is to say
that people are employed but not in a very efficient way. They have skills that are being left on the
table, are working jobs that do not fit their skills (possibly due to an inefficiency in the market that
fails to recognize their skills), or are working but not as much as they would like.
www.achieveias.co.in, YouTube Channel: http://youtube.com/c/AchieveIAS Telegram Channel: http://t.me/Achieve_Ias,
Mail: achieveias21@gmail.com, Contact Number: 8968920720

10. Consider the following statements


1. The Biological Demand for oxygen is directly proportional to the quantity of discharge of waste
in water bodies.
2. Increase in water temperature decreases dissolved oxygen in water which adversely affects
aquatic life.

Select the correct answer from the following codes


A. 1 Only B. 2 Only C. Both 1 and 2 D. Neither 1 nor 2

Answer: C

Explanation: Aquatic organisms are adopted to a uniform steady temperature of environment and
any fluctuation in water temperature severely affects aquatic plants and animals. The biological
demand for oxygen (BOD) increases with the increase in aquatic organisms. Biochemical oxygen
demand (BOD) represents the amount of oxygen consumed by bacteria and other microorganisms
while they decompose organic matter under aerobic (oxygen is present) conditions at a specified
temperature. When you look at water in a lake the one thing you don't see is oxygen. In a way, we
think that water is the opposite of air, but the common lake or stream does contain small amounts
of oxygen, in the form of dissolved oxygen. Although the amount of dissolved oxygen is small, up to
about ten molecules of oxygen per million of water, it is a crucial component of natural water bodies;
the presence of a sufficient concentration of dissolved oxygen is critical to maintaining the aquatic
life and aesthetic quality of streams and lakes. The presence of a sufficient concentration of
dissolved oxygen is critical to maintaining the aquatic life and aesthetic quality of streams and lakes.
Determining how organic matter affects the concentration of dissolved oxygen (DO) in a stream or
lake is integral to water- quality management. The decay of organic matter in water is measured as
biochemical or chemical oxygen demand. Oxygen demand is a measure of the amount of oxidizable
substances in a water sample that can lower DO concentrations. Certain environmental stresses
(hot summer temperatures) and other human-induced factors (introduction of excess fertilizers to a
water body) can lessen the amount of dissolved oxygen in a water body, resulting in stresses on the
local aquatic life. One water analysis that is utilized in order to better understand the effect of bacteria
and other microorganisms on the amount of oxygen they consume as they decompose organic
matter under aerobic (oxygen is present) is the measure of biochemical oxygen demand (BOD).
Determining how organic matter affects the concentration of dissolved oxygen in a stream or lake is
integral to water-quality management. BOD is a measure of the amount of oxygen required to
remove waste organic matter from water in the process of decomposition by aerobic bacteria (those
bacteria that live only in an environment containing oxygen). The waste organic matter is stabilized
or made unobjectionable through its decomposition by living bacterial organisms which need oxygen
to do their work. BOD is used, often in wastewater-treatment plants, as an index of the degree of
organic pollution in water.
www.achieveias.co.in, YouTube Channel: http://youtube.com/c/AchieveIAS Telegram Channel: http://t.me/Achieve_Ias,
Mail: achieveias21@gmail.com, Contact Number: 8968920720

ACHIEVE IAS GEOGRAPHY MCQ SERIES, DAY 46, SOLUTIONS

1. Which of the following is not related to the human geography?


A. Bio-climatic B. Way of Human life
C. Economic way of Human life D. Social way of Human life

Answer: A

Explanation: Human Geography is concerned with the study of Cultural or Man-made features such
as houses, villages, towns, cities, railways, roads, bridges etc. Human Geography is a vast subject
and has the largest number of branches. Some of the important branches of human geography are
1. Social/Cultural
2. Population and Settlement
3. Historical
4. Political
5. Economic

2. Which of the following statement is not related to the dichotomy of Human Geography?
A. It is a substantive concept and once it is achieved it will address all the socio-cultural and
environmental ills of the society.
B. Neo-Malthusians, environmentalists and radical ecologists advocated the gap between the
resources and population has widened after eighteenth century.
C. Sir Robert Malthus was the first Scholar who raises his voice his concern on the growing
insecurity in social justice.
D. Both A & B

Answer: D

3. According to Census 2011, which of the following state has the lowest population density
in India?
A. Mizoram B. Nagaland C. Arunachal Pradesh D. Himanchal Pradesh

Answer: C

Explanation: According to Census-2011, the population density of Arunachal Pradesh is


only 17 people per square km.

4. What is the unit of population density measurement?


A. Family / meter B. Person/ sq. km C. Family / cm D. Person / Hector

Answer: B

Explanation: Population density is measured in person per km. It is measured that how many
people live in one km.

5. Which is the correct sequence of the decreasing population density of union territories?
A. Andaman & Nicobar, Delhi, Chandigarh, Puducherry
B. Delhi, Chandigarh, Puducherry, Daman & Diu
www.achieveias.co.in, YouTube Channel: http://youtube.com/c/AchieveIAS Telegram Channel: http://t.me/Achieve_Ias,
Mail: achieveias21@gmail.com, Contact Number: 8968920720

C. Daman and Diu, Puducherry, Chandigarh, Lakshadweep


D. Andaman & Nicobar Islands, Delhi, Daman & Diu, Chandigarh

Answer: B

Explanation: Delhi: 11320, Chandigarh: 9528, Puducherry: 2547, Daman and Diu: 2191

6. According to Census 2011, what was the population growth rate of India from 2001 to
2011?
A. 7.28% B. 21.8% C. 11.25% D.17.72%

Answer D

Explanation: According to Census 2011, the population of India has increased from 17.71% in 2001
to 2011.

7. Consider the following statement (s) is/ are related to the Population composition
1. Population is divided into two parts-rural and urban on the basis of the size and occupation of
settlements.
2. The distribution within a group of people of specified individual attributes such as sex, age,
marital status, education, occupation, and relationship to the head of household.

Which of the following statement(s) is true?


A. 1 Only B. 2 Only C. Both 1 & 2 D. Neither 1 nor 2

Answer: C

Explanation: The distribution within a group of people of specified individual attributes such as sex,
age, marital status, education, occupation, and relationship to the head of household is called
Population composition. Population is divided into two parts-rural and urban on the basis of the size
and occupation of settlements. The rural population consists of small sized settlements scattered
over the countryside. Urban population is one that lives in large size settlements i.e. towns and
cities.

Rural – Urban Composition: An important indicator of social and economic characteristics is the
composition of population by their respective places of residence. For the first time since
Independence, the absolute increase in population is more in urban areas that in rural areas. Rural
– Urban distribution: 68.84% & 31.16%. Level of urbanization increased from 27.81% in 2001
Census to 31.16% in 2011 Census. The proportion of rural population declined from 72.19% to
68.84%

Religious Composition: Religion is one of the most dominant forces affecting the cultural and
political life of the most of Indians. Since religion virtually permeates into almost all the aspects of
people’s family and community lives, it is important to study the religious composition in
detail. Population Growth rate of various religion has come down in the last decade (2001-2011).
Hindu Population Growth rate slowed down to 16.76 % from previous decade figure of 19.92% while
Muslim witness sharp fall in growth rate to 24.60% (2001-2011) from the previous figure of 29.52 %
(1991-2001). Such sharp fall in population growth rate for Muslims didn't happened in the last 6
www.achieveias.co.in, YouTube Channel: http://youtube.com/c/AchieveIAS Telegram Channel: http://t.me/Achieve_Ias,
Mail: achieveias21@gmail.com, Contact Number: 8968920720

decades. Christian Population growth was at 15.5% while Sikh population growth rate stood at 8.4%.
The most educated and wealthy community of Jains registered least growth rate in 2001-2011 with
figure of just 5.4%. The Growth rate of Hindus, Muslims and Christian is expected to fall more in
upcoming 2021 census while other religions like Sikhism, Jainism and Buddhism are expected to
remain stable for next 2 decades considering already slowed down growth rate of these religions.

8. Which of the following attributes supplement when the geographer defines patterns and
processes of spatial interaction?
A. Accessibility and connectivity. B. Density and dispersion.
C. Diffusion and pattern. D. Pedestrian cities.

Answer: D

Explanation: SPATIAL INTERACTION IS A dynamic flow process from one location to another. It
is a general concept that may refer to the movement of human beings such as intraurban commuters
or intercontinental migrants, but may also refer to traffic in goods such as raw materials or to flows
of intangibles such as information. While the origin of the term may be traced to French geographers
of the early 20th century, Edward Ullman's Geography as Spatial Interaction is normally cited as the
seminal statement of the concept. In Ullman's conception there were “three bases for spatial
interaction” or more fundamentally, three reasons for why things move: complementarity,
transferability, and intervening opportunity. Complementarity refers to the presence of a demand or
deficit at one location and a supply or surplus at another without which there is no economic
rationale for any movement. A workplace such as a factory or office tower is an example of a place
with a demand for labor, while a residential neighborhood provides a source of workers. A sawmill
requires logs, while a forest provides them. To adapt a metaphor from physics, complementarity is
like a potential gradient with goods and people flowing from a higher energy state, where they are
in surplus, to a lower energy state, where they are in deficit. From the realm of PHYSICAL
GEOGRAPHY, wind is the flow of air between complementary atmospheric zones: from a high-
pressure cell to a low-pressure cell.

9. Which of the following states has the highest population growth rate?
A. West Bengal B. Uttar Pradesh C. Maharashtra D. Uttarakhand

Answer: B

Explanation: Uttar Pradesh has the highest population growth rate of 20.23% among all the state

10. Consider the following statement (s) related to composition of population.


1. The population is usually divided into three age groups- children (0-14yrs), adults (15-59yrs) and
aged (60 and over).
2. Population of males and females, children, young and old comprises the population of a country.

Which is / are correct option?


A. 1 Only B. 2 Only C. Both 1 and 2 D. Neither 1 nor 2

Answer: C
www.achieveias.co.in, YouTube Channel: http://youtube.com/c/AchieveIAS Telegram Channel: http://t.me/Achieve_Ias,
Mail: achieveias21@gmail.com, Contact Number: 8968920720

Explanation: Population of males and females, children, young and old comprises the population
of a country. The population is usually divided into three age groups- children (0-14yrs), adults (15-
59yrs) and aged (60 and over). This is called age-group of population. The proportion of adult
population is the least variable in the three groups. The main difference is found in the population of
children and old people. The proportion of children is quite low in the first world countries like
Sweden whereas in countries like Japan, the proportion of old people is high. When it comes to
India, the sex ratio, the number of females per thousand males is very low. Only Kerala is an
exception with a higher number of females per thousand males and also have good literacy rate. A
person who is above 7yrs and can read and write any language with understanding is called a
literate. Literacy, the percentage of literate people, is one of the indicators of the quality of
population.
www.achieveias.co.in, YouTube Channel: http://youtube.com/c/AchieveIAS Telegram Channel: http://t.me/Achieve_Ias,
Mail: achieveias21@gmail.com, Contact Number: 8968920720

ACHIEVE IAS GEOGRAPHY MCQ SERIES, DAY 47, SOLUTIONS

1. Which one of the following is NOT one of the indicators for the Human Development Index
dimensions?
A. Real GNP per capita (PPP$) B. Birth rates
C. Life expectancy at birth D. Educational attainment

Answer: B

Explanation: UNDP has released the annual HDI 2019 report. The focus of the 2019 Report is on
‘Inequality in Human Development’. India’s position: India’s rank- 129. Last year’s rank- 130.
Despite lifting 271 million people out of poverty between 2005-15, India still remains home to 28%
(364 million) of the world’s poor. Between 1990 and 2018, India’s HDI value increased by 50 per
cent (from 0.431 to 0.647), which places it above the average for countries in the medium human
development group (0.634) and above the average for other South Asian countries (0.642).
This means that in the last three decades, life expectancy at birth in India increased by 11.6 years,
whereas the average number of schooling years increased by 3.5 years. Per capita incomes
increased 250 times. India is only marginally better than the South Asian average on the Gender
Development Index (0.829 vs 0.828), and ranks at a low 122 (of 162) countries on the 2018 Gender
Inequality Index.

India’s neighbours: Sri Lanka (71) and China (85),Bhutan (134), Bangladesh (135), Myanmar
(145), Nepal (147), Pakistan (152) and Afghanistan (170).

Global scenario: Norway, Switzerland, Ireland occupied the top three positions in that order.
Globally, there are 1.3 billion poor people. Around 661 million of these poor people live in Asia and
the Pacific. South Asia constitutes 41% of the world’s poor.
Changing nature of inequality: As the number of people coming out of poverty is increasing, the
world is veering towards another type of poverty. The old inequalities were based on access to
health services and education whereas the next generation of poverty is based on technology,
education and climate.
What is HDI: Published by the United Nations Development Programme (UNDP), it is a statistical
tool used to measure a country’s overall achievement in its social and economic dimensions. The
social and economic dimensions of a country are based on the health of people, their level of
education attainment and their standard of living. It is part of the Human Development Report.

The other indices that form the part of the 2019 Report are:
1. Inequality-adjusted Human Development Index (IHDI),
2. Gender Development Index (GDI),
3. Gender Inequality Index (GII) and
4. Multidimensional Poverty Index (MPI).

HDI measures average achievement of a country in three basic dimensions of human development:
1. A long and healthy life.
2. Access to knowledge.
3. A decent standard of living.
www.achieveias.co.in, YouTube Channel: http://youtube.com/c/AchieveIAS Telegram Channel: http://t.me/Achieve_Ias,
Mail: achieveias21@gmail.com, Contact Number: 8968920720

2. Which of the following statements is correct regarding Economic Development and Human
Development?
A. The basic difference between Economic development and Human development is that Human
development entirely focuses on the increase of income whereas the economic development
believes in expanding and widening of all aspects of human life be it economic, social, political,
cultural, etc.
B. In economic aspect human development is one of the essential elements.
C. The basic idea behind Human development is that it is the use of income and not the income
itself that decides the human choices.
D. The real wealth of a nation is its Economy; therefore, the goal of development should be the
enrichment of the whole economy.

Answer: B

Explanation: The basic difference between economic development and human development is that
economic development entirely focuses on the increase of income whereas the human development
believes in expanding and widening of all aspects of human life be it economic, social, political,
cultural, etc. In economic aspect human development is one of the essential elements. The basic
idea behind this is that it is the use of income and not the income itself that decides the human
choices. Since, the real wealth of a nation is its people; therefore, the goal of development should
be the enrichment of human life.

3. Consider the following reasons which are responsible to keep India at the bottom of the
Human Development:
1. Rapid increase in population
2. Large number of adult illiterates and low gross enrolment ratio
3. Inadequate government expenditure on education and health

Which of the following statement(s) is/are correct?


A. 1 Only B. 1 and 2 Only C. 1, 2 and 3 D. 1 and 3 Only

Answer: C

Explanation: The following are the reasons to keep India at the bottom of human development (a)
rapid increase in population (b) large number of adult illiterates and low gross enrolment ratio (c)
high drop-out rates (d) inadequate government expenditure on education and health, (e) large
proportion of underweight children as well as under nourished people (f) very poor sanitation
facilities and low access to essential lifesaving medicines.

4. Consider the following statement (s) related to pillars of human development.


1. Equity means making equal access to opportunities available to everybody that opportunities
available to people must be equal irrespective of their gender, race, income and in the Indian case,
caste.
2. Sustainability to the human labour productivity or productivity in terms of human work that must
be constantly enriched by building capabilities in people.

Which is / are correct option?


A. 1 Only B. 2 Only C. Both 1 and 2 D. Neither 1 nor 2
www.achieveias.co.in, YouTube Channel: http://youtube.com/c/AchieveIAS Telegram Channel: http://t.me/Achieve_Ias,
Mail: achieveias21@gmail.com, Contact Number: 8968920720

Answer: A

Explanation: The term ‘human development’ may be defined as an expansion of human


capabilities, a widening of choices, ‘an enhancement of freedom, and a fulfilment of human rights.At
the beginning, the notion of human development incorporates the need for income expansion.
However, income growth should consider expansion of human capabilities. Hence development
cannot be equated solely to income expansion. The first UNDP Human Development Report
published in 1990 stated that: “The basic objective of development is to create an enabling environ-
ment for people to enjoy long, healthy and creative lives.” It also defined human development as “a
process of enlarging people’s choices”, “and strengthen human capabilities” in a way which enables
them to lead longer, healthier and fuller lives.

Components of Human Development:

These are:
1. Equality,
2. Sustainability,
3. Productivity, and
4. Empowerment.

1. Equality: If development is viewed in terms of enhancing people’s basic capabilities, people must
enjoy equitable access to opportunities. Such may be called equality-related capabilities. To ensure
equality-related capabilities or access to opportunities what is essential is that the societal
institutional structure needs to be more favourable or progressive. In other words, the unfavourable
initial asset distribution, like land, can be made more farmer-friendly through land reform and other
redistributive measures. In addition, uneven income distribution may be addressed through various
tax-expenditure policies. Economic or legislative- measures that interferes with market exchange
may enable people to enlarge their capabilities and, hence, well-being. Further, to ensure basic
equality, political opportunities need to be more equal. In the absence of effective political
organisation, disadvantaged groups are exploited by the ‘rich’ to further their own interests rather
than social goals. However, participatory politics gets a beating by the inequality in opportunities in
having basic education. It is to be added here that basic education serves as a catalyst of social
change. Once the access to such opportunity is opened up in an equitable way, women or religious
minorities or ethnic minorities would be able to remove socio-economic obstacles of development.
This then surely brings about a change in power relations and makes society more equitable.

2. Sustainability: Another important facet of human development is that development should ‘keep
going’, should ‘last long’. The concept of sustainable development focuses on the need to maintain
the long term protective capacity of the biosphere. This then suggests that growth cannot go on
indefinitely; there are, of course, ‘limits to growth. Here we assume that environment is an essential
factor of production. In 1987, the Bruntland Commission Report (named after the then Prime Minister
Go Harlem Bruntland of Norway) defined sustainable development as ‘… development that meets
the needs of the present without compromising the ability of future generations to meet their basic
needs.’ This means that the term sustainability focuses on the desired balance between future
economic growth and environ-mental quality. To attain the goal of sustainable development, what
is of great impor-tance is the attainment of the goal of both intra- generation and inter-generation
equality. This kind of inequality includes the term ‘social well-being’ not only for the present
www.achieveias.co.in, YouTube Channel: http://youtube.com/c/AchieveIAS Telegram Channel: http://t.me/Achieve_Ias,
Mail: achieveias21@gmail.com, Contact Number: 8968920720

generation but also for the people who will be on the earth in the future. Any kind of environmental
decline is tantamount to violation of distributive justice of the disadvan-taged peoples. Social well-
being thus, then, depends on environ-mental equality.

3. Productivity: Another component of human development is productivity which requires investment


in people. This is commonly called investment in human capital. Investment in human capital—in
addition to physical capital—can add more productivity. The improvement in the quality of human
resources raises the productivity of existing resources. Theodore W. Schultz—the Nobel Prize-
winning economist—articulated its importance: “The decisive factors of production in improving the
welfare of poor people are not space, energy, and crop land; the decisive factor is the improvement
in population quality.” Empirical evidence from many East Asian countries corroborate this view.

4. Empowerment: The empowerment of people—particularly women—is another com-ponent of


human development. In other words, genuine human development requires empower-ment in all
aspects of life. Empowerment implies a political democracy in which people themselves make the
decisions about their lives. Under it, people enjoy greater political and civil liberties and remain free
from excessive controls and regu-lations. Empowerment refers to decentralisation of power so that
the benefits of governance are reaped by all peoples.

It focuses on grassroots participation which promotes democracy by enfranchising the


disadvantaged groups. Unfortu-nately, benefits are cornered by the elites because of lack of
empowerment of people. Participation as a goal is a feature of ‘bottom-up’ development strategy
rather than ‘top-down’. Further, develop­ment policies and strategies male-dominated. But the
benefits of development are to be made ‘gender-sensitive’. Discrimination against women in health
and education is very costly from the viewpoint of achieving development goals. Education of women
can lead to a virtuous circle of lower fertility, better care of children, more educational oppor-tunity,
and higher productivity. Above all, as women’s education rises, women’s independence in making
their own choices also increase. Anyway, decentralization and participation empower people,
specially the women and the poor. It then breaks the ‘deprivation trap’. Mahbub ul Haq asserts: “If
people can exercise their choices in the political, social and economic spheres, there is a good
prospect that growth will be strong, democratic, participatory and durable.”

5. Consider the following statements about the population density/growth in India. Which are
Correct?
1. Assam and most of the peninsular states have moderate population densities
2. The Northern Plains and Kerala in the south have high to very high population densities
3. Since 1981 the rate of population growth started declining gradually in India
4. Meghalaya is having the highest decadal growth in the 2011 census

A. 1 & 2 Only B. 1, 2 & 4 Only C. 1, 2 & 3 Only D. All the above

Answer: D

Explanation: Areas of Moderate Density: Gujarat (258), Karnataka (276), Andhra Pradesh (277),
Tripura (305), Maharashtra (315), Jharkhand (338), Assam (340), Goa (364), Dadra and Nagar
Haveli (449), Haryana (478), Tamil Nadu (480) and Punjab (484) are included in this category.
Since 1981, the rate of growth started declining gradually, because of popularized.
(a) Family planning measures were adopted, leading to decline in the birth rate.
www.achieveias.co.in, YouTube Channel: http://youtube.com/c/AchieveIAS Telegram Channel: http://t.me/Achieve_Ias,
Mail: achieveias21@gmail.com, Contact Number: 8968920720

(b) Awareness about advantages of small family came to be recognized.


(c) There was a growth of nuclear families occured which adopted the small family norm.
(d) Promotion of family planning programme by the government.
The population of Meghalaya is only 0.25 percent of the total population of India. The decadal
growth rate of India between 2001 and 2011 census is found to be 17.68, whereas decadal
growth rate of Meghalaya in the decade is 27.95. ... Population between 2001 and 2011 is 28.25
which is higher than the State growth rate.

6. Which of the following statements is/are true?


1. From 1901 to 1911 India’s population registered a four-fold growth.
2. From 1901 to 1951, India’s population grew only one and half times.

A.1 only B only C. Both 1 and 2 D. Neither 1 nor 2

Answer: C

Explanation: India’s population has grown steadily from 1901, except for a decrease in 1921. It
increased from 23.8 crore in 1901 to 36.1 crore in 1951 and further to 102.7 crore in 2001, as shown
in Table 31.1. It reveals that during the first 50 years from 1901 to 1951, population grew by 12.3
crore while during the next 50 years from 1951 to 2001, it increased by more than 5 times, that is,
by 66.6 crore. The growth of population in India can be divided into four phases as under:

1. Phase of Stagnant Population: 1901-21: During this period, population increased by 1.29 crore
only. If we were to break this period in accordance with the two censuses, we find that during 1901-
11, the decadal growth rate was only 5.75 per cent, as the natural growth rate of population was low
(6.6).But in the next decade (1910-21), the growth rate of population declined to (-) 0.3 per cent.
This was due to high death rate (47.2) on account of famines and epidemics like plague, cholera
and malaria which took a heavy toll of human lives. The year 1921 is, therefore, known as the year
of Great Divide.

2. Phase of Steady Population Growth: 1921-51: From 1921 onwards till 1951 there was a steady
growth of population. During these years, population increased by 2.8 crore in 1931, to 4 crore in
1941 and to 5 crore in 1951. But the decadal growth rate in 1951 census was 0.9 percentage less
than in the 1941 census due to the Partition of India in 1947.The main reason for the steady increase
in population was the decline in death rate from 36.3 to 27.4 and of birth rate from 46.4 to 39.9 over
the period. This resulted in an average natural growth rate of 12.2 during this phase of 30 years.

3. Phase of Rapid Population Growth or of Population Explosion: 1951-81: This was the period of
rapid population growth when population increased by 32.5 crore as against about 12 crore during
the last 50 years from 1901 to 1951. The average annual growth rate of population increased from
1.25 per cent to 2.2 per cent in 1981. The main reason was a large decline in the death rate from
22.8 to 15 due to better medical facilities while the birth rate fell slowly from 41.7 to 37.2 over the
period. This led to population explosion in the country.

4. Phase of High Growth with Declining Trend: 1981-2001: During this phase, a definite declining
trend of population growth is visible. Total population increased by 34.37 crore during 20 years. But
the average annual growth rate had definitely declined to 1.93 per cent in 2001 as against 2.2 per
cent in 1981. However, the trend of population explosion still persists even now.
www.achieveias.co.in, YouTube Channel: http://youtube.com/c/AchieveIAS Telegram Channel: http://t.me/Achieve_Ias,
Mail: achieveias21@gmail.com, Contact Number: 8968920720

7. During which decade did India see a negative population growth?


A. 1901-11 B. 1931-41 C. 1911-21 D. 1951-61

Answer: C

8. Who among the following economists introduced the term Human Development?
A. Mehbub Al Haque B. Amartya Sen C. Adam Smith D. Both a and b

Answer: D

Explanation: In 1990 two economists - Prof. Mehbub Al Haque and Prof. Amartya Sen introduced
the concept of Human Development. From 1990 onwards, United Nations Development Programme
(UNDP), each year calculate Human Development Index (HDI) and publish as a report which is
known as Human Development Report (HDR).

9. What is the rank of India in the Human Development Index 2018?


A. 142nd B. 136th C. 140th D. 130th

Answer: D

Explanation: India climbed one spot to 130 out of 189 countries in the latest human development
rankings released today by the United Nations Development Programme (UNDP). India’s HDI value
for 2017 is 0.640, which put the country in the medium human development category. Between 1990
and 2017, India’s HDI value in case from 0.427 to 0.640, an increase of nearly 50 percent – and an
indicator of the country’s remarkable achievement in lifting millions of people out of poverty. Norway,
Switzerland, Australia, Ireland and Germany lead the ranking, while Niger, the Central African
Republic, South Sudan, Chad and Burundi have the lowest scores in the HDI’s measurement of
national achievements in health, education and income. Within South Asia, India’s HDI value is
above the average of 0.638 for the region, with Bangladesh and Pakistan, countries with similar
population size, being ranked 136 and 150 respectively. Movements in the HDI are driven by
changes in health, education and income. Health has improved considerably as shown by life
expectancy at birth, which has increased by almost seven years globally, with Sub-Saharan Africa
and South Asia showing the greatest progress, each experiencing increases of about 11 years since
1990. And, today’s school-age children can expect to be in school for 3.4 years longer than those in
1990. Between 1990 and 2017, India’s life expectancy at birth too increased by nearly 11 years,
with even more significant gains in expected years of schooling. Today’s Indian school-age children
can expect to stay in school for 4.7 years longer than in 1990. Whereas, India’s GNI per capita
increased by a staggering 266.6 percent between 1990 and 2017.

10. Which of the following is/are considered as the indicator (s) of Human Development?
A. HPI-1 B. GEM C. GDI D. All of the above

Answer: D

Explanation: Apart from Human Development Index (HDI), the other four indicators of human
development have been selected which were used by the Human Development Report. These are:
www.achieveias.co.in, YouTube Channel: http://youtube.com/c/AchieveIAS Telegram Channel: http://t.me/Achieve_Ias,
Mail: achieveias21@gmail.com, Contact Number: 8968920720

1. Human Poverty Index for developing countries (HPI-1)


2. Human Poverty Index for selected DECD Countries (HPI-2)
3. Gender - related Development Index (GDI)
4. Gender Empowered Measurement (GEM)
www.achieveias.co.in, YouTube Channel: http://youtube.com/c/AchieveIAS Telegram Channel: http://t.me/Achieve_Ias,
Mail: achieveias21@gmail.com, Contact Number: 8968920720

ACHIEVE IAS GEOGRAPHY MCQ SERIES, DAY 48, SOLUTIONS

1. Which of the following statements is/are correct?


1. Tertiary sector economic activities, by themselves, do not produce a good but they are an aid or
a support for the production process.
2. Tertiary sector is also called the service sector.

Select the correct answer using the code given below:


A. 1 only B. 2 only C. Both 1 and 2 D. Neither 1 nor 2

Answer: C

Explanation: Tertiary sector economic activities, by themselves, do not produce a good but they
are an aid or a support for the production process. Transport, storage, communication, banking,
trade are some examples of tertiary activities. Since these activities generate services rather than
goods, the tertiary sector is also called the service sector.

2. What percentage of Indian GDP is contributed by the service sector?


A. 25 % of GDP B. 54.40 % of GDP C. 53% of GDP D. 49 % of GDP

Answer: B

Explanation: Services sector is the largest sector of India. Gross Value Added (GVA) at current
prices for Services sector is estimated at 92.26 lakh crore INR in 2018-19. Services sector accounts
for 54.40% of total India's GVA of 169.61 lakh crore Indian rupees. With GVA of Rs. 50.43 lakh
crore, Industry sector contributes 29.73%. While, Agriculture and allied sector shares 15.87%. At
2011-12 prices, composition of Agriculture & allied, Industry, and Services sector are 14.39%,
31.46%, and 54.15%, respectively. Share of primary (comprising agriculture, forestry, fishing and
mining & quarrying), secondary (comprising manufacturing, electricity, gas, water supply & other
utility services, and construction) and tertiary (services) sectors have been estimated as 18.57 per
cent, 27.03 per cent and 54.40 per cent. Sector wise Indian GDP composition in 2017 are as follows
: Agriculture (15.4%), Industry (23%) and Services (61.5%). With production of agriculture activity
of $375.61 billion, India is 2nd larger producer of agriculture product. India accounts for 7.39 percent
of total global agricultural output. India is way behind china which has $991 bn GDP in agriculture
sector. GDP of Industry sector is $560.97 billion and world rank is 6. In Services sector, India world
rank is 8 and GDP is $1500 billion.
Contribution of Agriculture sector in Indian economy is much higher than world's average (6.4%).
Contribution of Industry and Services sector is lower than world's average 30% for Industry sector
and 63% for Services sector.

3. Which two of the following are likely to result in the tertiary sector taking a higher
proportion of total output and employment?
A. A shift of consumer tastes in favour of manufactured goods
B. A shift of consumer tastes in favour of services
C. A high income elasticity of demand for services at a time of rapid economic growth
D. Both B & C

Answer: D
www.achieveias.co.in, YouTube Channel: http://youtube.com/c/AchieveIAS Telegram Channel: http://t.me/Achieve_Ias,
Mail: achieveias21@gmail.com, Contact Number: 8968920720

4. Mixed economy means an economy where there is


A. Existence of capitalism
B. Privatization, liberalization and globalization
C. Existence of both public and private sectors
D. Growing crops along with rearing animals

Answer: C

Explanation: In India, there is existence of both public and private sectors working together. There
is a blend of socialism and capitalism A mixed economic system is a system that combines aspects
of both capitalism and socialism. A mixed economic system protects private property and allows a
level of economic freedom in the use of capital, but also allows for governments to interfere in
economic activities in order to achieve social aims. According to neoclassical theory, mixed
economies are less efficient than pure free markets, but proponents of government interventions
argue that the base conditions required for efficiency in free markets, such as equal information and
rational market participants, cannot be achieved in practical application.

KEY TAKEAWAYS:
1. A mixed economy is an economy organized with some free market elements and some socialistic
elements, which lies on a continuum somewhere between pure capitalism and pure socialism.
2. Mixed economies typically maintain private ownership and control of most of the means of
production, but often under government regulation.
3. Mixed economies socialize select industries that are deemed essential or that produce public
goods.
4. All known historical and modern economies are examples of mixed economies, though some
economists have critiqued the economic effects of various forms of mixed economy.

5. Consider the following statements.


1. National Highways constitute less than 2% of the total road length but carry about 40% of the
traffic.
2. National Highway No. 7 is the longest highway in India.

A. 1 Only B. 2 Only C. Both 1 and 2 D. Neither 1 nor 2

Answer: C

Explanation: A major part of NH 7 covers the North-South Corridor of NHDP and it is officially listed
as running over 2,369 km (1,472 mi) from Varanasi to Kanyakumari. It is the longest national
highway in India. The National Highways constitute only two per cent of the total road length but
carry 40 per cent of the road traffic. The National Highways Authority of India (NHAI) is the apex
body to improve the quality of the National Highways. It was constituted by an act of Parliament,
the National Highways Authority of India Act, 1988. It is responsible for the development,
maintenance and management of National Highways entrusted to it and for matters connected or
incidental thereto.

6. On which river National Waterway 1 (NW1) is located?


A. Ganga B. Mahanadi C. Barak D. Narmada
www.achieveias.co.in, YouTube Channel: http://youtube.com/c/AchieveIAS Telegram Channel: http://t.me/Achieve_Ias,
Mail: achieveias21@gmail.com, Contact Number: 8968920720

Answer: A

Explanation: The National Waterway 1 or NW-1 or Ganga-Bhagirathi-Hooghly river system is


located in India and runs from Haldia (Sagar) to Prayagraj across
the Ganges, Bhagirathi and Hooghly river systems.[1] It is 1,620 km (1,010 mi) long,[2] making it the
longest waterway in India.[3] It is of prime importance amongst all the national waterways
considering its locational advantages. The NW-1 passes through West
Bengal, Jharkhand, Bihar and Uttar Pradesh and serves major cities and their industrial hinterlands.
Transportation plays an important role in the development of a country and it is of great significance
for a developing country like India. The country is bestowed with a plethora of diverse topography
which enables different kinds of transportation. India has about 14500 km of navigable waterways.
This includes rivers, backwaters, canals, creeks and so on. National Waterways Act came into effect
in 2016. It proposed 106 additional National Waterways and merges 5 existing Acts which were
declared the 5 National Waterways. In 1986, the Government of India created Inland Waterways
Authority of India (IWAI) for regulation and development of Inland Waterways for navigation and
shipping. Out of the 111 National Waterways declared under the National Waterways Act, 2016, 13
are operational for shipping and navigation and cargo/passenger vessels are moving on them.
About the IWAI Inland Waterways Authority of India: This body was created by the government of
India in 1986 for regulating and developing inland waterways for shipping and navigation. The body
chiefly undertakes development and maintenance projects of IWT infrastructure on national
waterways. It undertakes these projects through grants from the Shipping Ministry. Its headquarters
is in Noida. It also has regional offices in various other cities and towns across the country.

7. Which of the following waterways transports raw materials of eastern countries to western
countries and transport industrial products to eastern countries?
A. South Atlantic Oceanic Route B. North Atlantic Oceanic Route
C. Routes of Mediterranean Sea D. Cape of Good Hope route

Answer: C

Explanation: The Routes of Mediterranean Sea transports raw materials of eastern countries to
western countries and transport industrial products to eastern countries.

8. Which of the following sea route gives maximum benefits to the countries like China,
Korea, Japan, Philippines, Indonesia, Malaysia, Singapore and Hong Kong?
A. South Atlantic Oceanic Route B. North Atlantic Oceanic Route
C. Routes of Mediterranean Sea D. North Pacific Oceanic Route

Answer: D

Explanation: North Pacific Oceanic Route join East Asia and North America with each other. This
waterway is very long. From this route the countries like China. Korea, Japan, Philippines, Indonesia,
Malaysia, Singapore and Hong Kong take maximum benefits.

9. Which of the following port is considered as a gateway to golden opportunities for trade
by India, Iran and Afghanistan with central Asian countries?
A. Jebel Ali Port, Dubai B. Tianjin, China
www.achieveias.co.in, YouTube Channel: http://youtube.com/c/AchieveIAS Telegram Channel: http://t.me/Achieve_Ias,
Mail: achieveias21@gmail.com, Contact Number: 8968920720

C. Guangzhou Harbor, China D. Chabahar Port

Answer: D

Explanation: The Chabahar port in the Sistan-Balochistan province in the southern coast of Iran
is easily accessible from India's western coast and is increasingly seen as a counter to Pakistan's
Gwadar Port located at distance of around 80 km from Chabahar. It is being considered as a
gateway to golden opportunities for trade by India, Iran and Afghanistan with central Asian
countries.

10. What body of water does the Panama Canal connect to the Pacific Ocean?
A. Indian Ocean B. Atlantic Ocean C. Southern Ocean D. Red Sea

Answer: B

Explanation: Panama Canal, Spanish Canal de Panamá, lock-type canal, owned and administered
by the Republic of Panama, that connects the Atlantic and Pacific oceans through the narrow
Isthmus of Panama. The length of the Panama Canal from shoreline to shoreline is about 40 miles
(65 km) and from deep water in the Atlantic (more specifically, the Caribbean Sea) to deep water in
the Pacific about 50 miles (82 km). The canal, which was completed in August 1914, is one of the
two most strategic artificial waterways in the world, the other being the Suez Canal. Ships sailing
between the east and west coasts of the United States, which otherwise would be obliged to round
Cape Horn in South America, shorten their voyage by about 8,000 nautical miles (15,000 km) by
using the canal. Savings of up to 3,500 nautical miles (6,500 km) are also made on voyages between
one coast of North America and ports on the other side of South America. Ships sailing between
Europe and East Asia or Australia can save as much as 2,000 nautical miles (3,700 km) by using
the canal. From its opening in 1914 until 1979, the Panama Canal was controlled solely by the
United States, which built it. In 1979, however, control of the canal passed to the Panama Canal
Commission, a joint agency of the United States and the Republic of Panama, and complete control
passed to Panama at noon on December 31, 1999. Administration of the canal is the responsibility
of the Panama Canal Authority (Spanish: Autoridad del Canal de Panamá [ACP]), which answers
solely to the government of Panama. The Panama Canal lies at a latitude of 9° N, at a point where
the North American Continental Divide dips to one of its lowest points. The canal does not, as is
generally supposed, cross the isthmus from east to west. It runs due south from its entrance at
Colón on the Atlantic side through the Gatún Locks to a point in the widest portion of Gatún Lake; it
then turns sharply toward the east and follows a course generally to the southeast until it reaches
the Bay of Panama, on the Pacific side. Its terminus near Balboa is some 25 miles (40 km) east of
its terminus near Colón. Parallel to the canal are the Panama Canal Railway and the Boyd-Roosevelt
Highway. In passing from the Atlantic to the Pacific, vessels enter the approach channel in Limón
Bay, which extends a distance of about 7 miles (11 km) to the Gatún Locks. At Gatún a series of
three locks lifts vessels 85 feet (26 metres) to Gatún Lake. The lake, formed by Gatún Dam on the
Chagres River and supplemented by waters from Alajuela Lake (Lake Madden; formed by the
Madden Dam), covers an area of 166 square miles (430 square km). The channel through the lake
varies in depth from 46 to 85 feet (14 to 26 metres) and extends for about 23 miles (37 km) to
Gamboa. Gaillard (Culebra) Cut begins at Gamboa and passes through the Continental Divide. The
channel through the cut has an average depth of about 43 feet (13 metres) and extends some 8
miles (13 km) to the Pedro Miguel Locks. The locks lower vessels 30 feet (9 metres) to Miraflores
Lake, at an elevation of 52 feet (16 metres) above sea level. Vessels then pass through a channel
www.achieveias.co.in, YouTube Channel: http://youtube.com/c/AchieveIAS Telegram Channel: http://t.me/Achieve_Ias,
Mail: achieveias21@gmail.com, Contact Number: 8968920720

almost 1.2 miles (2 km) long to the two-stepped locks at Miraflores, where they are lowered to sea
level. The final segment of the canal is a dredged approach passage 7 miles long through which
ships pass into the Pacific. Throughout its length the canal has a minimum bottom width of 500 feet
(150 metres); in Gatún Lake the width of the channel varies between 500 and 1,000 feet (150 and
300 metres), and in Miraflores Lake the width is 740 feet (225 metres).
www.achieveias.co.in, YouTube Channel: http://youtube.com/c/AchieveIAS Telegram Channel: http://t.me/Achieve_Ias,
Mail: achieveias21@gmail.com, Contact Number: 8968920720

ACHIEVE IAS GEOGRAPHY MCQ SERIES, DAY 49, SOLUTIONS

1. Consider the following statements:


1. When we produce a good by exploiting natural resources, it is an activity of the primary sector.
2. The secondary sector covers activities in which natural products are changed into other forms
through ways of manufacturing that we associate with industrial activity.

Which of the statements given above is/are correct?


A. 1 only B. 2 only C. Both 1 and 2 D. Neither 1 nor 2

Answer: C

Explanation: When we produce a good by exploiting natural resources, it is an activity of


the primary sector, because it forms the base for all other products that we subsequently make.
Since most of the natural products we get are from agriculture, dairy, fishing, forestry, this sector is
also called agriculture and allied sector. The secondary sector covers activities in which natural
products are changed into other forms through ways of manufacturing that we associate with
industrial activity. It is the next step after primary. The product is not produced by nature but has to
be made and therefore some process of manufacturing is essential. This could be in a factory, a
workshop or at home.

2. In this farming the land is used for growing food and fodder crops and rearing livestock?
A. Intensive Farming B. Plantation Farming
C. Primitive Farming D. Mixed Farming

Answer: D

Explanation: Mixed farming exists in many forms depending on external and internal factors.
External factors are weather patterns, market prices, political stability, technological developments,
etc. Internal factors relate to local soil characteristics, composition of the family and farmers'
ingenuity. Farmers can decide to opt for mixed enterprises when they want to save resources by
interchanging them on the farm - because these permit wider crop rotations and thus reduce
dependence on chemicals, because they consider mixed systems closer to nature, or because they
allow diversification for better risk management. There is wide variation in mixed systems. Even
pastoralists practise a form of mixed farming since their livelihood depends on the management of
different feed resources and animal species. At a higher level, a region can consist of individual
specialized farms and service systems that together act as a mixed system. Other forms of mixed
farming include cultivation of different crops on the same field, such as millet and cowpea or millet
and sorghum, or several varieties of the same crop with different life cycles, which uses space more
efficiently and spreads risks more uniformly. Mixed farming systems can be classified in many ways
- based on land size, type of crops and animals, geographical distribution, market orientation, etc.
Three major categories, in four different modes of farming, are distinguished here. The categories
are:
 On-farm versus between-farm mixing
 Mixing within crops and/or animal systems
 Diversified versus integrated systems.

3. The type of agriculture practiced in India is:


www.achieveias.co.in, YouTube Channel: http://youtube.com/c/AchieveIAS Telegram Channel: http://t.me/Achieve_Ias,
Mail: achieveias21@gmail.com, Contact Number: 8968920720

A. Intensive Farming B. Extensive Farming C. Primitive Farming D. Mixed Farming

Answer: A

Explanation: Intensive agriculture, in agricultural economics, system of cultivation using large


amounts of labour and capital relative to land area. Large amounts of labour and capital are
necessary to the application of fertilizer, insecticides, fungicides, and herbicides to growing crops,
and capital is particularly important to the acquisition and maintenance of high-efficiency machinery
for planting, cultivating, and harvesting, as well as irrigation equipment where that is required.
Optimal use of these materials and machines produces significantly greater crop yields per unit of
land than extensive agriculture, which uses little capital or labour. As a result, a farm using
intensive agriculture will require less land than an extensive agriculture farm to produce a similar
profit. In practice, however, the increased economies and efficiencies of intensive agriculture often
encourage farm operators to work very large tracts in order to keep their capital investments in
machinery productively engaged—i.e., busy. On the level of theory, the increased productivity of
intensive agriculture enables the farmer to use a relatively smaller land area that is located close to
market, where land values are high relative to labour and capital, and this is true in many parts of
the world. If costs of labour and capital outlays for machinery and chemicals, and costs of storage
(where desired or needed) and transportation to market are too high then farmers may find it more
profitable to turn to extensive agriculture. However, in practice many relatively small-scale farmers
employ some combination of intensive and extensive agriculture, and many of these operate
relatively close to markets. Many large-scale farm operators, especially in such relatively vast and
agriculturally advanced nations as Canada and the United States, practice intensive agriculture in
areas where land values are relatively low, and at great distances from markets, and farm enormous
tracts of land with high yields. However, in such societies overproduction (beyond market demands)
often results in diminished profit as a result of depressed prices.

4. Which of following is not true regarding India’s first Green Revolution:


A. It is the introduction of new techniques of agriculture which became popular by the name of the
Green Revolution (GR) around the world in early 1960s.
B. It revolutionised the very traditional idea of food production by giving a boost by more than 250
per cent to the productivity level.
C. The Green Revolution was centred on the use of the Genetically Modified variety (GMV) of
seeds developed by the US agro-scientist Norman Borlaug doing research on a British Rockfellor
Foundation Scholarship in Mexico by the early 1960s.
D. By 1965, the seeds were successfully tested and were being used by farmers in food deficient
countries such as Mexico, Taiwan.

Answer: C

Explanation: It is the introduction of new techniques of agriculture which became popular by the
name of the Green Revolution (GR) around the world in early 1960s—at first for wheat and by the
next decade for rice, too. It revolutionised the very traditional idea of food production by giving a
boost by more than 250 per cent to the productivity level. The Green Revolution was centred around
the use of the High Yielding variety (HYV) of seeds developed by the US agro-scientist Norman
Borlaug doing research on a British Rockfellor Foundation Scholarship in Mexico by the early
1960s.The new wheat seeds which he developed in vivo claimed to increase its productivity by more
www.achieveias.co.in, YouTube Channel: http://youtube.com/c/AchieveIAS Telegram Channel: http://t.me/Achieve_Ias,
Mail: achieveias21@gmail.com, Contact Number: 8968920720

than 200 per cent. By 1965, the seeds were successfully tested and were being used by farmers in
food deficient countries such as Mexico, Taiwan.

5. With reference to micro-irrigation, which of the following statements is/are correct?


1. Fertilizer/nutrient loss can be reduced.
2. It is the only means of irrigation in dry land farming.
3. In some areas of farming, receding of ground water table can be checked.

Select the correct answer using the codes given below:


A. 1 only B. 2 and 3 only C. 1 and 3 only D. 1, 2 and 3

Answer: C

Explanation: Micro irrigation is a modern method of irrigation; by this method water is irrigated
through drippers, sprinklers, foggers and by other emitters on surface or subsurface of the land.
Major components of a micro irrigation system is as follows. Water source, pumping devices (motor
and pump), ball valves, fertigation equipments, filters, control valves, PVC joining accessories (Main
and sub main) and emitters. In this system water is applied drop by drop nearer the root zone area
of the crop. The drippers are fixed based on the spacing of crop. Many different types of emitters
are available in the market. They are classified as Inline drippers, on line drippers, Micro tubes,
Pressed compensated drippers. Drip irrigation is most suitable for wider spacing crops. Micro
sprinkler irrigation system is mostly followed in sandy or loamy soils. This system is most suitable
to horticultural crops and small grasses. In this method water is sprinkled in a lower height at various
directions. Portable micro sprinklers are also available. They distribute slightly more water than
drippers and micro sprinklers. They spray water in not more than one meter. It is used for preparing
nursery and lawns in soils with low water holding capacity.

Advantages of drip irrigation system:


1. Water saving and higher yield
2. High quality and increased fruit size
3. Suitable for all types of soil
4. Easy method of fertigation and chemigation
5. Saving in labour and field preparation cost

Disadvantage of drip irrigation system:


1. High initial investment
2. Clogging of emitters
3. Possible damage of system components due to animals, etc.,

Investment cost mostly differs based on spacing of the crops:


1. Generally, the reasons for clogging are solid particles (sand, rust), soft dirt (organic matter, algae,
microorganism, salt), sediments (salt in the fertilizers).
2. Filtration is the main key factor to the success or failure of the system. The main of filtration is to
stop dirt particles which damage any components of the system.
3. To remove salt encrustation, 30 per cent commercial hydrochloric acid can been used at the rate
of one liter per one m3 area. (One part HCl mixed with 5 parts of water)
4. To remove algae and fungal clogging 5 to 500 ppm sodium hydrochloride (10 per cent chlorine)
can be used.
www.achieveias.co.in, YouTube Channel: http://youtube.com/c/AchieveIAS Telegram Channel: http://t.me/Achieve_Ias,
Mail: achieveias21@gmail.com, Contact Number: 8968920720

Maintenance of drip system:


1. Back washing and sand filters has to be cleaned
2. Frequent cleaning of emitters and drippers
3. Flushing at every irrigation
4. Cleaning of sub main and main pipes
5. Cleaning of PVC pipes and laterals and acid or chlorine may be used to remove clogging.

6. What percentage of Indian population is dependent on agriculture?


A. 65% of total population B. 53% of total population
C. 40% of total population D. 65% of total population

Answer: B

7. How much percentage is contributed by the industry sector in the economy of india?
A. 48% B. 10% C. 29.6% D. 36%

Answer: C

Explanation: Agriculture & Allied Sector: This sector includes forestry and fishing also. This sector
is also known as the primary sector of the economy. At the time of Indian independence, this sector
had the biggest share in the Gross Domestic Product of India. But year by year its contribution goes
on declining and currently, it contributes only 17% of Indian GDP at current prices. It is worth to
mention that the agriculture sector provides jobs to around 53% population of India.
2. Industry Sector: This sector includes 'Mining & quarrying', Manufacturing (Registered &
Unregistered), Gas, Electricity, Construction, and Water supply. This is also known as the secondary
sector of the economy. Currently, it is contributing around 29.6 % of the Indian GDP (at current
prices) in 2018-19.

8. How many setters are covered in the calculation of the Index of Industrial Production (IIP)?
A. 5 B. 6 C. 7 D. 8

Answer: D

Explanation: The Central Statistics Office (CSO) which comes under the Ministry of Statistics and
Programme Implementation, revised the base year of the Index of Industrial Production (IIP) from
2004-05 to 2011-12.It is revised to not only reflect the changes in the industrial sector but to also
align it with the base year of the Wholesale Price Index (WPI) and Gross Domestic Product (GDP).
In the calculation of Index of Industrial Production (IIP), the data of the following 8 sectors is
measured. These sectors are;
1. Coal: Its weight is 10.33%.
2. Crude oil: Its weight is 8.98%.
3. Natural gas: Its weight is 6.88%.
4. Refinery Product: Its weight is 28.04%.
5. Steel: Its weight is 17.92%.
6. Cement: Its weight is 5.37%.
7. Fertilizer: Its weight is 2.63%.
8. Power: Its weight is 19.85%.
www.achieveias.co.in, YouTube Channel: http://youtube.com/c/AchieveIAS Telegram Channel: http://t.me/Achieve_Ias,
Mail: achieveias21@gmail.com, Contact Number: 8968920720

9. Which among the following does not belong to India's major large scale industries?
A. Cotton textile industry B. Iron and steel industry
C. Jute industry D. Khadi and village industry

Answer: D
Explanation: Industries which requires huge infrastructure and manpower with an influx of
capital assets are Large Scale Industries. In India, large-scale industries are the ones with a fixed
asset of more than one hundred million rupees or Rs. 10 crores. The Indian economy relies heavily
on such industries for economic growth, generation of foreign currency, and the creation of job
opportunities for millions of Indians. Here are some advantages of large scale industries:
1. They provide an impetus to the industrialization of the country.
2. Large scale industries, usually, produce capital and basic goods (instruments,
machines, chemicals, etc.)
3. They are capable of generating funds for the research and development of new technologies.
4. Due to the large scale of operations, they have the potential to lower the cost of goods.
5. Further, they create opportunities for small-scale and cottage industries to evolve and flourish.
6. Also, the employment opportunities created by large scale industries are huge.

10. Consider the following statements and identify the right ones.
1. The 1991 industrial reforms exempted all industries from compulsory licensing
2. There are six industries under compulsory licensing today

A. 1 only B. 2 only C. Both A & B D. None of the above

Answer: B

Explanation: The Industrial Development and Regulation Act (IDRA), 1951 provides a basic
framework for the growth and development of industries in India. The Act mandates every existing
or new industrial undertaking to register itself with the federal government. According to the law, an
‘industrial undertaking’ is a planned industry that is carried on in one or more factories owned by an
individual or authority, including government.

Industrial licensing in India: Since the liberalization and deregulation of the Indian economy in
1991, most industries have been exempt from obtaining an industrial license to start manufacturing
in India. Government attention is reserved only for those industries that may impact public health,
safety, and national security. In India, industrial licenses are regulated by the IDRA, 1951 Act, and
are approved by the Secretarial of Industrial Assistance (SIA) on the recommendation of the
licensing committee. The provisions of the Act restrict a licensed industrial undertaking from
manufacturing a new article unless the license has been renewed or a new license has been
obtained to include the new article. Industries that require industrial licensing for manufacturing in
India include:
1. Industries under compulsory licensing; and,
2. Industrial undertakings attracting locational restrictions. The licensing provision also applies to
the expansion of the existing industrial units.

Recent amendment to industrial licensing rule: Earlier, large industries that manufactured items that
were exclusively reserved for Micro, Small, and Medium Enterprises (MSME) also needed to obtain
www.achieveias.co.in, YouTube Channel: http://youtube.com/c/AchieveIAS Telegram Channel: http://t.me/Achieve_Ias,
Mail: achieveias21@gmail.com, Contact Number: 8968920720

an industrial license. MSMEs were previously known as Small Scale Industry (SSI). The provision
was aimed at protecting indigenous manufacturers from unequal competition with large scale
industries. However, in April 2015, the government de-reserved these items to encourage greater
investment, incorporate better technologies, and enhance competition in the Indian and global
market for the products. Large industries are now permitted to manufacture items such as – bread,
wood, firework, pickles and chutneys, mustard oil, groundnut oil, steel chairs and tables, padlocks,
stainless steel and aluminum utensils, without obtaining an industrial license.

Industries subject to compulsory licensing in India:


Businesses planning to establish industries to produce any of the following items in India must obtain
a compulsory license:
1. Distillation and brewing of alcoholic drinks;
2. Cigars and cigarettes of tobacco and manufactured tobacco substitutes;
3. Electronics and aerospace and defense equipment;
4. Industrial explosives including detonating fuses, safety fuses, gun powder, nitrocellulose and
matches; and
5. Hazardous chemicals including items hazardous to human safety and health and thus fall for
mandatory licensing.
These industries are under compulsory licensing mainly because of environmental, safety and
strategic considerations. Compulsory licensing is regulated by the Ministry of Industrial
Development.
www.achieveias.co.in, YouTube Channel: http://youtube.com/c/AchieveIAS Telegram Channel: http://t.me/Achieve_Ias,
Mail: achieveias21@gmail.com, Contact Number: 8968920720

ACHIEVE IAS GEOGRAPHY MCQ SERIES, DAY 50, SOLUTIONS

1. Consider the following statement (s) related to Hamleted rural settlement.


1. Units are locally called panna, para, palli, nagla, dhani, etc. in various parts of the country.
2. A pattern may also result from segregation or fragmentation of a large compact village.

Which is / are correct option?


A. Only 1 B. Only 2 C. Both 1 and 2 D. Neither 1nor 2

Answer: A

Explanation: Sometimes settlement is fragmented into several units physically separated from each
other bearing a common name. These units are locally called panna, para, palli, nagla, dhani, etc.
in various parts of the country. This segmentation of a large village is often motivated by social and
ethnic factors. A hamlet is a small settlement, smaller than a village. Usually, all settlers in a hamlet
are centered around a single economic activity. A hamlet may consist of a farm, a mill, a mine or
a harbor. All the people living there would be workers on that farm, mill, mine or harbour. Hamlets,
especially those with a medieval church may have resulted from a medieval village, that was
abandoned for some reason. Because of the small size of the settlement, there are usually no
buildings which have a central or admninistrative function.5-6 families may live there usually within
a 2km radius area.

2. Consider the following statement (s) related to dispersed or isolated rural settlement.
1. Pattern of settlement appears in the form of isolated huts or hamlets of few huts in remote jungles,
or on small hills with farms or pasture on the slopes.
2. Extreme dispersion of settlement is often caused by extremely fragmented nature of the terrain
and land resource base of habitable areas.

Which is / are correct option?


A. 1 Only B. 2 Only C. Both 1 and 2 D. Neither 1 nor 2

Answer: C

Explanation: A dispersed settlement, also known as a scattered settlement, is one of the main
types of settlement patterns used by landscape historians to classify rural settlements found in
India and other parts of the world. Typically, there are a number of separate farmsteads scattered
throughout the area. In these settlements, houses are spaced far apart and often interspersed with
fields. A cultural feature such as a place of worship or a market, binds the settlement together. Such
settlements are found in the western Malwa Plateau, where they occur in association with dispersed
clusters. These are also found in areas of Western Ghats stretching from Satara towards Kerala
high lands and also in some portions of Himalayan ranges in Kashmir and Uttar Pradesh. These
isolated homesteads are also found unoccupied due to frequent floods in eastern Uttar Pradesh and
Bihar districts. Thus, isolated homesteads seem to grow according to the exigencies of local
circumstances. This pattern of settlement appears in the form of isolated huts or hamlets of few huts
in remote jungles, or on small hills with farms or pasture on the slopes. Extreme dispersion of
settlement is often caused by extremely fragmented nature of the terrain and land resource base of
habitable areas.
www.achieveias.co.in, YouTube Channel: http://youtube.com/c/AchieveIAS Telegram Channel: http://t.me/Achieve_Ias,
Mail: achieveias21@gmail.com, Contact Number: 8968920720

3. Consider the following statement (s) related to rural settlements.


1. On the basis of setting: The main types are plain villages, plateau villages, coastal villages, forest
villages and desert villages.
2. On the basis of functions: There may be farming villages, fishermen’s villages, lumberjack
villages, pastoral villages etc.

Which is / are correct option?


A. Only 1 B. Only 2 C. Both 1 and 2 D. Neither1 nor 2

Answer: C

Explanation: Rural Settlement Patterns Patterns of rural settlements contemplate the way the
houses are sited in relation to each other. The site of the village, the surrounding topography and
terrain influence the shape and size of a village. Rural settlements may be classified on the basis
of a number of criteria:
(i) On the basis of setting: The main types are plain villages, plateau villages, coastal villages,
forest villages and desert villages.
(ii) On the basis of functions: There may be farming villages, fishermen’s villages, lumberjack
villages, pastoral villages etc.
(iii) On the basis of forms or shapes of the settlements: These may be a number of geometrical
forms and shapes such as Linear, rectangular, circular star like, T-shaped village, double village,
cross-shaped village etc.
• Linear pattern: In such settlements houses are located along a road, railway line, and river, canal
edge of a valley or along a levee.
• Rectangular pattern: Such patterns of rural settlements are found in plain areas or wide inter
montane valleys. The roads are rectangular and cut each other at right angles.
• Circular pattern: Circular villages develop around lakes, tanks and sometimes the village is
planned in such a way that the central part remains open and is used for keeping the animals to
protect them from wild animals.
• Star like pattern: Where several roads converge, star shaped settlements develop by the houses
built along the roads.
• T-shaped, Y-shaped, Cross-shaped or cruciform settlements: T –shaped settlements develop at
tri-junctions of the roads while –shaped settlements emerge as the places where two roads
converge on the third one and houses are built along these roads. Cruciform settlements develop
on the cross-roads and houses extend in all the four direction.
• Double village: These settlements extend on both sides of a river where there is a bridge or a ferry.

4. Consider the following statement (s) related to Rural Urban Settlement Dichotomy.
1. The term settlement is accepted but when it comes to its existence that can be differentiated in
terms of rural and urban, but there is no consensus on what exactly defines a village or a town.
2. The basic difference between towns and villages is that in towns the main occupation of the
people is related to secondary and tertiary sectors, while in the villages most of the people are
engaged in primary occupations.

Which is / are correct option?


A. 1 Only B. 2 Only C. Both 1 and 2 D. Neither 1 nor 2

Answer: C
www.achieveias.co.in, YouTube Channel: http://youtube.com/c/AchieveIAS Telegram Channel: http://t.me/Achieve_Ias,
Mail: achieveias21@gmail.com, Contact Number: 8968920720

Explanation: Human Settlement is a form of human habitation which ranges from a single dowelling
to large city. In other words, it is a process of opening up and settling of a previously uninhabited
area by the people. People live in clusters of houses that might be a village, a town or a city. The
study of human settlements is basic to human geography because the form of settlement in any
particular region reflects human relationship with the environment. A human settlement is defined
as a place inhabited more or less permanently. The houses may be designed or redesigned,
buildings may be altered, functions may change but settlement continues in time and space. There
may be some settlements which are temporary and are occupied for short periods, may be a season.

Rural Urban Settlement Dichotomy: The term settlement is accepted but when it comes to its
existence that can be differentiated in terms of rural and urban, but there is no consensus on what
exactly defines a village or a town. Although population size is an important criterion, it is not a
universal criterion since many villages in densely populated countries of India and China have
population exceeding that of some towns of Western Europe and United States. At one time, people
living in villages pursued agriculture or other primary activities, but presently in developed countries,
large sections of urban populations prefer to live in villages even though they work in the city. The
basic difference between towns and villages is that in towns the main occupation of the people is
related to secondary and tertiary sectors, while in the villages most of the people are engaged in
primary occupations such as agriculture, fishing, lumbering, mining, animal husbandry, etc.
Differentiations between rural and urban on the basis of functions are more meaningful even though
there is no uniformity in the hierarchy of the functions provided by rural and urban settlements. Petrol
pumps are considered as a lower order function in the United States while it is an urban function in
India. Even within a country, rating of functions may vary according to the regional economy.
Facilities available in the villages of developed countries may be considered rare in villages of
developing and less developed countries.

5. Which of the following is not the objective of the WTO?


A. To improve the standard of living of peoples of the member countries
B. To enlarge production and trade of goods
C. To protect environment
D. To improve the Balance of Payment situation of the member countries

Answer: D

Explanation: WTO is not responsible of the improvement in the Balance of Payment of the member
countries while other options given in the question are objectives of the WTO.
Created in 1995, the World Trade Organization (WTO) is an international institution that
oversees the global trade rules among nations. It superseded the 1947 General Agreement on
Tariffs and Trade (GATT) created in the wake of World War II. The WTO is based on agreements
signed by the majority of the world’s trading nations. The main function of the organization is to help
producers of goods and services, exporters, and importers protect and manage their businesses.
As of 2019 the WTO has 164 member countries, with Liberia and Afghanistan the most recent
members, having joined in July 2016, and 23 “observer” countries. The WTO is essentially an
alternative dispute or mediation entity that upholds the international rules of trade among nations.
The organization provides a platform that allows member governments to negotiate and
resolve trade issues with other members. The WTO’s main focus is to provide open lines of
communication concerning trade between its members.
www.achieveias.co.in, YouTube Channel: http://youtube.com/c/AchieveIAS Telegram Channel: http://t.me/Achieve_Ias,
Mail: achieveias21@gmail.com, Contact Number: 8968920720

For example, the WTO has lowered trade barriers and increased trade among member countries.
On the other hand, it has also maintained trade barriers when it makes sense to do so in the global
context. Therefore, the WTO attempts to provide negotiation mediation that benefits the
global economy. Once negotiations are complete and an agreement is in place, the WTO then offers
to interpret that agreement in the event of a future dispute. All WTO agreements include a settlement
process, whereby the organization legally conducts neutral conflict resolution.
No negotiation, mediation, or resolution would be possible without the foundational WTO
agreements. These agreements set the legal ground rules for international commerce that the WTO
oversees. They bind a country’s government to a set of constraints that must be observed
when setting future trade policies. These agreements protect producers, importers, and
exporters while encouraging world governments to meet specific social and environmental
standards. Advantages and Disadvantages of the World Trade Organization (WTO): The history of
international trade has been a battle between protectionism and free trade, and the WTO has fueled
globalization with both positive and adverse effects. The organization’s efforts have increased global
trade expansion, but a side effect has been a negative impact on local communities and human
rights. Proponents of the WTO, particularly multinational corporations (MNCs), believe that the
organization is beneficial to business, seeing the stimulation of free trade and a decline in trade
disputes as beneficial to the global economy. Skeptics believe that the WTO undermines the
principles of organic democracy and widens the international wealth gap. They point to the decline
in domestic industries and increasing foreign influence as negative impacts on the world economy.
As part of his broader attempts to renegotiate U.S. international trade deals, President Trump has
threatened to withdraw from the WTO, calling it a “disaster.” A U.S. withdrawal from the WTO could
disrupt trillions of dollars in global trade.

6. Which of the following factor forms the invisible account of the Balance of Payments of a
country?
A. International trade in services.
B. Income associated with non-resident assets and liabilities.
C. Remittance of worker income.
D. All the above.

Answer: D

Explanation: Balance of payment broadly comprises of current and capital accounts. Current
account records export and import of goods (visible items), export and import of services (invisible
items) and unilateral transfers from one country to another.

7. What is the objective of the ASEAN?


A. To increase competition among the members countries for better products.
B. To integrate the economies of the region to make region more attractive for investors
C. To promote economic cooperation in South East Asia and ensure economic stability in the region.
D. b and c both

Answer: D
Explanation: ASEAN nations want to promote economic cooperation in South East Asia and ensure
economic stability in the region and to integrate the economies of the region to make region more
attractive for investors. The Association of Southeast Asian Nations (more commonly known as
ASEAN) is an intergovernmental organization aimed primarily at promoting economic growth and
www.achieveias.co.in, YouTube Channel: http://youtube.com/c/AchieveIAS Telegram Channel: http://t.me/Achieve_Ias,
Mail: achieveias21@gmail.com, Contact Number: 8968920720

regional stability among its members.There are currently 10 member states: Indonesia, Malaysia,
Philippines, Singapore, Thailand, Brunei, Laos, Myanmar, Cambodia and Vietnam.

Why was it set up?


ASEAN was founded half a century ago in 1967 by the five Southeast Asian nations of Indonesia,
Malaysia, Philippines, Singapore and Thailand. This was during the polarized atmosphere of the
Cold War, and the alliance aimed to promote stability in the region. Over time, the group expanded
to include its current 10 members. Regional cooperation was further extended with the creation of
the ASEAN Plus Three forum in 1997, which included China, South Korea and Japan. And then
the East Asia Summit, which began taking place in 2005 and has expanded to include India,
Australia, New Zealand, Russia and the United States.

What does it do now?


ASEAN aims to promote collaboration and cooperation among member states, as well as to advance
the interests of the region as a whole, including economic and trade growth. It has negotiated a free
trade agreement among member states and with other countries such as China, as well as eased
travel in the region for citizens of member countries. In 2015, it established the ASEAN Economic
Community (AEC), a major milestone in the organization’s regional economic integration agenda.
The AEC envisions the bloc as a single market with free flow of goods, services, investments and
skilled labour, and freer movement of capital across the region.True to its original mission, the
organization strives towards peace and stability in the region: members have signed a treaty
pledging not to develop nuclear weapons, and most have agreed to a counter-terrorism pact, which
includes sharing intelligence and easing the extradition process of terror suspects.

How important is the region economically? If ASEAN were a country, it would be the seventh-largest
economy in the world, with a combined GDP of $2.6 trillion in 2014. By 2050 it's projected to rank
as the fourth-largest economy. Home to more than 622 million people, the region has a larger
population than the European Union or North America. It also has the third-largest labour force in
the world, behind China and India.

8. Which of the following statement is NOT correct about the EU?


A. Germany is the founder member of the EU
B. Croatia is the latest member of the EU.
C. Among all the members of the EU, Malta has smallest area
D. All the member countries of the EU use EURO as the currency of their state.

Answer: D

Explanation: The EURO is the official currency of the European Union. Currently 19 of 28 member
states use the euro: this group of states is known as the Euro zone. It is the second most traded
currency in the Foreign Exchange Market after the US dollar. The European Union is a unified trade
and monetary body of 28 member countries. It eliminates all border controls between members.
That allows the free flow of goods and people, except for random spot checks for crime and
drugs. The EU transmits state-of-the-art technologies to its members. The areas that benefit are
environmental protection, research and development, and energy. Public contracts are open to
bidders from any member country. Any product manufactured in one country can be sold to any
other member without tariffs or duties. Taxes are all standardized. Practitioners of most services,
www.achieveias.co.in, YouTube Channel: http://youtube.com/c/AchieveIAS Telegram Channel: http://t.me/Achieve_Ias,
Mail: achieveias21@gmail.com, Contact Number: 8968920720

such as law, medicine, tourism, banking, and insurance, can operate in all member countries. As a
result, the cost of airfares, the internet, and phone calls have fallen dramatically.
Purpose: Its purpose is to be more competitive in the global marketplace. At the same time, it must
balance the needs of its independent fiscal and political members.
What Countries Are EU Members: The EU's 28 member countries are: Austria, Belgium, Bulgaria,
Croatia, Cyprus, Czech Republic, Denmark, Estonia, Finland, France, Germany, Greece, Hungary,
Ireland, Italy, Latvia, Lithuania, Luxembourg, Malta, Netherlands, Poland, Portugal, Romania,
Slovakia, Slovenia, Spain, Sweden, and the United Kingdom. That will drop to 27 after Brexit causes
the United Kingdom to leave the EU in 2019.
How It Is Governed: Three bodies run the EU. The EU Council represents national governments.
The Parliament is elected by the people. The European Commission is the EU staff. They make
sure all members act consistently in regional, agricultural, and social policies. Contributions of 120
billion euros a year from member states fund the EU.
Here's how the three bodies uphold the laws governing the EU. These are spelled out in a series of
treaties and supporting regulations:
1. The EU Council sets the policies and proposes new legislation. The political leadership, or
Presidency of the EU, is held by a different leader every six months.
2. The European Parliament debates and approves the laws proposed by the Council. Its
members are elected every five years.
3. The European Commission staffs and executes the laws. Jean-Claude Juncker is the
president until October 2019.
Currency: The euro is the common currency for the EU area. It is the second most commonly held
currency in the world, after the U.S. dollar. It replaced the Italian lira, the French franc, and the
German deutschmark, among others.
The value of the euro is free-floating instead of a fixed exchange rate. As a result, foreign exchange
traders determine its value each day. The most widely-watched value is how much the euro's value
is compared to the U.S. dollar. The dollar is the unofficial world currency.
The Difference Between the Eurozone and the EU:
The eurozone consists of all countries that use the euro.1 All EU members pledge to convert to the
euro, but only 19 have so far. They are Austria, Belgium, Cyprus, Estonia, Finland, France,
Germany, Greece, Ireland, Italy, Latvia, Lithuania, Luxembourg, Malta, Netherlands, Portugal,
Slovakia, Slovenia, and Spain. The eurozone was created in 2005. The European Central Bank is
the EU's central bank. It sets monetary policy and manages bank lending rates and foreign
exchange reserves. Its target inflation rate is less than 2%. The EU's trade structure has propelled
it to become the world's second-largest economy after China.6 In 2018, its gross domestic
product was $22 trillion, while China's was $25.3 trillion. These measurements use purchasing
power parity to account for the discrepancy between each country's standard of living. The United
States was third, producing $20.5 trillion, according to estimates by the International Monetary Fund.
But the EU's success is not evenly distributed. Italy, Greece, and Cyprus have high levels of public
and private debt, including bad bank loans. Italy also has high unemployment while France suffers
from low productivity. Germany has a large trade surplus. Many countries need reforms of their
pension systems and labor markets.

9. Consider the following statements


1. In India, Semi-Clustered Settlements is a rather universal feature and particularly in the northern
plains
2. Dominant community and lower strata of society is clearly visible in Semi-Clustered Settlements
www.achieveias.co.in, YouTube Channel: http://youtube.com/c/AchieveIAS Telegram Channel: http://t.me/Achieve_Ias,
Mail: achieveias21@gmail.com, Contact Number: 8968920720

Which is/are CORRECT?


A. 1 Only B. 2 Only C. Both 1 & 2 D. None of the above

Answer: B

Explanation: Semi-Clustered Settlements Semi-clustered or fragmented settlements may result


from tendency of clustering in a restricted area of dispersed settlement. More often such a pattern
may also result from segregation or fragmentation of a large compact village. In this case, one or
more sections of the village society choose or is forced to live a little away from the main cluster or
village. In such cases, generally, the land-owning and dominant community occupies the central
part of the main village, whereas people of lower strata of society and menial workers settle on the
outer flanks of the village. Such settlements are widespread in the Gujarat plain and some parts of
Rajasthan.

10. With reference to Asian Infrastructure Investment Bank (AIIB), consider the following
statements
1. AIIB has more than 80 member nations.
2. India is the largest shareholder in AIIB.
3. AIIB does not have any members from outside Asia.

Which of the statements given above is / are correct?


A. 1 only B. 2 and 3 only C. 1 and 3 only D. 1, 2 and 3

Answer: A

Explanation: Headquartered in Beijing, the Asian Infrastructure Investment Bank (AIIB) is a


development bank established to boost lending for projects in sectors like energy, transportation,
urban construction, logistics, education and healthcare. AIIB presently has 97 approved members
worldwide. The Bank has invested in 13 member regions; it has members from outside Asia in Egypt
(Middle East), Azerbaijan (Asia and Europe) and others. The three largest shareholders of the bank
are China, India and Russia.

You might also like